You are on page 1of 179

THE FRATERNAL ORDER OF SAINT THOMAS

MORE
TAU MU Fraternity and SIGMA TAU MU
Sorority
Ateneo de Davao University College of Law
-PRAYER by SAINT THOMAS MORE-

TAU MU
TAU MU
TAU MU
TAU MU
TAU MU
TAU MU
TAU MU
TAU MU

Bill of Rights (Definition)


It is the set of prescriptions setting forth the
fundamental civil and political rights of the
individual, and imposing the limitations on the
powers of government as a means of securing the
enjoyment of those rights. The Bill of Rights is
designed to preserve the ideals of liberty,
equality and security against the assaults of
opportunism, the expediency of the passing hour,
the erosion of small encroachments, and the

TAU MU

CONSTITUTIONAL LAW II

TAU MU
TAU MU

The above was written while St. Thomas


was imprisoned in the Tower of London.

The Fraternal
Ateneo de Davao

TAU MU
TAU MU

Give me the grace, Good Lord


To set the world at naught. To set the mind firmly
on You and not to hang upon the words of men's
mouths.
To be content to be solitary. Not to long for
worldly pleasures. Little by little utterly to cast of
the world and rid my mind of all its business.
Not to long to hear of earthly things, but that the
hearing of worldly fancies may be displeasing to
me.
Gladly to be thinking of God, piteously to call for
His help. To lean into the comfort of God. Busily to
labor to love Him.
To know my own vileness and wretchedness. To
humble myself under the mighty hand of God. To
bewail my sins and, for the purging of them,
patiently to sufer adversity.
Gladly to bear my purgatory here. To be joyful in
tribulations. To walk the narrow way that leads to
life.
To have the last thing in remembrance. To have
ever before my eyes my death that is ever at
hand. To make death no stranger to me. To
foresee and consider the everlasting fire of Hell.
To pray for pardon before the judge comes.
To have continually in mind the passion that
Christ sufered for me. For His benefits
unceasingly to give Him thanks.
To buy the time again that I have lost. To abstain
from vain conversations. To shun foolish mirth
and gladness. To cut of unnecessary recreations.
Of worldly substance, friends, liberty, life and all,
to set the loss at naught, for the winning of
Christ.
To think my worst enemies my best friends, for
the brethren of Joseph could never have done him
so much good with their love and favor as they
did him with their malice and hatred.
These minds are more to be desired of every man
than all the treasures of all the princes and kings,
Christian and heathen, were it gathered and laid
together all in one heap.
Amen

KITY

TAU MU TAU MU TAU MU


TAU MU TAU MU TAU MU

CONSTITUTIONAL LAW II
Order of Saint Thomas More
Atty. Philip John Pojas/Atty. Rovyne G. Jumao-as, RN
University College of Law

scorn and derision of those who have no patience


with general principles. [Nachura, Outline
Reviewer in Political Law 2006, p. 81]
Significance of the Bill of Rights
Government is powerful. When unlimited, it
becomes tyrannical. The Bill of rights is a
guarantee that there are certain areas of a
persons life, liberty, and property which
governmental power may not touch. All powers of
the government are limited by the Bill of Rights.
[Bernas, The 1987 Philippine Constitution: A
Comprehensive Reviewer, p. 22]
Classification of Rights
1. Political Rights granted by law to
members of community in relation to their
direct or indirect participation in the
establishment or administration of the
government [San Beda Reviewer 2006, p.
7]. They refer to the right to participate,
directly or indirectly, in the establishment
or administration of government, e.g. the
right of sufrage, the right to hold public
office, etc. [Nachura, Outline Reviewer in
Political Law 2006, p. 81];
2. Civil Rights rights which municipal law
will enforce at the instance of private
individuals for the purpose of securing
them the enjoyment of their means of
happiness [San Beda Reviewer 2006, p.
7]. Those rights that belong to every
citizen of the state or country, or, in a
wider sense, to all its inhabitants, and are
not connected with the organization or
administration
of government.
They
include the rights to property, marriage,
equal protection of the laws, freedom of
contract, etc. [Nachura, Outline Reviewer
in Political Law 2006, p. 81] ;
3. Social and Economic Rights;
4. Human Rights;
The diference between the guarantees of the Bill
of Rights and the guarantees that are found in
Article XIII on social justice: the Bill of Rights
focuses on civil and political rights, whereas
Article XIII focuses on social and economic rights.
Moreover, the guarantees in the Bill of Rights are
generally self-implementing, i.e. they can be
appealed to even in the absence of implementing
legislation. On the other hand, the social and
economic rights guaranteed in Article XIII
generally
require
implementing
legislation.
[Bernas, The 1987 Philippine Constitution: A
Comprehensive Reviewer, p. 22]
*Human Rights vs. Property Rights: Human Rights
will prevail over Property Rights (Phil. Blooming
Mills case)
*In a democracy,
the preservation
and
enhancement of the dignity and worth of a
person is the central core as well as the cardinal
article of ?faith? in our civilization. The inviolable
character of man as an individual must be
protected to the largest possible extent in his
thoughts and in his beliefs as the citadel of his
person.

ACADCOM 2010; Contributors: Gene Geocaniga, Jarissa Guiani, Darlene Magabilen


TAU MU Page 1 of 179

CONSTITUTIONAL LAW II
Order of Saint Thomas More
Atty. Philip John Pojas/Atty. Rovyne G. Jumao-as, RN
University College of Law

KITY

Ateneo de Davao

are

superior

to

PBMEO vs. PBMCI


51 SCRA 189 (1973)

TAU MU
TAU MU
TAU MU
TAU MU
TAU MU
TAU MU

Facts: Private respondent was dismissed by her


employer for dishonesty. The evidence against
her consisted of a check worth P640.00 which
was discovered by her co-employee who opened
an envelope addressed to respondent. The NLRC

TAU MU

WATEROUS DRUG vs. NLRC


280 SCRA 735 (1997)

TAU MU

Facts: Andre Marti and his wife wanted to have


some packages delivered to Switzerland by a
forwarding company. In accordance with the SOP
of the company, the company inspected the
package. It turned out that the package
contained marihuana which was neatly stashed to
avoid detection. A case was filed against Marti by
the State for violation of the Dangerous Drugs
Act. Marti contested that there was an illegal
search
and
invoked
his
right
against
unreasonable search, therefore, the evidences
should be held inadmissible in court. TAU MU
Issue: W/N BOR can be invoked against a private
entity
Held: The court ruled that he cannot invoke this
right because the Bill of Rights can only be
invoked against the State. It governs the
relationship of the State and its citizens. It does
not apply to issues between two individuals (Marti
and the forwarding company). The search was
conducted by a private individual and not a peace
officer. The police were just looking as the
proprietor did the search of the package.

TAU MU

PEOPLE vs. MARTI


193 SCRA 57 (1991)

TAU MU
TAU MU

Bill of Rights can only be invoked against


the State

TAU MU
TAU MU

Facts: Union wanted to participate in a


demonstration. The management allowed them
on the condition that it should not be during their
shift so that operation will not be hampered. But
union wanted all of the members to participate at
the same time and they did. As a consequence,
the management dismissed the officers of the
union. TAU MU
Issue: Whether the rights of the workers (right to
assembly, expression and petition for redress of
grievances) should be given more importance
over the rights of the management-property
rights
Held: The court ruled in favor of the workers.
Human rights enjoy primacy over property rights.
Not all rights are equal. There is a hierarchy of
rights in the Bill.

TAU MU TAU MU TAU MU


TAU MU TAU MU TAU MU

BASIC PRINCIPLES
Basic Human Rights
Property Rights

The Fraternal

ruled that the dismissal was illegal as the only


evidence supporting the charge is inadmissible
for having been obtained in violation of her right
against unreasonable searches and seizures.
Held: The evidence is inadmissible. The search
was made by a private individual. [As ruled in
People v. Marti, the Constitution, in laying down
the principles of government and fundamental
liberties of the people, does not govern
relationship
between
individuals.
The
constitutional
prescription
against
unlawful
searches and seizures therefore applies as a
restraint directed against the government and its
agencies tasked with the enforcement of the law.
Thus, it could be invoked only against the State
to whom the restraint against arbitrary and
unreasonable exercise of power is imposed.] The
recourse of citizens against such assaults by
private individuals is through criminal and civil
proceedings, not the exclusionary rule.
PEOPLE vs. DOMANTAY
307 SCRA 1 (1999)
Facts: Accused was charged with rape with
homicide. While detained in a municipal jail, he
was interviewed by a radio reporter during which
he confessed to the crime. Two or three meters
away from the reporters were policemen, but no
lawyer assisted the accused. Is the confession of
accused to the reporter admissible?
Held: Yes. The confession is not covered by Sec.
12 (1) of the Bill of Rights. The Bill of Rights does
not concern itself with the relation between a
private individual and another individual.
It
governs the relationship between the individual
and the State.
The prohibitions therein are
primarily addressed to the State and its agents.
In this case, the presence of the police officers 23 meters away did not exert undue pressure or
influence on accused or coerced him into giving
his confession. Accused could have refused to be
interviewed, but instead he agreed.
PEOPLE vs. MENDOZA
301 SCRA 66 (1999)
Facts: Accused is charged with parricide. After
the death of his wife, her father came to the
house and took her personal belongings as well
as a Mission Order and a Memorandum Receipt
for a .38 caliber Colt Revolver issued in favor of
accused. Accused claims that the documents
were procured in violation of his constitutional
right against unreasonable searches.
Held: the right invoked by accused is a restraint
directed only against the government and its
agencies. The constitutional protection against
unreasonable searches and seizures refers to the
immunity of ones person from interference by
government and it cannot be extended to acts
committed by private individuals so as to bring it
within the ambit of alleged unlawful intrusion. In
the instant case, the memorandum receipt and
mission order were discovered by accuseds
father-in-law. No search warrant is necessary.

ACADCOM 2010; Contributors: Gene Geocaniga, Jarissa Guiani, Darlene Magabilen


TAU MU Page 2 of 179

SERRANO vs. NLRC


323 445 (2000)

TAU MU
TAU MU
TAU MU
TAU MU
TAU MU
TAU MU
TAU MU

Facts: Accused committed a crime on May 3,


1982. he was investigated by the police during
which he waived his right to a lawyer in writing
but without the assistance of counsel. He is now
before the Supreme Court challenging the
admissibility of his confession on the ground that
under the 1987 Constitution the waiver of the
right to counsel can only be made with the
assistance of counsel.
Held: Petitioners contention that Art. III, Sec. 12
of the 1987 Constitution should be given
retroactive efect for being favorable to him as
accused, cannot be sustained. While Art. 22 of
the Revised Penal Code provides that penal laws
shall have a retroactive efect insofar as they
favor the person guilty of a felony who is not a
habitual criminal, what is being constructed here
is a constitutional provision specifically contained
in the Bill of Rights which is obviously not a penal
statute. A bill of rights is a declaration of the
individual rights and privileges which the
Constitution is designed to protect against
violations by government, or by individuals or
groups of individuals. TAU MU

TAU MU

FILOTEO, JR. vs. SANDIGANBAYAN


263 SCRA 222 (1996)

TAU MU
TAU MU

Bill of Rights has no retroactive application

The Fraternal
Ateneo de Davao

TAU MU
TAU MU

Facts: Serrano was head of the Security


Checkers Section of Isetann Department Store.
In 1999, as a cost-cutting measure Isetann
phased out the entire security section and
engaged the services of an independent security
agency. Thus, it wrote a notice of termination to
Serrano efective on the same day, contrary to
the provisions of Art. 283 of the Labor Code which
requires a one-month notice.
Was the
constitutional right of Serrano violated? TAU MU
Held: No. The employers failure to comply with
the notice requirement does not constitute a
denial of due process, but a mere failure to
observe a procedure for termination. The reason
is that the due process clause is a limitation on
government power, not on private power such as
the termination of employment under the Labor
Code. Secondly, the notice and hearing are
required under the process clause before the
power of organized society are brought to bear
upon the individual. Under Art. 283, the failure to
comply with the notice requirement makes the
termination of his employment merely inefectual,
but not illegal. Consequently, he is not entitled to
reinstatement. However, he must be paid
backwages from the time his employment was
terminated until it is determined that his
termination is for just cause because the failure
to hear him before he is dismissed renders the
termination without any legal efect.

KITY

TAU MU TAU MU TAU MU


TAU MU TAU MU TAU MU

CONSTITUTIONAL LAW II
Order of Saint Thomas More
Atty. Philip John Pojas/Atty. Rovyne G. Jumao-as, RN
University College of Law

INHERENT POWERS OF THE STATE


Fundamental/Inherent Powers (Definition)
The fundamental powers of the State are the
police power, the power of eminent domain, and
the power of taxation. These powers are inherent
and do not need to be expressly conferred by
constitutional provision on the State. They are
supposed to co-exist with the State. The moment
the State comes into being, it is deemed invested
with these three powers as its innate attributes.
[Cruz, Constitutional Law 2007, p.37].
These powers are considered inherent because
they belong to the very essence of government
and without them no government can exist. A
constitution can only define and delimit them and
allocate
their
exercise
among
various
government agencies. A constitution does not
grant them. [Bernas, The 1987 Philippine
Constitution: A Comprehensive Reviewer, p. 22]
Briefly, the police power is the power of the State
to regulate liberty and property for the promotion
of the general welfare. The power of eminent
domain enables the State to forcibly acquire
private property, upon payment of just
compensation, for some intended public use. By
the power of taxation, the State is able to
demand from the members of society their
proportionate share or contribution in the
maintenance
of
the
government
[Cruz,
Constitutional Law 2007, p.37]. TAU MU
Similarities
The three inherent powers of the State are similar
in the following respects:
1. They are inherent in the State and may be
exercised by it without need of express
constitutional grant;
2. They are not only necessary but
indispensable. The State cannot continue
or be efective unless it is able to exercise
them;
3. They are methods by which the State
interferes with private rights;
4. They all presuppose an equivalent
compensation for the private rights
interfered with.
5. They are exercised primarily by the
legislature. [Cruz, Constitutional Law
2007, p.37].

TAU MU

Differences
The three inherent powers of the State difer from
each other in the following ways:
1. The police power regulates both liberty
and property. The power of eminent
domain and the power of taxation afect
only property rights;
2. The police power and the power of
taxation may be exercised only by the
government. The power of eminent
domain may be exercised by some private
entities;
3. The property taken in the exercise of the
police power is destroyed because it is
noxious or intended for a noxious purpose.
The property taken under the power of

ACADCOM 2010; Contributors: Gene Geocaniga, Jarissa Guiani, Darlene Magabilen


TAU MU Page 3 of 179

4.

eminent domain and the power of taxation


is intended for a public use or purpose and
is therefore wholesome; TAU MU
The compensation of the person subjected
to the police power is the intangible
altruistic feeling that he has contributed to
the general welfare. The compensation
involved in the other powers is more
concrete, to wit, a full and fair equivalent
of the property expropriated or protection
and public improvements for the taxes
paid. [Cruz, Constitutional Law 2007, p.38]

TAU MU
TAU MU
TAU MU
TAU MU
TAU MU
TAU MU
TAU MU

Scope/Characteristics
Police power has been characterized as the most
essential, insistent and the least limitable of
powers, extending as it does to all the great
public needs. Negatively, it has been defined as
that inherent plenary power in the State which
enables it to prohibit all that is hurtful to the
comfort, safety, and welfare of society. [Bernas,
The
1987
Philippine
Constitution:
A
Comprehensive Reviewer, p. 23]

TAU MU

As thus defined, the police power easily outpaces


the other two inherent powers as instruments of
the State in interfering with private rights. The
power of eminent domain affects not all of
the people directly but only those whose
property is needed for conversion to public
use. The power of taxation, while imposed on
most of the people directly, demands only part of
their money as their contribution to the upkeep of
the government. Both these powers involve only
property rights. By contrast, the police power
regulates not only the property but, more
importantly, the liberty of private persons, and
virtually all the people. It is in this sense that
police power may be regarded as infinitely more
important than eminent domain and taxation.
[Cruz, Constitutional Law 2007, p. 40]

TAU MU

Police Power (Definition)


It is the power of promoting public welfare by
restraining and regulating the use of liberty and
property [Nachura, Outline Reviewer in Political
Law 2006, p. 43].

TAU MU
TAU MU

POLICE POWER

The Fraternal
Ateneo de Davao

TAU MU
TAU MU

Limitations
Although
inherent
and
indispensible,
the
fundamental powers of the State are not without
restrictions. As ours is a government of limited
powers, even these prerogatives may not be
exercised arbitrarily, to the prejudice of the Bill of
Rights. The presumption in libertarian societies is
in favor of private rights and against attempts on
the part of the State to interfere with them.
Constitutional provisions for the security of
persons and property should be liberally
construed. Hence, the exercise of these
fundamental powers is subject at all times to the
limitations and requirements of the Constitution
and may in proper cases be annulled by the
courts of justice. [Cruz, Constitutional Law 2007,
p.38]

KITY

TAU MU TAU MU TAU MU


TAU MU TAU MU TAU MU

CONSTITUTIONAL LAW II
Order of Saint Thomas More
Atty. Philip John Pojas/Atty. Rovyne G. Jumao-as, RN
University College of Law

Police power is the most pervasive, the least


limitable, and the most demanding of the three
powers [Nachura, Outline Reviewer in Political
Law 2006, p. 43]. It may be exercised as long as
the activity or the property sought to be
regulated has some relevance to the public
welfare. The reach is virtually limitless. The
persons acts and acquisitions are hemmed in by
the police power. The justification is found in the
ancient Latin maxims, salus populi est suprema
lex (the welfare of the people is the supreme law)
and sic utere tuo ut alienum non laedas (So use
your own as not to injure another's property),
which calls for the subordination of individual
benefit to the interests of the greater number
[Cruz, Constitutional Law 2007, p.40]. TAU MU
Owing to the need to protect society from the
inordinate assertion of individual liberty, it has
been held that the police power may not be
bargained away through the medium of a
contract or even a treaty. The impairment clause
must yield to the police power whenever the
contract deals with a subject afecting the public
welfare [Cruz, Constitutional Law 2007, p.41].
A law enacted in the exercise of police power to
regulate certain government activities or
transactions could be given retroactive efect and
may reasonably impair vested rights or contracts.
Police power legislation is applicable not only to
future contracts, but equally to those already in
existence. Non-impairment of contracts or vested
rights clauses will have to yield to the superior
and legitimate exercise by the State of the police
power [Nachura, Outline Reviewer in Political Law
2006, p. 43].
The taxing power may be used as an implement
of police power [Nachura, Outline Reviewer in
Political Law 2006, p. 43].
Eminent domain may be used as an implement to
attain the police objective [Nachura, Outline
Reviewer in Political Law 2006, p. 43]. TAU MU
Exercise of Police Power
The national government, through the legislative
department, exercises police power. [Bernas, The
1987 Philippine Constitution: A Comprehensive
Reviewer, p. 23]. However, Congress may validly
delegate this power to the President, to
administrative bodies and to lawmaking bodies of
local government units. Local government units
exercise the power under the general welfare
clause (Sec. 16 of RA 7160 and under Secs. 391,
447, 458 and 468 of RA 7160) [Nachura, Outline
Reviewer in Political Law 2006, p. 44].
The exercise of police power lies in the discretion
of the legislative department. Given a police
problem, it is entirely up to the legislature to
decide, whether or not, in the first place, it should
act against the problem. If it does, well and good;
but if it does not, it may not be compelled to do
so by judicial process. No mandamus is available
to coerce the exercise of the police power [Cruz,
Constitutional Law 2007, p.46].

ACADCOM 2010; Contributors: Gene Geocaniga, Jarissa Guiani, Darlene Magabilen


TAU MU Page 4 of 179

Tests for Valid Exercise of Police Power


1. Lawful Subject
The interests of the public in general, as
distinguished from those of a particular class,
require the exercise of the power. This means
that the activity or property sought to be
regulated afects the general welfare; if it does,
then the enjoyment of the rights flowing
therefrom may have to yield to the interests of
the greater number [Nachura, Outline Reviewer
in Political Law 2006, p. 45]. TAU MU

TAU MU
TAU MU
TAU MU
TAU MU
TAU MU
TAU MU
TAU MU

Municipal
Ordinances
(Requisites
for
Validity):
a. Must not contravene the Constitution or
any statute;
b. Must not be unfair or oppressive;
c. Must not be partial or discriminatory;
d. Must not prohibit, but may regulate, trade;
e. Must not be unreasonable;
f. Must be general in application and
consistent with public policy [Nachura,
Outline Reviewer in Political Law 2006, p.
46].

TAU MU

Additional Limitations (When exercised by


delegate):
a. Express grant by law ( Secs. 16, 391, 447,
458 and 468, RA 7160 for LGUs);
b. Within territorial limits (for LGUs, except
when exercised to protect water supply);
c. Must not be contrary to law (activity
prohibited by law cannot in the guise of
regulation be allowed; an activity allowed
by law may be regulated, but not
prohibited) [Nachura, Outline Reviewer in
Political Law 2006, p. 46].

TAU MU

Even if the purpose be within the scope of the


police power, the law will still be annulled if the
subject is sought to be regulated in violation of
the second requirement. In Constitutional Law,
the end does not justify the means. The lawful
objective, in other words, must be pursuant to a
lawful method; that is, both the end and the
means must be legitimate. Lacking such
concurrence, the police measure shall be struck
down as an arbitrary intrusion into private rights
[Cruz, Constitutional Law 2007, p.49].

TAU MU
TAU MU

2. Lawful Means
The means employed are reasonably necessary
for the accomplishment of the purpose, and not
unduly oppressive on individuals [Nachura,
Outline Reviewer in Political Law 2006, p. 46].

The Fraternal
Ateneo de Davao

TAU MU
TAU MU

The first requisite simply means that the subject


of the measure is within the scope of the police
power, that is, that the activity or property sought
to be regulated afects the public welfare. If it
does, the enjoyment of private rights may be
subordinated to the interests of the greater
number, on the time-honored principle that the
welfare of the people is the supreme law [Cruz,
Constitutional Law 2007, p.49].

KITY

TAU MU TAU MU TAU MU


TAU MU TAU MU TAU MU

CONSTITUTIONAL LAW II
Order of Saint Thomas More
Atty. Philip John Pojas/Atty. Rovyne G. Jumao-as, RN
University College of Law

CASES ON POLICE POWER


ERMITA-MALATE HOTEL vs. CITY OF MANILA
July 31, 1967
Facts: The city of Manila promulgated an
ordinance which regulated the operation of hotels
& motels. The ordinance requires the following:
1. Registration in the lobby should be open
to public view
2. That the mayor can conduct inspection of
the establishment anytime
3. Prohibits the renting of the room more
than once within 24 hours
4. Minors should be accompanied by parents
or adults
Held: the ordinance is valid. The reason of the
law is to safeguard public morals due to the
alarming increase in prostitution traceable to
hotel/motel businesses.
The 24 hour limit is
reasonable considering that clandestine meetings
are usually shorter than legitimate ones. TAU MU
TAXICAB OPERATORS vs. BOT
119 SCRA 597 (1982)
Facts: The law concerns the phasing out of
dilapidated taxicabs. The lawful subject here is
public welfare. The lawful method is to phase out
old taxis to ensure public safety. Taxicab owners
and operators challenged this by saying that it
afects their property rights.
Held: The court held that the law is not unduly
oppressive, therefore valid. Six years cut-of time
is sufficient for the owners to have substantial
return of their investment.
ICHONG vs. HERNANDEZ
101 PHIL 1055
Held: There is substantial distinction between
aliens and Filipinos since aliens loyalty to the
country is transitory in nature which is motivated
only by some kind of personal interest.
The purpose of the law is to prevent aliens from
engaging in business in the Philippines. It is
observed that there is alien dominance in retail
trade. That is the reason why the law intends to
give it back to the Filipinos.
It also applies to all members of the class, except
to Americans, since they are covered by the
Parity
Amendment
found
in
the
1935
Constitution. This provision authorizes Americans
to engage in all trade in the country. The
classification also has relevance to the lawful
subject and method. TAU MU
LUTZ vs. ARANETA
98 PHIL 148
Facts: Walter Lutz, as Judicial Administrator of
the Intestate Estate of Antonio Jayme Ledesma,
sought to recover the sum of P14,6666.40 paid
by the estate as taxes from the Commissioner
under Section e of Commonwealth Act 567 (the

ACADCOM 2010; Contributors: Gene Geocaniga, Jarissa Guiani, Darlene Magabilen


TAU MU Page 5 of 179

TAU MU
TAU MU
TAU MU
TAU MU
TAU MU
TAU MU

It is the ultimate right of sovereign power to


appropriate, not only the public, but [even] the
private property of all citizens within the
territorial sovereignty, to public purposes

TAU MU

Eminent Domain (Definition)


Also known as the power of expropriation
[Nachura, Outline Reviewer in Political Law 2006,
p. 46].

TAU MU

EMINENT DOMAIN (Sec. 9)

TAU MU

Facts: Pres. Marcos promulgated an EO which


prohibited the inter-provincial movement of
carabaos and carabeefs. Anyone caught violating
the law will have his carabao and carabeef
confiscated. The purpose of the law is to prevent
indiscriminate slaughter of carabaos so that they
could be preserved for agricultural use by the
farmers. The lawful subject is general welfare
since the Philippines is an agricultural country.
The farmers need them for farming instead of
using machineries. (The country was in an energy
crisis)
Held: The court held that there was a lawful
subject which is general welfare. However, the
method chosen by the government has no logical
connection with the purpose of the law.
Prohibiting the inter-provincial transfer of
carabaos would not prevent their indiscriminate
slaughter. The carabaos can still be killed
anywhere without even transferring them. TAU MU

TAU MU
TAU MU

YNOT VS IAC
148 SCRA 659

The Fraternal
Ateneo de Davao

TAU MU
TAU MU

Sugar Adjustment Act), alleging that such tax is


unconstitutional as it levied for the aid and
support of the sugar industry exclusively, which is
in his opinion not a public purpose.
Issue: Whether the tax is valid in supporting an
industry.
Held: The tax is levied with a regulatory purpose,
i.e. to provide means for the rehabilitation and
stabilization of the threatened sugar industry. The
act is primarily an exercise of police power, and is
not a pure exercise of taxing power. As sugar
production is one of the great industries of the
Philippines; and that its promotion, protection and
advancement redounds greatly to the general
welfare, the legislature found that the general
welfare demanded that the industry should be
stabilized, and provided that the distribution of
benefits therefrom be readjusted among its
component to enable it to resist the added strain
of the increase in tax that it had to sustain.
Further, it cannot be said that the devotion of tax
money to experimental stations to seek increase
of efficiency in sugar production, utilization of byproducts, etc., as well as to the improvement of
living and working conditions in sugar mills and
plantations, without any part of such money
being channeled diectly to private persons,
constitute expenditure of tax money for private
purposes.
The tax is valid.

KITY

TAU MU TAU MU TAU MU


TAU MU TAU MU TAU MU

CONSTITUTIONAL LAW II
Order of Saint Thomas More
Atty. Philip John Pojas/Atty. Rovyne G. Jumao-as, RN
University College of Law

[Bernas, The 1987 Philippine Constitution: A


Comprehensive Reviewer, p. 101].
Eminent domain is described as the highest and
most exact idea of property remaining in the
government that may be acquired for some
public purpose through a method in the nature
of a compulsory sale to the State.
Being inherent, the power of eminent domain
does not need to be specifically conferred on the
government by the Constitution. As it happens,
however, it is expressly provided in Article III,
Section 9, that private property shall not be
taken for public use without just compensation.
This provision is not a grant but indeed a
limitation of the power as its negative and
restrictive language clearly suggests [Cruz,
Constitutional Law 2007, p.62].
Strict Construction against Expropriator
The limiting function imposed by the Bill is in
keeping with the philosophy of the Bill of Rights
against the arbitrary exercise of governmental
powers to the detriment of individual rights.
Given this function, the provision of Article III,
Section 9 should therefore be strictly interpreted
against the expropriator and liberally in favor of
the property owner.
The exercise of the right of eminent domain,
whether directly by the State, or by its authorized
agents, is necessarily in derogation of private
rights, and the rule in that case is that the
authority must be strictly construed. [Cruz,
Constitutional Law 2007, p.63] TAU MU
Who May Exercise the Power
Congress, and by delegation, the President,
administrative bodies, LGUs, and even private
enterprises performing public services [Nachura,
Outline Reviewer in Political Law 2006, p. 48].
Under existing laws, the following may exercise
the power of expropriation:
a. The Congress;
b. The President of the Philippines;
c. The various local legislative bodies;
d. Certain public corporations, like the Land
Authority and the National Housing
Authority;
e. Quasi-public corporations like Philippine
National
Railways,
PLDT
and
the
MERALCO.
Eminent Domain vs. Destruction from
Necessity
EMINENT DOMAIN
DESTRUCTION FROM
NECESSITY
The right of eminent The right of necessity
domain is a public arises under the laws of
right; it arises from the society or society itself.
laws of society and is It is the right of selfvested in the state or defense,
of
selfits
grantee,
acting preservation, whether
under the right and applied to persons or to
power of the state, or property. It is a private
benefit of the state, or right vested in every
those acting under it.
individual, and with
which the right of the
state or state necessity

ACADCOM 2010; Contributors: Gene Geocaniga, Jarissa Guiani, Darlene Magabilen


TAU MU Page 6 of 179

of

Property must be for


public use

Scope of the Power of Eminent Domain


In the hands of Congress the scope of the power
is, like the scope of legislative power itself,
plenary. It is as broad as the scope of police
power itself. It can thus reach every form of
property which the State might need for public
use. It can reach even private property already
dedicated to public use or even property devoted
to religious worship [Bernas, The 1987 Philippine
Constitution: A Comprehensive Reviewer, p. 22].

Ateneo de Davao

properties [Cruz, Constitutional Law 2007, p.67].


All private property capable of ownership may be
expropriated, except money and choses in action
[Nachura, Outline Reviewer in Political Law 2006,
p. 50]. Expropriation of money would be a futile
act because of the requirement for the payment
of just compensation, usually also in money. A
chose in action is a personal right not reduced
into possession but recoverable by a suit at law, a
right to receive, demand or recover a debt,
demand or damages on a cause of action ex
contractu or for a tort or omission of duty. It is
essentially conjectural both as to its validity and
its value [Cruz, Constitutional Law 2007, p.68].
Even services may be subject to eminent domain
[Nachura, Outline Reviewer in Political Law 2006,
p. 50].

TAU MU

TAU MU
TAU MU
TAU MU
TAU MU
TAU MU

b.
Private Property
Anything that can come under the dominion of
man is subject to expropriation. This will include
real and personal, tangible and intangible

TAU MU

The issue of the necessity of the expropriation is


a matter properly addressed to the RTC in the
course of the expropriation proceedings. If the
property owner objects to the necessity of the
takeover, he should say so in his Answer to the
Complaint. The RTC has the power to inquire into
the legality of the exercise of the right of eminent
domain and to determine whether there is a
genuine necessity for it. [Nachura, Outline
Reviewer in Political Law 2006, p. 49].

TAU MU

MU

TAU MU

Requisites for the Exercise of the Power of


Eminent Domain
a.
Necessity
Questions of necessity or wisdom are essentially
political when decided by the national legislature
and are usually not subject to judicial review
[Cruz, Constitutional Law 2007, p.65]. But when
exercised by a delegate, the determination of
whether there is genuine necessity for the
exercise is a justiciable question [Nachura,
Outline Reviewer in Political Law 2006, p. 49]. TAU

TAU MU

Private property may be expropriated for public


use and upon payment of just compensation;
condemnation of property is justified only if it is
for the public good and there is genuine necessity
therefor of a public character. Consequently, the
courts have the power to inquire into the legality
of the right of eminent domain and to determine
whether or not there is a genuine necessity
therefor [Cruz, Constitutional Law 2007, p.66].

TAU MU
TAU MU

Foundation of Right to Exercise the Power


of Eminent Domain
The foundation of the right to exercise eminent
domain is genuine necessity and that necessity
must be of public character. Government may not
capriciously or arbitrarily choose which private
property should be expropriated [Nachura,
Outline Reviewer in Political Law 2006, p. 49].

The Fraternal

TAU MU
TAU MU

Requires payment
just compensation

has nothing to do.


Does
not
require
payment
of
just
compensation
Cannot
require
the
conversion
of
the
property taken to public
use

KITY

TAU MU TAU MU TAU MU


TAU MU TAU MU TAU MU

CONSTITUTIONAL LAW II
Order of Saint Thomas More
Atty. Philip John Pojas/Atty. Rovyne G. Jumao-as, RN
University College of Law

Property already devoted to public use is still


subject to expropriation, provided this is done
directly by the national legislature or under a
specific grant of authority to the delegate. A mere
general authority may not suffice. In such a case,
the courts will have authority to inquire into the
necessity of the expropriation and, finding none,
refuse to allow it [Cruz, Constitutional Law 2007,
p.68].
c.
Taking
Taking, as the term is commonly understood,
imports a physical dispossession of the owner, as
when he is ousted from his land or relieved of his
watch or his car and is thus deprived of all
beneficial use and enjoyment of his property. In
law, however, the term has a broader
connotation. Taking may include trespass without
actual eviction of the owner, material impairment
of the value of the property or prevention of the
ordinary uses for which the property was
intended [Cruz, Constitutional Law 2007, p.70].
Taking, in the constitutional sense, may include
trespass without actual eviction of the owner,
material impairment of the value of the property
for which the property was intended. In People
vs. Fajardo (104 Phil 44), a municipal ordinance
prohibiting a building which would impair the
view of the plaza from the highway was
considered taking. The property owner was
entitled to payment of just compensation
[Nachura, Outline Reviewer in Political Law 2006,
p. 50].
Requisites of Taking (Republic vs. Castellvi):
1. The expropriator must enter a private
property;
2. The entry must be for more than a
momentary period;
3. The entry must be under warrant or color
of legal authority;
4. The property must be devoted to public
use or otherwise informally appropriated
or injuriously afected;
5. The utilization of the property for public
use must be in such a way as to oust the
owner and deprive him of the beneficial
enjoyment of the property.

ACADCOM 2010; Contributors: Gene Geocaniga, Jarissa Guiani, Darlene Magabilen


TAU MU Page 7 of 179

When municipal property is taken by the


State
There is compensable taking depending on the
nature of the property. If it is patrimonial property
of the municipality, that is, property acquired by
the municipality with its private funds in its
corporate or private capacity, compensation is
required. However, if it is any other property such
as public buildings or legua communal held by
the municipality for the State in trust for the
inhabitants, the State is free to dispose of it at
will [Bernas, The 1987 Philippine Constitution: A
Comprehensive Reviewer, p. 104]. TAU MU

TAU MU
TAU MU
TAU MU
TAU MU
TAU MU
TAU MU
TAU MU
TAU MU

Private property condemned for public use


sold to private user effect (Heirs of Moreno
vs. Mactan-Cebu, 2005):
The
predominant
precept
is
that
upon
abandonment of real property condemned for
public purpose, the party who originally
condemned the property recovers control of the
land if the condemning party continues to use the
property for public purpose; however, if the
condemning authority ceases to use the property
for a public purpose, property reverts to the
owner in fee simple. The governments taking of
private property, and then transferring it to
private persons under the guise of public use or
purpose is the despotism found in the immense
power of eminent domain. Moreover, the direct
and unconstitutional states power to oblige a
landowner to renounce his productive and
invaluable possession to another citizen, who will
use it predominantly for his own private gain, is
ofensive to our laws [Bernas, The 1987
Philippine
Constitution:
A
Comprehensive
Reviewer, p. 104]. TAU MU

TAU MU

This original meaning of the phrase has now been


broadened to cover uses which, while not directly
available to the public, redound to their indirect
advantage or benefit [Cruz, Constitutional Law
2007, p.76]. TAU MU

TAU MU
TAU MU

d.
Public Use
As a requirement of eminent domain, public use
is the general concept of meeting public need or
public exigency. It is not confined to actual use by
the public in its traditional sense. The idea that
public use is strictly limited to clear cases of
use by the public has been abandoned. The
term public use has not been held to be
synonymous with public interest, public benefit,
public welfare, and public convenience [Nachura,
Outline Reviewer in Political Law 2006, p. 51].

The Fraternal
Ateneo de Davao

TAU MU
TAU MU

Effect of Police Power and Eminent Domain


on Private Property*
EMINENT DOMAIN
POLICE POWER
Property is taken
Property is regulated
Transfer of ownership
No
transfer
of
ownership
The term taking must The term regulation is
be compensated.
not compensable
*[Bernas, The 1987 Philippine Constitution: A
Comprehensive Reviewer, p. 104]

KITY

TAU MU TAU MU TAU MU


TAU MU TAU MU TAU MU

CONSTITUTIONAL LAW II
Order of Saint Thomas More
Atty. Philip John Pojas/Atty. Rovyne G. Jumao-as, RN
University College of Law

e.
Just Compensation
Just compensation is described as a full and fair
equivalent of the property taken from the private
owner by the expropriator. This is intended to
indemnify the owner fully for the loss he has
sustained as a result of the expropriation. The
measure of this compensation is not just the
takers gain but the owners loss. The word just
is used to intensify the meaning of the word
compensation, to convey the idea that the
equivalent to be rendered for the property taken
shall be real, substantial, full, ample [Cruz,
Constitutional Law 2007, p.76].
Expressed diferently, the compensation given to
the owner is just if he receives for his property a
sum equivalent to its market value [Bernas,
The
1987
Philippine
Constitution:
A
Comprehensive Reviewer, p. 104].
Market Value (Definition)
Market value has been described in a variety of
ways. It is the price fixed by the buyer and seller
in the open market in the usual and ordinary
course of legal trade and competition; the price
and value of the article established or shown by
sale, public or private, in the ordinary way of
business; the fair value of property as between
one who desires to purchase and one who desires
to sell; the current price; the general or ordinary
price for which property may be sold in that
locality
[Bernas,
The
1987
Philippine
Constitution: A Comprehensive Reviewer, p. 104].
Owner (Concept)
According to Knecht vs. CA, the term owner as
applied in eminent domain cases refers to all
those who have lawful interest in the property to
be condemned, including a mortgagee, a lessee
and a vendee in possession under an executory
contract [Cruz, Constitutional Law 2007, p.78].
When a parcel of land is taken by eminent
domain, the owner of the fee is not necessarily
the only person who is entitled to compensation.
Every person having interest at law or in equity in
the land taken is entitled to share in the award. If
a person claiming an interest in the land sought
to be condemned is not made a party, he is given
the right to intervene and lay claim to the
compensation [Bernas, The 1987 Philippine
Constitution: A Comprehensive Reviewer, p. 108].
Title to the property does not pass until after
payment, except in agrarian reform [Nachura,
Outline Reviewer in Political Law 2006, p. 57]. TAU
MU

Consequential
Damages/Consequential
Benefits
Consequential damages consist of injuries directly
caused on the residue of the private property
taken by the reason of expropriation. Where, for
example, the expropriator takes only a part of a
parcel of land, leaving the remainder with an odd
shape or area as to be virtually unusable, the
owner can claim consequential damages. On the
other hand, if the remainder is as a result of the

ACADCOM 2010; Contributors: Gene Geocaniga, Jarissa Guiani, Darlene Magabilen


TAU MU Page 8 of 179

expropriation placed in a better location, such as


fronting a street where it used to be an interior
lot, the owner will enjoy consequential benefits
which should be deducted from the consequential
damages.
Consequential
benefits,
like
consequential damages, must be direct and
particular and not merely shared with the rest of
the properties in the area, as where there is a
general appreciation of lad values because of the
public use to which the condemned properties are
devoted [Cruz, Constitutional Law 2007, p.79].

TAU MU
TAU MU
TAU MU
TAU MU
TAU MU
TAU MU
TAU MU

The general rule is that the value must be that as


of the time of the filing of the complaint for
expropriation (Section 4, Rule 67, Rules of Court).

TAU MU

Property Assessment
The property taken should be assessed as of the
time of the taking, which usually coincides with
the
commencement
of
the
expropriation
proceedings. Where entry precedes the filing of
the complaint for expropriation, the assessment
should be made as of the time of the entry [Cruz,
Constitutional Law 2007, p.83].

TAU MU

Without prompt payment, compensation cannot


be considered just, for the property owner is
made to sufer the consequence of being
immediately deprived of his land while being
made to wait for a decade or more before
actually receiving the amount necessary to cope
with his loss.
To allow the taking of the
landowners properties and in the meantime
leave them empty-handed by withholding
payment of just compensation while the
government speculates on whether or not it will
pursue expropriation, or worse, for government to
subsequently decide to abandon the property and
return it to the landowner when it has already
been rendered useless by force majeure, it is
undoubtedly an oppressive exercise of eminent
domain that must never be sanctioned [Nachura,
Outline Reviewer in Political Law 2006, p. 54].

TAU MU
TAU MU

Reasonable Period
Just compensation includes not only the correct
determination of the amount to be paid to owner
of the land but also the payment of the land
within a reasonable period of time from its taking
[Bernas, The 1987 Philippine Constitution: A
Comprehensive Reviewer, p. 105]. TAU MU

The Fraternal
Ateneo de Davao

TAU MU
TAU MU

In such a case, the owner is not restricted to


payment of the market value of the portion
actually taken. In addition to the market value of
the portion taken, he is also entitled to payment
of consequential damages, if any to the
remaining part of the property. At the same time,
from the total compensation must be deducted
the value of consequential benefits, if any,
provided consequential benefits shall not exceed
consequential
damages
[Nachura,
Outline
Reviewer in Political Law 2006, p. 51]. If the
consequential benefits exceed the consequential
damages, these items should be disregarded
altogether as the basic value of the property
should be paid in every case [Cruz, Constitutional
Law 2007, p.79].

KITY

TAU MU TAU MU TAU MU


TAU MU TAU MU TAU MU

CONSTITUTIONAL LAW II
Order of Saint Thomas More
Atty. Philip John Pojas/Atty. Rovyne G. Jumao-as, RN
University College of Law

Moreover, the filing of the case generally


coincides with the taking. When, however, the
filing of the case comes later than the time of
taking and meanwhile the value of the property
has increased because of the use to which the
expropriator has put it, the value is that of the
time of the earlier taking. Otherwise, the owner
would gain undeserved profit. But if the value
increased independently of what the expropriator
did, then the value is that of the later filing of the
case [Bernas, The 1987 Philippine Constitution: A
Comprehensive Reviewer, p. 109].
When eminent domain is exercised by a local
government unit, the amount to be paid for the
expropriated property shall be determined by the
proper court, based on the fair market value at
the time of the taking of the property (Sec. 19,
RA7160) precisely because the Rules of Court
cannot prevail over RA7160, a substantive law
[Nachura, Outline Reviewer in Political Law 2006,
p. 56]. TAU MU
Determination of Just Compensation
The principal criterion in determining just
compensation is the character of the land at the
time of the taking. The tax declaration is only one
of the factors to be used in determining the
market value of the property for purposes of
arriving at the amount to be paid by way of just
compensation
[Nachura, Outline Reviewer in
Political Law 2006, p. 55]
Form of Compensation
As explicitly provided by Sec. 16(e), RA 6657, the
deposit of compensation must be in cash or in
Land Bank bonds, not in any other form, and
certainly not in a trust account.
Entitlement of Owner to Interest
When there is delay in the payment of just
compensation, the owner is entitled to payment
of interest, if claimed; otherwise, interest is
deemed waived. In NAPOCOR vs. Angas, the
Supreme Court held that the interest due the
property owner is at the rate of 6% per annum,
prescribed in Article 2209 of the Civil Code, and
not 12% per annum under Central Bank Circular
No. 416, because the latter applies to loans or
forbearance of money, goods or credits, or
judgments involving such loans or forbearance of
money goods or credits. The kind of interest
involved here is by way of damages, hence Art.
2209 of the Civil Code applies.
In some expropriation cases, the Court imposed
an interest of 12% per annum on the just
compensation due the landowner. It must be
stressed, however, that in these cases, the
imposition of interest was in the nature of
damages for delay in payment which, in efect,
makes the on the part of the government one of
forbearance. It follows that the interest in the
form of damages cannot be applied where there
was prompt and valid payment of just
compensation. Conversely, where there was
delay in tendering a valid payment of just
compensation, imposition of interest is in order

ACADCOM 2010; Contributors: Gene Geocaniga, Jarissa Guiani, Darlene Magabilen


TAU MU Page 9 of 179

[Nachura, Outline Reviewer in Political Law 2006,


p. 56].

TAU MU
TAU MU
TAU MU
TAU MU
TAU MU
TAU MU

Temporary takeover of private business or


utility
Temporary takeover by the government extends
only to the operation of the business and not to
the ownership thereof. As such, the government
is not required to compensate the private entityowner of the said business as there is not transfer
of ownership, whether permanent or temporary.
The private entity-owner afected cannot,
likewise, claim just compensation for the use of
said business and its properties, as the temporary
takeover by the government is in the exercise of

TAU MU

Right to repurchase or re-acquire the


property
The property owners right to repurchase the
property depends upon the character of the title
acquired by the expropriator, e.g. if land is
expropriated for a particular purpose with the
condition that when that purpose is ended or
abandoned, the property shall revert to the
former owner, then the former owner can
reacquire the property
[Nachura, Outline
Reviewer in Political Law 2006, p. 58].

TAU MU

MU

TAU MU

Plaintiffs right to dismiss the complaint in


eminent domain
In expropriation cases, there is no such thing as
the plaintifs matter-of-right to dismiss the
complaint, precisely because the landowner may
have already sufered damages at the start of the
taking. The plaintifs right to dismiss the
complaint has always been subject to Court
approval and to certain conditions [Nachura,
Outline Reviewer in Political Law 2006, p. 58]. TAU

TAU MU
TAU MU

Writ of Possession
The issuance of the writ of possession becomes
ministerial upon:
a. Filing of a complaint for expropriation
sufficient in form and substance;
b. Upon deposit made by the government of
the amount equivalent to fifteen percent
(15%) of the fair market value of the
property sought to be expropriated per
current tax declaration [Nachura, Outline
Reviewer in Political Law 2006, p. 58].

The Fraternal
Ateneo de Davao

TAU MU
TAU MU

Right of Owner in case of Non-payment of


Just Compensation
As a rule, non-payment of just compensation in
an expropriation proceeding does not entitle the
private landowner to recover possession of the
expropriated lots, but only to demand payment of
the fair market value of the property.
However, in Republic vs. Vicente Lim, the
Supreme Court said that the facts of the case do
not justify the application of the rule. It was held
that where the government failed to pay just
compensation within five (5) years from the
finality of the judgment in the expropriation
proceedings, the owners concerned shall have
the right to recover possession of their property
[Nachura, Outline Reviewer in Political Law 2006,
p.57]. TAU MU

KITY

TAU MU TAU MU TAU MU


TAU MU TAU MU TAU MU

CONSTITUTIONAL LAW II
Order of Saint Thomas More
Atty. Philip John Pojas/Atty. Rovyne G. Jumao-as, RN
University College of Law

the police power and not the power of eminent


domain [Nachura, Outline Reviewer in Political
Law 2006, p. 59].
RA7279 (Urban Development and Housing
Act of 1992)
Under RA7279, lands for socialized housing are to
be acquired in the following order:
1. Government lands
2. Alienable lands of the public domain
3. Unregistered, abandoned, or idle lands
4. Lands within the declared Areas for
Priority Development, Zonal Improvement
Program sites, Slum Improvement and
Resettlement sites which have not yet
been acquired
5. BLISS site which have not yet been
acquired
6. Privately owned lands
The mode of expropriation is subject to two
conditions, namely:
a. It shall be resorted to only when the other
modes of acquisition have been exhausted
b. Parcels owned by small property owners
are exempt from such acquisition
[Nachura, Outline Reviewer in Political Law 2006,
p. 60]
Judicial Review [Bernas, The 1987 Philippine
Constitution: A Comprehensive Reviewer, p.112]
The exercise of the power of eminent domain is
subject to judicial review. The following aspects of
the exercise of the power have been subject to
judicial scrutiny:
1. The adequacy of the compensation
2. The necessity of the taking
3. The public use character of the purpose
of the taking
Instance where exercise of the power of eminent
domain is not subject to judicial review
When land is expropriated for subdivision and
resale for social justice purposes directly by the
legislature and not through an inferior agency of
the state, the necessity and public purpose of the
taking are not subject to judicial review. Article
XIII, Section 4, constitutes a textual commitment
of discretion on the subject to the legislature.
However, if the power is exercised by an agency
possessing power delegated by the legislature or
when the issue is the adequacy of the
compensation, judicial review is still proper.

Res Judicata
The principle of res judicata does not bar the
right of the State or its agent to expropriate
private property. The very nature of the eminent
domain, as an inherent power of the State,
dictates that the right to exercise the power be
absolute and unfettered by a prior judgment or
res judicata. The scope of eminent domain is
plenary and, like police power, can reach every
form of property which the State might need for
public use.

ACADCOM 2010; Contributors: Gene Geocaniga, Jarissa Guiani, Darlene Magabilen


TAU MU Page 10 of 179

While the principle of res judicata does not


denigrate the right of the State to exercise
eminent domain, it does not apply to specific
issues decided in a previous case. For example, a
final judgment dismissing an expropriation suit on
the ground that there was not prior ofer
precluded another suit raising the same issue; it
cannot, however, bar the State or its agent from
thereafter complying with this requirement, as
prescribed by law, and subsequently exercising
its power of eminent domain over the same
property. (Municipality of Paranaque vs. VM
Realty Corporation) TAU MU

TAU MU
TAU MU
TAU MU
TAU MU
TAU MU
TAU MU
TAU MU

Essential requisites for LGU to validly


exercise the power of eminent domain
(Section 19, RA7160):
a. An ordinance is enacted by the local
legislative council authorizing the local
chief executive, in behalf of the LGU, to
exercise the power of eminent domain or

TAU MU

Ordinance
(Requisite
for
Valid
Expropriation)
In Municipality of Paranaque vs. VM Realty
Corporation, the Supreme Court declared that
there was lack of compliance with Sec. 19,
RA7160, where the Municipal Mayor filed a
complaint for eminent domain over two parcels of
land on the strength of a resolution passed by the
Sangguniang Bayan, because what is required by
law is an ordinance.

TAU MU

Exercise of Eminent Domain by Local


Government Unit
By express legislative authority granted by
Congress
in
Section
19,
RA7160,
local
government units may expropriate private
property for public use, or purpose, or welfare, for
the benefit of the poor and the landless. Thus, in
Moday vs. CA, the Supreme Court held that the
Sangguniang Panlalawigan of Agusan del Sur was
without authority to disapprove Bunawan
Municipal Reso 43-89 because clearly, the
Municipality of Bunawan has the authority to
exercise the power of eminent domain and its
Sangguniang Bayan the capacity to promulgate
the assailed resolution
[Nachura, Outline
Reviewer in Political Law 2006, p. 52].

TAU MU
TAU MU

The State may not enter into a contract which in


efect binds it not to exercise the power of
eminent domain. Like police power and the power
of taxation, the power of eminent domain is
inalienable. There can be no right to restrain by
contract the power of eminent domain, it must
also of necessity to follow that any contract by
which it was sought to accomplish that result
would be inefficacious for want of power.

The Fraternal
Ateneo de Davao

TAU MU
TAU MU

Expropriation
as
substitute
for
the
enforcement of a valid contract
Expropriation lies only when it is made necessary
by the opposition of the owner to the sale or by
the lack of any agreement as to price. Where
there is a valid and subsisting contract, between
the owners of the property and the expropriating
authority, there is no reason for the expropriation.

KITY

TAU MU TAU MU TAU MU


TAU MU TAU MU TAU MU

CONSTITUTIONAL LAW II
Order of Saint Thomas More
Atty. Philip John Pojas/Atty. Rovyne G. Jumao-as, RN
University College of Law

pursue expropriation proceedings over a


particular private property;
b. The power of eminent domain is exercised
for public use, purpose or welfare, or for
the benefit of the poor and the landless;
c. There is payment of just compensation as
required under Sec. 9, Article III of the
Constitution, and other pertinent laws;
d. A valid and definite ofer has been
previously made to the owner of the
property sought to be expropriated, but
said ofer was not accepted.
Limitations on power of eminent domain of
LGU: (Estate of JBL Reyes vs. City of Manila)
Since local governments possess only delegated
power of eminent domain, it is subject to
limitations specified by law on the delegated
power. Thus there are mandatory limits with
respect to:
1. The order of priority in acquiring land for
socialized housing; and
2. The resort to expropriation proceedings as
a means to acquiring it.
Private lands rank last in the order of priority for
purposes of socialized housing. In the same vein,
expropriation proceedings may not be resorted to
only after the other modes of acquisition are
exhausted. Compliance with these conditions are
mandatory because these are the only
safeguards of often time helpless owners of
private property against what may be tyrannical
violation of due process when their property is
forcibly taken from them allegedly for public use.
TAU MU

CASES ON EMINENT DOMAIN


PPI vs. COMELEC
244 SCRA 272 (1995)
Facts: The Commission on Elections passed a
resolution authorizing itself to procure free space
in newspapers which it shall allocate among
candidates or for dissemination of information
during the 1995 national election. Thereafter,
Commissioner Maambong wrote to newspapers
letters directing them to provide free space of no
less that one-half page form March 6 to May 6,
1995. PPI contends that the resolution and the
letters constitute unlawful taking of private
property.
Held: The letters form the constitutional body
have coercive efect as to compel the addressee
to donate space.
As such they constitute
unlawful taking of private property for public use.
First, there appears to be no necessity for the
taking as it was not shown that the newspapers
are not willing to sell space at the usual rate.
Second, the taking is without compensation.
Third, COMELEC is not empowered under the
Constitution or under any legislation to take over
private property.
TELECOMMUNICATIONS vs. COMELEC
289 SCRA 337 (1998)

ACADCOM 2010; Contributors: Gene Geocaniga, Jarissa Guiani, Darlene Magabilen


TAU MU Page 11 of 179

TAU MU
TAU MU
TAU MU
TAU MU
TAU MU
TAU MU
TAU MU
TAU MU
TAU MU

Facts: Petition for review of a decision of the CA.


The SB of the Municipality of Bunawan passed a
resolution authorizing the municipal mayor to
initiate the petition for expropriation of a parcel of
land,
belonging
to
the
petitioners.
The
Sangguniang Panlalawigan disapproved said
resolution and returned it with the comment that
expropriation is unnecessary considering that
there are still other lots available.
Issue: W/N a municipality may expropriate
private property by virtue of a municipal
resolution which was disapproved by the SP
Held: The Court finds no merit in the petition and
affirms the decision of the CA. Eminent domain is
a fundamental State power that is inseparable
from sovereignty. It is the governments right to
appropriate in the nature of a compulsory sale to
the State, private property for public use or
purpose. Inherently possessed by the national
legislature, the power of eminent domain may be
validly delegated to local governments, other
public entities and public utilities. For the taking
of private property by the government to be
valid, the taking must be for public use and there
must be just compensation. The SPs disapproval
does not render said resolution null and void.
Their power to declare a municipal resolution
invalid is on the sole ground that it is beyond the
power of the SB or the mayor to issue. Said

TAU MU
TAU MU

MODAY vs. CA
268 SCRA 586 (February 20, 1997)

The Fraternal
Ateneo de Davao

TAU MU
TAU MU

Facts: GMA Network, Inc., is challenging Sec. 52


of BP Blg. 881, which provides:
The Commission shall procure radio and
television time to be known as Comelec Time
which shall be allocated equally and impartially
among candidates For this purpose, the
franchise of all radio broadcasting and television
stations are hereby amended so as to provide
radio or television time, free of charge, during the
election period.
Petitioner claims that this is a form of taking or
property without compensation. TAU MU
Held:
The argument has no merit.
All
broadcasting, whether by radio or television
stations, is licensed by the government. Airwave
frequencies have to be allocated as there are
more individuals who want to broadcast than
there are frequencies assign. A franchise is thus
a privilege subject, among other things, to
amendment by Congress in accordance with the
Constitution if the common good so requires. In
truth,
radio
and
television
broadcasting
companies, which are given franchises, do not
own the airwaves and frequencies though which
they transmit broadcast signal and images.
Airtime is not a finished product which become
the property of the company, like oil produced
form refining. Thus, no private property is taken.
As the government spends public funds in
granting broadcasting companies the privilege
and in supervising them, it would be strange if
they cannot be required to render public service
by giving free airtime.

KITY

TAU MU TAU MU TAU MU


TAU MU TAU MU TAU MU

CONSTITUTIONAL LAW II
Order of Saint Thomas More
Atty. Philip John Pojas/Atty. Rovyne G. Jumao-as, RN
University College of Law

resolution is valid and binding and could be used


as lawful authority to petition for the
condemnation of petitioners property. The
limitations on the power of eminent domain are
that the use must be public, compensation must
be made and due process of law must be
observed. The necessity of exercising eminent
domain must be genuine and of a public
character.
MUNICIPALITY vs. VM REALTY CORP.
292 SCRA 678 (July 20, 1998)
Facts: Municipality of Paraaque filed a
complaint for expropriation against respondent
VM Realty over two parcels of land. The complaint
was filed for the purpose of alleviating the living
conditions of the underprivileged by providing
homes for the homeless through a socialized
housing project. Parenthetically, it was also for
this stated purpose that petitioner previously
made an ofer to enter into a negotiated sale of
the property with private respondent, which the
latter did not accept. The trial dismissed the case.
CA affirmed the said decision.
Issue: W/N the resolution of the Paraaque
Municipal Council is a substantial compliance of
the statutory requirement of Sec. 19, RA 7180 in
the exercise of the power of eminent domain by
the plaintif-appellant. TAU MU
Held: The power of eminent domain is lodged in
the legislative district branch of the government,
which may delegate the exercise thereof to LGUs,
other public entities and public utilities. An LGU
may therefore exercise the power to expropriate
private property only when authorized by
Congress and subject to the latters control and
restraints, imposed through the law conferring
the power or in other legislations. In this case,
Sec. 19 of RA 7160, which delegates to LGUs the
power of eminent domain , also lays down the
paramenters for its exercise. Thus, the following
essential requisites must concur before an LGU
may exercise the power of eminent domain:
1. An ordinance is enacted by the local
legislative council authorizing the local
chief executive, in behalf of the LGU, to
exercise the power of eminent domain or
pursue expropriation proceedings over a
particular private property;
2. The power of eminent domain is exercised
for public use, purpose or welfare, or for
the benefit of the poor and the landless;
3. There is payment of just compensation, as
required under Sec. 9, Art. III of the
Constitution and other pertinent laws;
4. A valid and definite ofer has been
previously made to the owner of the
property sought to be expropriated, but
said ofer was not accepted.
The local chief executive sought to exercise the
power of eminent domain pursuant to a
resolution of the municipal council. Thus, there
was no compliance with the first requisite that
the mayor be authorized through an ordinance. A
municipal ordinance is diferent from a resolution.
An ordinance is a law, but a resolution is merely a

ACADCOM 2010; Contributors: Gene Geocaniga, Jarissa Guiani, Darlene Magabilen


TAU MU Page 12 of 179

TAU MU
TAU MU
TAU MU
TAU MU
TAU MU
TAU MU
TAU MU

Facts: In 1947, The PAF leased the property of


Castellvi renewable year to year. The lease was
renewed yearly until 1956, Castellvi refused to
renew the contract with PAF any longer. In 1959,
the government instituted an action to
expropriate the property previously leased. Also,
in the same year, the property was placed under
the possession of the government by virtue of a
court order. TAU MU
Issue: What are being disputed in this case are
two issues:
1. Castellvi and the government cannot agree on
the price
Government P 0.20 per sq.meter
Castellvi P15.00 per sq.meter
2. They do not also agree on the date which will
be the basis for the determination of the value of
the land.
Governement value in 1947
Castellvi value in 1959
Held: The court held that the value should be
determined at the time of the taking. The taking
here would mean 1959, where all the elements of
taking were present. It cannot be 1947 since it
did not satisfy the 2nd and 5th elements.

TAU MU

REPUBLIC vs. CASTELLVI


58 SCRA 336 (1974)

TAU MU
TAU MU

TAKING

The Fraternal
Ateneo de Davao

TAU MU
TAU MU

declaration of the sentiment or opinion of a


lawmaking body on a specific matter. The power
of eminent domain necessarily involves a
derogation of a fundamental or private right of
the people. An LGU, like the municipality of
Paraaque, cannot authorize an expropriation of
private property through a mere resolution of its
lawmaking body. The LGC expressly and clearly
requires an ordinance of a local law for that
purpose. A resolution that merely expressed the
sentiment or opinion of the Municipal Council will
not suffice.
The Court holds that the principle of res judicata,
which finds application in generally all cases and
proceedings cannot bar the right of the State or
its agent to expropriate private property. The very
nature of eminent domain, as an inherent power
of the State, dictates that the right to exercise
the power be absolute and unfettered even by a
prior judgment or res judicata. The scope of
eminent domain is plenary and, like police power,
can reach every from of property which the State
might need for public use. While the principle of
res judicata does not denigrate the right of the
State to exercise eminent domain, it does not
apply to specific issues decided in a previous
case. The State or its authorized agent may still
subsequently exercise its right to expropriate the
same property, once all legal requirements are
complied with. To rule otherwise will not only
improperly diminish the power of eminent
domain, but also clearly defeat social justice.

KITY

TAU MU TAU MU TAU MU


TAU MU TAU MU TAU MU

CONSTITUTIONAL LAW II
Order of Saint Thomas More
Atty. Philip John Pojas/Atty. Rovyne G. Jumao-as, RN
University College of Law

NPC vs. GUTTIEREZ


193 SCRA 1 (1991)
Facts: NPC erected transmission lines passing
through the property of Gutierrez. As a
consequence, NPC asked for a 3-meter right of
way. In this portion, NPC prohibited Gutierrez from
planting anything above 3 meters. According to
NPC, they will only pay the rental of the right for
way. Gutierrez argued that there is actually
impairment of ordinary usage of his property.
Therefore, he should be paid for the full value of
the property traversed by the line.
Issue 1: Whether there was taking under the
concept of eminent domain here
Issue 2: Should there be a transfer of title
Held 1: The court held that there is taking as to
the sense that there is impairment of ordinary
use, so that the owner is entitled to full
compensation for the portion afected. As the
property is injuriously afected by an easement of
right of way, there is expropriation.
Held 2: NPC argued that since they will pay for
the whole property afected, then ownership of
that property should be transferred to them. The
title remains with Gutierrez but NPC should still
pay the full amount for the property afected
because there was impairment of usage.
REPUBLIC vs. PLDT
26 SCRA 620 (1969)

TAU MU

Facts: Republic, through the Bureau of


Telecommunications, rented the facilities of PLDT
for the use of the government and its offices.
Thereafter, the Bureau extended its services to
the public. PLDT filed a complaint alleging that
said bureau was violating the conditions under
which their Private Branch Exchange (PBX) is
inter-connected with the PLDT's facilities,
referring to the rented trunk lines, for the Bureau
had used the trunk lines not only for the use of
government offices but even to serve private
persons or the general public, in competition with
the business of the PLDT, and thereafter
disconnected the services of the Bureau. Republic
then filed a suit praying in its complaint for
judgment commanding the PLDT to execute a
contract with plaintif, through the Bureau, for the
use of the facilities of defendant's telephone
system throughout the Philippines under such
terms and conditions as the court might consider
reasonable, and for a writ of preliminary
injunction against the defendant company to
restrain the severance of the existing telephone
connections and/or restore those severed. TAU MU
Issue: W/N the Republic may compel PLDT to
execute a contract to use of its facilities
Held: While the Republic may not compel the
PLDT to celebrate a contract with it, the Republic
may, in the exercise of the sovereign power of
eminent domain, require the telephone company
to permit interconnection of the government
telephone system and that of the PLDT, as the
needs of the government service may require,
subject to the payment of just compensation to
be determined by the court. Nominally, of course,

ACADCOM 2010; Contributors: Gene Geocaniga, Jarissa Guiani, Darlene Magabilen


TAU MU Page 13 of 179

TAU MU
TAU MU
TAU MU
TAU MU
TAU MU
TAU MU
TAU MU
TAU MU
TAU MU

Facts: In 1978, the National Power Corporation


(NPC) took possession of private respondents
land under the mistake in belief that it forms part
of the public land reserved for hydroelectric
power purposes, and it paid financial assistance
to he the City of Marawi.
In 1979 private
respondent raised his claim of ownership, but it
was only in 1990, after more than ten years of
beneficial use, when NPC acceded to the fact that
the property belonged to respondent. In 1992,
NPC instituted expropriation proceedings.
At
what time should the value of the land be
computed, at the taking or the date of the filing
of the complaint for eminent domain?
Held: The general rule is that in determining
just compensation the value of the property at
the time of the filing of the complaint is
controlling.
Normally, the time of taking
coincides with the filing of the complaint.
However, where the taking precedes the
institution of the case, the time of taking is
critical in determining just compensation. In this
case, however, while there was entrance by NPC
into the property in 1978, taking was not really
made at that time. Taking has the following
elements: (1) the expropriator must enter a
private property; (2) the entrance must be for
more than a momentary period; (3) the entry into
the property should be under warrant or color of
legal authority; (4) the property must be devoted
to public use or otherwise injuriously afected;
and (5) it must oust owner and deprive him of all
beneficial enjoyment of the property.
In this case, NPCs entrance in 1978 was without
intent to expropriate or was not made under
warrant or color of legal authority, for it believed
that the property was public land. Thus, just
compensation should be the value at the time of
the filing of the complaint in 1992.

TAU MU
TAU MU

NPC vs. CA
254 SCRA 577 (1996)

The Fraternal
Ateneo de Davao

TAU MU
TAU MU

the power of eminent domain results in the taking


or appropriation of title to, and possession of, the
expropriated property; but no cogent reason
appears why the said power may not be availed
of to impose only a burden upon the owner of
condemned property, without loss of title and
possession. It is unquestionable that real property
may, through expropriation, be subjected to an
easement of right of way. The use of the PLDT's
lines and services to allow inter-service
connection between both telephone systems is
not much diferent. In either case private
property is subjected to a burden for public use
and benefit. If, under the Constitution, the State
may, in the interest of national welfare, transfer
utilities to public ownership upon payment of just
compensation, there is no reason why the State
may not require a public utility to render services
in
the
general
interest,
provided
just
compensation is paid therefor. Ultimately, the
beneficiary of the interconnecting service would
be the users of both telephone systems, so that
the condemnation would be for public use.

KITY

TAU MU TAU MU TAU MU


TAU MU TAU MU TAU MU

CONSTITUTIONAL LAW II
Order of Saint Thomas More
Atty. Philip John Pojas/Atty. Rovyne G. Jumao-as, RN
University College of Law

MARINE RADIO vs. REYES


191 SCRA 205 (November 6, 1990)
Facts: Petitioners are self-described Filipino
entrepreneurs deeply involved in the business of
marine radio communications in the country.
They are also operators of shore-to-ship and shipto-shore public marine coastal radio stations, and
are holders of certificates of public convenience
duly issued by the NTC. Among other things, they
handle
correspondence
between
vessel
passengers or crew and the public. DOTC
unveiled a maritime coastal communications
system project, designed to ensure safety of lives
at sea. It was set out to provide, among other
things, ship-to-shore and shore-to-ship public
corresponding, free of charge. Sec. Reyes replied
that MARCAPIs main business concern is public
correspondence, which is only fourth in the order
of priority of services to be ofered by the present
maritime project. Primarily, it will ofer distress
and safety communications service which is
obligatory in the maritime mobile service. The
petitioners brought the instant suit, alleging in
essence, that Secretary Rainerio Reyes had been
guilty of grave abuse of discretion. TAU MU
Issue: W/N the act complained of is equivalent to
taking without just compensation.
Held: There is no merit in the petition. There can
hardly be any valid argument against providing
for public corresponding, free of charge. It is
compatible with State aims to serve the people
under the Constitution, and certainly, amid these
hard times, the State can do no less. The Court is
not of the thinking that the act complained of is
equivalent to a taking without just compensation.
It does not seem that the DOTC, by providing for
free public correspondence, is guilty of an
uncompensated taking. Rather, the Government
merely built a bridge that made the boat
obsolete, although not entirely useless. Certainly,
the owner of the boat cannot charge the builder
of the bridge for lost income. And certainly, the
Government has all the right to build the bridge.
DIDIPIO EARTH SAVERS vs. GUZON
485 SCRA 586 (2006)
Facts: Sec. 76 of RA 7942 otherwise known as
the Philippine Mining Act of 1995, provides:
Entry into private lands and concession areas
Subject to prior notification, holders of mining
rights shall not be prevented from entry into
private lands and concession areas by surface
owners, occupants, or concessionaires when
conducting mining operations therein.
Issue:Is the provision a form of taking which is
subject to just compensation? TAU MU
Held: Yes. The entry referred to in Sec. 76 is not
just a simple right-of-way which is ordinarily
allowed under the provisions of the Civil Code.
Here, the holders of mining rights enter private
lands for purposes of conducting mining activities
such as exploration, extraction and processing of
minerals. Mining right holders build mine
infrastructure, dig mine shafts and connecting

ACADCOM 2010; Contributors: Gene Geocaniga, Jarissa Guiani, Darlene Magabilen


TAU MU Page 14 of 179

tunnels, prepare tailing ponds, storage areas and


vehicle
depots,
install
their
machinery,
equipment and server systems. On top of this,
under Sec 75, easement rights are accorded to
them where they may build warehouses, port
facilities, electric transmission, railroads and
other infrastructures necessary for mining
operations. All these will definitely oust the
owners or occupants of the afected areas the
beneficial ownership of their lands. Without a
doubt taking occurs once mining operations
commence.

TAU MU
TAU MU
TAU MU
TAU MU
TAU MU
TAU MU
TAU MU
TAU MU
TAU MU
TAU MU
TAU MU

Facts: In 1978, NAPOCOR, without the


landowners knowledge and prior consent, took
possession of the sub-terrain area of their lands
and constructed therein underground tunnels.
When discovered by the owners in 1992, they
asked for compensation. NAPOCOR denied
arguing that:
1. The owners rights extend to the subsoil
only insofar as necessary for their
practical interests, and
2. The tunnels merely constitute and
easement upon their property, which does
not involve any loss or title or possession.
Hence, they were not denied the beneficial
use of their subject properties as to entitle
them to just compensation by way of
damages.
Issue: Was there taking as to entitle the owners
to compensation?
Held: Yes, there was taking. The ownership of
land extends to the surface as well as to the
subsoil under it.
First, the argument that the landowners right
extends to the subsoil insofar as necessary for
their practical interests serves only to further
weaken the NPCs case. The theory would limit
the right to the subsoil upon the economic utility
which such area ofers to the surface owners.
Presumably, the landowners right extends to
such height or depth where it is possible for them
to obtain some benefit or enjoyment, and it is
extinguished beyond such limit as there would be
no more interest protected by law.
The owners could have dug upon their property
motorized deep wells but were prevented from
doing so by NPC precisely because of the
construction and existence of the tunnels
underneath the surface of their property. They,
therefore, still had a legal interest in the subterrain portion insofar as they could have
excavated the same for the construction of the
deep well.
Second, the manner in which the easement was
created by NPC, however, violates the due
process rights of the owners as it was, without
notice and indemnity to them and did not go
through proper expropriation proceedings.
NPC could have, at any time, validly exercised the
power of eminent domain to acquire the
easement over the property as this power
encompasses not only the taking or appropriation

The Fraternal
Ateneo de Davao

TAU MU
TAU MU

NPC vs. IBRAHIM


526 SCRA 149 (2007)

KITY

TAU MU TAU MU TAU MU


TAU MU TAU MU TAU MU

CONSTITUTIONAL LAW II
Order of Saint Thomas More
Atty. Philip John Pojas/Atty. Rovyne G. Jumao-as, RN
University College of Law

of title to and possession of the expropriated


property but likewise covers even the imposition
of a mere burden upon the owner of the
condemned property.
The underground tunnels impose limitations on
the use of the property for an indefinite period
and deprives them of its ordinary use. Based
upon the foregoing, the owners are clearly
entitled to the payment of just compensation. TAU
MU

REPUBLIC vs. ANDAYA


524 SCRA 671 (2007)
Facts: Ismael Andaya is the owner of two parcels
of land in Butuan City, which were subject to a
60-meter wide perpetual easement for public
highways, irrigation ditches, aqueducts, and other
similar works of the government or public
enterprise. Out of the 10,380 sq. m. property, 701
sq. m. was subject of the easement. During the
expropriation proceedings, Andaya demanded
that the consequential damages he must be paid
should be passed on the remaining area of 9,679
sq. m. which is the entire property minus the
easement. Andaya alleged that the easement
would render his entire property unusable and
unhabitable. Is he correct?
Held: Yes. Taking, in the exercise of the power of
eminent domain, occurs not only when the
government actually deprives or dispossesses the
property owner of his property or of its ordinary
use, but also when there is a practical destruction
or material impairment of the value of his
property. Using this standard, there was
undoubtedly a taking of the remaining area of
Andayas property. True, no burden was imposed
thereon and Andaya still retained title and
possession of the property. But the nature and
the efect of the floodwalls would deprive Andaya
of the normal use of the remaining areas. It would
prevent ingress and egress to the property and
turn it into a catch basin for the floodwaters
coming from the Agusan River.
Hence, Andaya is entitled to payment of just
compensation, which must neither more nor less
than the monetary equivalent of the land. TAU MU
CARLOS SUPERDRUG vs. DSWD
526 SCRA 130 (2007)
Facts:
Carlos
Superdrug
questioned
the
constitutionality of the Expanded Senior Citizens
Act of 2003, which grants to senior citizens 20%
discount on their purchase of medicines.
According to CS, the law is confiscatory because
it compels drugstore owners to grant the discount
and will result in a loss of profit and capital since
it tailed to provide a scheme whereby drugstores
will be justly compensated for the discount.
Issue: Was there a valid taking?
Held: Yes. The Senior Citizens Act was enacted
primarily to maximize the contribution of senior
citizens to nation-building, and to grant benefits
and privileges to them for their improvement and

ACADCOM 2010; Contributors: Gene Geocaniga, Jarissa Guiani, Darlene Magabilen


TAU MU Page 15 of 179

well-being as the State considers them an


integral part of our society.
The law is a legitimate exercise of police power
which, similar to the power of eminent domain,
has general welfare for its object.
Police power is not capable of an exact definition
but has been purposely veiled in general terms to
underscore its comprehensiveness to meet all
exigencies and provide enough room for an
efficient and flexible response to conditions and
circumstances, thus assuring the greatest
benefits.
PUBLIC USE

TAU MU
TAU MU
TAU MU
TAU MU
TAU MU
TAU MU
TAU MU
TAU MU

Facts: The President of the Philippines issued a


Letter of Instruction instituting an nationwide
slum improvement and resettlement program and
to adopt slum improvement as a national housing
policy. The Metro Manila Zonal Improvement
Program included the properties known as the
Tambunting Estate and the Sunog-Apog area in its
priority list for a zonal improvement program. The
President also designated the NHA to negotiate
with the owners of the property for the
acquisition of the same. This, however, did not
materialize as the negotiations for the purchase
of the property failed. The President issued Proc.
No. 1810 declaring all sites identified by the
Metro Manila local governments and approved by
the Ministry of Human Settlements to be included
in the ZIP upon proclamation of the President.
The Tambunting Estate and the Sunog-Apog area
were among the sites included.
Issue: W/N the government had the right to
expropriate the said areas. TAU MU
Held: The power of eminent domain is inherent
in every state and the provisions in the
Constitution pertaining to such power only serve
to limit its exercise in order to protect the
individual against whose property the power is
sought to be enforced. The State is not subject to

TAU MU

MANOTOK vs. CA
150 SCRA 87 (May 21, 1987)

TAU MU
TAU MU

Facts: The NHA wanted to use Sumulongs


property for socialized housing for the lower and
middle class. The owner contended that
socialized housing is not public use because not
everyone can benefit from this, only the handful
of people who to be given the houses. TAU MU
Held: The court held that the socialized housing
is within the context of public use. Public use has
acquired a more comprehensive meaning. That is
whatever would result to indirect public benefit or
welfare is also public use. It also ruled that it will
benefit everyone in the sense that it will afect
the safety, health and environment. Providing
housing to these people will help in lessening the
incidence of violence and problems concerning
health. In the end, it will benefit everybody in a
way. In short, socialize housing falls within the
meaning of public use.

The Fraternal
Ateneo de Davao

TAU MU
TAU MU

SUMULONG vs. GUERRERRO


154 SCRA 461 (1987)

KITY

TAU MU TAU MU TAU MU


TAU MU TAU MU TAU MU

CONSTITUTIONAL LAW II
Order of Saint Thomas More
Atty. Philip John Pojas/Atty. Rovyne G. Jumao-as, RN
University College of Law

any limitation other than those imposed by the


Constitution which are:
First, the taking must be for a public use;
Second, the payment of just compensation must
be made and;
Third, due process must be observed in the
taking.
The due process clause cannot be rendered
nugatory everytime a specific decree or law
orders the expropriation of somebodys property
and provides its own peculiar manner of taking
the same. Neither should the courts adopt a
hands-of policy just because the public use has
been ordained as existing by the decree of the
just compensation has been fiexed and
determined beforehand by a statute.
Although
due
process
does not
always
necessarily demand that a proceeding be had
before a court of law, it still mandates some form
of proceeding wherein notice and reasonable
opportunity to be heard are given to the owner to
protect his property rights. There are exceptional
situations when, in the exercise of the power of
eminent domain, the requirement of due process
may not necessarily entail judicial process. But
where it is alleged that in the taking of a persons
property, his right to due process of law has been
violated, the courts will have to step in and probe
into such an alleged violation. A necessity must
exist for the taking of private property for the
proposed uses and purposes but accepted the
fact that modern decisions do not call for
absolute necessity. It is enough if the condemnor
can show a reasonable or practical necessity,
which of course, varies with the time and peculiar
circumstances of each case.
In the instant petitions, there is no showing
whatsoever as to why the properties involved
were singled out for expropriation through
decrees or what necessity impelled the particular
choices or selections. The provision of P.D. 1669
which allows NHA, at its sole option, to put
portions of the expropriated area to commercial
use in order to defray the development costs of
its housing projects cannot stand constitutional
scrutiny. P.D. Nos. 1669 and 1670 is violative of
the petitioners' right to due process of law and,
therefore,
they
must
fail
the
test
of
constitutionality. The decrees, do not by
themselves, provide for any form of hearing or
procedure by which the petitioners can question
the propriety of the expropriation of their
properties or the reasonableness of the just
compensation. The fixing of the maximum
amounts of compensation and the bases thereof
which are the assessed values of the properties in
1978 deprive the petitioner of the opportunity to
prove a higher value because, the actual or
symbolic taking of such properties occurred only
in 1980 when the questioned decrees were
promulgated. The decision of the government to
acquire a property through eminent domain
should be made known to the property owner
through a formal notice wherein a hearing or a
judicial proceeding is contemplated. This shall be
the time of reckoning the value of the property
for the purpose of just compensation. The
questioned decrees transgress the petitioners'

ACADCOM 2010; Contributors: Gene Geocaniga, Jarissa Guiani, Darlene Magabilen


TAU MU Page 16 of 179

right to just compensation. Having violated the


due process and just compensation guarantees, P.
D. Nos. 1669 and 1670 are unconstitutional and
void.
ESTATE vs. PHILIPPINE EXPORT
349 SCRA 240 (2000)

TAU MU
TAU MU
TAU MU
TAU MU
TAU MU
TAU MU
TAU MU
TAU MU
TAU MU

Facts: In 1977, the National Housing Authority


(NHA) field complaints to expropriate sugarcane
lands in Cavite. The purpose of the expropriation
was
the
expansion
of
the
Damarias
Resettlement Project to accommodate the
squatters who were relocated from the
Metropolitan Manila Area. The trial court rendered
judgment ordering the expropriation, after
payment of just compensation, of the subject lots
which decision was affirmed by the SC in 1987. In
1992, the previous owners of the expropriated
properties filed a case against NHA for their
return on the ground that NHA deviated from the
public purpose of the taking when it entered into
a contract for the construction of low cost
housing units to be sold to low-income
beneficiaries. TAU MU
Held: The act of NHA of entering into a contract
with a real estate developer for the construction
of low cost housing on the expropriated lots
cannot be taken to mean as a deviation from the
stated
public
purpose
of
their
taking.
Jurisprudence has it that the expropriation of
private land for slum clearance and urban

TAU MU
TAU MU

REYES vs. NHA


395 SCRA 495 (2003)

The Fraternal
Ateneo de Davao

TAU MU
TAU MU

Facts: Philippine Export Processing Zone (PEZA)


initiated
before
the
RTC
expropriation
proceedings on 3 parcels of irrigated rice land.
The power of eminent domain of PEZA is
contained in its original charter, PD No. 66, which
allows it to expropriate any property for export
processing zones, low-cost housing, or for the
construction
of
dams,
terminal
facilities,
structures and approaches thereto. Considering
that portions of the expropriated property would
be leased to banks, is it still for a public
purpose?
Held: Yes. The term public use has acquired a
more comprehensive coverage. To the literal
import of the term signifying strict use by the
public has been added the broader notion of
indirect public benefit or advantage. It should be
pointed out that PEZA was created to be a viable
commercial, industrial and investment area. The
expropriation of the lot for the purpose of being
leased to banks and for the construction of a
terminal has the purpose of making banking and
transportation facilities easily accessible to
persons working at industries located at PEZA. It
therefore comes as a matter of necessity to bring
life to the purpose of the law, as reaffirmed by
RA No. 9716. Moreover, Congress can determine
the necessity of expropriating private property. In
such a case, it is a question exclusively for the
legislature to decide.

KITY

TAU MU TAU MU TAU MU


TAU MU TAU MU TAU MU

CONSTITUTIONAL LAW II
Order of Saint Thomas More
Atty. Philip John Pojas/Atty. Rovyne G. Jumao-as, RN
University College of Law

development is for a public purpose even if the


developed area is later sold to homeowners,
commercial firms, service companies and other
private concerns. Moreover, the Constitution itself
allows the State to undertake, for the common
good, and in cooperation with the private sector,
a continuing program of urban land reform and
housing which will make at afordable cost decent
housing to homeless citizens. It follows that the
low cost housing of the NHA on the expropriated
lots is consistent with the public use requirement.
MANOSCA vs. CA
252 SCRA 412 (1996)
Facts: Petitioners own a piece of land consisting
of 492 square meters, which was later
ascertained by the National Historical Institute to
be the birth site of Felix Y. Manalo, the founder of
the Iglesia Ni Cristo.
Consequently, it was
declared a national historical landmark, and the
Republic instituted expropriation proceedings.
Issue: Would the expropriation constitute an
application of funds for the use, benefit or
support of a religious entity?
Held:
The attempt to give some religious
perspective
to
the
case
deserves
little
consideration, for what should be significant is
the principal objective of, not the casual
consequences that might follow from, the
exercise of power. The purpose in setting up a
marker is essentially to recognize the distinctive
contribution of the late Felix Manalo to the culture
of the Philippines, rather than to commemorate
his founding and leadership of the Iglesia Ni
Cristo.
The practical reality that the greater
benefit may be derived by its members than by
most others could well be true but such a peculiar
advantage still remains to be merely incidental
and secondary in nature. Indeed that only a few
would actually benefit from the expropriation of
the property does not necessarily diminish the
essence and character of public use.
HEIRS OF MORENO vs. MACTAN CEBU
413 SCRA 502 (2003)
Facts: In 1949, the National Airport Corporation
as the predecessor agency of respondent MactanCebu International Airport Authority (MCIAA)
wanted to acquire Lot Nos. 916 and 920 among
other parcels of land for the proposed expansion
of Lahug Airport. To entice the landowners to
cede their properties, the government assured
them that they could repurchase their lands once
Lahug Airport was closed or its operations
transferred to Mactan Airport. TAU MU
At the end of 1991, Lahug Airport ceased
operations as the Mactan Airport was opened for
incoming and outgoing flights. Lot Nos. 916 and
920 which had been expropriated for the
extension of Lahug Airport were not utilized.
Hence, petitioners filed a complaint for
reconveyance and damages to compel the
repurchase of Lot Nos. 916 and 920.

ACADCOM 2010; Contributors: Gene Geocaniga, Jarissa Guiani, Darlene Magabilen


TAU MU Page 17 of 179

The Fraternal
Ateneo de Davao

TAU MU
TAU MU
TAU MU
TAU MU
TAU MU
TAU MU
TAU MU
TAU MU
TAU MU
TAU MU
TAU MU
TAU MU
TAU MU

Petitioners argue that Fery vs. Municipality of


Cabanatuan does not apply to the case at bar
since what was involved therein was the right of
reversion and not the right of repurchase
which they are invoking. They also diferentiate
Mactan-Cebu International Airport Authority vs.
CA from the instant case in that the landowners
in the MCIAA case ofered inadmissible evidence
to show their entitlement to a right of repurchase,
while petitioners herein ofered evidence based
on personal knowledge for which reason MCIAA
did not object and thus waived whatever
objection it might have had to the admissibility
thereof. Finally, petitioners allege that their right
to equal protection of the laws would be infringed
if some landowners are given the right to
repurchase their former properties even as they
are denied the exercise of such prerogative.
Held: in Fery, we declared that the government
acquires only such rights in the expropriated
parcels of land as may be allowed by the
character of its title over the properties:
If x x x land is expropriated for a particular
purpose, with the condition that when that
purpose is ended or abandoned, the property
shall return to its former owner, then or course,
when the purpose is terminated or abandoned
the former owner reacquires the property so
expropriated. If x x x land is expropriated for a
public street and the expropriation is granted
upon condition that the city can only use it for a
public street, it returns to the former owner,
unless there is a statutory provision to the
contrary x x x if, upon the contrary, however, the
decree if expropriation gives to the entity a fee
simple title, then, of course, the land becomes
absolute property of the expropriator, whether it
be a State, province, or municipality, and in that
case the non-use does not have the effect of
defeating the title acquired by the expropriation
proceedings x x x
As for the public purpose of the expropriation
proceeding, it cannot now be doubted. Although
Mactan Airport is being constructed, it does not
take away the actual usefulness and importance
of the Lahug Airport: it is handling the air traffic
both civilian and military. From it aircrafts fly to
Mindanao and Visayas and past through it or their
flights to the North and Manila. Then, no evidence
was adduced to show how soon is the Mactan
Airport to be placed in operation and whether the
Lahug Airport will be closed immediately
thereafter. It is up to the other departments of the
Government to determine said matters. The Court
cannot substitute its judgment for those of the
said departments or agencies. In the absence of
such showing, the Court will presume that the
Lahug Airport will continue to be in operation.
The predicament of petitioners involves a
constructive trust, one that is akin to the implied
trust referred to in Art. 1454 of the NCC.
If an absolute conveyance of property is made in
order to secure the performance of an obligation
of the grantor toward the grantee, a trust by
virtue of law is established. If the fulfillment of
the obligation is offered by the grantor when it
becomes due, he may demand the reconveyance
of the property to him.

KITY

TAU MU TAU MU TAU MU


TAU MU TAU MU TAU MU

CONSTITUTIONAL LAW II
Order of Saint Thomas More
Atty. Philip John Pojas/Atty. Rovyne G. Jumao-as, RN
University College of Law

In the case at bar, petitioners conveyed Lot Nos.


916 and 920 to the government with the latter
obliging itself to use the realties for the
expansion of Lahug Airport; toiling to keep its
bargain, the government can be compelled by
petitioners to reconvey the parcels of land to
them, otherwise, petitioners would be denied the
use of their properties upon a state of afairs that
was not conceived nor contemplated when the
expropriation was authorized. TAU MU
Hence, respondent MCIAA as representative of
the State is obliged to reconvey Lot Nos. 916 and
920 to petitioners who shall hold the same
subject to existing liens thereon, ie leasehold
right of DPWH.
HEIRS OF MORENO vs. MACTAN-CEBU
466 SCRA 288 (2005)
Facts: On MFR by Mactan International Airport
Authority, it was argued that the decision of the
Court which granted the owners right of
repurchase efectively overturns the ruling in
Fery, which requires that for an expropriation
judgment to be conditional, it must clearly spell
out said condition.
Held: Nothing in the Fery case bespeaks that
there should be an express condition in the
dispositive portion of the decision before the
condemned property can be returned to its
former owner after the purpose for its taking has
been abandoned or ended.
The indisputable certainty in the present case is
that there was a prior promise by the predecessor
of Mactan that the expropriated properties may
be recovered by the former owners since the
airport is transferred to Mactan, Cebu. In fact, the
witness for Mactan testified that 15 lots were
already reconveyed to their previous owners.
MASIKIP vs. CITY OF PASIG
479 SCRA (2006)
Facts: The City of Pasig, notified Masikip of its
intention to expropriate a portion of her property
to be used for the sports development and
recreational activities. She replied that the
intended expropriation of her property is
unconstitutional, invalid, and oppressive, as the
area of her lot is neither sufficient nor suitable to
provide land opportunities to deserving poor
sectors of our community.
She contended that the City failed to establish a
genuine
necessity
which
justifies
the
condemnation of her property. While she does not
dispute the intended public purpose, nonetheless,
she insists that there must be a genuine
necessity for the proposed use and purposes.
Held: LGUs have no inherent power of eminent
domain and may exercise it only when expressly
authorized by statute. Section 19 of the LGC
prescribes the delegation by Congress of the
power of eminent domain to LGUs and lays down
the parameters for its exercise.
SEC. 19. Eminent Domain. - A local government
unit may, through its chief executive and acting

ACADCOM 2010; Contributors: Gene Geocaniga, Jarissa Guiani, Darlene Magabilen


TAU MU Page 18 of 179

TAU MU
TAU MU
TAU MU
TAU MU
TAU MU
TAU MU
TAU MU
TAU MU

Facts: EPZA argues that the commissioners


report should no longer be set for hearing
because under PD Nos. 76, 464, 794 and 1533,
the basis for just compensation shall be the fair
and current market value declared by the owner
of the property sought to be expropriated or such
market value as determined by the assessor,
whichever is lower.
Hence, there is no more need to appoint
commissioners as prescribed by Rule 67 of the
Rules of Court and for them to consider other
highly variable factors in order to determine just
compensation.
Issue: Are PD Nos. 76, 464, 794 and 1533 valid
and constitutional, such that in determining the
just compensation of property in an expropriation
case, the only basis should be its market value as
declared by the owner or as determined by the
assessor, whichever is lower?
Held: No. the method of ascertaining just
compensation under the afore-cited decrees
constitutes impermissible encroachment on
judicial prerogatives. It tends to render this Court

TAU MU

EPZA vs. DULAY


149 SCRA 305 (1987)

TAU MU
TAU MU

JUST COMPENSATION

The Fraternal
Ateneo de Davao

TAU MU
TAU MU

pursuant to an ordinance, exercise the power of


eminent domain for public use, or purpose, or
welfare for the benefit of the poor and the
landless, upon payment of just compensation,
pursuant to the provisions of the Constitution and
pertinent laws: Provided, however, That the
power of eminent domain may not be exercised
unless a valid and definite offer has been
previously made to the owner, and such offer was
not accepted: Provided, further, That the local
government
unit
may
immediately
take
possession of the property upon the filing of the
expropriation proceedings and upon making a
deposit with the proper court of at least fifteen
percent (15%) of the fair market value of the
property based on the current tax declaration of
the property to be expropriated: Provided, finally,
That, the amount to be paid for the expropriated
property shall be determined by the proper court,
based on the fair market value at the time of the
taking of the property.
According to Masikip, there is already an
established sports development and recreational
activity center. The City does not dispute this.
Evidently, there is no genuine necessity to
justify the expropriation.
Where the taking by the State of private property
is done for the benefit of a small community
which seeks to have its own sports and
recreational facility, notwithstanding that there is
such a recreational facility only a short distance
away, such taking cannot be considered to be for
public use. Its expropriation is not valid.
The right to take private property for public
purposes necessarily originates from the
necessity and the taking must be limited to such
necessity. TAU MU

KITY

TAU MU TAU MU TAU MU


TAU MU TAU MU TAU MU

CONSTITUTIONAL LAW II
Order of Saint Thomas More
Atty. Philip John Pojas/Atty. Rovyne G. Jumao-as, RN
University College of Law

inutile in a matter which under the Constitution is


reserved to it for final determination.
Thus, although in an expropriation proceeding the
court technically would still have the power to
determine the just compensation for the
property, following the applicable decrees, its
task would be relegated to simply stating the
lower value of the property as declared either by
the owner or the assessor. As a necessary
consequence, it would be useless for the court to
appoint commissioners under Rule 67 of the ROC.
The strict application of the decrees during the
proceedings would be nothing short of a mere
formality or charade as the court has only to
choose between the valuation of the owner and
that of the assessor, and its choice is always
limited to the lower of the two. The court cannot
exercise its discretion or independence in
determining what is just or fair. Even a grade
school pupil could substitute for the judge insofar
as the determination of constitutional just
compensation is concerned.
SANTOS vs. LAND BANK
340 SCRA 59 (2000)
Facts: An RTC ordered Land Bank to pay P49M to
Santos for the taking of some 77 hectares of
agricultural land. It was also ordered that the
compensation shall be in the manner provided
under the CARL, which allows compensation in
cash and in bonds.
Issue: Is the mode of payment under the CARL
constitutional?
Held: Yes. Traditionally, the medium of payment
if just compensation is money and no other.
However, in land reform, we do not deal with the
traditional exercise of the power of eminent
domain. TAU MU
This is a revolutionary kind of expropriation that
does not afect only a specific property or a
limited area. Accepting the theory that payment
of just compensation need not always be made
fully in money, we find that the proportion of cash
payment is not unduly oppressive. No less
important, the government financial instruments
in making up the balance of the payment are
negotiable at any time.
ROBERN vs. QUITAIN
315 SCRA 150 (1999)
Facts: Robern Development Corporation owns
parcels of land in Davao City intended for a lowcost housing project.
The National Power
Corporation filed a complaint to expropriate
them.
Upon deposit of P6,121.20 with the
Philippine National Bank representing the
assessed value of the property, the Regional Trial
Court issued a writ of possession in NPCs favor
as authorized by PD No. 42. Can the judge issue
the writ without first conducting a hearing on the
amount of just compensation? TAU MU
Held: In Panes v. Visayas, 264 SCRA 708 (1996),
we ruled that the determination of the amount of
provisional deposit for the issuance of the writ of
possession is a judicial function. Consequently, a

ACADCOM 2010; Contributors: Gene Geocaniga, Jarissa Guiani, Darlene Magabilen


TAU MU Page 19 of 179

TAU MU
TAU MU
TAU MU
TAU MU
TAU MU
TAU MU
TAU MU

Facts: Respondent owns a piece of land with an


area of 39,512 square meters. On October 6,
1981, with the consent of respondent provided
they would be paid for the area taken after the
processing of documents by the Commission on
Audit, the National Irrigation Administration
constructed an irrigation canal.
The canal
afected 24,660 square meters. Since NIA failed
to pay for the property, on Dec. 10,1990,
respondent filed a complaint praying that NIA be
ordered to pay compensation for the property
used in the canal constructed. What year should

CITY OF CEBU vs. DEDAMO


387 SCRA 754 May 7, 2002

TAU MU

ESLABAN vs. VDA DE ONORIO


360 SCRA 230 (2001)

be used as basis for determining just


compensation?
Held: Sec. 4, of Rule 67 of the Rules of Court
states that just compensation should be
determined as of the date of the taking of the
property or the filing of the complaint, whichever
comes first. The price of the land at the time of
the taking, not its value after the passage of
time, represents the true value to be paid as just
compensation. Thus, the just compensation to be
paid respondent should be determined as of its
taking by the NIA in 1981, not the filing of the
complaint in 1990. TAU MU

TAU MU

Facts: In 1994, the Sangguniang Panglungsod ng


Mandaluyong City issued a resolution authorizing
the mayor to institute expropriation proceedings
over the lot of Saguitan. Saguitan filed a motion
to dismiss arguing that the Local Government
Code requires an ordinance, not a mere
resolution. The trial court, however, denied the
motion reasoning that an ordinance would only
be required to appropriate funds in payment of
the expropriated property once the trial court has
determined its value. Is the court correct?
Held: No. Under the Local Government Code,
the exercise of eminent domain must comply with
the following requisites: 1) an ordinance enacted
by the council authorizing the chief executive; 2)
the power is exercised for public use, purpose or
welfare, or for the benefit of the poor and the
landless; 3) payment of just compensation; and
4) a definite ofer was previously made which was
not accepted by the property owner.
An
examination of the law shows that an ordinance
is necessary to authorize the filing of the
complaint with the proper court since, beginning
at that point, the power of eminent domain stats
to be exercised. The determination of the award
of just compensation is but the last stage which
cannot be arrived at without an initial finding by
the court that plaintif has a right to take
property. TAU MU

Ateneo de Davao

TAU MU
TAU MU

HEIRS OF SUGUITAN vs. CITY


328 SCRA 137 (2000)

The Fraternal

TAU MU
TAU MU

summary hearing must first be done by the judge


to determine provisionally the value of the
property, which full amount should be deposited
with the national or provincial treasurer.
However, under the 1987 Rules on Civil
Procedure, it is provided that upon filing of the
complain and after due notice to the defendant
the plaintif shall have the right to take or enter
upon the possession of the real property involved
if he deposits with the authorized government
depositary an amount equivalent to the assessed
value of the property for purposes of taxation to
be held by such bank subject to the orders of the
court. There is therefore no need of a hearing,
as the issuance of the writ of possession by the
court becomes ministerial once the provisional
compensation is deposited.

KITY

TAU MU TAU MU TAU MU


TAU MU TAU MU TAU MU

CONSTITUTIONAL LAW II
Order of Saint Thomas More
Atty. Philip John Pojas/Atty. Rovyne G. Jumao-as, RN
University College of Law

Facts: On Sept. 17, 1993, the City of Cebu filed


expropriation proceedings against respondents
who owned parcels of land which the former
wanted to use for the construction of a public
road. On Sept. 21, 1994, the court issued a writ
of possession in favor of the City. Sec. 19 of the
Local Government Code of 1991 provides that
just compensation shall be determined as of the
time of actual taking. However, Sec. 4, Rule 67 of
the 1997 Rules of Procedure provides that just
compensation shall be determined at the time of
the filing of the complaint for expropriation.
Which shall prevail?
Held: The applicable law as to the point of
reckoning for the determination of just
compensation is the Local Government Code.
The 1997 Rules on Civil Procedure cannot prevail
over RA No. 7160, which is a substantive law.
However, since the parties voluntarily agreed to
be bound by the report of the commission and
approved by the trial court, said valuation of the
property has the force of law between the parties
and should be complied with in good faith. TAU MU
REPUBLIC vs. CA
383 SCRA 611 July 2, 2002
Facts: In 1969, the government instituted
expropriation proceedings against tracts of land
in Malolos, Bulacan, to be utilized for broadcast
operation and use of radio transmitter facilities of
the Voice of the Philippines. In 1979, the trial
court rendered judgment condemning the
property and ordering the government to pay as
just compensation P6.00 per square meter, which
included the 76,589 square meter subject of the
dispute. Eventually, portions of the land were
used for the expansion of Bulacan State
University. Despite the judgment however, and
despite utilization for public use by the
government, as of Sept. 16, 1999, the owners of
the property were not compensated. May the
owners demand the return of the property? If
not, may they demand compensation based on
the current value of the property?
Held: By final and executory judgment in the
expropriation proceedings, the previous owners
are not entitled to recover possession of their

ACADCOM 2010; Contributors: Gene Geocaniga, Jarissa Guiani, Darlene Magabilen


TAU MU Page 20 of 179

expropriated land which are still devoted to


public use but only to demand the fair market
value.
It is of no moment that the present
property use difers from the original purpose of
the expropriation. Surely, the government, as
condemnor and owner of the property, is well
within its rights to alter and decide the use, the
only, limitation being that it is for public use. The
compensation for the property should be
computed at its market value at the time it was
taken and appropriated by the State. However,
the owners are entitled to an interest of 12% per
annum computed from the date of the taking of
the property, i.e. September 1969, until the due
amount shall have been fully paid.

TAU MU
TAU MU
TAU MU
TAU MU
TAU MU
TAU MU
TAU MU

Facts: The plan to extend EDSA to Roxas


Boulevard to be ultimately linked to the Cavite
Coastal Road Project, originally called for the
expropriation of properties along Cuneta Avenue
in Pasay City. Later on, however, the Ministry of
Public Highways decided to make the proposed
extension pass through Fernando Rein and Del
Pan Streets. Because of the protests of residents
of the latter, the Commission on Human
Settlements recommended the reversion to the
original plan, but the Ministry argued the new
route withh save the government P2 million. The
government filed expropriation proceedings

TAU MU

DE KNECHT vs. BAUTISTA


100 SCRA 660 (1980)

TAU MU

JUDICIAL REVIEW

TAU MU
TAU MU

Facts: On April 5, 1948 an entry of judgment was


made in the expropriation case filed by the
Republic against the Denzons. The judgment
allowed the Republic to expropriate their property
after payment of just compensation in the
amount of P4,062.10. After the lapse of more
than 50 years, the Republic failed to pay the
amount, though it took possession of the
property. May the landowner be allowed to
recover?
Held: Yes. While we are aware of the doctrine
that non-payment of just compensation does not
entitle the landowner to recover possession, the
facts of the present case do not justify its
application. It bears stressing that the Republic
was ordered to pay just compensation twice, the
first was in the expropriation proceedings and the
second in, Valdehueza. Fifty-seven years have
passed since then. We cannot but construe the
Republics failure to pay just compensation as a
deliberate refusal on its part. We thus rule that
the special circumstances prevailing in this case
entitle respondent to recover possession of the
expropriated lot from the Republic. Unless this
form of swift an efective relief is granted to him,
the grave injustice committed against his
predecessors-in-interest, though no fault or
negligence on their part, will be perpetuated.

The Fraternal
Ateneo de Davao

TAU MU
TAU MU

REPUBLIC vs. LIM


462 SCRA 265 (2005)

KITY

TAU MU TAU MU TAU MU


TAU MU TAU MU TAU MU

CONSTITUTIONAL LAW II
Order of Saint Thomas More
Atty. Philip John Pojas/Atty. Rovyne G. Jumao-as, RN
University College of Law

against the owners of Fernando Rein and Del Pan


streets, among whom was petitioner.
Held: The choice of Fernando Rein and Del Pan
streets is arbitrary and should not receive judicial
aprpoval. The Human Settlements Commission
concluded that the cost factor is so minimal that
it can be disregarded in making a choice between
the two lines. The factor of functionality strongly
militates against the choice of Fernando Rein and
Del Pan streets, while the factor of social and
economic impact bears grievously on the
residents of Cuneta Avenue. While the issue
would seem to boil down to a choice between
people, on one hand, and progress and
development, on the other, it is to be
remembered that progress and development are
carried out for the benefit of the people. TAU MU
POWER OF TAXATION
Taxes and Taxation (Definition)
Taxes are the enforced proportional contributions
from persons and property, levied by the State by
virtue of its sovereignty, for the support of
government and for all public needs. Taxation is
the method by which these contributions are
exacted [Cruz, Constitutional Law 2007, p.86].
Purpose
The obvious, primary, and specific purpose of the
power to tax is to raise revenue. However, from
the earliest days of the history of the power of
taxation, the power to tax has been recognized
as an instrument of national economic and social
policy. It has, for instance, been used as an
instrument for the extermination of undesirable
activities and enterprises. In the celebrated words
of Justice Marshall, the power to tax involves the
power to destroy [Bernas, The 1987 Philippine
Constitution: A Comprehensive Reviewer, p. 279].
TAU MU

Nature
The importance of taxation derives from the
unavoidable obligation of the government to
protect the people and extend them benefits in
the form of public projects and services. In
exchange for these, the people are subjected to
the reciprocal duty of sharing the expenses to be
incurred therefor through the payment by them
of taxes.
The obligation to pay taxes is not based on
contract. It is a duty imposed upon the individual
by the mere fact of his membership in the body
politic and his enjoyment of the benefits available
from such membership. Hence, except only in the
case of poll taxes, nonpayment of a tax may be
the subject of criminal prosecution and
punishment. The accused cannot invoke the
prohibition
against
prohibition
against
imprisonment for debt as taxes are not
considered debts [Cruz, Constitutional Law 2007,
p.87]. TAU MU

Levied

TAX
to

Tax vs. License*


LICENSE
raise Imposed for regulatory

ACADCOM 2010; Contributors: Gene Geocaniga, Jarissa Guiani, Darlene Magabilen


TAU MU Page 21 of 179

KITY

purposes only
Justified under police
power, and the amount
of fees required is
usually limited only to
the cost of regulation.
Exception:
business
that is licensed is nonuseful and is sought to
be discouraged by the
legislature, in which
case a high license fee
is required.
*[Cruz, Constitutional Law 2007, p.87]
Scope
So pervasive is the power of taxation that it
reaches even the citizen abroad and his income
earned from sources outside his State. In other
cases, all income earned in the taxing State,
whether by citizens or aliens, and all immovable
and intangible personal properties found in its
territory, as well as tangible personal property
owned by persons domiciled therein, are subject
to its taxing power. Even shares of stock issued
by a foreign corporation, but in action in the
local State may be taxed by it, as so too are the
proceeds from an insurance policy issued abroad
[Cruz, Constitutional Law 2007, p.87]. TAU MU

TAU MU
TAU MU
TAU MU
TAU MU
TAU MU
TAU MU
TAU MU
TAU MU
TAU MU
TAU MU
TAU MU

Who may Exercise


The power of taxation is inherent in the State.
Primarily vested in the national legislature, it may
now also be exercised by the local legislative
bodies, no longer by virtue of a valid delegation
as before but pursuant to a direct authority
conferred by Article X, Section 5, of the
Constitution, which provides that each local
government unit shall have the power to create
its own sources of revenue and to levy taxes,
fees and charges, subject to such guidelines and
limitations as the Congress may provide,
consistent with the basic policy of local
autonomy.

TAU MU
TAU MU

revenues

Power to Tax and Power to Destroy


The power to tax may include the power to
destroy if it is used validly as an implement of the
police power in discouraging and in efect
ultimately
prohibiting
certain
things
or
enterprises inimical to the public welfare. Thus, if
massage parlors are found to be mere fronts for
prostitution, they may be subjected to such
onerous taxes as to practically force them to stop
operating. A similar imposition, for the same
purpose, may be levied upon non-useful
businesses or things, like pool halls, slot
machines, or idle lands.
But where the power to tax is used solely for the
purpose of raising revenues, the modern view is
that it cannot be allowed to confiscate or destroy.
If this is sought to be done, the tax may be
successfully attacked as an inordinate and
unconstitutional exercise of the discretion that is
usually vested exclusively in the legislature in
ascertaining the amount of the tax [Cruz,
Constitutional Law 2007, p.88].

The Fraternal
Ateneo de Davao

TAU MU TAU MU TAU MU


TAU MU TAU MU TAU MU

CONSTITUTIONAL LAW II
Order of Saint Thomas More
Atty. Philip John Pojas/Atty. Rovyne G. Jumao-as, RN
University College of Law

Not only the amount of the tax but also such


other questions as whether or not to tax in the
first place, or whom or what to tax, or for what
purpose, are likewise subject to the discretion of
the legislature. As usual, however, such discretion
is not unlimited. Its exercise may be reversed in
proper cases, specifically when it violates the due
process and equal protection clauses or the
particular restrictions on the power of taxation as
prescribed in Article VI, Section 28, of the
Constitution [Cruz, Constitutional Law 2007,
p.88].
The President of the Philippines may also exercise
the power of taxation to a limited extent when
delegated tarif powers (Sec 28 (2), Article VI, of
the Constitution) [Nachura, Outline Reviewer in
Political Law 2006, p. 61]. TAU MU
General Limit on the Power to Tax
The power to tax exists for the general welfare.
Hence implicit on the power is the limitation that
it should be exercised only for a public purpose.
To lay, with one hand, the power of the
government on the property of the citizen, and
with the other to bestow it upon favored
individuals to aid private enterprises and build up
private fortunes, is none the less a robbery
because it is done under the forms of law and is
called taxation.
The power to tax is an attribute of sovereignty.
In fact, it is the strongest of all the powers of
government. But for all its plenitude, the power to
tax is not unconfined as there are restrictions.
Adversely afecting as it does property rights,
both due process and equal protections clauses
of the Constitution may properly be invoked to
invalidate in appropriate cases a revenue
measure. If it were otherwise, there would be
truth to the 1903 dictum of Chief Justice Marshall
that the power to tax involves the power to
destroy. The web of unreality spun from Justice
Marshalls famous dictum was brushed away by
one stroke of Mr. Justice Holmes pen, thus: The
power to tax is not the power to destroy while
this Court sits. So it is in the Philippines.
[Bernas, The 1987 Philippine Constitution: A
Comprehensive Reviewer, p. 280] TAU MU
Limitations on the Exercise of the Power of
Taxation
a. Due Process of Law
Taxes will not be allowed if they are confiscatory,
except where they are intended precisely for
destruction as an instrument of police power
[Cruz, Constitutional Law 2007, p.89].
With the legislature primarily lies the discretion to
determine the nature, object, extent, coverage
and situs of taxation. But where a tax measure
becomes so unconscionable and unjust as to
amount to confiscation of property, courts will not
hesitate to strike it down, for despite all its
plentitude, the power to tax cannot override
constitutional prescriptions [Nachura, Outline
Reviewer in Political Law 2006, p. 61].
From the procedural viewpoint, due process does
not require previous notice and hearing before a

ACADCOM 2010; Contributors: Gene Geocaniga, Jarissa Guiani, Darlene Magabilen


TAU MU Page 22 of 179

law prescribing fixed or specific taxes on certain


articles may be enacted. But where the tax to be
collected is to be based on the value of the
taxable property, the taxpayer is entitled to be
notified of the assessment proceedings and to be
heard therein on the correct valuation to be given
the property. Otherwise, the ad valorem tax may
be increased to the prejudice of the owner in the
ex parte appraisal to be made by the government
authorities [Cruz, Constitutional Law 2007, p.90].

TAU MU
TAU MU
TAU MU
TAU MU
TAU MU
TAU MU
TAU MU
TAU MU

d. Public Purpose
To sustain a tax, it is necessary to show that the
proceeds are devoted to a public purpose.

TAU MU

There is no provision in the Constitution


specifically prohibiting double taxation. Our
Supreme Court has not categorically held that
double or multiple taxation is prohibited in our
jurisdiction. Double taxation is no more than
doubled taxation, as when a thing is taxed once
at Php 250.00 and taxed again at another Php
250.00 while a similar thing is taxed only once at
Php 500.00. The power to tax twice, it is said,
is as ample as to tax twice.
There is double taxation when additional taxes
are laid on the same subject by the same taxing
jurisdiction during the same taxing period and for
the same purpose. Thus, if a persons properties
are each taxed separately and thereafter all of
them are again taxed, this time collectively, by
the same taxing jurisdiction for the same purpose
and during the same taxing period, the second
imposition would constitute double taxation
[Cruz, Constitutional Law 2007, p.92].

TAU MU
TAU MU

c. Double Taxation
Additional taxes are laid on the same subject by
the same taxing jurisdiction during the same
taxing period and for the same purpose
[Nachura, Outline Reviewer in Political Law 2006,
p. 62].

The Fraternal
Ateneo de Davao

TAU MU
TAU MU

b. Equal Protection
The rule of taxation shall be uniform and
equitable (Section 28, Article VI, Constitution).
Uniformity in taxation means that persons or
things belonging to the same class shall be taxed
at the same rate. Thus, if cigarettes are classified
into local and imported, there is observance of
the uniformity rule if all local cigarettes are taxed
at Php 12.00 per carton, regardless of value, and
all imported cigarettes are taxed at Php 20.00 per
carton, also regardless of value. TAU MU
This should be distinguished from equality in
taxation, which simply means that the tax shall
be strictly proportional to the relative value of the
property.
The above rules require a valid classification in
the selection of the object of taxation. Higher
taxes may be imposed on commercial or
industrial lands than on residential lands, or on
practitioners in urban centers than in rural areas.
It is also provided that the rule of taxation shall
be equitable. Equitable taxation connotes that
taxes should be apportioned among the people
according to their capacity to pay [Cruz,
Constitutional Law 2007, p.90].

KITY

TAU MU TAU MU TAU MU


TAU MU TAU MU TAU MU

CONSTITUTIONAL LAW II
Order of Saint Thomas More
Atty. Philip John Pojas/Atty. Rovyne G. Jumao-as, RN
University College of Law

Revenues derived from taxes cannot be used for


purely private purposes or for the exclusive
benefit of private persons.
Like public use in eminent domain, the phrase
public purpose as applied to taxation is now
given the broadest interpretation as to include
even indirect public advantage or benefit. The
mere fact that the tax will be directly enjoyed by
a private individual does not make it invalid so
long as some link to the public welfare is
established [Cruz, Constitutional Law 2007, p.92].
TAU MU

Tax Exemptions [Cruz, Constitutional Law 2007,


p.92-96]
Tax exemptions are either constitutional or
statutory.
The constitutional exemption from taxes is
provided for in Article VI, Section 28(3):
Charitable
institutions,
churches,
and
parsonages or convents appurtenant thereto,
mosques, non-profit, cemeteries, and all lands,
buildings, and improvements actually, directly,
and exclusively used for religious, charitable or
educational purposes shall be exempt from
taxation.
Exemption is granted religious and charitable
institutions because they give considerable
assistance to the State in the improvement of the
morality of the people and the care of the
indigent and the handicapped. There are
ministrant functions of the government which,
however, it is not able to fully discharge. The
above provision is intended to make it easier for
these institutions to pursue these laudable
objectives without the impediment of taxes that
they otherwise would have to shoulder. The
added justification in the case of religious
institutions is the principle to give full rein to the
freedom of religious profession and worship.
The provision now requires that the lands,
buildings or improvements be directly, actually
and exclusively devoted to religious, charitable or
educational purposes to be entitled to exemption.
Under the 1935 Constitution, it was enough that
they were at least incidentally devoted to these
purposes.
Statutory exemptions are granted in the
discretion of the legislature. However, the
Constitution provides that no law granting any
tax exemption shall be passed without the
concurrence of a majority of all the Members of
the Congress. This is because tax exemptions
should not be lightly extended since they will
represent a loss of revenue to the government.
TAU MU

LLADOC vs. CIR


14 SCRA 292
Facts: A parish priest accepted a donation of
10,000, to be used for the construction of a

ACADCOM 2010; Contributors: Gene Geocaniga, Jarissa Guiani, Darlene Magabilen


TAU MU Page 23 of 179

TAU MU
TAU MU
TAU MU
TAU MU
TAU MU

Meaning of Life, Liberty and Property

TAU MU

Who are Protected


It is universal in application to all persons,
without regard to any diference in race, color, or
nationality. Artificial persons are covered by the
protection but only insofar as their property is
concerned. The guarantee extends to aliens and
includes the means of livelihood (Villegas vs Hui
Chong) [Nachura, Outline Reviewer in Political
Law 2006, p. 82].

TAU MU

Due process is a guaranty against any


arbitrariness on the part of the government,
whether committed by the legislature, the
executive, or the judiciary. If the law itself
unreasonably deprives a person of hi life or his
liberty or his property, he is denied the protection
of due process. Of the enjoyment of his rights is
conditioned on an unreasonable requirement, due
process is likewise violated. Any government act
that militates against the ordinary norms of
justice or fair play is considered an infraction of
the greatr guaranty of due process; and this is
true whether the denial involves violation merely
of the procedure prescribed by the law or afects
the very validity of the law itself [Cruz,
Constitutional Law 2007, p.100]. TAU MU

TAU MU

Due Process of Law Definition


A law which hears before it condemns, which
proceeds upon inquiry and renders judgment only
after trial. It is the responsiveness to the
supremacy of reason, obedience to the dictates
of justice (Ermita-Malate Hotel vs City of Manila).
The embodiment of the sporing idea of fair play
[Nachura, Outline Reviewer in Political Law 2006,
p. 82].

TAU MU

Section 1. No person shall be deprived of


life, liberty or property without due process
of law, nor shall any person be denied the
equal protection of the laws.

TAU MU
TAU MU

DUE PROCESS AND EQUAL PROTECTION

The Fraternal
Ateneo de Davao

TAU MU
TAU MU

church. The money was spent for the purpose


specified. Later, the Bureau of Internal Revenue
imposed a donees tax on his successor, who
objected, invoking the constitutional exemption of
church properties from taxes.
Held: The SC rejected his contention, holding
that the exemption referred only to property
taxes
imposed
on
lands,
buildings
and
improvements used for religious purposes. The
tax in this case was not an ad valorem tax on the
church itself but an excise tax imposed on the
priest for his exercise of the privilege to accept
the donation. The Commission assessed the
donees gift tax; the assessment was not the
properties themselves. Manifestly, a gift tax is not
within the exemption provisions. A gift tax is not a
property tax, but an excise tax imposed on the
transfer of property by way of gift inter vivos, the
imposition of which on property used exclusively
for religious purposes does not constitute an
impairment of the Constitution.

KITY

TAU MU TAU MU TAU MU


TAU MU TAU MU TAU MU

CONSTITUTIONAL LAW II
Order of Saint Thomas More
Atty. Philip John Pojas/Atty. Rovyne G. Jumao-as, RN
University College of Law

Life
The constitutional protection of the right to life is
not just a protection of the right to be alive or to
the security of ones limb against physical harm.
The right to life is the right to a good life. The
emphasis on the quality of living is found in
Article II where Section 6 commands the State to
promote a life of dignity and where Section 7
guarantees a decent standard of living.
[Bernas, The 1987 Philippine Constitution: A
Comprehensive Reviewer, p. 24] TAU MU
Life includes the right of an individual to his body
in its completeness, free from dismemberment,
and extends to the use of God-given facilities
which make life enjoyable [Justice Malcolm,
Philippine Constitutional Law, p. 320]
The unborn have the constitutional right to life as
embodied in Section 12, Article II of the
Constitution, thus:
The State recognizes the sanctity of family life
and shall protect and strengthen the family as a
basic autonomous social institution. It shall
equally protect the life of the mother and the life
of the unborn from conception. The natural and
primary right and duty of parents in the rearing
of the youth for civic efficiency and the
development of moral character shall receive the
support of the Government.
Liberty
According to Mabini, liberty is the freedom to do
right and never wrong. Liberty, as guaranteed
under the due process clause, is not unbridled
license; it is liberty regulated by law. A person is
free to act but he may exercise his rights only in
such manner as not to injure the rights of others.
Ones own liberty must be enjoyed consistently
with the enjoyment of a like liberty by others. In
other words, the individual, as a creature of
society, should be prepared to surrender part of
his freedom for the benefit of the greater number
in recognition of the time-honored principle of
salus populi est suprema lex. TAU MU
Thus, to illustrate, ones freedom of expression
cannot be unsed unfairly to destroy anothers
reputation, or to incite rebellion, or to ofend
public morals; neither may he abuse the sanctity
of his home by converting it into a den of
criminality or hotbed of disease; nor may he insist
on selling his goods at black market prices, if they
be prime necessities, to the detriment of the
consuming public. TAU MU
Subject only to reasonable restrictions of the law,
a person is free to do as he pleases. He may
marry for love or for money, pursue a profession
or engage in manual labor, establish his own
business or merely hire out as an employee,
isolate himself from the community or mix with
his neighbors, profess a religion or embrace
atheism in short, do anything that does not
ofend the public welfare [Cruz, Constitutional
Law 2007, p.104].
Property
Property is anything what can come under the
right of ownership and be the subject of contract.

ACADCOM 2010; Contributors: Gene Geocaniga, Jarissa Guiani, Darlene Magabilen


TAU MU Page 24 of 179

This will include all things real, personal,


tangible and intangible that are within the
commerce
of
man,
like
lands,
jewelry,
automobiles, buildings, goodwill, inheritance,
intellectual creations, future earnings, works of
art, animals, mortgages, proceeds, etc [Cruz,
Constitutional Law 2007, p105].

TAU MU
TAU MU
TAU MU
TAU MU
TAU MU
TAU MU
TAU MU
TAU MU

Life and Property do NOT Enjoy Identical


Protection from the Constitution
The primacy of human rights over property rights
is recognized. In the hierarchy of civil liberties,
the rights of free expression and of assembly
occupy a preferred position as they are essential
to the preservation and vitality of our civil and
political institutions. TAU MU
The superiority of these freedoms over property
rights is underscored by the fact that a mere
reasonable or rational relation between the
means employed by the law and its object or
purpose that the law is neither arbitrary nor
discriminatory nor oppressive would suffice to
validate a law which restricts or impairs property
rights. On the other hand, a constitutional or valid
infringement of human rights requires a more
stringent criterion, namely existence of a grave
and immediate danger of a substantive evil which
the State has the right to prevent. (Philippine
Blooming Mills Employees Organization vs
Philippine Blooming Mills, Inc.)
[Bernas, The
1987 Philippine Constitution: A Comprehensive
Reviewer, p. 26]. TAU MU

TAU MU

Public office is not property; but one unlawfully


ousted from it may institute an action to recover
the same, flowing from the de jure officers right
to office. Indeed, the Court has recognized that
while public office is not property to which one
may acquire a vested right, it is nevertheless a
protected right [Nachura, Outline Reviewer in
Political Law 2006, p. 82].

TAU MU
TAU MU

A mining license that contravenes a mandatory


provision of law under which it is granted is void.
Being a mere privilege, a license does not vest
absolute rights in the holder. Thus, without
ofending the due process and the nonimpairment clauses of the Constitution, it can be
revoked by the State in the public interest. Mere
privileges, such as the license to operate
cockpits, are not property rights and are
revocable at will [Nachura, Outline Reviewer in
Political Law 2006, p. 83].

The Fraternal
Ateneo de Davao

TAU MU
TAU MU

Protected property includes all kinds of property


found in the Civil Code. A license to operate a
cockpit is not considered protected property. It is
deemed merely a privilege withdrawable when
the public interest require its withdrawal.
A mere privilege, however, may evolve into some
form of property right protected by due process,
as for instance when a privilege, in this case an
export quota, has been enjoyed for so long, has
been subject of substantial investment and has
become the source of employment for thousands
(American Interfashion Corporation vs Office of
the President) [Bernas, The 1987 Philippine
Constitution: A Comprehensive Reviewer, p. 25].

KITY

TAU MU TAU MU TAU MU


TAU MU TAU MU TAU MU

CONSTITUTIONAL LAW II
Order of Saint Thomas More
Atty. Philip John Pojas/Atty. Rovyne G. Jumao-as, RN
University College of Law

Two Kinds of Due Process


Due process has both a procedural and a
substantive aspect.
As a substantive requirement, it is a prohibition of
arbitrary laws; because, if all that the due process
clause required were proper procedure, then life,
liberty, or property could be destroyed arbitrarily
provided proper formalities are observed.
As a procedural requirement, it relates chiefly to
the mode of procedure which government
agencies must follow in the enforcement and
application of laws. It is a guarantee of
procedural fairness. Its essence was expressed by
Daniel Webster as a laws which hears before it
condemns
[Bernas,
The 1987 Philippine
Constitution: A Comprehensive Reviewer, p. 27].
PROCEDURAL DUE PROCESS
Requisites of Civil Procedural Due Process
(Judicial Due Process):
a. An impartial court clothed with judicial
power to hear and determine the matters
before it;
b. Jurisdiction properly acquired over the
person of the defendant and over property
which is the subject matter of the
proceeding;
c. Opportunity to be heard;
d. Judgment rendered upon lawful hearing
and based on evidence adduced. (Banco
Espaol Filipino vs Palanca) TAU MU
Impartial and Competent Court
It is clear that a court afected by bias or
prejudice cannot be expected to render a fair and
impartial decision. As our Supreme Court has
declared, every litigant is entitled to the cold
neutrality
of
an
impartial
judge
[Cruz,
Constitutional Law 2007, p.110].
In Rivera vs CSC and in Singson vs NLRC, the
Supreme Court reiterated the rule that a public
officer who decided the case should not be the
same person to decide it on appeal because he
cannot be an impartial judge [Nachura, Outline
Reviewer in Political Law 2006, p. 84].
In the Ynot case, the Supreme Court said:
the closed mind has no place in the open
society. It is part of the sporting idea of fair play
to hear the other side before an opinion is formed
or a decision is made by those who sit in
judgment
And in Javier vs COMELEC, the imperative of
impartiality was described, thus:
This court has repeatedly and consistently
demanded the cold neutrality of an impartial
judge as the indispensible imperative of due
process. To bolster that requirement, we have
held that the judge must not only be impartial
but must also appear to be impartial as an added
assurance to the parties that his decision will be
just. The litigants are entitled to no less than
that. They should be sure that when their rights
are violated they can go to a judge who shall give

ACADCOM 2010; Contributors: Gene Geocaniga, Jarissa Guiani, Darlene Magabilen


TAU MU Page 25 of 179

TAU MU
TAU MU
TAU MU
TAU MU
TAU MU
TAU MU
TAU MU
TAU MU

In actions in personam, such as a complaint for


recovery of a loan, jurisdiction over the defendant
is acquired by the court by his voluntary
appearance or through service of summons upon
him. This may be efected personally, or by
substituted service, or, in exceptional cases, by
publication.
In actions in rem or quasi in rem such as land
registration proceedings or the foreclosure of a
real estate mortgage, the jurisdiction of the court
is derived from the power it may exercise over
the property. Jurisdiction over the person is not
essential, provided the relief granted by the court
is limited to such as can be enforced against the
property itself. Notice by publication is sufficient
in these cases. TAU MU
The reason is that property is always presumed to
be in the possession of the owner or his agent,

TAU MU

Jurisdiction
It should be emphasized that the service of
summons is not only required to give the court
jurisdiction over the person of the defendant but
also to aford the latter the opportunity to be
heard on the claim made against him. Thus,
compliance with the rules regarding the service of
summons is as much an issue of due process as
of jurisdiction.
While jurisdiction over the person of the
defendant can be acquired by the service of
summons, it can also be acquired by voluntary
appearance before the court, which includes
submission of pleadings in compliance with the
order of the court or tribunal [Nachura, Outline
Reviewer in Political Law 2006, p. 86].

TAU MU
TAU MU

By competent court is meant one vested with


jurisdiction over a case as conferred upon it by
law. For example, a regional trial court is
competent to try a prosecution for murder but not
for violation of a municipal ordinance. The
competent court in the latter case would be the
metropolitan or the municipal trial court. Only the
Supreme Court is competent to review a decision
of the Commission on Audit, but jurisdiction over
ordinary appealed cases involving only questions
of fact is vested in the Court of Appeals [Cruz,
Constitutional Law 2007, p.112].

The Fraternal
Ateneo de Davao

TAU MU
TAU MU

them justice. They must trust the judge,


otherwise they will not go to him at all. They
must believe in his sense of fairness, otherwise
they will not seek his judgment. Without such
confidence, there would be no point in invoking
his action for the justice they expect.
Due process is intended to insure that confidence
by requirements with what Justice Frankfurter
calls the rudiments of fair play. Fair play calls for
equal justice. There cannot be equal justice
where a suitor approaches a court already
committed to the other party and with a
judgment already made and waiting only to be
formalized after the litigants shall have
undergone the charade of a formal hearingthe
judge will reach his conclusions only after the
evidence is in and all the arguments are filed, on
the basis of the established facts and pertinent
law. TAU MU

KITY

TAU MU TAU MU TAU MU


TAU MU TAU MU TAU MU

CONSTITUTIONAL LAW II
Order of Saint Thomas More
Atty. Philip John Pojas/Atty. Rovyne G. Jumao-as, RN
University College of Law

who may be safely held under certain conditions


to know that proceedings have been instituted
against it. According to Cooley, if the owners are
named in the proceedings and personal notice is
provided for, it is rather from tenderness to their
interests, and in order to make sure that the
opportunity for a hearing shall not be lost to
them, than from any necessity that the case shall
assume that form [Cruz, Constitutional Law 2007,
p.113].
Hearing/Opportunity to be Heard
Due process is satisfied as long as the party is
accorded with the opportunity to be heard. If it is
not availed of, it is deemed waived or forfeited
without violating the constitutional guarantee
(Bautista vs CA) [Nachura, Outline Reviewer in
Political Law 2006, p. 86].
Notice to a party is essential to enable it to
adduce its own evidence and to meet and refute
the evidence submitted by the other party. Every
litigant is entitled to his day in court. He has a
right to be notified of every incident of the
proceeding and to be present at every stage
thereof so that he may be heard by himself and
counsel for the protection of his interests.
As held in David vs Aquilizan, a decision rendered
without a hearing is null and void ab initio and my
be attacked directly or collaterally.
If it were otherwise, the Supreme Court
declared, then the cardinal requirement that no
party should be made to sufer in person or
property without being given a hearing would be
brushed aside. The doctrine consistently adhered
to by this Court when such a question arisesis
that a denial of due process suffices to cast on
the official act taken by whatever branch of the
government the impress of nullity. [Cruz,
Constitutional Law 2007, p.114]
Not all cases require a trial-type hearing. Due
process in labor cases before a Labor Arbiter is
satisfied when the parties are given the
opportunity to submit their position papers to
which they are supposed to attach all the
supporting documents or documentary evidence
that would support their respective claims
(Mariveles Shipyard vs CA) [Nachura, Outline
Reviewer in Political Law 2006, p. 87].
In Zaldivar vs Sandiganbayan, the Supreme Court
declared that to be heard does not only mean
verbal arguments in court. One may be heard
also through pleadings. Where the opportunity to
be heard, either through oral arguments or
pleadings, is accorded, there is no denial of
procedural due process.
ANG TIBAY vs. CIR
69 PHIL 635 (140)
The court held in this case the requisites for
administrative due process:
1. The right to a hearing which includes the
right to present ones case and submit the
evidences in support thereof.

ACADCOM 2010; Contributors: Gene Geocaniga, Jarissa Guiani, Darlene Magabilen


TAU MU Page 26 of 179

TAU MU
TAU MU
TAU MU
TAU MU
TAU MU
TAU MU
TAU MU

1. The students must be informed in writing


of the nature and cause of any accusation
against them.
2. They shall have the right to answer the
charges against them with the assistance
of counsel if desired
3. They shall be informed of the evidences
against them.
4. They shall have the right to adduce
evidence in their own behalf
5. The evidences must be duly considered by
the investigating committee or official
designated by the school authorities to
hear and decide the case.
The students invoked the Ang Tibay case but the
court held that the applicable case is the Guzman
case.

TAU MU

Important points in this case:


Five requirements to satisfy due process in the
discipline of students:

TAU MU

Facts: There was a hazing conducted as part of


the initiation rites of the Aquila Legis Fraternity of
ADM where the victim, Villa, was an applicant,
and was killed as a result thereof. Students who
acted as master auxiliaries were refused
admittance to the school after having been found
guilty of participating in hazing. The decision of
the school to dismiss the students was based on
the findings submitted by the Board. The accused
contested that they were not given copies of the
schools rules and regulations. They were not
warned on the conduct to avoid, thus, they were
not aforded due process.
Held: The court held that this is an exception to
due process. Since they are law students, they
cannot use that argument. It was presumed that
they have received a copy or have diligently
asked for one. If they were undergraduates, they
would have been treated diferently. Hazing was
not defined in the manual but is the same
punishable.

TAU MU
TAU MU

ADM vs. CAPULONG


222 SCRA 644 (1993)

The Fraternal
Ateneo de Davao

TAU MU
TAU MU

2. The tribunal must consider the evidences


presented.
3. The decision must have something to
support itself
4. The evidence must be substantial
5. The decision must be rendered on the
evidence presented at the hearing or at
least contained in the record and disclosed
to the parties afected.
6. The tribunal or any of its judges must act
on its own independent consideration of
the law and facts of the controversy and
not simply accept the views of a
subordinate.
7. The body should, in all controversial
questions, render its decision in such a
manner that the parties to the proceeding
can know the various issues involved and
the reason for the decision.

KITY

TAU MU TAU MU TAU MU


TAU MU TAU MU TAU MU

CONSTITUTIONAL LAW II
Order of Saint Thomas More
Atty. Philip John Pojas/Atty. Rovyne G. Jumao-as, RN
University College of Law

They also argued that the fifth requirement was


not satisfied: their evidence was not considered.
The court ruled that whether or not the evidence
entered the process of decision making or
consideration is very hard to prove and that is
why the court said that the fact that it was
mentioned as having been considered is
sufficient.
LAO GI vs. CA
180 SCRA 756 (1989)
Facts: On Sept 3, 1958, the Secretary of Justice
found Filomeno Chia, Jr., alis Sia Pieng Hui, to be a
Filipino Citizen as it appears that hid father is a
Filipino citizen. However, on October 3, 1980, the
Minister of Justice set aside his citizenship on the
ground hat it was founded on fraud and
misrepresentation. On March 9, 1981, a charge
for deportation was filed with the CID against Lao
Gi alias Filomeno Chia, Jr., his wife and children. In
this case, it appears that petitioners are charged
with having entered the Philippines by means of
false and misleading statements or without
inspection or admission by the immigration
authority at a designated port of entry.
Issue: Whether due process was denied from the
petitioners
Held: While it is not disputed that it is also within
the power and authority of the commissioner to
require an alien to register, such a requirement
must be predicated in a positive finding that the
person who is so required is an alien. In this case
where the very citizenship of the petitioner is in
issue, there should be a previous determination
by the CID that they are aliens before the
petitioners may be directed and required to
register as aliens. Although a deportation
proceeding does not partake of the nature of a
criminal action, however, considering that it is a
harsh
and
extraordinary
administrative
proceeding afecting freedom and liberty of a
person, the constitutional right of such person to
due process should not be denied. The charge
against an alien must specify the acts or
omissions complained of which must be stated in
ordinary and concise language to enable a person
of common understanding to know on what
ground he is intended to be deported and enable
the CID to pronounce a proper judgment before
any charge should be filed in the CID. A
preliminary investigation must be conducted to
determine if there is sufficient cause to charge
the respondent for deportation.
Petition is GRANTED and the question
order of the respondent CID is set aside. CID is
directed to continue hearing the deportation case
against the petitioners and thereafter, based on
the evidence before it, resolve the issue of
citizenship of petitioners, and if found to be
aliens, to determine whether or not the
petitioners should be deported and/or otherwise
ordered to register as aliens.
TAADA vs. TUVERA
146 SCRA 446 (1986)

ACADCOM 2010; Contributors: Gene Geocaniga, Jarissa Guiani, Darlene Magabilen


TAU MU Page 27 of 179

The case involves the requirement that all laws


must be published in order to take efect. The
court stated that publication of laws is an
element of due process because how can it be
efective if people do not know about its
existence.
Laws would entail publication of all laws,
including statutes of local application that are of
public interest. Only regulations that are internal
in nature or which regulate the personnel of an
administrative agency are not required to be
published.

TAU MU
TAU MU
TAU MU
TAU MU
TAU MU
TAU MU
TAU MU

PHILCOMSAT vs. ALCUAZ

TAU MU

Facts: Petitioner was sentenced to forced


resignation without separation benefits by the
Merit Systems Protection Board which was
chaired by one Thelma Gaminde. He appealed to
the Civil Service Commission which sustained the
appealed decision.
He filed a motion for
reconsideration which was also denied by the
CSC. However, Gaminde participated in resolving
the motion for reconsideration after her
appointment as CSC Commissioner. Is there a
violation of due process?
Held: Yes. In order that the review of the
discussion of a subordinate officer might not turn
out to be a farce, the reviewing officer must
perforce be other than the officer whose decision
is under review; otherwise there could be no
diferent view or there would be no real review of
the case. The decision of the reviewing officer
would be a biased view; inevitably it would be the
same view since being human he would not
admit that he was mistaken in his first view of the
case.

TAU MU

RIVERA vs. CSC


240 SCRA 3 (1995)

TAU MU
TAU MU

Facts:Petitioner here was charged with violation of ordinances


which requires managers of fishponds to pay operation tax.
He is a lessee of a forest land converted to a fishpond. He is
charged because of non-payment of tax. He admits that he
has not paid said tax but alleges that the ordinances are
unconstitutional or that it does not apply to him. He anchors
his allegation on the vagueness of the law. The ordinance
provides: if youre the owner or manager of a fishpond, you
pay P3.00 tax. You should be taxed 3 years after the approval
of the measure. Petitioner argues that the tax is vague
because there is no specific date as to its effectivity. Assuming
that it is not vague, it does not apply to him because he is not
the owner or manager of the fishpond but lessee.
Held: The law is not vague. A law is only vague if it is utterly
vague on its face. A vague law violates due process since it
will deny people of fair notice of what conduct to avoid. A
statute is only vague if people of common intelligence guess as
to its meaning or would differ in the application.
In the case at bar, the petitioner is included in the term
manager because even if he is just a lessee, he still invests in
it. Even if there is no specific date as to the effectivity of the
tax measure, tax would accrue on a specific time, that is 3
years after the approval which is sure to happen.

The Fraternal
Ateneo de Davao

TAU MU
TAU MU

PEOPLE vs. NAZARIO


165 SCRA 186 (1988)

KITY

TAU MU TAU MU TAU MU


TAU MU TAU MU TAU MU

CONSTITUTIONAL LAW II
Order of Saint Thomas More
Atty. Philip John Pojas/Atty. Rovyne G. Jumao-as, RN
University College of Law

180 SCRA 218 (1989)


Facts: PHILCOMSAT was given franchise to
operate facilities for satellite communications.
After a few years it was placed under the
jurisdiction of the NTC which had the power of
fixing rates. NTC ordered the petitioner to reduce
its rate by 15%. Since the order was issued
without notice and hearing, it was challenged as
a violation of due process.
Held: The court held that the order issued by
NTC applies to one individual. It is quasi-judicial
in nature. Hence, notice and hearing are essential
for due process. If the order applies to all, it is
rule-making and therefore quasi-legislative. It
does not need notice and hearing. In this case,
since it is only PHILCOMSAT that is afected by
the order, it is entitled to notice and hearing to
have due process. It is quasi-judicial. Likewise the
order was based on petitioners financial
statement. Therefore, it should have been given
an opportunity to dispute the findings.
RADIO vs. NTC
184 SCRA 517 (1990)
Facts: In January 1984, PLDT applied to NTC
approval of rates for Digital Telecommunication
Service
Facilities
which
was
provisionally
approved. In a hearing, petitioners were not
included in the list of afected parties.
Issue: Whether NTC gravely abused its discretion
amounting to excess or lack of jurisdiction of
issuing a provisional authorization in favor of
PLDT, without prior notice to petitioners.
Held: Petitioners alleged that the application is
not for approval of rates but for authorization to
engage in new services not covered by PLDTs
franchise and certificate of public convenience
and necessity. PLDT maintains that its franchise
authorization is to operate not only telephone
system but transmission service facilities.
Petition is devoid of merit. Well-settled is the rule
that Public Service Commission is empowered to
approve provisionally rates without the necessity
of a prior hearing. NTC did not grant PLDT any
authority to engage in any new common service.
As to the required notice, it is impossible for the
respondent commission to give personal notice to
all parties afected, not all of them being known
to it. There is no dispute that the notice was
published and one of them received the notice
which in turn informed the others. The petitioners
have timely opposed the petition in question, lack
of notice was deemed cured.
ESTATE VS CA
Facts: Petitioner owned a Quonset building
located on a lot of the PPA. The Quonset was
being used to store copra and petitioner had a
lease contract over the lot with PPA. Respondent,
the mayor, demolished the building on the
ground that it did not conform with the zoning
ordinance which located the zone for warehouse
elsewhere.
Held: The court held that while the zoning
ordinance authorizes the removal of any property

ACADCOM 2010; Contributors: Gene Geocaniga, Jarissa Guiani, Darlene Magabilen


TAU MU Page 28 of 179

TAU MU
TAU MU
TAU MU
TAU MU
TAU MU

SUMMARY DISMISSAL vs. TORCITA


330 SCRA 153 (2000)

TAU MU

Facts:
Mollaneda was a DECS Division
Superintendent in Davao City charged before the
Civil Service Commission with grave misconduct
and conduct grossly prejudicial to the service
arising from an act of sexual harassment. The
Commission assigned a hearing officer to conduct
the hearing including the taking of the testimony
of witnesses.
Thereafter, based on the
recommendation of the hearing officer, the
Commission dismissed Mollaneda from the
service. Considering that the testimony was not
actually heard by the Commission, was the right
of Mollaneda to due process violated?
Held:
No. Due process of law or the
requirements of fair hearing does not require that
the actual taking of testimony be before the same
officer who will make the decision in the case. As
long as a party is not deprived of his right to
present his own case and submit evidence in
support thereof, and the decision is supported by
the evidence in the record, there is no question
that the requirements of due process and fair trial
are fully met. In short, there is no abnegation of
responsibility on the part of the tribunal
concerned as the actual decision remains with
and is made by it.

TAU MU

MOLLANEDA vs. UMACOB


358 SCRA 537 (2001)

TAU MU

Facts: Respondents were public school teachers


who were dismissed by the Secretary of
Education for participating in the walk-outs and
strikes in 1990. The Magna Carta provides that a
member or a representative of any local or
national teachers organization must be included
in the investigating committee. This was not the
case.
Held: Violation of due process impartiality

TAU MU
TAU MU

FABELLA vs. CA
282 SCRA 256 (1997)

The Fraternal
Ateneo de Davao

TAU MU
TAU MU

which does not conform to it. It should not be


interpreted as authorizing the summary removal
of the Quonset building. If it does, it would be a
contravention of the requirements of due process.
Violation of an ordinance does not empower the
mayor to avail of extrajudicial remedies. On the
contrary the LGC imposes upon him the duty to
institute judicial proceedings for violation of
ordinance. The authority to abate nuisances
without judicial proceedings applies to nuisances
per se. While the Sangguniang Bayan may
provide for the abatement of a nuisance per se
when it is not, the nuisance can only be so
adjudged by a judicial determination. Petitioner
ws entitled to an impartial hearing to determine
whether the Quonset building was a nuisance per
se.

KITY

TAU MU TAU MU TAU MU


TAU MU TAU MU TAU MU

CONSTITUTIONAL LAW II
Order of Saint Thomas More
Atty. Philip John Pojas/Atty. Rovyne G. Jumao-as, RN
University College of Law

Facts: Respondent policeman was charged


before the Summary Dismissal Board with 7
counts of Abuse of Authority, 3 counts of Conduct
Unbecoming of a Police Officer, 1 Grave Threats
and 1 Illegal Search in connection with a single
incident. During the pre-trial, the 12 cases were
consolidated into one major complaint of
conduct unbecoming of a police officer. After
hearing, however, the charge was dismissed for
insufficiency of evidence.
Instead, the Board
found him guilty only of Simple Irregularity in the
Performance of Duty for taking alcoholic drinks
during the incident in question and sentenced
him to 20 days suspension. Did the Board violate
respondents right to due process?
Held: Yes, while the definition of conduct
unbecoming of a police officer, the more serious
ofense, is broad and almost all-encompassing, a
finding of guilt for an ofense, no matter how
light, for which one is not properly charged and
tried cannot be countenanced without violation
the rudimentary requirements of due process.
None of the 12 administrative cases made
reference to the specific act of being drunk while
in the performance of official duty. There is also
no indication in the summary dismissal
proceedings that respondent was also being
charged with breach of internal discipline
consisting of taking alcoholic drinks while in the
performance of his duties.
Respondent was
entitled to know what was he being charged of so
that he could traverse the accusation and
squarely adduce evidence in his defense.
SECRETARY vs. LANTION
343 SCRA 377 (2000)

TAU MU

Facts: The Department of Justice received from


the Department of Foreign Afairs U.S. Note
verbale No. 0522 requesting for the extradition of
Mark Jimenez. Together with the request was a
Grand Jury Indictment and Warrant of Arrest
issued by the United States District Court for the
Southern District of Florida for tax evasion, wire
fraud, conspiracy, giving false statements and
illegal campaign contributions to President
Clinton. The Secretary immediately formed a
panel to conduct a technical evaluation of the
request. Meanwhile, Jimenez demanded a copy
of the documents and asked for ample time to
comment of the request.
The DOJ refused
contending that the documents included grand
jury proceedings which are covered by secrecy
rules and may only be released upon orders of
the United States Court. The Philippine Supreme
Court initially ruled that as part of due process
Jimenez was entitled to the documents as well as
to oppose the request even before the petition
reaches the court.
Hence, the Motion for
Reconsideration.
Is Jimenez entitled to the
documents as a requirement of due process?
Held: No. there is no provision in the RP-US
Extradition Treaty of PD No. 1069 which gives an
extraditee the right to demand from the Justice
Secretary copies of the extradition request from
the United States government and its supporting
documents and to comment thereon while the

ACADCOM 2010; Contributors: Gene Geocaniga, Jarissa Guiani, Darlene Magabilen


TAU MU Page 29 of 179

TAU MU
TAU MU
TAU MU
TAU MU
TAU MU
TAU MU
TAU MU

Facts: Sangguniang Bayan member Nicolas


Montesclaros was 72 years old widower when he
married Milagros on July 10, 1983. He retired on

TAU MU

GSIS vs. MONTESCLAROS


434 SCRA 441 (2004)

TAU MU

Facts: President Macapagal-Arroyo issued EO


156 on Dec. 12, 2002, prohibiting the importation
of all types of used motor vehicles into the
country. Importers of said vehicles challenged the
order on the ground that it was issued without
notice and hearing. Is the executive order valid?
Held: Yes. As in the enactment of laws, the
general rule is that, the promulgation of
administrative issuances requires [no] previous
notice and hearing, the only exception being
where that legislature itself requires it and
mandates that the regulation shall be based on
certain facts as determined at an appropriate
investigation. This exception pertains to the
issuances of legislative rules as distinguished
from interpretative rules which give no real
consequences more than what the law itself has
already prescribed; and are designed merely to
provide guidelines to the law which the
administrative agency is in charge of enforcing. A
legislative rule, on the other hand, is in the
nature of subordinate legislation, crafted to
implement a primary legislation.
In this case, EO 156 is obviously a legislative rule
as it seeks to implement or execute primary
legislative enactments intended to protect the
domestic industry by imposing a ban on the
importation of a specified product not previously
subject to such prohibition. The due process
requirements in the issuance thereof are
embodied in Section 401 of the Tarif and
Customs Code, which essentially mandate the
conduct of investigation and public hearings
before the regulatory measure or importation ban
may be issued. But since respondents did not
raise the issue, the presumption is that said
executive issuance duly complied with the
procedures and limitations imposed by law.
SUBSTANTIVE DUE PROCESS

TAU MU
TAU MU

EXECUTIVE vs. SOUTHWING


482 SCRA 673 (2006)

The Fraternal
Ateneo de Davao

TAU MU
TAU MU

request is still undergoing evaluation. The court


cannot write a provision in the treaty giving
private respondent that right where there is none.
An equally compelling factor is that other
countries with similar extradition treaties with the
Philippines
have
expressed
the
same
interpretation adopted by the Philippines and US
governments. It is not an international practice
to aford a potential extraditee with a copy of the
extradition papers during the evaluation stage.
An extradition proceeding is not a criminal
proceeding to determine his guilt or innocence
which will call into operation all the rights of an
accused under the Bill of Rights. Besides, under
the Treaty, he is aforded opportunity to meet the
evidence against him once the petition is filed in
court.

KITY

TAU MU TAU MU TAU MU


TAU MU TAU MU TAU MU

CONSTITUTIONAL LAW II
Order of Saint Thomas More
Atty. Philip John Pojas/Atty. Rovyne G. Jumao-as, RN
University College of Law

February 17, 1985 and got his retirement benefits


fro the GSIS. When he died seven years later,
GSIS denied the claim for survivorship pension of
Milagros because under PD 1146 the surviving
spouse is not qualified if the marriage with the
pensioner was contracted within 3 years before
the pensioner qualified for the pension. Does the
law violate the equal protection clause?
Held: Yes. A statue based on reasonable
classification does not violate the equal
protection clause provided it satisfies the
following: 1. It must rest on a substantial
distinction 2. It must be germane to the purpose
of the law 3. It must not be limited to existing
conditions only 4. It must apply equally to all
embers of the same class
The above provision discriminates against the
dependent spouse who contracts marriage within
3 years befre the petitioner qualified for the
pension. Even if the dependent souse married the
pensioner more than three years before the
pensioners death, the dependent spouse would
not receive the pension if the marriage took place
within three years before the pensioner qualified
for the pension. If the purpose of the prohibition
is to prevent deathbed marriages, there is no
reason why the 3-year prohibition is reckoned
from the date the pensioner qualified for the
pension and not from the date he died. The
classification does not rest on substantial
distinction. It lumps all those marriages
contracted within three years before the
pensioner qualified for the pension as having
contracted within three years before the
pensioner qualified for the pension as having
been
contracted
primarily
for
financial
convenience.
CHAVEZ vs. COMELEC
437 SCRA 415 (2004)
Comelec resolution n. 6520, sec. 32, states:
All propaganda materials such as posters,
streamers, stickers or paintings on walls and
other materials showing the picture, image or
name of a person, and all advertisements on
print, in radio or on television showing the image
or mentioning the name of a person, who
subsequent to the placement or display thereof
becomes a candidate for public office shall be
immediately removed by said candidate and
radio station, print media or television within 3
days after the afectivity of these implementing
rules.
Issue: is the resolution a valid exercise of police
power?
Held: Yes. To determine the validity of a police
power measure, two questions must be asked: 1.
Does the interest of the public in general, as
distinguished fron those of a particular class,
require the exercise of police power? 2. Are the
means employed reasonably necessary for the
accomplishment of the purpose and not unduly
oppressive upon individuals?
A close examination of the assailed provision
reveals that its primary objectives are to prohibit
premature campaigning and to level the playing

ACADCOM 2010; Contributors: Gene Geocaniga, Jarissa Guiani, Darlene Magabilen


TAU MU Page 30 of 179

field for candidates of public office, to equalize


the situation of rich candidates and poorer
candidates by preventing the former from
enjoying undue advantage in exposure and
publicity. The latter is valid reason for the
exercise of police power.
CHAVEZ vs. ROMULO
431 SCRA 430 (2004)

TAU MU
TAU MU
TAU MU
TAU MU
TAU MU
TAU MU

Facts: The law concerns the phasing out of


dilapidated taxicabs. The lawful subject here is
public welfare. The lawful method is to phase out
old taxis to ensure public safety. Taxicab owners
and operators challenged this by saying that it
afects their property rights.
Held: The court held that the law is not unduly
oppressive, therefore valid. Six years cut-of time
is sufficient for the owners to have substantial
return of their investment.

TAU MU

TAXICAB VS BOT

TAU MU

Held: The ordinance regulating motels is not a


violation of due process as this consists merely a
manifestation of police power measure to
safeguard public morals. Similarly, in another
case (Ericta vs. City), the prohibition to check in
for more than 24 hours in a motel is valid
exercise of police power, the purpose being to
promote good morals.

TAU MU
TAU MU

ERMITA vs. CITY MAYOR


20 SCRA 849 (1967)

The Fraternal
Ateneo de Davao

TAU MU
TAU MU

Facts: On January 31, 2003, the Chief of the


Philippine National Police issued the Guidelines in
the Implementation of the Ban on the Carrying of
Firearms Outside of Residence (PTCFOR). Among
others, the guidelines suspended the issuance of
permits to carry firearms outside of ones
residence. Does the Constitution protect the right
to bear arms? Does the ban on carrying firearms
constitute deprivation of property without due
process?
Held: The right to bear arms is a mere statutory
privilege, not a constitutional right. The Philippine
Constitution contains no provision similar to the
Second Amendment. Similarly, a license to permit
authorizing a person to enjoy certain privileges is
neither a property nor a property right. Just like
ordinary licenses in other regulatory fields, a
license to carry firearm may be revoked anytime.
It does not confer an absolute right, but only
personal privilege to be exercised under existing
restrictions, ad such as may thereafter be
reasonably imposed. Even if it is assumed that a
PTCFOR constitutes a property right protected by
the Constitution, the same cannot be considered
as absolute to be placed beyond the reach of the
States police power. All property in the State is
held subject to its general regulations, necessary
to the common good and general welfare.

KITY

TAU MU TAU MU TAU MU


TAU MU TAU MU TAU MU

CONSTITUTIONAL LAW II
Order of Saint Thomas More
Atty. Philip John Pojas/Atty. Rovyne G. Jumao-as, RN
University College of Law

TOMAS VELASCO vs. HON. ANTONIO J.


VILLEGAS
Facts: This is an appeal from an order of the
lower court dismissing a suit for declaratory relief
challenging the constitutionality based on
Ordinance No. 4964 of the City of Manila, the
contention being that it amounts to a deprivation
of property of petitioners-appellants of their
means of livelihood without due process of law.
The assailed ordinance is worded thus: "It shall be
prohibited for any operator of any barber shop to
conduct the business of massaging customers or
other persons in any adjacent room or rooms of
said barber shop, or in any room or rooms within
the same building where the barber shop is
located as long as the operator of the barber
shop and the room where massaging is
conducted is the same person." The issue in the
case is whether or not the ordinance is
unconstitutional.
Held: Even if such were not the case, the attack
against the validity cannot succeed. As pointed
out in the brief of respondents-appellees, it is a
police power measure. The objectives behind its
enactment are: "(1) To be able to impose
payment of the license fee for engaging in the
business of massage clinic under Ordinance No.
3659 as amended by Ordinance 4767, an entirely
diferent measure than the ordinance regulating
the business of barbershops and, (2) in order to
forestall possible immorality which might grow
out of the construction of separate rooms for
massage of customers." 3 This Court has been
most liberal in sustaining ordinances based on
the general welfare clause. As far back as U.S. v.
Salaveria, 4 a 1918 decision, this Court through
Justice Malcolm made clear the significance and
scope of such a clause, which "delegates in
statutory form the police power to a municipality.
As above stated, this clause has been given wide
application by municipal authorities and has in its
relation to the particular circumstances of the
case been liberally construed by the courts. Such,
it is well to really is the progressive view of
Philippine jurisprudence." 5 As it was then, so it
has continued to be. 6 There is no showing,
therefore, of the unconstitutionality of such
ordinance. WHEREFORE, the appealed order of
the lower court is affirmed. No costs.
CITY OF MANILA vs. LAGUIO, JR.
455 SCRA 308 (2005)

TAU MU

Facts: On March 30, 1993, the City of Manila


approved an ordinance prohibiting and person
from operating in the Ermita-Malate area any
business providing certain forms of amusement,
entertainment, services and facilities where
women are used as tools in entertainment,
including sauna parlors, beerhouses, discoteques,
cabarets, dance halls, motels and inns. Those
already engaged in said business were given
three months to wind up their operations or
transfer elsewhere.
Issue: Is the ordinance a valid exercise of police
power?

ACADCOM 2010; Contributors: Gene Geocaniga, Jarissa Guiani, Darlene Magabilen


TAU MU Page 31 of 179

TAU MU
TAU MU
TAU MU
TAU MU
TAU MU
TAU MU

Facts: This concerns the Generic Act which was


quite an issue in the medical world. The Act
requires doctors to prescribe medicine using
generic names and not to use brand names,
which was the common practice before the
passage of the law. Doctors contended that the
law burdened their freedom to contract. The
lawful subject is health. The Supreme Court also
cited some constitutional policies wherein the
State will promote health and make it afordable
to the people.
Held: The court held that it is too hard to
challenge the presumption of validity. And since it
has constitutional basis, the more it would be
difficult to challenge. With regards the doctors

TAU MU

DEL ROSARIO vs. BENGZON


180 SCRA 521 (1989)

TAU MU

Held: The court held that the fee paid to enter


movie houses represents property rights. The
theater owners should have the right to sell their
tickets in whatever way they want to.
Interference on the right to sell tickets was not
validated. As to #2 and #3, the court ruled them
to be valid because they afect public safety
which fell within the scope of police power. What
is being avoided is the overcrowding of movie
houses. They comply with Substantive Due
Process.

TAU MU

Facts: The ordinance: 1) reduced by the fee


for children aged 7-12 years old to enter into a
movie house; 2) prohibited theater owners from
selling tickets beyond the seating capacity; 3)
prohibited owners from selling a ticket to two
persons. The lawful subject used as basis is
general welfare. What is burdened is the property
right of the theater owners.

TAU MU
TAU MU

BALACUIT vs. CFI


163 SCRA 182 (1988)

The Fraternal
Ateneo de Davao

TAU MU
TAU MU

Held: The promotion of morals is undoubtedly


one of the fundamental duties of the City of
Manila. However, the worthy aim of fostering
public morals and the eradication of the
communitys social ills can be achieved through
means less restrictive of private rights. It can be
attained by reasonable restrictions rather than by
absolute prohibition. The closing down and
transfer of businesses or their conversion into
businesses allowed under the ordinance have
no reasonable relation to the accomplishment of
its purposes. Otherwise stated, the prohibition of
the enumerated establishments will not per se
protect and promote the social and moral welfare
of the community; it will not in itself eradicate the
alluded social ills of prostitution, adultery,
fornication nor will it arrest the spread of sexual
disease
in
Manila.
The
enumerated
establishments are lawful pursuits which are not
per se ofensive to the moral welfare of the
community. The City Council instead should
regulate human conduct that occurs inside the
establishments.

KITY

TAU MU TAU MU TAU MU


TAU MU TAU MU TAU MU

CONSTITUTIONAL LAW II
Order of Saint Thomas More
Atty. Philip John Pojas/Atty. Rovyne G. Jumao-as, RN
University College of Law

argument, the court said that there is no contract


between the doctor and the patient. In fact, the
patient can refuse to buy the medicine prescribed
by his doctor. Therefore the freedom of contract
is not involved here. In fact, there seems to be no
case wherein the doctors sued his patient for not
following his prescription.
DEPARTMENT vs. SAN DIEGO
180 SCRA 533 (1989)
Facts: San Diego challenged the NMAT rule that
one cannot take a medical course after failing the
test three times. The lawful subject here is health.
The rational relation is that the medical
profession is intimately related to life and health
and is impressed with public interest. Restriction
of the medical profession is related to health
(because doctors promote health). What is being
burdened here according to San Diego is the right
to education.
Held: The court held that the right to education
is not unduly burdened because if a person fails
the NMAT three times, it seems that he is not fit
to be in the medical profession at all.

YNOT VS IAC
Facts: Pres. Marcos promulgated an EO which
prohibited the inter-provincial movement of
carabaos and carabeefs. Anyone caught violating
the law will have his carabao and carabeef
confiscated. The purpose of the law is to prevent
indiscriminate slaughter of carabaos so that they
could be preserved for agricultural use by the
farmers. The lawful subject is general welfare
since the Philippines is an agricultural country.
The farmers need them for farming instead of
using machineries. (The country was in an energy
crisis)
Held: The court held that there was a lawful
subject which is general welfare. However, the
method chosen by the government has no logical
connection with the purpose of the law.
Prohibiting the inter-provincial transfer of
carabaos would not prevent their indiscriminate
slaughter. The carabaos can still be killed
anywhere without even transferring them.
CORONA VS. UNITED HARBOR PILOTS
ASSOCIATION OF THE PHIL
Facts: Respondents were holders of a pilot
license. An order was issued by Dayan, general
manager of the Phil Ports Authority, which
provided that license previously issued is
efective only until Dec 31, 1992, subject to a
yearly renewal after a rigid evaluation of
performance. This was questioned by the pilots
because it deprives them of their property
without due process.
Held:
On procedural due process no violation

ACADCOM 2010; Contributors: Gene Geocaniga, Jarissa Guiani, Darlene Magabilen


TAU MU Page 32 of 179

Notice and hearing are essential only


when an administrative body exercises
quasi-judicial function not when it
exercises quasi-legislative function such
as the issuance of rules and regulation.

On substantive due process there is violation


The fact that renewal of licenses are
subject to a rigid evaluation violates right
to property because it would mean that
your license may not be renewed if you do
not pass the evaluation.

TAU MU
TAU MU
TAU MU
TAU MU
TAU MU
TAU MU
TAU MU

Held: The scope of the function of MMDA as an


administrative, coordinating and policy-setting
body has been settled in Metropolitan Manila
Development Authority (MMDA) v. Bel-Air Village
Association, Inc.41 In that case, the Court stressed
clearly, the scope of the MMDAs function is
limited to the delivery of the seven (7) basic
services. One of these is transport and traffic
management which includes the formulation and
monitoring of policies, standards and projects to
rationalize the existing transport operations,
infrastructure
requirements,
the
use
of
thoroughfares and promotion of the safe
movement of persons and goods. It also covers
the mass transport system and the institution of a
system of road regulation, the administration of
all
traffic
enforcement
operations,
traffic
engineering services and traffic education
programs, including the institution of a single
ticketing system in Metro Manila for traffic
violations. Under this service, the MMDA is
expressly authorized to "to set the policies
concerning traffic" and "coordinate and regulate

TAU MU

Issues: the issues were narrowed down to


whether 1) the MMDAs power to regulate traffic
in Metro Manila included the power to direct
provincial bus operators to abandon and close
their duly established and existing bus terminals
in order to conduct business in a common
terminal; (2) the E.O. is consistent with the Public
Service Act and the Constitution.

TAU MU

The first assailed Order of September 8,


2005,2
which
resolved
a
motion
for
reconsideration filed by herein respondents,
declared Executive Order (E.O.) No. 179,
hereafter referred to as the E.O., "unconstitutional
as it constitutes an unreasonable exercise of
police power." The second assailed Order of
November 23, 20053 denied petitioners motion
for reconsideration.

TAU MU
TAU MU

The present petition for review on


certiorari, rooted in the traffic congestion
problem, questions the authority of the
Metropolitan Manila Development Authority
(MMDA) to order the closure of provincial bus
terminals along Epifanio de los Santos Avenue
(EDSA) and major thoroughfares of Metro Manila.

The Fraternal
Ateneo de Davao

TAU MU
TAU MU

THE METROPOLITAN MANILA DEVELOPMENT


AUTHORITY VS. VIRON TRANSPORTATION
CO., Inc.

KITY

TAU MU TAU MU TAU MU


TAU MU TAU MU TAU MU

CONSTITUTIONAL LAW II
Order of Saint Thomas More
Atty. Philip John Pojas/Atty. Rovyne G. Jumao-as, RN
University College of Law

the implementation of all traffic management


programs." In addition, the MMDA may install and
administer a single ticketing system," fix, impose
and collect fines and penalties for all traffic
violations. Even then, for reasons which bear
reiteration, the MMDA cannot order the closure of
respondents terminals not only because no
authority to implement the Project has been
granted nor legislative or police power been
delegated to it, but also because the elimination
of the terminals does not satisfy the standards of
a valid police power measure. No matter how
noble the intentions of the MMDA may be then,
any plan, strategy or project which it is not
authorized to implement cannot pass muster
In the second issue the court ruled that
the establishment, as well as the maintenance of
vehicle parking areas or passenger terminals, is
generally considered a necessary service to be
provided by provincial bus operators like
respondents, hence, the investments they have
poured into the acquisition or lease of suitable
terminal sites. Eliminating the terminals would
thus run counter to the provisions of the Public
Service Act. Finally, an order for the closure of
respondents terminals is not in line with the
provisions of the Public Service Act.
Romeo Gerochi vs. Department of Energy
Facts: Petitioners Romeo P. Gerochi, Katulong Ng
Bayan (KB), and Environmentalist Consumers
Network, Inc. (ECN) (petitioners), come before
this Court in this original action praying that
Section 34 of Republic Act (RA) 9136, otherwise
known as the Electric Power Industry Reform Act
of 2001 (EPIRA), imposing the Universal Charge,
and Rule 18 of the Rules and Regulations (IRR)
which seeks to implement the said imposition, be
declared unconstitutional. Petitioners also pray
that the Universal Charge imposed upon the
consumers be refunded and that a preliminary
injunction and/or temporary restraining order
(TRO) be issued directing the respondents to
refrain from implementing, charging, and
collecting the said charge. The universal charge
shall be a non-bypassable charge which shall be
passed on and collected from all end-users on a
monthly basis by the distribution utilities.
Issues : Whether or not, the Universal
Charge imposed under Sec. 34 of the
EPIRA is a tax; and Whether or not there is
undue delegation of legislative power to
tax on the part of the ERC.
Ruling:
Evidently, the establishment and
maintenance of the Special Trust Fund, under the
last paragraph of Section 34, R.A. No. 9136, is
well within the pervasive and non-waivable power
and responsibility of the government to secure
the physical and economic survival and wellbeing of the community, that comprehensive
sovereign authority we designate as the police
power of the State. This feature of the Universal
Charge further boosts the position that the same

ACADCOM 2010; Contributors: Gene Geocaniga, Jarissa Guiani, Darlene Magabilen


TAU MU Page 33 of 179

TAU MU
TAU MU
TAU MU
TAU MU
TAU MU
TAU MU
TAU MU
TAU MU

There is substantial distinction between


aliens and Filipinos since aliens loyalty to the
country is transitory in nature which is motivated
only by some kind of personal interest.
The purpose of the law is to prevent aliens
from engaging in business in the Philippines. It is
observed that there is alien dominance in retail
trade. That is the reason why the law intends to
give it back to the Filipinos.
It also applies to all members of the class,
except to Americans, since they are covered by
the Parity Amendment found in the 1935
Constitution. This provision authorizes Americans

TAU MU

ICHIONG VS. HERNANDEZ

TAU MU
TAU MU

EQUAL PROTECTION

The Fraternal
Ateneo de Davao

TAU MU
TAU MU

is an exaction imposed primarily in pursuit of the


State's police objectives. The STF reasonably
serves and assures the attainment and perpetuity
of the purposes for which the Universal Charge is
imposed, i.e., to ensure the viability of the
country's electric power industry. Ergo, Universal
Charge is considered a tax.
The second issue governs the principle of
separation of powers ordains that each of the
three branches of government has exclusive
cognizance of and is supreme in matters falling
within its own constitutionally allocated sphere. A
logical corollary to the doctrine of separation of
powers is the principle of non-delegation of
powers In the face of the increasing complexity of
modern life, delegation of legislative power to
various specialized administrative agencies is
allowed as an exception to this principle.
All that is required for the valid exercise of
this power of subordinate legislation is that the
regulation be germane to the objects and
purposes of the law and that the regulation be
not in contradiction to, but in conformity with, the
standards prescribed by the law. These
requirements
are
denominated
as
the
completeness test and
the sufficient
standard test. Under the first test, the law must
be complete in all its terms and conditions when
it leaves the legislature such that when it reaches
the delegate, the only thing he will have to do is
to enforce it. The second test mandates adequate
guidelines or limitations in the law to determine
the boundaries of the delegate's authority and
prevent the delegation from running riot. The
Court finds that the EPIRA, read and appreciated
in its entirety, in relation to Sec. 34 thereof, is
complete in all its essential terms and conditions,
and that it contains sufficient standards.
Finally, every law has in its favor the
presumption of constitutionality, and to justify its
nullification, there must be a clear and
unequivocal breach of the Constitution and not
one
that
is
doubtful,
speculative,
or
argumentative. Indubitably, petitioners failed to
overcome this presumption in favor of the EPIRA.
We find no clear violation of the Constitution
which would warrant a pronouncement that Sec.
34 of the EPIRA and Rule 18 of its IRR are
unconstitutional and void.

KITY

TAU MU TAU MU TAU MU


TAU MU TAU MU TAU MU

CONSTITUTIONAL LAW II
Order of Saint Thomas More
Atty. Philip John Pojas/Atty. Rovyne G. Jumao-as, RN
University College of Law

to engage in all trade in the country. The


classification also has relevance to the lawful
subject and method.
PASEI VS DRILON
What was questioned was the order by the
DOLE which prohibited temporarily the sending of
female domestic abroad. The law was challenged
as violative of the equal protection clause since it
singled out domestic helpers who are female only.
Held: The SC ruled that there is valid
classification since it is the females who
experience the various abuses abroad. The
female domestic workers are those who live with
their employers, unlike entertainers, and it is
their houses where most abuses occur. They are
very susceptible to maltreatment. The ban is
germane to the purpose of the law which is to
protect them. The law applies to all members of
the class. It will apply as long as there are female
domestic workers abroad.
NUEZ VS SANDIGANBAYAN
Nuez claimed that the PD minimizes the
constitutional rights of the public officers. What
the law imposes is that public officials are to be
tried by the Sandiganbayan for cases concerning
their position or office. If an appeal is made by
the official, it goes straight to the SC unlike
ordinary citizens, whose cases can still be
appealed in the CA. Nuez claimed that the law is
discriminatory since public officials have only one
chance to appeal (certiorari), while others have
more remedies. It should be noted that the SC
only deals with appeals with questions of law.
Held: The court held that the law did not violate
the equal protection clause. There is a substantial
distinction since public officers occupy a position
diferent from others: Public office is a public
trust. It is also germane to the purpose of the law.
It can also apply inefinitely and it applies to all
members of the class.
ORMOC SUGAR COMPANY VS TREASURER
An ordinance was passed imposing tax
specifically on sugar produced by Ormoc Sugar
Co.
It violated the equal protection clause
because there is a possibility that somebody
might put up a new sugar company. In efect, the
law will still apply singly to Ormoc Sugar
Company because it was specifically sated so.
Held: The ordinance is a violation of the equal
protection clause. The law was limited to existing
conditions only. Although Ormoc Sugar was the
only sugar miller at that time, the ordinance
should also cover other future sugar millers. The
ordinance should have stated that all sugar mills
in the municipality should be taxed so that
anybody who comes in and starts the same

ACADCOM 2010; Contributors: Gene Geocaniga, Jarissa Guiani, Darlene Magabilen


TAU MU Page 34 of 179

business can now be covered by the ordinance.


There was no valid classification.
The court held that the ordinance violated
the equal protection clause. It did not satisfy
requirement #3. The law was only limited to
existing conditions. Although Ormoc Sugar was
the only sugar miller at the time, the ordinance
should also cover for other future sugar millers.
Presently, the law does not cover other future
millers.
VILLEGAS VS. HIU CHIONG

TAU MU
TAU MU
TAU MU
TAU MU
TAU MU

Petitioner is a policeman charged with


murder and attempted murder. Upon acceptance
of the case, the trial court suspended him until
the case is terminated by virtue of the provision
of RA 6975 sec. 47 (LGC). If you are a policeman

TAU MU

HIMAGAN VS. PEOPLE

TAU MU

Held: The court ruled in favor of Chua, the law


violates the equal protection clause. However,
the court did not declare the law null and void for
being unconstitutional. Usually, it will. The court
merely explained the law in another manner, it
injected contractual workers as included in the
law. There is no substantial distinction between a
co-terminous employee and a casual employee
when in fact the former should enjoy better
treatment than the casuals since they are in
plantilla.

TAU MU

In 1988, in order to bring about the


reorganization in the government, an early
retirement law was enacted. The law covered
regular, temporary casual and emergency
employees who have rendered at least 2 years of
consecutive service. Chua, who was hired and
rehired 4 times, has worked with the government
continuously for 15 years as a contractual
employee. And she applied for early retirement.
She was refused the benefits of the law because
under existing civil service law she was classified
as a contractual, and her tenure is co-terminous
with the project. She was not considered as
included in the law. She contested this being
violative of the equal protection clause having no
substantial distinction among casual temporary,
emergency and contractual workers.

TAU MU

CHUA VS. CIVIL SERVICE COMMISSION

TAU MU
TAU MU

Held: The court held that it violated the equal


protection clause because it did not indicate as to
the particular class of aliens who are required to
pay for working permits. The ordinance was
applicable to all, whether the alien is a lowly
worker or a highly paid executive or a rich
businessman.

The Fraternal
Ateneo de Davao

TAU MU
TAU MU

An ordinance was passed imposing a


P50.00 working fee for permits to aliens who wish
to work in Manila. This was a revenue raising
program of the City of Manila.

KITY

TAU MU TAU MU TAU MU


TAU MU TAU MU TAU MU

CONSTITUTIONAL LAW II
Order of Saint Thomas More
Atty. Philip John Pojas/Atty. Rovyne G. Jumao-as, RN
University College of Law

charged with an ofense whose penalty is 6yrs


and 1 day or more, you will be suspended from
the service until the termination of the case.
According to the petitioner, the 90-day
suspension under the Civil Service Law should
apply because allegedly RA 6975 unduly
prejudices members of the PNP described above.
Held:
Petitioners allegation is misplaced
because the Civil Service Law applies to
administrative cases. Sec 47 of LGC, a later
enactment, repealed this part of the CSL.
Besides on the aspect of equal protection, there
is a valid distinction between members of the PNP
and other accused because PNP members
charged with serious ofenses might influence the
victims and witnesses.
Police officers carry
weapon and the badge of the law which can be
used to harass witnesses. Members of the PNP
are diferent from other accused by reason of the
position they occupy.
ALMONTE V. VASQUEZ
Petitioners allege that the procedure which
allows the Office of the Ombudsman to
investigate anonymous complaints violates the
equal protection clause. They point out that in all
forums and tribunals, aggrieved parties can only
hale respondents via verified complaints or sworn
statements with their identities fully disclosed.
Held:
There can be no objection to the
procedure because the Constitution itself
authorizes it. Art. XI, Sec. 12. of the Constitution
requires the Ombudsman and his deputies to act
promptly on complaints filed in any form or
manner against public officials or government
employees.
In addition, the Office of the
Ombudsman is diferent from other agencies
because those subject to its jurisdiction are public
officials, who, through official pressure and
influence,
can
quash,
delay
or
dismiss
investigations held against them.
TIU V. COURT OF APPEALS
Pursuant to R.A. No. 7227, the President
issued A.O. No. 97-A grading tax and duty
incentives only to businesses and residents within
the secured area of the Subic Special Economic
Zone. Does the administrative order violate the
equal protection clause?
Held:
No.
The classification rests on a
substantial distinction which is germane to the
purpose law. R.A. No. 7227 aims primarily to
accelerate the conversion of military reservations
into productive uses.
Obviously, the lands
covered
under
the
1947
Military
Base
Agreement are its object. It is this specific area
which the government intends to transform and
develop from its status quo ante as an
abandoned naval facility into a self-sustaining
industrial and commercial zone, particularly for
big foreign and local investors to use as

ACADCOM 2010; Contributors: Gene Geocaniga, Jarissa Guiani, Darlene Magabilen


TAU MU Page 35 of 179

operational bases. The bias for big investors may


be justified because they are the ones who can
pour huge investments to spur economic growth.
INTERNATIONAL V. QUISUMBING

TAU MU
TAU MU

LACSON V. EXECUTIVE
Chief Superintendent Lacson of the
Philippine National Police was charged as an
accessory in a multiple murder case filed before
the Sandiganbayan. Under R.A No. 7975, said
court has jurisdiction where one of the principal
accused has the rank of Chief Superintendent or
higher. Since Lacson was charged as a mere
accessory, he moved to quash the information for
lack of jurisdiction.
While the motion was
pending, the Congress enacted R.A. No. 8249
which deleted the word principal.
The
amendment was made to apply to all pending
cases over which trial had not begun. Is the law
discriminatory?
Held:
No.
the classification between those
pending cases involving public officials and those
whose trials have not commended as of the
approval of the law rests on substantial
distinction that makes real diferences. In the
first instance, evidence against them were not
yet presented, whereas in the latter parties had
already submitted their respective proofs. Thus,
accuse cannot claim that the law placed them
under a diferent category form those similarly
situated as they are.
DE GUZMAN V. COMELEC
Sec 44 of RA no. 8189, otherwise known
as The Voters Registration Act states:
No Election Officer shall hold office in a
particular city or municipality for more than 4
years. Any election officer who, either at the time
of the approval of this Act or subsequent thereto,
has served for at least 4 years in a particular city
or municipality shall automatically be reassigned
by the Commission to a new station outside the
original congressional district.
Does it violate the equal protection clause?

TAU MU
TAU MU
TAU MU
TAU MU
TAU MU

Held: never has the call of a particular duty lifted


a prisoner into a diferent classification from
those others who are validity restrained by law.
The performance of a legitimate and even
essential duties by public officers has never been
an excuse to free a person in prison. After all ,
accused is only one of 250 members of the House
of Representatives, which can continue to
function without him. Election to the position of
Congressman is not a reasonable classification in
criminal law enforcement. The functions and
duties of the office are not substantial distinctions
which remove him from the class of prisoners
restricted in their liberty of movement.

TAU MU

Accused is a full- fledged member of


Congress who is now confined at the national
penitentiary while his convictions for statutory
rape on two counts acts of lasciviousness are
pending appeal. Accused filed a motion asking
that he be allowed to fully discharge the duties of
a
Congressman,
including
attendance
at
legislative sessions and committee meetings
despite his having been convicted of a nonbailable ofense. Does being an elective official
result in a substantial distinction that allows a
diferent treatment?

TAU MU

PEOPLE V. JALOSLOS

TAU MU
TAU MU

Held: There is no reasonable distinction between


the services rendered by foreign-hires and local
hires. Persons who work with substantially equal
qualifications, efort and responsibility, under
similar conditions, should be paid similar salaries.
If an employer accords employees the same
position and rank, the presumption is that these
employees perform equal work.
There is no
evidence that foreign-hires perform 25% more
efficiently than local hires. Both groups have
similar functions and responsibilities, which they
perform under similar working conditions. The
dislocation factor and their limited tenure
cannot serve as valid basis for the distinction in
salary rate. They are adequately compensated
by
certain
benefits
such
as
housing,
transportation, taxes and home leave travel
allowance.

The Fraternal
Ateneo de Davao

TAU MU
TAU MU

International School is a domestic


educational
institution
established
for
dependents of foreign diplomatic personnel. As
much, it is authorized to employ aliens and these
personnel are exempt from employment laws.
Foreign-hires receive benefits not accorded to
local hires including housing, transportation,
taxes as well as a 25% higher salary rates to ofset the dislocation factor. The labor union is
now challenging the salary rate diference on the
ground that it is discriminatory.

KITY

TAU MU TAU MU TAU MU


TAU MU TAU MU TAU MU

CONSTITUTIONAL LAW II
Order of Saint Thomas More
Atty. Philip John Pojas/Atty. Rovyne G. Jumao-as, RN
University College of Law

Held: The singling out of election officers in order


to ensure the impartiality of election officials by
preventing them form developing familiarity with
the people of their place of assignment does not
violate the equal protection clause. It can be
discerned that the legislature thought the noble
purpose of the law would be served by breaking
an important link in the chain of corruption than
by breaking up each and every link thereof.
Election officers are the highest officials or
authorized representatives of the COMELEC in the
city or municipality. It is safe to say that without
the complicity of such officials, large scale
anomalies in the registration of voters can hardly
be carried out.
ELENA P. DYCAICO vs SOCIAL SECURITY
SYSTEM

ACADCOM 2010; Contributors: Gene Geocaniga, Jarissa Guiani, Darlene Magabilen


TAU MU Page 36 of 179

TAU MU
TAU MU
TAU MU
TAU MU
TAU MU
TAU MU
TAU MU
TAU MU
TAU MU

Ruling: The classification of dependent spouses


on the basis of whether their respective
marriages to the SSS member were contracted
prior to or after the latters retirement for the
purpose of entitlement to survivors pension does
not rest on real and substantial distinctions. It is
arbitrary and discriminatory. It is too sweeping
because the proviso as of the date of his
retirement, which efectively disqualifies the
dependent spouses whose respective marriages
to the retired SSS member were contracted after
the latters retirement as primary beneficiaries,
unfairly lumps all these marriages as sham
relationships or were contracted solely for the
purpose of acquiring benefits accruing upon the
death of the other spouse. The proviso thus
unduly prejudices the rights of the legal surviving
spouse, like the petitioner, and defeats the
avowed policy of the law to provide meaningful
protection to members and their beneficiaries
against the hazards of disability, sickness,
maternity,
old
age,
death,
and
other
contingencies resulting in loss of income or
financial burden. In the petitioners case, for
example, she asserted that when she and
Bonifacio got married in 1997, it was merely to
legalize their relationship and not to commit

TAU MU
TAU MU

Issue: Whether or not Rep. Act 8282 is violative


of equal protection and due process claus and the
petitioner qualified to the benefits.

The Fraternal
Ateneo de Davao

TAU MU
TAU MU

Facts: Bonifacio S. Dycaico became a member


of the SSS on January 24, 1980. In his selfemployed data record (SSS Form RS-1), he named
the petitioner, Elena P. Dycaico, and their eight
children as his beneficiaries. At that time,
Bonifacio and Elena lived together as husband
and wife without the benefit of marriage. In June
1989, Bonifacio was considered retired and began
receiving his monthly pension from the SSS. He
continued to receive the monthly pension until he
passed away on June 19, 1997. A few months
prior to his death, however, Bonifacio married the
petitioner on January 6, 1997.
Shortly after Bonifacios death, the
petitioner filed with the SSS an application for
survivors pension. Her application, however,
was denied on the ground that under Section 12B(d) of Republic Act (Rep. Act) No. 8282 or the
Social Security Law she could not be considered a
primary beneficiary of Bonifacio as of the date of
his retirement. The said proviso reads:
Sec. 12-B. Retirement Benefits.
(d) Upon the death of the retired member,
his primary beneficiaries as of the date of his
retirement shall be entitled to receive the
monthly pension.
Applying this proviso, the petitioner was
informed that the records show that the member
[referring to Bonifacio] was considered retired on
June 5, 1989 and monthly pension was cancelled
upon our receipt of a report on his death on June
19, 1997. In your death claim application,
submitted marriage contract with the deceased
member shows that you were married in 1997 or
after his retirement date; hence, you could not be
considered his primary beneficiary.

KITY

TAU MU TAU MU TAU MU


TAU MU TAU MU TAU MU

CONSTITUTIONAL LAW II
Order of Saint Thomas More
Atty. Philip John Pojas/Atty. Rovyne G. Jumao-as, RN
University College of Law

fraud. This claim is quite believable. After all,


they had been living together since 1980 and, in
fact, during that time their eldest child was
already twenty-four (24) years old. However, the
petitioner was not given any opportunity to prove
her claim that she was Bonifacios bona fide legal
spouse as she was automatically disqualified from
being considered as his primary beneficiary. In
efect, the petitioner was deprived of the
survivors benefits, a property interest, accruing
from the death of Bonifacio without any
opportunity to be heard.
Finally, the Court concedes that the
petitioner did not raise the issue of the validity of
the proviso as of the date of his retirement in
Section 12-B(d) of Rep. Act No. 8282. The rule is
that the Court does not decide questions of a
constitutional nature unless absolutely necessary
to a decision of the case.[29] However, the
question of the constitutionality of the proviso is
absolutely necessary for the proper resolution of
the present case. Accordingly, the Court required
the parties to present their arguments on this
issue and proceeded to pass upon the same in
the exercise of its equity jurisdiction and in order
to render substantial justice to the petitioner who,
presumably in her advanced age by now,
deserves to receive forthwith the survivors
pension accruing upon the death of her husband.
JAMES MIRASOL vs DEPARTMENT OF PUBLIC
WORKS AND HIGHWAYS
The facts:
The petition sought the declaration of
nullity of the following administrative issuances
for being inconsistent with the provisions of
Republic Act 2000, entitled Limited Access
Highway Act enacted in 1957:
1.
Previously, pursuant to its mandate under
R.A. 2000, DPWH issued on June 25, 1998
Department Order (DO) No. 215 declaring the
Manila-Cavite (Coastal Road) Toll Expressway as
limited access facilities.
2. Accordingly, petitioners filed an Amended
Petition on February 8, 2001 wherein petitioners
sought the declaration of nullity of the aforesaid
administrative issuances. Moreover, petitioners
prayed for the issuance of a temporary
restraining order and/or preliminary injunction to
prevent the enforcement of the total ban on
motorcycles along the entire breadth of North and
South Luzon Expressways and the Manila-Cavite
(Coastal Road) Toll Expressway under DO 215.
3. And AO1 which limited access highways, AO1
States that it is unlawful for any person or group
of persons to: Drive any bicycle, tricycle, pedicab,
motorcycle or any vehicle (not motorized);
4. On July 18, 2001, the DPWH acting thru the
TRB, issued Department Order No. 123 allowing
motorcycles with engine displacement of 400
cubic centimeters inside limited access facilities
(toll ways).

ACADCOM 2010; Contributors: Gene Geocaniga, Jarissa Guiani, Darlene Magabilen


TAU MU Page 37 of 179

Issues: WHETHER DO 74 DO 215 AND


THE TRB REGULATIONS CONTRAVENE RA
2000; AND
WHETHER AO 1 AND DO 123 ARE
UNCONSTITUTIONAL.
Ruling:
First Issue

The Fraternal
Ateneo de Davao

TAU MU
TAU MU
TAU MU
TAU MU
TAU MU
TAU MU
TAU MU
TAU MU
TAU MU
TAU MU
TAU MU
TAU MU
TAU MU

DPWH issued DO 74 and DO 215 declaring


certain expressways as limited access facilities on
5 April 1993 and 25 June 1998, respectively.
Later, the TRB, under the DPWH, issued the
Revised Rules and Regulations on Limited Access
Facilities. However, on 23 July 1979, long before
these department orders and regulations were
issued,
the
Ministry
of
Public
Works,
Transportation and Communications was divided
into two agencies the Ministry of Public Works
and
the
Ministry
of
Transportation
and
Communications by virtue of EO 546. The
question is, which of these two agencies is now
authorized to regulate, restrict, or prohibit access
to limited access facilities.
Even under Executive Order No. 125 (EO
125) and Executive Order No. 125-A (EO 125-A),
which further reorganized the DOTC, the authority
to administer and enforce all laws, rules and
regulations relative to transportation is clearly
with the DOTC.
Thus, DO 74 and DO 215 are void because
the DPWH has no authority to declare certain
expressways as limited access facilities. Under
the law, it is the DOTC which is authorized to
administer and enforce all laws, rules and
regulations in the field of transportation and to
regulate related activities.
Since the DPWH has no authority to
regulate activities relative to transportation, the
TRB cannot derive its power from the DPWH to
issue
regulations governing limited access
facilities. The DPWH cannot delegate a power or
function which it does not possess in the first
place. Since DO 74 and DO 215 are void, it
follows that the rules implementing them are
likewise void.
On the second issue
DO 123 is unconstitutional on the ground
that it violates the equal protection clause by
allowing only motorcycles with at least 400 cubic
centimeters engine displacement to use the toll
ways. The RTC reasoned that the creation of a
distinction within the class of motorcycles was
not based on real diferences. The RTCs Decision
dated 10 March 2003 declared DO 123
unconstitutional on the ground that it violates the
equal protection clause by allowing only
motorcycles with at least 400 cubic centimeters
engine displacement to use the toll ways. The
RTC reasoned that the creation of a distinction
within the class of motorcycles was not based on
real diferences.
We
need
not
pass
upon
the
constitutionality
of
the
classification
of
motorcycles under DO 123.
As previously
discussed, the DPWH has no authority to regulate
limited access highways since EO 546 has

KITY

TAU MU TAU MU TAU MU


TAU MU TAU MU TAU MU

CONSTITUTIONAL LAW II
Order of Saint Thomas More
Atty. Philip John Pojas/Atty. Rovyne G. Jumao-as, RN
University College of Law

devolved this function to the DOTC.


Thus, DO
123 is void for want of authority of the DPWH to
promulgate it.
On the other hand, the assailed portion of
AO 1 states it is unlawful for any person or group
of persons to drive any bicycle, tricycle, pedicab,
motorcycle or any vehicle (not motorized). They
also claim that AO 1 introduces an unreasonable
classification by singling-out motorcycles from
other motorized modes of transport. Finally,
petitioners argue that AO 1 violates their right to
travel.
The SC ruled on the negative. A toll way is
not an ordinary road. As a facility designed to
promote
the
fastest
access
to
certain
destinations, its use, operation, and maintenance
require close regulation. Public interest and safety
require the imposition of certain restrictions on
toll ways that do not apply to ordinary roads. As a
special kind of road, it is but reasonable that not
all forms of transport could use it.
The right to travel does not mean the right
to choose any vehicle in traversing a toll way.
The right to travel refers to the right to move
from one place to another.
Petitioners can
traverse the toll way any time they choose using
private
or
public
four-wheeled
vehicles.
Petitioners are not denied the right to move from
Point A to Point B along the toll way. Petitioners
are free to access the toll way, much as the rest
of the public can. The mode by which petitioners
wish to travel pertains to the manner of using the
toll way, a subject that can be validly limited by
regulation.
Finally, petitioners assert that their
possession of a drivers license from the Land
Transportation Office (LTO) and the fact that their
vehicles are registered with that office entitle
them to use all kinds of roads in the country.
Again, petitioners are mistaken. There exists no
absolute right to drive. On the contrary, this
privilege, is heavily regulated. Only a qualified
group is allowed to drive motor vehicles: those
who pass the tests administered by the LTO. A
drivers license issued by the LTO merely allows
one to drive a particular mode of transport. It is
not a license to drive or operate any form of
transportation on any type of road. Vehicle
registration in the LTO on the other hand merely
signifies the roadworthiness of a vehicle. This
does not preclude the government from
prescribing which roads are accessible to certain
vehicles. Therefore, Only AO 1 is constitutional,
the rest otherwise.
2nd LT. SALVADOR PARREO vs COMMISSION
ON AUDIT
Facts:
Salvador Parreo (petitioner) served in the
Armed Forces of the Philippines (AFP) for 32
years. On 5 January 1982, petitioner retired from
the Philippine Constabulary with the rank of 2nd
Lieutenant.
Petitioner availed, and received
payment, of a lump sum pension equivalent to
three years pay.
In 1985, petitioner started

ACADCOM 2010; Contributors: Gene Geocaniga, Jarissa Guiani, Darlene Magabilen


TAU MU Page 38 of 179

receiving his monthly pension amounting to


P13,680.

TAU MU
TAU MU
TAU MU
TAU MU
TAU MU
TAU MU
TAU MU

There is compliance with all these


conditions.
There is a substantial diference
between retirees who are citizens of the
Philippines and retirees who lost their Filipino
citizenship by naturalization in another country,
such as petitioner in the case before us. The
constitutional right of the state to require all
citizens to render personal and military service
necessarily includes not only private citizens but
also citizens who have retired from military
service.
A retiree who had lost his Filipino
citizenship already renounced his allegiance to
the state. Thus, he may no longer be compelled
by the state to render compulsory military service
when the need arises. Petitioners loss of Filipino

TAU MU

The constitutional right to equal protection of the


laws is not absolute but is subject to reasonable
classification. To be reasonable, the classification
(a) must be based on substantial distinctions
which make real diferences; (b) must be
germane to the purpose of the law; (c) must not
be limited to existing conditions only; and (d)
must apply equally to each member of the class.

TAU MU

Ruling: The jurisdiction of the COA over money


claims against the government does not include
the power to rule on the constitutionality or
validity of laws. The 1987 Constitution vests the
power of judicial review or the power to declare
unconstitutional a law, treaty, international or
executive agreement, presidential decree, order,
instruction, ordinance, or regulation in this Court
and in all Regional Trial Courts. Petitioners money
claim essentially involved the constitutionality of
Section 27 of PD 1638, as amended. Hence, the
COA did not commit grave abuse of discretion in
dismissing petitioners money claim. Taking into
consideration the exclusivity of the jurisdiction of
the branches of government the COA holds no
jurisdiction over the case.
On the second issue

TAU MU
TAU MU

Issues: Whether or not there is substantial


distinction between retirees who are citizens of
the Philippines and retirees who lost there Filipino
citizenship by naturalization in another country.
And whether or not COA ha jurisdiction to rule on
the constitutionality of sec 27 of PD 1638 as
amended.

The Fraternal
Ateneo de Davao

TAU MU
TAU MU

Petitioner migrated to Hawaii and became


a naturalized American citizen. In January 2001,
the AFP stopped petitioners monthly pension in
accordance with Section 27 of Presidential Decree
No. 1638 (PD 1638), as amended by Presidential
Decree No. 1650. Section 27 of PD 1638, as
amended, provides that a retiree who loses his
Filipino citizenship shall be removed from the
retired list and his retirement benefits terminated
upon loss of Filipino citizenship.
Petitioner
requested for reconsideration but the Judge
Advocate General of the AFP denied the request.
Petitioner filed a claim before the COA for the
continuance of his monthly pension.

KITY

TAU MU TAU MU TAU MU


TAU MU TAU MU TAU MU

CONSTITUTIONAL LAW II
Order of Saint Thomas More
Atty. Philip John Pojas/Atty. Rovyne G. Jumao-as, RN
University College of Law

citizenship constitutes a substantial distinction


that distinguishes him from other retirees who
retain their Filipino citizenship. If the groupings
are characterized by substantial distinctions that
make real diferences, one class may be treated
and regulated diferently from another.
Petitioner will be entitled to receive his
monthly pension should he reacquire his Filipino
citizenship since he will again be entitled to the
benefits and privileges of Filipino citizenship
reckoned from the time of his reacquisition of
Filipino citizenship. There is no legal obstacle to
the resumption of his retirement benefits from
the time he complies again with the condition of
the law, that is, he can receive his retirement
benefits provided he is a Filipino citizen.
Therefore, the petitioner cannot get hi benefits
not until he returns to being a Filipino citizen.
ARREST WARRANT
WEBB VS. DE LEON
The NBI filed a complaint before the DOJ
for the so called Vizconde massacre indicting
Webb and five others. The DOJ formed a panel of
prosecutors to conduct preliminary investigation,
finding that probable cause exist, information was
filed before the RTC of Paranaque under judge
Escano. It was judge DE Leon, however, who
issued the WA. De Leon was the pairing judge of
Esacano.
Escano then voluntarily inhibited
himself from the case because he had formerly
worked with the NBI. The case was reraffled and
landed on the sala of judge Tolentino. Tolentino
then issued another WA. Petitioners question the
issuance of WA because there was allegedly no
preliminary investigation conducted and there are
inconsistencies on the affidavit of the witness.
Held: There is no requirement for a preliminary
investigation since personal review of the
documents is sufficient. Inconsistencies in the
affidavit is immaterial.
When the judge
determines probable cause, what is required of
him is to determine probability not certainty of
the guilt. If by all probability they have to be held
for trial then WA should be issued. Determination
of probability is diferent from determination of
guilt. Upon determination of probable caue, guilt
or non-guilt should not be the issue.
PANGANDAMAN VS. CESAR
There was a shooting incident which killed
5 people. After the incident and after preliminary
investigation, the Lanao judge issued a warrant
for the arrest of 14 individuals and 50 John Does.
Not one of the 50 can be identified by the
witnesses, but it seems that the number was
about the number of people who participated in
the shooting.
ISSUE: Is a John Doe warrant valid?

ACADCOM 2010; Contributors: Gene Geocaniga, Jarissa Guiani, Darlene Magabilen


TAU MU Page 39 of 179

Held: According to the court, a John Doe warrant


is void. The Constitution requires that there must
be specific description or designation of the
person to be arrested. John Does warrants are
general warrants.
PEOPLE VS. VELOSO

ISSUE: Is this a violation of the Constitution?

TAU MU
TAU MU

Held: The court said that the judge need not


personally conduct the investigation because it
will duly burdensome on the part of the judges if
they will be required to do so. They will not have
enough time to conduct the trial, which is their
main responsibility.

TAU MU

This is the famous libel case filed by


former Pres. Aquino against Luis Beltran. Beltran
questioned the issuance of the warrant. He said
the judge did not personally examine the
witnesses, which is unconstitutional. In an arrest
warrant, the judge may rely on the evidences. He
need not call the witnesses.

TAU MU

SOLIVEN VS. MAKASIAR

TAU MU
TAU MU

Held: According to the court, this is not a


violation of the Constitution because it describes
the John Doe. The police can also identify him by
the following designation written in the warrant
who is the owner of the establishment.

LIM VS. FELIX

TAU MU
TAU MU
TAU MU

Held: The court ruled that the warrant is void.


Before the fiscal can conclude that there is

TAU MU

ISSUE: Whether the judge can issue a warrant


based on the certificate alone. Can it be
considered valid?

TAU MU

The killing of Congressman Espinosa


happened in Masbate. The fiscal conducted a
preliminary investigation to find out if there was
probable cause to hold the accused for trial. And
he found out that there was. During the pendency
of the case before the RTC of Masbate, the SC
approved a petition for a change in venue to
Makati. The judge, on the certificate alone, which
was issued by the prosecutor, determined that
there was probable cause. The accused
challenged the validity of the warrant. He argued
that the judge should make a personal
determination as to the existence of probable
cause to issue the warrant.

The Fraternal
Ateneo de Davao

TAU MU
TAU MU

A search warrant was issued for an


establishment designated as the Parliamentary
Club which was believed to be a gambling den.
The search warrant identified the owner. They
also recovered gambling paraphernalia hidden in
his pocket. The accused questioned the warrant
being a John Doe warrant. There was not exact
description of the person to be arrested.

KITY

TAU MU TAU MU TAU MU


TAU MU TAU MU TAU MU

CONSTITUTIONAL LAW II
Order of Saint Thomas More
Atty. Philip John Pojas/Atty. Rovyne G. Jumao-as, RN
University College of Law

probable cause, he should first examine all the


evidences, documents presented to him, and all
of those should be attached on the certificate
stating that there is sufficient ground that the
accused should be put on trial. In this case, there
was nothing attached to the certificate stating
that there is sufficient ground that the accused
should be put on trial. In this case, there was
nothing attached to the certificate. The
Constitution underscores that the judge should
determine if there is probable cause basing on all
the facts and evidences, and not on the
certificate alone.
In the case of arrest warrant, the judge
need not call the witnesses for questioning. He
may rely on the record submitted to him by the
fiscal. But he must not rely solely on the
resolution or certification of the fiscal. The
resolution must be supported by other evidence
which led to the filing of the case. In the case of a
search warrant, the judge must personally
examine the witnesses himself to determine
probable cause.
COJUANGCO V. SANDIGANBAYAN
The Ombudsman filed a criminal case for
violation of Sec.3 (e) of P.A. No. 3019 against
Cojuangco. On the basis of the Resolution of the
Panel of Investigators of the Office of the
Ombudsman and the Memorandum of the Office
of the Special Prosecutor denying the existence of
a prejudicial question which would warrant the
suspension
of
the
criminal
case,
the
Sandiganbayan issued the warrant of arrest even
without the supporting evidence. Is this proper?
Held: No. The Sandiganbayan committed grave
abuse of discretion when it issued the warrant of
arrest without the benefit of the records or
evidence supporting the prosecutors finding of
probable cause. The court failed to abide by the
constitutional mandate of personally determining
the existence of probable cause before issuing a
warrant of arrest. The two documents before the
court were the product of somebody elses
determination, insufficient to support a finding of
probable cause by the Sandiganbayan. However,
by posting bail and filing various motions,
accused has waived the defects that attended the
issuance of the warrant of arrest. There was
voluntary submission to the courts jurisdiction.
ABDULA V. GUIANI
In a murder case, accused filed a petition
for certiorari and prohibition claiming that the
warrant of arrest issued against them was null
and void because the judge did not personally
examine the evidence nor did he call the
complainant and the witnesses in the face of their
incredible accounts. The judge in his Comment
stated that there was no reason for [him] to
doubt the validity of the certification made by the
Assistant
Prosecutor
that
a
preliminary
investigation was conducted and that probable

ACADCOM 2010; Contributors: Gene Geocaniga, Jarissa Guiani, Darlene Magabilen


TAU MU Page 40 of 179

cause was found to exist as against those


charged in the Information filed, so that he
issued the warrant. Is the warrant valid?

TAU MU
TAU MU
TAU MU
TAU MU
TAU MU
TAU MU

The Government of the United States of


America, through the Department of Justice, filed
with the Regional Trial Court a petition for the
extradition of Mark Jimenez who was facing
various criminal charges in Florida. Petitioner
prayed fro his immediate arrest, and the judge
set it for hearing. In extradition proceedings, is
the potential extraditee entitled to notice and
hearing before a warrant for his arrest can
issue?

TAU MU

GOVERNMENT V. PURGANAN

TAU MU

Held: No. It is the exclusive and personal


responsibility of the judge to satisfy himself as to
the existence of probable cause before issuing an
arrest warrant. To this end he must personally
evaluate the report and the supporting
documents
submitted
by
the
prosecutor
regarding the existence of probable cause. We
have ruled that a judge fails in his bounden duty
if he relies merely on the certification of the
investigation officer as to the existence of
probable cause making him administratively
liable. We can do no less in this case where
respondent issued the subject warrant of arrest
without even such certification to rely upon, and
worse, merely at the instance of the clerk who
mechanically typed the warrant for his signature.

TAU MU

Upon the filing of a complaint for libel


directly with the Regional Trial Court, the judge
issued a warrant for the arrest of accused. It
appears that no preliminary investigation was
conducted, nor an information prepared, by the
Office of the Prosecutor. Asked to explain his
action, the judge said that he signed the warrant
typed by his Criminal Docket Clerk, who under
the Check List for Criminal Cases which the judge
prepared for her, was supposed to verify from the
records whether an information had already been
filed. Should the judge be absolved?

TAU MU
TAU MU

TALINGDAN V. EDUARTE

The Fraternal
Ateneo de Davao

TAU MU
TAU MU

Held: No. The statement is an admission that


the judge relied solely and completely on the
certification made by the prosecutor that
probable cause exists as against those charged in
the information, and issued the challenged
warrant of arrest. The Constitution commands
the judge to personally determine probable cause
in the issuance of warrants of arrest. To be sure it
cannot be determined before hand how cursory
or exhaustive the examination of the records
should be. The extent depends on the exercise of
sound discretion as the circumstances of the
case require. But he cannot adopt the judgment
of the prosecutor regarding the existence of
probable cause on his own without abdicating his
duty under the Constitution.

KITY

TAU MU TAU MU TAU MU


TAU MU TAU MU TAU MU

CONSTITUTIONAL LAW II
Order of Saint Thomas More
Atty. Philip John Pojas/Atty. Rovyne G. Jumao-as, RN
University College of Law

Held: No. PD No. 1069, the Extradition Law uses


the word immediate to qualify the arrest of
accused. Arrest subsequent to a hearing can no
longer be considered immediate. Immediately
upon receipt of the petition, the judge shall make
a prima facie finding whether the petition is
sufficient in form and substance, whether it
complies with the Extradition Law, and whether
the person sought is extraditable. If the judge is
convinced, he issues a warrant for the arrest of
the person summons him to appear at scheduled
hearings.
Even Sec. 2 of Art. III of the
Constitution does not require a notice or a
hearing before the issuance of a warrant of
arrest.
What it requires is for the judge to
determine probable cause by examination of
complainants and the witnesses they may
produce. There is no requirement to notify and
hear the accused before the issuance of warrants
of arrest.
SEARCH WARRANT
SILVA VS. PRESIDING JUDGE
The deposition of the witness in support of
the application for search warrants proceeded as
follows:
Q: Do you personally know M/Sgt. Ranulfo
Villamor Jr., the applicant of a search warrant?
A: Yes sir.
Q: Do you have personal knowledge that the said
premises subject of the ofense stated above and
other proceeds or fruit of the ofense used or
obtained are intended to be used as means of
committing the ofense?
A: Yes sir.
Q: Do you know personally the person(s) who
have the property in his/their possession and
control?
A: Yes sir.
Q: How did you know all their(sic) things?
A: Through discreet surveillance
Held: The court held that the abuse deposition
did not only contain long questions but it was also
very broad. The questions propounded to the
witness were not probing but were merely
routinary.
The
deposition
was
already
mimeographed and all that the witnesses had to
do was to fill in their answers on the blanks
provided.
Asking of leading questions to the
deposition and conducting of an examination in a
general manner would satisfy the requirements
for issuance of a valid search warrant.
CORRO VS. LISING
Search of the Philippine Times with search
warrant based on two affidavits filed by the
witnesses
Affidavit of Col. Castillo submitted to the
judge contained:
We found that the said publication in fact
commenced distrust and hatred against the
government of the Philippines and its duly

ACADCOM 2010; Contributors: Gene Geocaniga, Jarissa Guiani, Darlene Magabilen


TAU MU Page 41 of 179

constituted authorities defined and penalized


under Art. 142 of the RPC as amended by
PD1835.
The other witness, Lt. Ignacio stated in his
affidavit:
The said periodical published by Corro contains
article leading to cite distrust and hatred for the
authority of the Republic of the Philippines or any
of the duly concerned authorities.
The judge issued the search warrant
based on the 2 affidavits. This was challenged
before the SC arguing that there was no probable
cause.

TAU MU
TAU MU

PRUDENTE VS. DAYRIT

The Fraternal
Ateneo de Davao

TAU MU
TAU MU

Held: The court held that the above statements


do not amount to probable cause. The statements
do not contain facts for conclusions of law. The
statements do not contain facts for conclusions of
law. The statements are only conclusions
determined by the witnesses and not by the
judge.

KITY

TAU MU TAU MU TAU MU


TAU MU TAU MU TAU MU

CONSTITUTIONAL LAW II
Order of Saint Thomas More
Atty. Philip John Pojas/Atty. Rovyne G. Jumao-as, RN
University College of Law

TAU MU

Held 1: In the first issue, the Court ruled that this


is not a general search warrant. It appears that if
the general place follows the particular place,
there is no violation of the Constitution as long as
the place is under the control of the same person.
It would be diferent if the search is to be
conducted on a condo building, you cannot
search anymore the other condo units which are
not owned by the same person. Usually a search
warrant must be specific of the place to search
and the object to be seized. This is to prevent the
peace officers from exercising discretion as to
what are to be seized. Otherwise, he can seize
anything he wants which are void and
unconstitutional.
Held 2: As to the 2nd issue, the court ruled that
the warrant is void because the judge did not ask
searching questions, but leading questions. It
would seem that the judge knows more about the
alleged crime than the witness. In addition, was
found that the witness did not hve personal
knowledge of the facts regarding the crime than
the witness. In addition, it was found that the
witness did not have personal knowledge of the
facts regarding the crime. His affidavit contained
statements like I have been informed and have
good reason to believe, with regards to the

TAU MU

TAU MU

During evacuation in Leyte, Prudente


allowed the evacuees to be housed on the
Polytechnic University. The police applied for a
search warrant alleging the existence of firearms
in the school. The judge commanded to make an
immediate search anytime in the day or night of
the premises of Polytechnic Univ. of the
Philippines, more particularly in the office of the
Dept. of Science and Tactics at the ground floor
and other rooms at the second floor.
Lawyers of Prudente got hold of a copy of
the examination of the witness by the judge.
They discovered that the judge did not ask
searching questions, but only leading questions in
which the witness only answered: yes sir.

statement, the undersigned has verified the


report therefore acquiring personal knowledge,
the court held that nothing in the verification was
made. It did not satisfy the requirement of
searching questions.
COLUMBIA VS. FLORES
The judge issued a search warrant for
violation of the Decree on Protection of
Intellectual Property against FGT Video Network,
a licensed video tape distributor. In addition to
video tapes, posters, and journal, it is also
ordered the seizure of:
Xxx
c.)
television
sets,
video
cassette
recorders, rewinders, tape head cleaners,
accessories, equipment and other machines and
paraphernalia or materials used or intended to be
used in the unlawful sale, lease, distribution, or
possession for purpose of sale, lease, distribution,
circulation or public exhibition of the abovementioned pirated video tapes.
Held: The court ruled that these articles and
appliances are generally connected with, or
related to a legitimate business not necessarily
involving piracy of intellectual property or
infringement of copyright laws. Hence, including
these articles without specification and/or
particularly that they were really instruments in
violating the Anti-Piracy Law makes the search
warrant too general which could result in the
confiscation of all items found in any video store.

TAU MU

COLUMBIA V. CA
261 SCRA 144 (1996)

TAU MU

The search warrant issued for violation of


Presidential Decree No. 49 directed the seizure,
among
others,
of
pirated
video
tapes.
Considering that there are as many or ofenses of
infringement as there are rights protected in the
various movie titles involved, does the warrant
violate the rule that a search warrant must be
issued only in connection with one specific
ofense?

TAU MU
TAU MU
TAU MU

Held: No. The search warrant itself indicates it


was issued for violation of Sec. 56, PD No. 49, as
amended only. The specifications therein merely
refer to the titles of the copyrighted motion
pictures belonging to private complainants which
defendants were in possession for sale or lease in
violation of PD No. 49. That there were several
counts of the ofense of copy right infringement
and the search warrant uncovered several
contraband items in the form of pirated video
tapes in not to be confused with the number of
ofenses charged.

TAU MU

COLUMBIA PICTURES V. CA
262 SCRA 219 (1996)

ACADCOM 2010; Contributors: Gene Geocaniga, Jarissa Guiani, Darlene Magabilen


TAU MU Page 42 of 179

Held:
Judges can issue warrants efective
outside of their jurisdiction because the Rules of
Court does not prohibit it. Sec. 3 of BP Blg. 129
states that all other processes may be served
anywhere in the Philippines. A search warrant is
a judicial process. This ruling applies when the
issuance is necessitated and justified by
compelling consideration of urgency, subject,
time and place. However, if a case is pending,
the court where such case is pending has the
primary jurisdiction to issue a warrant, except
under extreme and compelling circumstances.

An RTC judge of Kaloocan issued a search


warrant for alleged violation of PD 1866. He
directed the search of a house in Quezon City
which is outside his territorial jurisdiction. The
validity of the search is now questioned.

SALAZAR VS. ACHACOSO


In line with the power granted by Art. 38
of the Labor Code, the Secretary issued a warrant
to conduct a search in a particular property, in
connection with the crime of illegal recruitment.

TAU MU

Held: The court held that the PD which was later


incorporated in to the Labor Code as Art. 38 was
unconstitutional. Under the Constitution, only
judges can issue search warrants.

TAU MU

PICOP V. ASUNCION

TAU MU

PEOPLE VS. DICHOSO

TAU MU

TAU MU

Held:
No.
The above items could not be
anymore specified as the circumstances will allow
since they are all used or intended to be used in
the unlawful sale or lease of pirated tapes. [But
see, Columbia v. Flores, 223 SCRA 761 (1993),
where another division of the Supreme Court
voided a similarly-worded search warrant as a
general warrant.]

TAU MU
TAU MU

The court issued a search warrant for


violation of the Dangerous Drugs Act. It directed
the peace officer to search and seize the
following:
a.)marijuana,
b.)shabu,
c.)and
paraphernalia. The accused challenged the
constitutionality of the warrant on the basis that
it covers three articles, making it a general
search warrant. He claimed that the warrant
failed to specify the thing to be seized.

ILANO V. CA

TAU MU

For violation of Sec. 56 of the Decree on the


Protection of Intellectual Property, the judge
issued a search warrant for the seizure of:
(c) Television sets, video cassettes records,
rewinders, tape head cleaners, accessories,
equipment
and
other
machines
and
paraphernalia on material used or intended to be
used in the unlawful sale, lease, distribution, or
possession for purpose of sale, lease, distribution,
circulation
or
public
exhibition
of
the
abovementioned pirated video tapes which he is
keeping and concealing in the premises abovedescribed. Is the warrant a general warrant?

Held: The court ruled that this is not a general


search warrant. The ofenses involved, or the
objects subject for seizure belong to the same
class. Therefore, the officer does not need one
warrant for each item.

TAU MU

COLUMBIA PICTURES V. CA
supra

Ateneo de Davao

TAU MU
TAU MU

Held: The ruling in 20th Century Fox has no


retroactive application. The lower court could not
have expected more evidence from the applicant
other that what the law and jurisprudence
required at that time with respect to finding of
probable cause. In addition, the Court did not
rule therein that representation of the master
tapes is an absolute requirement for a search
warrant to issue. It should serve only as a guide
post when there is doubt as to the nexus between
the master tapes and the pirated copies. It does
not rule out the use of testimonial, documentary
or other evidence tending to establish probable
cause.

The Fraternal

TAU MU
TAU MU

On July 28, 1986, the judge issued a


search warrant for Violation of the Decree on the
Protection of Intellectual Property ordering the
seizure of pirated video tapes, without requiring
the production of the master tapes. In August 19,
1988, the Supreme Court voided a search warrant
in 20th Century v. CA, 162 SCRA 655 (1988), on
the ground of lack of probable cause because the
master tapes of the alleged pirated tapes were
not shown to the lower court. Is the search
warrant valid?

KITY

TAU MU TAU MU TAU MU


TAU MU TAU MU TAU MU

CONSTITUTIONAL LAW II
Order of Saint Thomas More
Atty. Philip John Pojas/Atty. Rovyne G. Jumao-as, RN
University College of Law

The Philippine National Police (PNP)


applied for a search warrant in Quezon City for
the search of Paper Industries Corporation of the
Philippines (PICOP) in Bislig, Surigao del Sur. The
search was for the seizure of 70 Armalite rifles
and other assorted firearms. The applicant did
not allege before the trial court that PICOP has no
license to possess the firearms nor attached a
certification from the Firearms and Explosives
Unit on the lack of license.
Upon application by the Philippines
National Police (PNP), the judge ordered the
search and seizure of firearms in possession of
the Paper Industries Corporation of the
Philippines located at PICOP Compound,
Barangay Tabon, Bislig, Surigao del Sur.
Issues: (1) Was there probable cause for the
issuance of the warrant? ; (2) Is the description of
the place to be searched sufficient?
Held: First Issue: There was no probable cause
for the issuance of the search warrant. The facts
and circumstances that would show probable

ACADCOM 2010; Contributors: Gene Geocaniga, Jarissa Guiani, Darlene Magabilen


TAU MU Page 43 of 179

Are the warrants valid?

TAU MU
TAU MU
TAU MU

KENNETH ROY V. TAYPAN

TAU MU

Held: Yes. The law does not require that the


things to be seized must be described in precise
and minute detail as to leave no room for doubt
on the part of the searching authorities.
Otherwise, it will be virtually impossible for the
applicants to obtain a warrant as they would not
know exactly what kind of things they are looking
for. Since the element of time is very crucial in
criminal cases, the efort and time spent in
researching the details to be embodied in the
warrant would render the purpose of the search
nugatory. Consequently, the above descriptions
could be adjudged in substantial compliance with
the requirements of law.

TAU MU

Chop-chop

TAU MU

Search Warrant No. 90-14:


vehicles and other spare parts.

TAU MU

Search Warrant No 90-12:


Unlicensed
firearms of various calibers and ammunitions for
the said firearms.

TAU MU

The judge issued 3 search warrants describing


the items to be seized as follows:
Search Warrant No. 90-11: Unlicensed radio
communications equipment such as transmitters,
transceivers, handsets, scanners, monitoring
devices and others.

TAU MU
TAU MU

KHO V. MAKALINTAL

The Fraternal
Ateneo de Davao

TAU MU
TAU MU

cause must be the best evidence that could be


obtained under the circumstances.
The
introduction of such evidence is necessary in
cases where the issue is the existence of the
negative ingredient of the ofense charged --- for
instance, the absence of a license required by
law, as in the present case --- and such evidence
is within the knowledge and control of the
applicant who could easily produce the same.
But if the best evidence could not be secured at
the time of the application, the applicant must
show a justifiable reason therefore during the
examination by the judge.
2nd issue: No. The warrant failed to describe the
place with particularity. Is simply authorizes a
search of the aforementioned premises, but it
did not specify such premises. It identifies only
one place, and that is the PICOP Compound. The
compound, however, is made up to 200
offices/buildings, 15 plants, 84 staf houses, 1
airstrip, 3 piers/wharves, 23 warehouses, 6 POL
depots/quick service outlets and some 300
miscellaneous, all of which are spread out over
one hundred fifty-five hectares. Obviously, the
warrant gives the police officers unbridled and
thus illegal authority to search all the structures
found inside the PICOP Compound.

KITY

TAU MU TAU MU TAU MU


TAU MU TAU MU TAU MU

CONSTITUTIONAL LAW II
Order of Saint Thomas More
Atty. Philip John Pojas/Atty. Rovyne G. Jumao-as, RN
University College of Law

Upon application by the National Bureau


of Investigation, the Regional Trial Court of Cebu,
Branch 12, ordered the seizure of several pieces
of furniture from petitioner. The ofense was for
violation of Art. 189 of the Revised Penal Code on
unfair competition involving design patents.
Does RTC, Branch 12, have the authority to issue
the warrant considering that it was not
designated as a special court for Intellectual
Property Rights? In applying for a search warrant
must the applicant execute a certificate of nonforum shopping?
Held: A search warrant is merely a process
issued by the court in the exercise of its ancillary
jurisdiction and not a criminal action which it may
entertain pursuant to its original jurisdiction. The
authority to issue search warrants is inherent in
all courts and may be efected outside their
territorial jurisdiction.
AO No. 113-95 merely
specified which court could try and decide cases
involving intellectual property rights. It did not
vest exclusive jurisdiction with regard to all
matters in any one court. Jurisdiction is conferred
upon courts by substantive law, not by an
administrative order. Nor is a certificate of nonforum shopping necessary. It is required only for
initiatory pleadings, not applications.
UY V. BIR
For the crime of selling cartons of sardines
without issuing receipts in violation of Sec. 253 of
the National Internal Revenue Code, the Regional
Trial Court issued a Search Warrant against
petitioner at Unifish Packing Corporation. In the
caption of the Search Warrant, his address was
indicated as Hernan Cortes St., Cebu City, while
the body of warrant states the address as
Hernan Cortes St., Mandaue City. Is there a
violation of the constitutional requirement that
there be a particular description of the place to
be search?
Held: No. The Rule is that a description of a
place to be searched is sufficient if the officer
with the warrant can, with reasonable efort,
ascertain and identify the place intended and
distinguish it from other places in the community.
In this case, it was not shown that a street
similarly named Hernan Cortes could be found in
Cebu City.
Nor was it established that the
enforcing officers had any difficulty in locating the
premises described in the warrant. That the
warrant, therefore, inconsistently identified the
city where the premises to be searched is, is not
a defect that would spell the warrants
invalidation.
UY V. BIR
supra

TAU MU

For violation of Sec. 253 of the National


Internal Revenue Code, the RTC issued 2 Search
Warrants, the second apparently to correct the
inconsistencies in the first. The first warrant was
issued solely against Uy Chin Ho alias Frank Uy,

ACADCOM 2010; Contributors: Gene Geocaniga, Jarissa Guiani, Darlene Magabilen


TAU MU Page 44 of 179

and the second against Uy Chin Ho alias Frank


Uy, and Unifish Packing Corporation. Is there a
defect in the description of the place to be
searched?

TAU MU
TAU MU
TAU MU

The warrant for the ofense of selling


cartons of sardines without issuing receipts in
violation of Sec. 253 of the National Internal
Revenue Code issued by the Regional Trial Court
described the objects to be seized as follows: 1.
Multiple sets of Books of Accounts, Ledgers,
Journals, Provisional & Official Receipts; 2.
Production Record Books/Inventory Lists; 3.
Unregistered Delivery Receipts; 4. Unregistered
Purchase & Sales Invoice; 5. Sales Records, Job
Orders.
Are the objects described with
particularity?

TAU MU
TAU MU

UY V. BIR
supra.

The Fraternal
Ateneo de Davao

TAU MU
TAU MU

Held: No. The Constitution does not require the


warrant to name the person who occupies the
named premises. Where the warrant is issued for
the search of a specifically described premises
only and not for the search of a person, the
failure to name the owner or occupant of such
property in the warrant does not invalidate the
warrant; and where the name of the owner of the
premises is incorrectly inserted, it is not a fatal
defect if the legal description of the premises is
otherwise correct so that no discretion is left to
the officer making the search as to the place to
be searched. Since in this case the warrant was
issued not for the search of the persons owning
the premises but only a search of the premises
occupied by them, the search could not be
declared unlawful because of the inconsistencies
in stating their names.

KITY

TAU MU TAU MU TAU MU


TAU MU TAU MU TAU MU

CONSTITUTIONAL LAW II
Order of Saint Thomas More
Atty. Philip John Pojas/Atty. Rovyne G. Jumao-as, RN
University College of Law

TAU MU

PEOPLE V. SALANGUIT

TAU MU

TAU MU

Held: Most of the items listed fail to meet the


test of particularity, especially since the judge
was furnished with the photocopies of the
documents sought to be seized. The use of
generic term or a general description is
acceptable only when a more specific description
of the things is unavailable. The use of terms like
multiple sets of books of account ledgers
journals, cash register booksprovisional &
official receipts stock delivery receipts and
unregistered purchase & sales invoices, they
need not be specified as it is not possible to do so
precisely because they are unregistered. The
general description of most of the documents
listed in the warrant does not render the entire
warrant void. Insofar as the warrant authorized
the seizure of unregistered delivery receipts and
unregistered purchase and sales invoices, the
warrant remain valid.
The search warrant is
severable, and those items not particularly
described may be cut of without destroying the
whole warrant.
Accordingly, the items not
particularly described ought to be returned to
petitioner.

The Search Warrant issued by the judge states:


It appearing to the satisfaction of the
undersigned after examining under oath SR. INSP.
Rodolfo V. Aguilar, PNP and his witness SPO1
Edmund M. Badua, PNP, that there is probable
cause to believe that Robert Salanguit has in his
possession and control in his premises Binhagan
St., San Jose, Quezon City as shown in Annex A,
the properties to wit: UNDETERMINED QUANTITY
OF SHABU AND DRUG PARAPHERNALIA.
Considering that during the depositiontaking, no witness testified on anything about
drug paraphernalia, should the warrant be
nullified for having been issued without probable
cause?
Held: No. The fact that there was no probable
cause to support the application for the seizure of
drug paraphernalia does not warrant the
conclusion that the search warrant is void. This
fact would be material only if drug paraphernalia
were seized by the police. But none was taken by
virtue of the search warrant. If at all, therefore,
the search warrant is void only insofar as it
authorized the seizure of drug paraphernalia, but
it is valid as to the seizure of shabu about which
evidence was presented showing probable cause
as to its existence. It would be a drastic remedy
indeed if a warrant, which was issued on probable
cause and particularly describing the items to be
seized on the basis thereof, is to be invalidated in
to because the judge erred in authorizing a
search for other items not supported by evidence.
PEOPLE V. SALANGUIT

TAU MU

The Search Warrant issued by the judge


ordered the search of the residence of Robert
Salanguit at Binhagan St., San Jose, Quezon City
as shown in Annex A. Among the documents in
the records of the applications for the warrant
are: 1) the application itself which stated that the
premises to be searched was located in between
No. 7 and 11 of Binhagan St.; 2) deposition of
witness which describe the premises as a house
without a number located at Binhagan St.; and
3) the pencil sketch of the location of the
premises to be searched. In addition, the leader
of the police learn who conducted the search
resides in the neighborhood. Did the warrant
satisfy the requirement of particularity of
description of the place to be searched?

TAU MU
TAU MU

Held: Yes. The rule is that a description of the


place to be searched is sufficient if the officer
with the warrant can, with reasonable efort,
ascertain and identify the place intended to be
searched.
For example, a search warrant
authorized a search of Apartment No. 3 of a
building at 83 Peasant St., Malborough,
Massachusettss. As is turned out, there were 5
apartments in the basement and 6 apartments on
both the ground and top floors and that there was
an Apartment No. 3 on each floor. However, the
description was made determinate by a reference
to the affidavit supporting the warrant that the

ACADCOM 2010; Contributors: Gene Geocaniga, Jarissa Guiani, Darlene Magabilen


TAU MU Page 45 of 179

apartment was occupied by Morris Ferrante of 83


Pleasant St., Malboro, Mass. In this case, the
location of Salanguits house being indicated by
the evidence on record, there can be no doubt
that the warrant described the place to be search
with sufficient particularity.
AL-GHOUL V. CA
364 SCRA 363 (2001)

TAU MU
TAU MU
TAU MU
TAU MU

A team from the Police Assistance and


Reaction Against Crime (PARAC) of the DILG was

TAU MU

Facts:

TAU MU

Case: Quelnan vs. People


(526 SCRA 353)

TAU MU

Held: There was no specific ofense mentioned.


There is a need to mention only one specific
ofense. Otherwise, it will be considered a general
warrant.

TAU MU

Note: This case was also in the exclusionary rule.


Stated in the warrant: The above items are
subject to the ofense, stolen or embezzled, or
intended to be used as a means to commit
ofenses violating CB laws, tarifs and customs
laws, The Internal Revenue Code, and the RPC.

TAU MU

STONEHILL VS. DIOKNO


(discussed by Ms. Jumao-as General
Warrant)

TAU MU
TAU MU

Held: Yes. The articles seized are of the same


kind and nature as those items enumerated in
the search warrant. The items ordered seized in
the warrant were described with specificity. The
nature of the items ordered to be seized did not
require a technical description. Moreover, the
law does not require that the things to be seized
must be described in precise and minute details
as to leave no room for doubt, otherwise, it would
be virtually impossible for the applicants to
obtain a search warrant as they would not know
exactly what kind of things they are looking for.
Once describe, however, the articles subject of
seizure need not be so invariant as to require
absolute concordance between those seized and
those described in the warrant.
Substantial
similarity of those articles described as a class or
species would suffice.

The Fraternal
Ateneo de Davao

TAU MU
TAU MU

The judge ordered the search of


Apartment No. 2 and the seizure of the following:
1 .45 caliber pistol; 1 5.56 M16 Rifle with
ammunitions; 1 9MM pistol with ammunitions; 3
boxes of explosives; 10 sticks of dynamite; 30
pieces of blasting caps. The police, however,
found and seized the following: 2 M-16 rifles with
2 magazines and 20 ammunitions; 1 bar
demolition charge; 1 caliber Pistol, with magazine
of Caliber .45 and 3 .45 ammunitions; 1 22
caliber handgun with 5 ammunitions nitrate; 22
detonating cords; and pound of TNT; 1 timer
alarm clock; and 2 batteries 9 volts. Are the
objects admissible?

KITY

TAU MU TAU MU TAU MU


TAU MU TAU MU TAU MU

CONSTITUTIONAL LAW II
Order of Saint Thomas More
Atty. Philip John Pojas/Atty. Rovyne G. Jumao-as, RN
University College of Law

formed to implement a search warrant. The team


proceeded to the CityLand Condo in Makati and
upon arrival, they went to the Security Office of
the said building to seek assistance in serving the
warrant and they were accompanied to Unit 615.
The team presented the search warrant to
petitioner and the police operatives searched the
unit. In the presence of the petitioner, the team
started searching the place and eventually found
on top of the bedroom table 3 pcs. of transparent
plastic sachets containing white crystalline
substances later confirmed by the NBI as shabu,
plastic tubings, weighing scales, an improvised
burner and empty transparent plastic sachets.
Thereafter, the team prepared a receipt of the
properties seized and an Affidavit of Orderly
Search allegedly signed by petitioner in their
presence and that of security officer of the
building. The team also went to Unit 418 of the
same building to serve the warrant and search
the place. The police operatives did not find any
occupant in the room. The following day,
petitioner was charged for violation of Sec. 16,
Article III of R.A. 6245.
In his defense, petitioner testified that he
is the registered owner of Unit 615 of CityLand
Condo which he leased to Sung Kok Lee. He also
said that at the time the police operatives went to
the said unit, he was supposed to collect
payment of rental from Lee. He heard somebody
knocking in the door when he opened it. The
police operatives asked for a certain Bernard Kim
and he introduced himself as the owner of the
condo unit. The police operatives then searched
the house and he was forced to sign some
documents at gunpoint.
Issue:

WON the search warrant was validly


implemented.
Ruling:
Section 4, Rule 126 of the Revised Rules of
Criminal Procedure provides for the requisites for
the issuance of search warrant which states that:
a search warrant shall not issue except upon
probable cause in connection with one specific
ofense to be determined personally by the judge
after examination under oath or affirmation of the
complaint and the witness he may produce, and
particularly describing the place to be searched
and the things to be seized which may be
anywhere in the Philippines. Nowhere in said rule
or any other provision in the Revised Rules of
Criminal Procedure is it required that the search
warrant must name the person who occupies the
described premises. A cursory reading of the
search warrant reveals that the police officers
were ordered to make an immediate search of the
premises mentioned and to seize and take
possession of shabu. Furthermore, they were
directed to bring persons to be dealt with as the
law may direct. While petitioner may not be the
person subject of the search, the fact that he was
caught in flagrante delicto necessitated his valid
warrantless arrest. Therefore, the fact that
petitioners name was not indicated in the search
warrant is immaterial.

ACADCOM 2010; Contributors: Gene Geocaniga, Jarissa Guiani, Darlene Magabilen


TAU MU Page 46 of 179

Case: People vs. Francisco


(387 SCRA 569)
Facts:

TAU MU
TAU MU
TAU MU
TAU MU
TAU MU
TAU MU
TAU MU
TAU MU

Ruling:
The basic guarantee to the protection of
the privacy and sanctity of a person, his home
and his possessions against unreasonable
intrusions of the State is articulated in Section 2,
Article III of the Constitution. For the validity of
the search warrant, the Constitution requires that
there be a particular description of the place to
be searched and the persons or things to be
seized. The rule is that a description of the place
to be searched is sufficient if the officer with the
warrant can, with reasonable efort, ascertain and
identify the place intended and distinguish it from
other places in the community. Any designation
or description known to the locality that leads the
officer unerringly to it satisfies the constitutional
requirement.
The requisites for the issuance of a valid
search warrant are: (1) probable cause is present;

TAU MU

WON the search conducted by the police


officers at the accused-appellants residence was
reasonable.

TAU MU
TAU MU

Issue:

The Fraternal
Ateneo de Davao

TAU MU
TAU MU

Verona and his live-in girlfriend Francisco,


were placed under surveillance after the police
confirmed that they were engaged in selling
shabu. SPO2 Teneros and SPO4 San Juan applied
for a search warrant to authorize them to search
the premises at 122 M. Hizon St., Caloocan City.
Attached to such application was the AfterSurveillance Report that one of Veronas runners
in the illegal drugs operations, allegedly sought
the assistance of SPO4 Teneros for the arrest of
Verona.
According to Francisco, who was then 9
months pregnant at the time the police
operatives suddenly entered her bedroom, while
she was resting inside their bedroom at 120 M.
Hizon St., Caloocan City, she heard a loud bang
downstairs and 8 policemen entered and
conducted a search for an hour. She inquired
about their identities but it was only at the police
station where she found out that the team of
searchers was led Teneros. The police team
enforced the warrant and seized the following: (1)
1 pyrex salad set wrapped in a plastic containing
white crysthalline substance or shabu with
markings by the undersigned inside the house of
subjects residence weighing 230 grams of shabu:
(2) several plastics in diferent sizes: (3) 2 roll of
strip aluminum foil: (4) 5 tooter water pipe and 2
improvised burner: (5) 2 measuring weight in
shabu: (6) 2 motorola cellular phones; (7) 1
monitoring device with cord: and (8) Php22, 990.
Consequently, Francisco was charged with
violation of RA 6425 (Dangerous Drug Act).
Francisco then filed a motion to quash the search
warrant asserting that she and her live-in partner
Verona had been leasing an apartment at 120 M.
Hizon St., District 2 Caloocan City. Upon
arraignment, she pleaded not guilty.

KITY

TAU MU TAU MU TAU MU


TAU MU TAU MU TAU MU

CONSTITUTIONAL LAW II
Order of Saint Thomas More
Atty. Philip John Pojas/Atty. Rovyne G. Jumao-as, RN
University College of Law

(2) such presence is determined personally by the


judge: (3) the complainant and the witnesses he
or she may produce are personally examined by
the judge, in writing, and under oath or
affirmation: (4) the applicant and the witnesses
testify on the facts personally known to them:
and (5) the warrant specifically describes the
place to be searched and the things to be seized.
The absence of any of these requisites will
cause the downright nullification of the search
warrants. The warrants will always be construed
strictly without, however, going the full length of
requiring technical accuracy. No presumptions of
regularity are to be invoked in aid of the process
when an officer undertakes to justify it.
In the case at bar, the trial court
conducted an ocular inspection of the area. It
turned out that No. 122 M. Hizon St., District 2
Caloocan City was a concrete two-storey
residential building with steel-barred windows
and a terrace. It was owned by certain Mr. Joseph
Ching. The house bore no house number. He
house marked No. 122 M. Hizon St., District 2
Caloocan City was actually two houses away from
Franciscos house at 120 M. Hizon St., District 2
Caloocan City. The particularity of the place
described is essential in the issuance of search
warrants to avoid the exercise by the enforcing
officers of discretion. The controlling subject of
search warrants is the place indicated in the
warrant itself and not the place identified by the
police. Hence, Francisco was acquitted.
Case: Prudente vs. Dayrit and PP
Facts:
Dimagmaliw, chief of ISAD, filed an
application for the issuance of a search warrant
for violation of PD 1866 against Prudente.
Dimagmaliw alleged that Prudente has in his
control or possession of firearms, explosives,
hand grenades and ammunitions which are
illegally possessed or intended to be used as the
means of committing an ofense which Prudente
is keeping and concealing at the premises of PUP
in the Office of the Department Military Science
and Tactics and Office of the President. In support
of his application, the OIC of the Intelligence
Section of ISAD, executed a Deposition of
Witness which made Dayrit to issue the search
warrant on the same day.
Thereafter, the search warrant was
enforced. A member searching team alleged that
he found in the drawer of the cabinet inside the
wash room of Prudente a bulging brown envelope
with 3 live fragmentation hand grenades
separately wrapped with old newspapers,
classified as 1 pc. M33 fragmentation hand
grenade, 1 pc. M26 fragmentation hand grenade
and 1 pc. PRB-423 fragmentation hand grenade.
Prudente moved to quash the search warrant
claiming that the complainants lone witness had
no personal knowledge of the facts which formed
the basis for the issuance of the search warrant,
that the examination of the said witness was not
in the form of searching questions and answers,
the search warrant was a general warrant for the

ACADCOM 2010; Contributors: Gene Geocaniga, Jarissa Guiani, Darlene Magabilen


TAU MU Page 47 of 179

reason that it did not particularly describe the


place to be searched and that it failed to charge
one specific ofense and the search warrant was
issued in violation of Circular No. 19 of the SC in
that the complainant failed to allege under oath
that the issuance of the search warrant on that
certain day was urgent.
Issue:
WON the search warrant was valid.

TAU MU
TAU MU
TAU MU
TAU MU
TAU MU
TAU MU
TAU MU
TAU MU

SONY filed a complaint against Supergreen


with the NBI for the reproduction and distribution
of counterfeit Playstation game software,
consoles and accessories in violation of Sony

TAU MU

Facts:

TAU MU
TAU MU

Case: SONY Computer Entertainment, Inc.


vs. Supergreen Incorporated

The Fraternal
Ateneo de Davao

TAU MU
TAU MU

Ruling:
For a valid search warrant to issue, there
must be a probable cause, which is to be
determined personally by the judge, after
examination under oath or affirmation of the
complainant and the witnesses he may produce,
and particularly describing the place to be
searched and the persons or things to be seized.
The probable cause must be in connection with
one specific ofense, and the judge must, before
issuing the warrant, personally examine in the
form of searching questions and answers, in
writing and under oath, the complainant and any
witness he may produce, on facts personally
known to them and attach to the record their own
statements
together
with
any
affidavits
submitted.
The probable cause for a valid search
warrant has been defined as such facts and
circumstances which would lead a reasonably
discreet and prudent man to believe that an
ofense has been committed and that objects
sought in connection with the ofense are in the
place sought to be searched. This probable
cause must be shown to be within the personal
knowledge of the complainant or the witnesses
he may produce and not based on mere hearsay.
In the case at bar, the evidence failed to
show the existence of probable cause to justify
issuance of the search warrant. The SC also notes
post facto that the search in the question yielded,
no armalites, handguns, pistols, assorted
weapons or ammunitions as stated in the
application for search warrant, the supporting
deposition, and the search warrant itself. Only 3
live fragmentation hand grenades were found in
the searched premises of the PUP, according to
the affidavit of an alleged member of the
searching party. The SC also avails of the decision
to
reiterate
the
strict
requirements
for
determination of probable cause in the valid
issuance of a search warrant. The requirements
are stringent but the purpose is to assure the
constitutional right of the individual against
unreasonable search and seizure shall remain
both meaningful and efective. Hence, the
petition for certiorari to annul and set aside the
order of Dayrit was granted.

KITY

TAU MU TAU MU TAU MU


TAU MU TAU MU TAU MU

CONSTITUTIONAL LAW II
Order of Saint Thomas More
Atty. Philip John Pojas/Atty. Rovyne G. Jumao-as, RN
University College of Law

Computers Intellectual Property Rights. Thus,


NBI applied with the RTC of Manila, Branch 1, for
warrants to search respondents premises in
Paranaque and Cavite. RTC issued search
warrants covering Supergreens premises at
Trece-Tanza Road, Purok 7, Brgy. De Ocampo,
Trece Martires City, Cavite and at Room 302, 3rd
Floor Chateau de Baie Condo, 149 Roxas Blvd.
corner Airport Road, Paranaque City.
NBI
simultaneously served the search warrants on the
subject premises and seized a replicating
machine and several units of counterfeit
Playstation consoles, joy pads, housing, labels
and game software. Thus, Supergreen filed a
motion to quash the search warrants on the
ground that the search warrant failed to
particularly describe the properties to be seized.
Issue:
WON the quashal of search warrants were
valid in this case.
Ruling:
In the case at bar, Supergreens premises
in Cavite, within the Fourth Judicial Region, is
definitely beyond the territorial jurisdiction of the
RTC of Manila in the NCR. Thus, the RTC of Manila
does not have the authority to issue a search
warrant for ofenses committed in Cavite.
However, SC agreed with Sony that this case
involves a transitory continuing ofense of unfair
competition under Section 168 of RA 8293.
Supergreens imitation of the general appearance
of petitioners goods was done allegedly in
Cavite. It sold the goods allegedly in
Mandaluyong. The alleged acts would constitute a
transitory or continuing ofense. Thus, under
Section 2 (b) of Rule 126, Section 168 of RA 8293
and Article 189 (1) of the RPC, Sony may apply
for a search warrant in any court where any
element of the alleged ofense was committed,
including any courts within NCR. Hence, the
petition for review sought by Sony was granted.
Case: Yao Sr. vs. PP
Facts:
Petitioners are incorporators and officers
of MASAGANA, an entity engaged in refilling, sale
and distribution of LPG products. PR Petron and
Shell are two of the largest bulk suppliers and
producers of LPG in the Phils. Their LPG products
are sold under the marks of GASUL and
SHELLANE
respectively.
NBI
filed
two
applications
for
search
warrants
against
petitioners and other occupants of MASAGANA
compound located at Governors Drive, Brgy.
Lapidario, Trece Martires, Cavite City for alleged
violation of RA 8293, otherwise knows as The
Intellectual Property Code of the Philippines. The
2 applications for search warrants uniformly
alleged that per info, belief and personal
verification of Oblanca (the NBI agent who
applied for search warrants), the petitioners are
actually producing, selling, ofering for sale
and/or distributing LPG products using steel
cylinders owned by, and bearing the tradenames,
trademarks and devices of Petron and Shell,

ACADCOM 2010; Contributors: Gene Geocaniga, Jarissa Guiani, Darlene Magabilen


TAU MU Page 48 of 179

TAU MU
TAU MU
TAU MU
TAU MU
TAU MU
TAU MU
TAU MU
TAU MU
TAU MU
TAU MU
TAU MU

Ruling:
Article III, Section 2 of the Constitution
states the requirements before a search warrant
may be validly issues such as: (1) probable cause
is present; (2) such presence is determined
personally by the judge: (3) the complainant and
the witnesses he or she may produce are
personally examined by the judge, in writing, and
under oath or affirmation: (4) the applicant and
the witnesses testify on the facts personally
known to them: and (5) the warrant specifically
describes the place to be searched and the things
to be seized.
A search warrant can be issued only upon
finding a of probable cause. Probable cause for
search
warrant
means
such
facts
and
circumstances which would lead a reasonably
discreet and prudent man to believe that an
ofense has been committed and that the objects
sought in connection with the ofense are in the
place to be searched.
The facts and
circumstances being referred thereto pertain to
facts, data or info personally known to the
applicant and the witnesses he may present. The
applicant or his witnesses must have personal
knowledge of the circumstances surrounding the
commission of the ofense being complained of.
Reliable Info is insufficient. Mere affidavits are
not enough, and the judge must depose in writing
the complainant and his witnesses.
In the case at bar, documentary and
object evidence is that Oblanca and his witness
have personal knowledge of the fact that
petitioners, through MASAGANA, have been using
the cylinders bearing the marks GASUL and
SHELLANE without permission from Petron and
Shell which is a probable case for trademark
infringement. Both Oblanca and his witness were
clear and insistent that they were the very same
persons who monitored the activities of

The Fraternal
Ateneo de Davao

TAU MU
TAU MU

without authority and in violation of the rights of


the
said
entities.
The
search
warrants
commanded any peace officer to make an
immediate search of the MASAGANA compound
and to seize the all empty or filled LPG cylinder
tanks/containers bearing the tradename SHELL
or SHELLANE and the trademarks and other
devices owned by Shell including sales invoices
as well as Petrons all empty or filled LPG cylinder
tanks/containers bearing the tradename GASUL
and all other devices owned by Petron.
Petitioners filed a motion to quash the
search warrants on the following grounds: (1) that
there is no probable cause for the issuance of
search warrant and the conditions for search
warrant were not complied with: (2) that NBI
agent Oblanca and his witness do not have any
authority to apply for a search warrant: (3) that
the place to be searched was not specified in the
search warrant: and (4) the search warrant is
characterized as a general warrant as the items
to be seized as mentioned in the search warrant
are being used in the conduct of the lawful
business and the same are not being used in
refilling Shellane and Gasul LPGs.
Issue:
WON the search warrant was valid.

KITY

TAU MU TAU MU TAU MU


TAU MU TAU MU TAU MU

CONSTITUTIONAL LAW II
Order of Saint Thomas More
Atty. Philip John Pojas/Atty. Rovyne G. Jumao-as, RN
University College of Law

MASAGANA: that they conducted test-buy


thereon: and that in order to avoid suspicion,
they used diferent names during the test-buys.
They also personally witnessed the refilling of LPG
cylinders bearing the marks GASUL and
SHELLANE inside the MASAGANA refilling plant
station and the deliveries of these refilled
containers to some outlets using mini trucks.
Indeed,
the
aforesaid
facts
and
circumstances are sufficient to establish probable
cause. It should be borne in mind that the
determination of the probable cause does not call
for the application of the rules and standards of
proof that a judgment of conviction requires after
trial on the merits. As the term implies, probable
cause is concerned with probability, not absolute
or even moral certainty. The standards of
judgment are those of a reasonably prudent man,
not the exacting calibrations of a judge after a full
blown trial. The facts that Oblanca and his
witness used diferent names in the purchase
receipts do not negate personal knowledge on
their part. It is common practice of the law
enforcers such as NBI agents during covert
investigations to use diferent names in order to
conceal their true identities. This is reasonable
and understandable so as not to endanger the life
of the undercover agents and to facilitate the
lawful arrest or apprehension of suspected
violators of the law.
Oblanca is a member of the AntiOrganized Crime Division of NBI and it does not
abrogate his authority to apply for search
warrant. The applicant law enforcer must be a
member of a division that is assigned or related
to the subject crime or ofense before the
application of the search warrant may be acted
upon. A description of the place to be searched is
sufficient if the officer with the warrant can, with
reasonable efort, ascertain and identify the place
intended and distinguish it from other places in
the community. Any description or designation
known to the locality that points out to the place
to the exclusion of others, and on inquiry leads
the officers unerringly to it satisfies the
constitutional requirement. Moreover, in the
determination of whether a search warrant
describes the premises to be searched with
sufficient particularity, it has been held that the
executing officers prior knowledge as to the
place intended in the warrant is relevant. This
would seem to be especially true where the
executing officer is the affiant on whose affidavit
the warrant had been issued, and when he knows
that the judge who issued the warrant intended
the compound described in the affidavit.
WARRANTLESS ARREST
UMIL VS. RAMOS
This is a consolidation of about 8 or 9
cases. What are important are the cases of
Nazareno and Dural.
Dural Case:
On Jan. 31, 1998, there were two CAPCOM
soldiers who were killed by Dural in a shootout in

ACADCOM 2010; Contributors: Gene Geocaniga, Jarissa Guiani, Darlene Magabilen


TAU MU Page 49 of 179

Quezon City. On Feb. 1, Dural was arrested based


on information that he was a member of the
sparrow unit. He was arrested in a hospital while
undergoing treatment. Only a few hours had
lapsed.
Nazareno Case:
Nazareno was charged with the killing of a
person which took place on Dec. 14, 1998. On
Dec. 28, he was arrested. 14 days had lapsed.
The arrest of Nazareno was based on the
statement by an arrested suspect who implicated
Nazareno.

TAU MU
TAU MU
TAU MU
TAU MU
TAU MU
TAU MU
TAU MU

At about midnight of May 29, the accused


shot and stabbed and agent of the NARCOM to
death. At around 7pm of May 30, Manlulu was
arrested by policemen based on information
given by an eyewitness. The police did not have a
warrant of arrest.

TAU MU

PEOPLE VS. MANLULU

TAU MU

NOTE: There seems to be an inconsistency here


because the personal knowledge given here in
the Umil case is derived from investigation and
personal knowledge of probable cause. Note
also that it has been observed that personal
knowledge in non-political cases is usually
derived from personal sensed perception. Also,
the 14 days that passed in the Nazareno case still
considered that a crime was just freshly
committed.

TAU MU
TAU MU

Held: The court ruled the arrests as valid. In the


Dural case, the police got the identity of the killer
at about 5am. The arrest was made by 7am. The
court said that the crime had just been
committed , only 2 hours and 20m minutes had
passed after learning of the suspects identity.
The authorities acted immediately after having
the knowledge. In this case, the court ruled that
personal knowledge can be made up of
personal knowledge of probable cause. It was
not personal knowledge of facts anymore.
Personal knowledge of probable cause can be
made up of 2:
1. That there were reasonable grounds of
suspicion based on actual facts.
Basis:
i. There was a confidential information which
led to the arrest of the accused
ii. There was actually a crime that happened: 2
soldiers were killed in the shootout.
iii. There was actually a wounded person, who
happened to be the accused, being treated
in the hospital.
2. That the police must be acting in good faith
when conducting the arrest. There is always a
presumption that the police are performing
their duties regularly.

The Fraternal
Ateneo de Davao

TAU MU
TAU MU

ISSUE: In the 2 cases, have the crime just freshly


been committed? Did the officers have personal
knowledge of the facts to the crime to constitute
the arrest as valid? (Take note that there is a
significant diference in the time element)

KITY

TAU MU TAU MU TAU MU


TAU MU TAU MU TAU MU

CONSTITUTIONAL LAW II
Order of Saint Thomas More
Atty. Philip John Pojas/Atty. Rovyne G. Jumao-as, RN
University College of Law

ISSUE: Can the State argue that the crime has


just been committed, considering that 19 hours
had lapsed?
RULING: According to the court you cannot
arrest a person after 19 hours without a valid
warrant of arrest. There was also no personal
knowledge since the info was from a witness.
PEOPLE V. ALAVARIO
The victim, a house leper, was allegedly
raped by her employer on Jan 22,23,25,27 & 28,
1993. On the last day she called her sister by
phone who reported to the police. The police
went to the address and when the victim opened
the door and saw the police, she pointed the
accused as the rapist.
Held: The arrest is legal. The arresting officers
had personal knowledge of facts as culled from
the information supplied by the victim herself
who pointed to accused as the man who raped
her at the time of his arrest.
PEOPLE V. SINOC
At 6:00 oclock in the morning of Sept. 20,
1991, a pajero was stopped by armed men at
Claver, Surigao del Norte. The occupants were
driven to Barobo, Surigao del Norte, bound,
brought to a coconut grove and gunned down. At
7:00 oclock in the morning of Sept. 21, the police
of Monkayo, Davao, learned from an informer that
the pajero was brought to Bliss Housing Project in
the town. Proceeding to the place, they saw the
pajero and were told by an apartment owner that
the person driving it would return. By 10:30
oclock that morning, he was back carrying the
key to the pajero. The police arrested him.
Held: The arrest is legal. Even if warrantless, an
arrest may be made when an ofense has in fact
been committed and the arresting officer has
personal knowledge of facts indicating that the
person to be arrested has committed it. There is
no question that the police were aware that some
twelve [?] hours earlier an ofense had been
committed. They received a report about the
pajero and went to the place to recover it.
Accused was identified by the apartment owner,
he was moving to the pajero and had the key to it
at the time of his arrest so that his link to the
crime was palpable.
PEOPLE V. JAYSON
At 10:00 oclock in the evening of March
16, 1991, the police received a radio message
directing them to report to Bonifacio St., Davao
City at Ihaw-Ihaw where there was a shooting
incident. They rushed to the scene and there
they saw the victim sprawled on the ground.
Bystanders present pointed to the accused as the
one who shot the victim. He was walking away

ACADCOM 2010; Contributors: Gene Geocaniga, Jarissa Guiani, Darlene Magabilen


TAU MU Page 50 of 179

but was only about ten meters from Ihaw-Ihaw


when they arrested him and seized his .38 caliber
revolver.

CUDIA V. CA

TAU MU
TAU MU
TAU MU
TAU MU
TAU MU
TAU MU
TAU MU

On July 2, 1991, Rolito Go shot Maguan, a


student. As a result of the investigation, the
police immediately identified Go as the killer. The
police conducted a manhunt for the arrest of Go.
On July 8, Rolito Go, after reading in the
newspaper that there was manhunt for him went
to the police station while being accompanied by
his two lawyers. It turned out that the witnesses
to the shooting were also there. The witness

TAU MU

PEOPLE VS. GO

TAU MU

Held: Yes. It has been ruled that personal


knowledge of facts in arrests without a warrant
must be based upon probable cause, an actual
belief or reasonable grounds of suspicion. In this
case, there was an initial report to the police
concerning the robbery. A radio dispatch was
then given to the arresting officers, who
proceeded to the place to verify. They met up
with the complainants who initiated the report
about the robbery. Upon the officers invitation,
both mother and daughter boarded the mobile
unit to join the search. The accused was spotted
in the vicinity. Based on the reported statements
of the complainants, he was identified as a logical
suspect in the ofense committed.
Moreover, at the time the police called out
to accused, he was in the act of drawing his gun.
Actual possession of an unlicensed firearm, which
accused attempted to draw out, amounts to
committing an ofense in the presence of the
arresting officers.

TAU MU
TAU MU

While the police were on patrol one


evening aboard a vehicle, they received a radio
call to proceed to a place where a hold-up had
taken place. There, the victims said that the
hold-uppers had fled so that the police requested
them to board the patrol vehicles to facilitate the
search. The victims described the color of the
clothing of the 2 perpetrators. They later came
upon 2 men walking whom the victims identified
as the culprits. As one of the 2 attempted to pull
something from his waist, a policeman pointed
his firearm and frisked the suspect. A gun was
taken from him. Was the arrest of accused valid?

The Fraternal
Ateneo de Davao

TAU MU
TAU MU

Held: A warrantless arrest is valid when the


ofense has in fact just been committed and the
arresting officer has personal knowledge of facts
indicating that the person to be arrested has
committed it.
In this case, the policemen
summoned to the crime scene saw the victim.
Accused was pointed to them only moments after
the shooting and he had not gone very far. The
arresting officers thus acted on the basis of
personal knowledge of the death of the victim
and of facts indication that accused was the
assistant.

KITY

TAU MU TAU MU TAU MU


TAU MU TAU MU TAU MU

CONSTITUTIONAL LAW II
Order of Saint Thomas More
Atty. Philip John Pojas/Atty. Rovyne G. Jumao-as, RN
University College of Law

pointed to him as the killer which prompted the


police to arrest him on the spot.
ISSUE: Whether Rolito Go was validly arrested.
Held: Applying the principles of the law, the
crime has not just been committed since a few
days had already lapsed. The police also had no
personal knowledge of the fact that he committed
the crime. Their knowledge was just based on the
statement of the witnesses. To say that the crime
has just been committed, there must be
immediacy of the time which is from the
commission of the crime up to the arrest.
Personal knowledge of facts means that his
knowledge must be derived from his own
personal sensed perception, not knowledge
derived from investigation, nor from witnesses,
nor from any secondary sources.
PEOPLE VS GERENTE
At 2pm of April 30, 1995, Clarito Blace was
killed. At 4pm, the police learned of the killing
and after which they immediately went to the
hospital only to find out that the victim was
pronounce DOA. The police went to the crime
scene where they found a piece of wood with
blood stains and a hollow block. They were able
to interview an eyewitness to the incident who
identified the accused as the killer. The police
went to the house of the accused where they
found him sleeping, and they arrested him.
ISSUE: Was the arrest valid? Did the police have
personal knowledge of the facts?
Held: The court ruled that the arrest was valid.
The crime has just been freshly committed. The
police had personal knowledge because they saw
the victim dead in the hospital and they also saw
the instruments of the crime. (the wood and the
hollow block). The police had also interviewed the
eyewitness.
PEOPLE V. OLIVARES
In the morning of Dec. 26, the body of Mr.
Sy was found by his employees. On Dec. 28,
through the police focused on Arellano as a
suspect.
They visited him and he readily
admitted participation in the crime. They invited
him to the police station and he revealed the
place where the Sanyo cassettes and a
wristwatch owned by the victim were hidden.
The police detained Arellano. Was the arrest
valid?
Held:
No.
At the time accused was
apprehended, two days had already lapsed since
the discovery of the crime. Accused had not just
committed a criminal act. Neither was he caught
in flagrante delicto or had escaped from
confinement. Probably aware of the illegality of
the arrest, the arresting officers testified that
accused was merely invited to the police station.

ACADCOM 2010; Contributors: Gene Geocaniga, Jarissa Guiani, Darlene Magabilen


TAU MU Page 51 of 179

Such invitation, however, when construed in he


light of the circumstances is actually in the nature
of an arrest designed for the purpose of
conducting an interrogation. Mere invitation is
covered by the proscription on warrantless
arrests because it is intended for no other reason
than to conduct an investigation. Thus by virtue
of Sec. 3, Art. IV, of the Constitution, any
evidence obtained, including the things allegedly
stolen by accused which were taken by the police
from the place of the illegal arrest, cannot be
used in evidence.
PEOPLE V. DEL ROSARIO

TAU MU
TAU MU
TAU MU
TAU MU
TAU MU
TAU MU

Held: No. Neneth was not caught red-handed


during the buy-bust operation to give ground for
her arrest. Neither could it be justified under the
theory that the arresting officers had personal
knowledge that she had just committed an
ofense. Personal knowledge of facts in arrest
must be based on probable cause which means
actual belief or reasonable ground of suspicion.
The grounds of suspicion are reasonable when

TAU MU

In the course of the buy-bust operation,


the police handed P1,600.00 to Doria who agreed
to get the marijuana. When he returned an hour
later with the marijuana, the police arrested him.
He told the police that the P1,600.00 was with his
co-accused Neneth. The police went to the house
of Neneth and arrested her. Was the arrest legal?

TAU MU

PEOPLE V. DORIA
supra.

TAU MU

In this case, the arrest of the accused did


not comply with these requirements since the
arrest came a day after the consummation of the
crime. Likewise, the arresting officers had no
personal knowledge of facts indicating that the
person to be arrested had committed the ofense
since they were not actual witnesses to the
crime.

TAU MU
TAU MU

Held: No. Under Sec. 5, para (b), Rule 113, of


the Rules of Court, a warrantless arrest can be
efected when: 1) an ofense had just been
committed; and 2) the person making the arrest
has personal knowledge of facts indicating that
the person to be arrested had committed it.
Hence, there must be a large measure of
immediacy between the time the ofense was
committed and the time of the arrest. Aside from
the sense of immediacy, it is also mandatory that
the person making the arrest must have personal
knowledge of certain facts indicating that the
person to be taken into custody had committed
the crime.

The Fraternal
Ateneo de Davao

TAU MU
TAU MU

After the robbers snatched the bag of a


woman and killed her, they boarded the tricycle
of accused that 5:00 oclock in the evening.
Another tricycle driver recognized accused and
reported to the police. Before lunch the following
day, accused was arrested. Was the arrest valid?

KITY

TAU MU TAU MU TAU MU


TAU MU TAU MU TAU MU

CONSTITUTIONAL LAW II
Order of Saint Thomas More
Atty. Philip John Pojas/Atty. Rovyne G. Jumao-as, RN
University College of Law

the suspicion that the person to be arrested is


probably guilty of committing the ofense is
based on actual facts, i.e., supported by
circumstances sufficiently strong in themselves to
create the probable cause of guilt of the person
to be arrested. A reasonable suspicion therefore
must be founded on probable cause, coupled with
good faith on the part of the person making the
arrest.
In this case, Neneth was arrested on the
basis of the alleged identification by Doria. Doria
did not point to her as associate in the drug
business, but as the person with whom he left the
money.
Doria may have left the money in
Neneths house with or without the latters
knowledge. Save for Dorias word, the police had
no reasonable ground to believe that Neneth was
engaged in drug business.
PEOPLE V. BOLASA
Tipped by an anonymous caller that a man
and woman were repacking prohibited drugs at a
certain house in Valenzuela, Metro Manila, the
police went to the place. When they reached the
house accompanied by their informer, they
peeped inside through a window and saw a man
and a woman repacking suspected marijuana.
They entered the house, confiscated the items
and arrested accused. Are the search and seizure
and arrest valid?
Held: No. The arrest was illegal. First, the
arresting officers had no personal knowledge that
at the time of the arrest accused had just
committed, were committing or were about to
commit an ofense. Second, the arresting officers
had no personal knowledge that a crime was
committed, nor did they have any reasonable
ground to believe that accused committed it. The
search was also illegal. Since the arrest was
illegal, the search was not incident to a lawful
arrest. Likewise, it was not also a search of
evidence in plain view since the marijuana was
not inadvertently discovered. Now was it a case
of search of a moving vehicle, a consented
search, customs search, stop and frisk or one
done
under
exigent
and
emergency
circumstances.
PEOPLE V. POSADAS
On December 8, 1994, a student of the
University of the Philippines was killed in a
rumble between fraternities. On December 12,
on the basis of identification by 2 eyewitnesses,
agents of the National Bureau of Investigation
attempted to arrest several students.
The
University President as well as the counsel of the
students objected to the arrest on the ground
that there was no warrant of arrest. For this, they
were charged with violation of PD No. 1829 or for
obstructing
the
apprehension
of
criminal
ofenders. Are they liable?

ACADCOM 2010; Contributors: Gene Geocaniga, Jarissa Guiani, Darlene Magabilen


TAU MU Page 52 of 179

Held: No. The attempted arrest of the students


could not be validly made without a warrant.
First, the NBI agents tried to make the arrest 4
days after the commission of the crime when the
law states that for a valid warrantless arrest it is
required that an ofense has in fact just been
committed.
Second, they had no personal
knowledge of any fact that might indicate that
the suspects were guilty of a crime. What they
had was the supposed positive identification of
alleged eye-witnesses, which is insufficient to
justify the arrest without a warrant. Indeed, at
the time of the killing, these agents where
nowhere near the scene of the crime.

TAU MU
TAU MU
TAU MU
TAU MU
TAU MU
TAU MU

Nolasco and another companion was


arrested while aon board on the jeepney at
around 11:30AM at the intersection of Mayon &
Margal St. At around 12:00nn of the same day,
there was also a search conducted on the
premises of 239-B Mayon St., which was believed
to be the residence of the two. There was no
mention of the distance where the two incidents
happened. When they were arrested, there was
only one warrant of arrest for one person. The

TAU MU

NOLASCO VS. PANO

TAU MU

INCIDENT TO ARREST:

TAU MU

Held: No. For a warrantless arrest to be valid, it


is required that the ofender has just committed
an ofense and the arresting officer had personal
knowledge of facts indicating that the person
arrested has committed it. In this case, the first
requisite is satisfied because the arrest of
accused was efected shortly after the victim was
killed. But they had no probable cause to believe
that accused committed the crime.
Their
knowledge of facts and circumstances from which
they inferred that accused was guilty was based
entirely on what they had been told by others: to
wit, and report of the killing, the information that
he was at the caf with the victim, and the
description given by the tricycle driver. At the
time he was arrested, he was not doing anything
overtly criminal.
WARRANTLESS SEARCHES

TAU MU
TAU MU

After being informed that a person has


been shot, the police send a team to investigate.
They saw the victim slumped on his tricycle
parked near the road. After talking to a waitress
and a tricycle driver who gave them the
description of the person last seen with the
victim, the police went to the house of accused.
They knocked and when accused opened the
door, they noticed that he fitted the description.
Entering the house, they found a bloodied
Hanes T-shirt placed over a divider. When the
police picked it up, two spent .38 caliber shells
fell from it. They then brought him to the caf
where the waitress identified him as the person
she saw drinking beer with the victim. Was the
arrest valid?

The Fraternal
Ateneo de Davao

TAU MU
TAU MU

PEOPLE V. CUBCUBIN

KITY

TAU MU TAU MU TAU MU


TAU MU TAU MU TAU MU

CONSTITUTIONAL LAW II
Order of Saint Thomas More
Atty. Philip John Pojas/Atty. Rovyne G. Jumao-as, RN
University College of Law

operatives also had a search warrant but it turned


out that the warrant was void.
ISSUE:
whether
the
subsequent
search
conducted can be justified as an incident to a
lawful arrest by virtue of an arrest warrant.
Held: In the first case, the court ruled that the
warrantless
search
conducted
was
valid.
According to the court, it was incident to a lawful
arrest because the search was conducted within
the general vicinity of the place where the arrest
had taken place this is as to the element of
place. As to the element of time, the 30 min gap
was still allowable. (This was based on the old
doctrine of incident to a lawful arrest).
In the motion for reconsideration, the
court already declared this doctrine illegal. The
court ruled that a search incident to a lawful
arrest is only valid if it is conducted on: (1) the
person of the accused; and (2) the premises
within his immediate control that is to search
him of weapons and evidences of the crime. With
regards to time, the search should be
contemporaneous with the arrest. Meaning, the
search should be conducted simultaneously with
the arrest. (Immediately after or during)
PEOPLE V. LUA
Policemen conducted a buy-bust operation
to entrap accused who was then standing outside
the door of his house.
After the sale was
consummated, they arrested him. When asked
by the police, accused informed them that the
rest of the marijuana was inside his house.
Accompanied by the accused they went in and
found a brick dried marijuana. Is it admissible in
evidence?
Held: No. While the arrest was lawful, the
search made inside the house of the accused
became unlawful since the police operatives were
not armed with a search warrant. Such search
cannot fall under search made incidental to a
lawful arrest the same being limited to body
search and to the point within reach or control of
the person arrested, or that which may furnish
him with the means of committing violence or of
escaping.
PEOPLE VS. MUSA
The
police conducted
a buy-bust
operation, an instance where in a warrantless
arrest is considered as legal (Selling marijuana is
a valid ground for a policeman to make an arrest,
even without a warrant). The incident took place
in the sala of the accused. The policeman used
marked money during the operation. While they
were looking for the money, they found a plastic
bag in the kitchen which after opening, contained
marijuana.
ISSUE: Can the arresting officers raise the issue
that this was incident to a lawful arrest? Since the

ACADCOM 2010; Contributors: Gene Geocaniga, Jarissa Guiani, Darlene Magabilen


TAU MU Page 53 of 179

accused was a dealer of marijuana. Therefore the


marijuana found should be admissible as
evidence.
Held: The search was not incident to a lawful
arrest because the marijuana was not obtained in
the person of the accused nor in the place within
his immediate control. It would be valid if Musa
were in the kitchen when the bag was found.
ESPANO V. CA

was passing by, the policeman called for


assistance from his precinct so that a team was
sent to the area.
Subsequently, a trisikad
carrying accused passed by.
The policemen
aboard a vehicle overtook it and ordered one of
the accused to open the bag he was carrying.
The bag contained marijuana. Was the search
incident to a lawful arrest?
Held: No. Reliable information alone, absent
any overt act indicative of a felonious enterprise
in the presence or within the view of the arresting
officers, are not sufficient to constitute probable
cause that would justify an in flagrant delicto
arrest.
In this case, accused manifested no
outward indication that would justify their arrest.
In holding a bag on board a trisikad, accused
could not be said to be committing an ofense.
Apart from the fact that they were pointed to by
an informer, accused could not be the subject of
any suspicion, reasonable or otherwise. Since the
arrest was illegal, the search conducted on their
person is likewise illegal.
Consequently, the
marijuana seized could not be admitted in
evidence.
Pp vs. Molina there was an information that a
certain man was carrying shabu. Search was
made and found were illegal drugs. There was no
overt act, however, indicative of a felonious act.
HELD: The search was invalid since there was no
valid arrest.
CONSENTED SEARCH:
LOPEZ VS. COMMISSIONER
The NBI searched a room in Skylark Hotel,
which was occupied by Veloso. The NBI did not
possess a warrant at that time. When they
knocked and entered the room, there was a
woman inside who introduced herself as the wife
of Veloso. When the agents stated their purpose,
the woman gave them consent to search the
place. In efect, they confiscated the illegal
documents and firearms, which they used as
evidence in court against Veloso. Veloso
contested this because the woman in the room
had no right to give the consent for the
policeman to enter the room.

TAU MU

Held: The court ruled that the consent given by


the woman is sufficient to validate the search. It
would be awkward if the agents would require the
woman to produce her marriage license to prove
his identity. On the other hand, this has been
modified by the next case.

TAU MU

TAU MU

PEOPLE VS. DAMASO

TAU MU
TAU MU

After receiving an information that a drug


pusher, whom he put under surveillance earlier,

TAU MU

PEOPLE V. MOLINA

TAU MU

Held: No. For a search incident to a lawful arrest


to be valid, the search must have been conducted
at about the time of the arrest or immediately
thereafter and only at the place where the
suspect was arrested, or the premises or
surrounding under his immediate control. In this
case, accused was arrested in the act of
delivering shabu outside Unit 122. Moreover,
Unit 122 was not even his residence but that of
his girlfriend and that he was merely a sojourner
there. Hence, it can hardly be said that the inner
portion of the home constituted a permissible
area within his reach or immediate control to
justify a warrantless search.

TAU MU

As soon as accepted stepped and handed


the shabu to a buyer in a buy bust operation
just outside the door of Unit 122 of Roxas
Seafront Garden in Pasay City, the police arrested
him. The police then proceeded to search Unit
122 and at the second floor found plastic bags
containing 5,578.8 grams of shabu. Unit 122
belonged to the girlfriend of accused. Was the
search incident to a lawful arrest valid?

TAU MU

PEOPLE V. CHE CHUN TING

TAU MU
TAU MU

Held: Accused was caught in flagranti handling


something to a buyer. He was in the act of
committing an ofense so that his arrest was
lawful, and the 2 plastic bags of marijuana are
admissible in evidence, being fruits of the crime.
It was search incident to a lawful arrest.
However, the ten plastic bags of marijuana found
in his residence are not admissible because the
policemen were not armed with a warrant. At the
time of his arrest, his house was beyond his reach
and control.

The Fraternal
Ateneo de Davao

TAU MU
TAU MU

To confirm reports of drug pushing,


policemen stationed themselves in a street
corner in Metro Manila. They saw accused selling
something to another person. When the buyer
left, they approached accused, frisked him and
found 2 tea bags of marijuana. When asked if he
had more, he replied that he had them in his
house. They went to his house and found 10
more plastic tea bags of marijuana.

KITY

TAU MU TAU MU TAU MU


TAU MU TAU MU TAU MU

CONSTITUTIONAL LAW II
Order of Saint Thomas More
Atty. Philip John Pojas/Atty. Rovyne G. Jumao-as, RN
University College of Law

The PC conducted a search on the house


rented by the accused, who was believed to be a
member of the NPA. The PC had no search
warrant. That time, only the helper was present.
She gave them the consent to enter and search
the house and rooms wherein they obtained
firearms.
The
accused
challenged
the
admissibility of the evidences.

ACADCOM 2010; Contributors: Gene Geocaniga, Jarissa Guiani, Darlene Magabilen


TAU MU Page 54 of 179

Held: The court stated that the evidences are


inadmissible. Only the person whose rights will be
invaded can give the consent to a search. Only
Damaso can waive his rights. Consent can also be
given by a person authorized to do so. Such
authorization must be expressly given by the
owner to a caretaker.
VEROY VS. LAYAGUE

TAU MU
TAU MU
TAU MU
TAU MU

Based on a tip supplied by an informant,


the police conducted a surveillance of the
residence and saw 2 tall marijuana plants in the
backyard of accused in San Carlos City. On July
12, 1995, the police applied for a warrant in
Bacolod City but the judge told them that he had
no territorial jurisdiction over the matter. They

TAU MU

PEOPLE V. COMPACION

TAU MU

Held: There was no probable cause for the


search. The radio communications from Gen.
Nazareno pertained to cases of robbery and
holdups, not drug pushing. While the acts of the
accused might have annoyed the police, or
caused them to suspect that something was
amiss, it does not constitute probable cause to
conduct an extensive search.
However, as
accused expressly consented to the search, the
marijuana is admissible in evidence under the
principle of waiver.

TAU MU

The police received an order form Gen.


Nazareno to monitor strategic places in the City
and barangays of Manila. While thus patrolling
on a police car, they noticed a taxi with two
passengers. One was beside the driver while the
other was at the back seat. The two bowed their
heads and slouched on their seats refusing to
look at the police. Their suspicion aroused, the
police stopped the taxi. They asked permission
to search the vehicle and the occupants agreed.
In the course of search, the found marijuana
inside a plastic bag.

TAU MU

PEOPLE V. LACERNA

TAU MU

Held: The court held that the firearms were not


admissible as evidence since the warrant was for
the search of rebels, not firearms. There was a
prior justification for the intrusion. On the other
hand, the police did not come across the guns
inadvertently since they were inside the drawer.

TAU MU
TAU MU

ISSUE: Are the firearms admissible in court, even


if the warrant was for the search of rebels?

The Fraternal
Ateneo de Davao

TAU MU
TAU MU

Here was a report that there were rebels


in the house of Veroy. During that time, Veroy was
out of town. To conduct the search, the police
asked the permission from Veroy, through a long
distance call to search the house for rebels. In
efect of the search, they were able to recover
firearms, but there were no rebels.

KITY

TAU MU TAU MU TAU MU


TAU MU TAU MU TAU MU

CONSTITUTIONAL LAW II
Order of Saint Thomas More
Atty. Philip John Pojas/Atty. Rovyne G. Jumao-as, RN
University College of Law

went to San Carlos City but the judge there told


them to go back the following day, it was already
night-time. At about 1:30 in the morning of the
following day, without a search warrant, the
police arrived at the house of accused who let
them into the gate without objection. The police
found the marijuana. Was there a waiver of the
right against warrantless searches?
Held: No. While the right against unreasonable
searches and seizures may be waived expressly
or impliedly, such waiver must be made
voluntarily, knowingly and intelligently. The acts
of the accused in allowing the police to enter the
premises and his consequent silence during the
search should not be construed as voluntary
submission
or
and
implied
acquiescence
especially so when members of the raiding team
were numerous. His implied consent, if any,
could not have been more than mere passive
conformity given under coercive circumstances,
and is, thus, no consent at all. Consequently, his
lack of objection is not tantamount to waiver of a
constitutional right.
PLAIN VIEW:
PEOPLE VS. EVARISTO
The police while on patrol heard gunfires.
They saw Rosillo in the act of firing his gun in the
air. When they tried to approach him, Rosillo ran
away and entered a house. The police followed
him. When they came to the house, they asked
Evaristo, the owner, about the suspects
whereabouts. Evaristo replied that Rosillo was no
longer in the house. Anyway, the police were able
to get Evaristos consent to conduct a search of
the house. It turned out that the house was full of
firearms.
ISSUE: Are the evidences recovered admissible
in court?
Held: The court ruled that they are. The police
had prior justification for the intrusion since they
were given consent by Evaristo. They also came
across the guns inadvertently, and the illegality
of the objects are readily apparent.
PEOPLE V. QUE MING KHAN
The police received an information that a
van with plate No. UPN 595 which was being used
in the transport of shabu was seen at the vicinity
of barangay Holy Spirit, Quezon City.
Three
teams were dispatched to the area, one of which
spotted the van, which hit a 7 year old boy. The
van stopped and the owner got of to bring the
boy to a hospital. A police officer approached the
van to apprehend the driver for reckless
imprudence. As he stood near the van, he saw
through the lightly tinted window several sacks.
One sack was open and he noticed white plastic
bags containing crystalline substance, which
turned out to be shabu. Is the shabu admissible
in evidence?

ACADCOM 2010; Contributors: Gene Geocaniga, Jarissa Guiani, Darlene Magabilen


TAU MU Page 55 of 179

Held: Yes. Objects falling in plain view of an


officer who has a right to be in a position to have
that view are subject to seizure even without a
search warrant. The plain view doctrine applies
when (a) the law enforcement officer has a prior
justification for an intrusion or is in a position
form which he can view a particular area; (b) the
discovery of the evidence in plain view is
inadvertent; and (c) the illegality of the object is
immediately apparent. Clearly, the prohibited
substance was in plain view of the police officer
who was in a position to be near the van at the
time. The substance is therefore not the product
of an illegal search.

TAU MU
TAU MU
TAU MU
TAU MU

Held: No. Seizure is limited to those items


particularly described in the warrant . In this
case, the firearm was not found inadvertently and
in plain view. It was found as a result of a
meticulous search in the kitchen. This firearm
was not mentioned in the search warrant. Hence,
the seizure was illegal. True that as an exception,
the police may seize without a warrant illegally
possessed
firearm
or
any
contraband
inadvertently found in plain view. However, the
seizure in plain view applies only where the police
officer is not searching for evidence against the
accused, but inadvertently comes across an
incriminating object, which is not the case here.

TAU MU

The judge issued a search warrant


authorizing the search and seizure of a .45 caliber
pistol at the residence of accused. In the course
of the search, during which they found the gun
which turned out to be licensed, the policemen
also seized a .22 caliber revolver, not described in
the warrant, from his daughters bedroom. Is
the .22 caliber revolver admissible in evidence?

TAU MU

DEL ROSARIO V. PEOPLE

TAU MU

NOTE: The possession of parts of a gun will also


hold a person liable under the crime of illegal
possession of firearms.

TAU MU

Held: Basing on the Musa case, they should be


held as admissible in court. But according to the
court, they are not. The authority given by the
court was only to obtain the shabu and
paraphernalia. They had no authority to get the
firearms
and
ammunitions.
The
warrant
specifically defines the object to be searched. It
would seem that there should be some
modification in the requisites of search in plain
view.

TAU MU

ISSUE: Are these admissible in court?

TAU MU
TAU MU

The policemen had a search warrant for


the seizure of shabu and their paraphernalia.
When the police entered the house, they found a
gun and a cup filled with bullets on top of the
television set.

The Fraternal
Ateneo de Davao

TAU MU
TAU MU

PEOPLE VS. DEL ROSARIO

KITY

TAU MU TAU MU TAU MU


TAU MU TAU MU TAU MU

CONSTITUTIONAL LAW II
Order of Saint Thomas More
Atty. Philip John Pojas/Atty. Rovyne G. Jumao-as, RN
University College of Law

PEOPLE V. SALANGUIT
The Search Warrant issued by the judge
ordered
the
seizure
of
UNDETERMINED
QUANTITY
OF
SHABU
AND
DRUG
PARAPHERNALIA. During the search, however, in
addition to the shabu, the police found and seized
2 bricks of dried marijuana leaves weighing 1,254
grams. Is the marijuana admissible in evidence?
Held: No. While police officers with a search
warrant may seize contraband items in plain view
even if not described in the warrant, the plain
view doctrine cannot apply here. The doctrine
requires (1) prior justification for the intrusion; (2)
inadvertent discovery; and (3) immediate
apparent illegality. Because the police knew that
the shabu was in the cabinet, it is reasonable to
assume that they found it first. Once the valid
portion of the warrant has been execute, the
plain view doctrine can no longer serve as valid
basis for admitting the other items subsequently
found. In addition, the marijuana bricks were
wrapped in newsprint. Its illegality, therefore,
was not immediately apparent to justify its
seizure.
PEOPLE V. DORIA,
In the course of the buy bust operation,
the police handed P1,600.00 to Doria who
proceeded to get the marijuana.
When he
returned an hour later with the marijuana, the
police arrested him. He told the police that he
left the buy bust money to Neneth and the police
went to the latters house. Standing by the door,
they saw a carton box under the dining table.
The box was partially open and revealed
something wrapped in plastic. As the plastic
looked similar to the one they seized from Doria,
the police opened it and found marijuana. Was
the marijuana seized in plain view?
Held: No. For the plain view doctrine to apply,
the following requisites must concur:
1) the
officer has prior justification for the intrusion or is
in a position from which he can view a particular
area; 2) the discovery of the evidence in plain
view must be inadvertent; and 3) it is
immediately apparent to the officer that the item
in contraband or otherwise subject to seizure. In
this case, it was not immediately apparent to the
police that the box contained marijuana. The
plastic wrapper was not colorless and transparent
as to manifest its contents to the viewer. Each of
ten bricks of marijuana in the box was individually
wrapped in old newspapers and place inside
plastic bags --- white, pink or blue in color. The
seizure of the marijuana therefore violated the
Constitution.
PEOPLE V. ELAMPARO
After the buy bust the police arrest
Spencer, who was able to free himself and run

ACADCOM 2010; Contributors: Gene Geocaniga, Jarissa Guiani, Darlene Magabilen


TAU MU Page 56 of 179

inside the house of appellant. The police pursued


Spence inside the house and found appellant
repacking 5 bricks of marijuana on top of a table.
They arrested appellant and confiscated the
marijuana.
Is the marijuana admissible in
evidence?

TAU MU
TAU MU
TAU MU
TAU MU
TAU MU
TAU MU

While a policeman was urinating at a


fence behind a bamboo school, he saw a garden
of about 70 square meters with corn and camote
tops. Concealed by the corn, however, were
marijuana plants. The policeman asked from a
storekeeper nearby as to who owned the garden
and he was told that it was accused. He reported
to the Chief of Police who dispatched a team of
policemen an hour later. Going straight to the
house of accused, they asked him to bring the
team to the backyard garden which was just 5

TAU MU

PEOPLE V. PASUDAG

TAU MU

Held: No. Considering that the informant had


revealed the name of accused as well as the
place were the marijuana was planted and the
police had at least one day to obtain a search
warrant, they had no reason not to obtain one.
The plain view doctrine cannot apply. The seizure
of evidence in plain view applies where the police
inadvertently came across the object. In this
case, the police team was dispatch precisely to
search the prohibited flora. It must also be noted
that they first had to look around the area
before they could spot the illegal plants.
Patently, the seized marijuana plants were not
immediately apparent. Thus, the plain view
doctrine cannot be made to apply.

TAU MU

At about 10:15 oclock in the morning, the


police were informed of the presence of a
marijuana plantation.
At 5:00 oclock in the
morning the following day, a police team,
accompanied by the informer, left for the site
were the marijuana plants were grown. After a 3hour uphill trek from the nearest barangay road,
they arrived at the place and found accused in his
nipa hut.
They looked around the unfenced
kaingin and saw 7 five-foot high marijuana
plants planted approximately 25 meters from his
hut.
Are the marijuana plants admissible in
evidence?

TAU MU
TAU MU

PEOPLE V. VALDEZ

The Fraternal
Ateneo de Davao

TAU MU
TAU MU

Held: Yes. The case falls under the plain view


doctrine. Under the said doctrine, the following
requisites must concur; 1) the law enforcer has a
prior justification for the intrusion; 2) the
discovery of the evidence in plain view is
inadvertent, and 3) the illegality of the object is
immediately apparent.
In this case, the
policemen were justified in running after Spencer
and entering the house without a search warrant
for they were not in the heels of a fleeing
criminal. They also caught appellant in flagrante
delicto repacking the marijuana bricks which
were in full view on top of the table.

KITY

TAU MU TAU MU TAU MU


TAU MU TAU MU TAU MU

CONSTITUTIONAL LAW II
Order of Saint Thomas More
Atty. Philip John Pojas/Atty. Rovyne G. Jumao-as, RN
University College of Law

meters away. The police team uprooted the 7


fully grown marijuana plants. Was the search and
seizure valid?
Held: No. The marijuana plants were seized in
violation of the rights against unreasonable
searches and seizures. The policeman had ample
time to secure a warrant. He knew who owned
the garden.
He was acquainted with the
marijuana plants and immediately recognized
them. Time was not of the essence to uproot and
confiscate the plants. They were three months
old and there was no sufficient reason to believe
that they would be uprooted on that same day.
PEOPLE VS. MUSA
This
was
a
buy-bust
operation.
Information was obtained that a certain Musa was
selling marijuana. A test-buy was conducted and
true enough marijuana was brought the following
day. A buy-bust was arranged. Sgt Ani posed as
a prospective buyer.
After Musa gave him
marijuana and after getting the marked P20.00
bill, Musa was arrested. In the living room, they
could not find the P20.00 bill.
Hence, they
proceeded to the other parts of the house and
found in the kitchen a plastic bag which when
opened contained dried marijuana leaves.
Held: No valid search. Although it was a valid
arrest, the subsequent search in the kitchen was
not. It cannot be considered as a surrounding
within his immediate control. The prosecutions
contention that it was evidence in plain view is
untenable because the evidence of illegality is
not apparent. They have to open the plastic bag
to know what was contained therein.
The
marijuana there in the plastic bag is inadmissible
in evidence.
PEOPLE VS. MUSA
(as discussed by Ms. Jumao-as under plain
view)
- (People vs. Musa, supra)
Held: The court ruled that this was not a search
in plain view. The police did not come across the
evidences
inadvertently.
There
was
prior
justification for the intrusion but it is limited only
within the immediate vicinity. Also, the illegality
was not readily apparent since it was inside a
plastic bag.
STOP AND FRISK:
POSADAS VS. CA
Posadas was carrying a buri bag. When he
was stopped by the police, he tried to run away
but he failed to escape. The police found guns
and ammunitions inside his bag. The police
reasoned that he was acting suspiciously and that
he was acting suspiciously and that is why they
searched him.

ACADCOM 2010; Contributors: Gene Geocaniga, Jarissa Guiani, Darlene Magabilen


TAU MU Page 57 of 179

Held: The court ruled that the search was valid.


There was probable cause(acting suspiciously) for
the police to conduct the search. The suspect was
looking side-to-side and he tried to flee when the
police stopped him.
PEOPLE VS. MENGOTE

TAU MU
TAU MU
TAU MU
TAU MU
TAU MU
TAU MU
TAU MU

Held: There was a valid stop-and-frisk search.


Where a police officer observes an unusual
conduct which leads him reasonably to conclude
in the light of his experience that criminal activity
may be afoot and that the person with whom he
is dealing may be armed and presently
dangerous, where in the course of investigating
this behavior he identified himself as a policeman
and makes reasonable inquiries, and where
nothing in the initial stage of the encounter
serves to dispel his reasonable fear for his own or
others safety, he is entitled for the protection of
himself and others in the area to conduct a
carefully limited search of the outer clothing of
such persons in an attempt to discover weapons
which might be used to assault him. [Terry b.
Ohio]. In this case, the policemen, based on the

TAU MU

At 2:00 oclock in the afternoon,


policemen form Caloocan were conducting a
surveillance in response to a report that drug
addicts were roaming in front of Caloocan
Cemetery. They chanced upon a male person
observed to have reddish eyes and walking in a
swaying manner as though he was high on drugs.
When he tried to avoid the policemen, they
approached him and introduced themselves.
They asked him what he was holding but he tried
to resist. He later showed the wallet he as
holding and the police found crushed marijuana
residue inside.

TAU MU

MANALILI VS. CA

TAU MU
TAU MU

Held: The court ruled that the firearm was not


admissible as evidence. The search was not valid
since there was no probable cause. When the
court tried to look at the facts how the police
arrived to the conclusion that the suspects were
acting suspiciously, they found it insufficient.
Looking side-to-side or holding ones abdomen
does
not
necessarily
constitute
acting
suspiciously.
As to the illegal possession of firearm, the
court ruled that the police had no personal
knowledge that the suspect was actually carrying
a gun at that time. Subsequent discovery of the
firearm will not cure the illegality of the search.

The Fraternal
Ateneo de Davao

TAU MU
TAU MU

The police received a phone call that there


were three suspiciously looking men at a street
corner in Tondo. The police found 2 men looking
side-to-side, and one had his hand on his
abdomen. When the police approached them, the
two tried to flee. The police caught up with them
and they were searched. The police found a .38
caliber pistol with 6 live ammunitions.

KITY

TAU MU TAU MU TAU MU


TAU MU TAU MU TAU MU

CONSTITUTIONAL LAW II
Order of Saint Thomas More
Atty. Philip John Pojas/Atty. Rovyne G. Jumao-as, RN
University College of Law

circumstances, had sufficient reason to stop


accused and investigate if he was high on drugs.
PEOPLE V. ENCINADA
At 4:00 oclock in the afternoon of May 20,
1992, the police received an intelligence report
that appellant was arriving in Surigao City aboard
M/V Sweet Pearly in the morning of May 21. The
following day, as accused was disembarking, the
police watched him. He walked briskly down the
gangplank carrying 2 plastic baby chairs. He took
a motorella which was stopped by the police who
then searched the chairs where they found 88
grams of marijuana leaves.
Held: the search was illegal. Accused was not
committing a crime in the presence of the
policemen. He was not acting suspiciously when
he disembarked from the ship or while he rode
the motorella. Moreover, the lawmen did not
have personal knowledge of facts indicating that
the person to be arrested had committed an
ofense.
The search cannot be said to be
incidental to a lawful arrest. Raw intelligence
information is not a sufficient ground for a
warrantless arrest.
Nor can the silence of
accused be taken as consent to the warrantless
search. The police cannot also claim that there
was no time to obtain a warrant since
Administrative Circular No. 19, Series of 1987,
allows application for search warrants even on
Sundays and holidays or after office hours.
PEOPLE V. CHUA HO SAN
In response to reports of smuggling, the
Chief of Police of Bacnotan, La Union, ordered the
patrolling of coastlines. One past midnight, they
were informed of an unfamiliar speedboat about
to dock. They came to the place in time to see a
male passenger disembark carrying a strawbag.
They accosted him when he changed direction
and tried to run. The man turned out to be a
Taiwanese who spoke no English and his bag
contained 28.7 kilos of shabu. Was the search
lawful?
Held: No. There was no probable cause for the
search. There was no report that a foreigner
would disembark bearing prohibited drug on that
date. The fact that the vessel that ferried him
looked unusual does not suggest that he was
perpetrating an ofense. True, accused entered
Philippine territory without a visa but that was not
obvious to the police. What was clear was that
accused was alighting from a speedboat, walking
casually towards the road and carrying a
strawbag. These acts did not convey that he
illegally entered Philippine shores.
Nor were
these manifestations of an on-going felonious
activity.
MOVING VEHICLES:
VALMONTE VS. DE VILLA

ACADCOM 2010; Contributors: Gene Geocaniga, Jarissa Guiani, Darlene Magabilen


TAU MU Page 58 of 179

PEOPLE VS. MALMSTEADT

TAU MU
TAU MU
TAU MU

There were several people including Exala


riding a jeepney and they were stopped in a
checkpoint. While the police were conducting a
routine check and asking questions, they noticed
a black leather bag with budging sides. An officer
asked what the contents of the bag were but
nobody responded. The police noticed the
demeanor of the accused hanged. They become
suspiciously quiet and nervous as if they were

TAU MU

PEOPLE VS. EXALA

Held: According to the court, the evidence is


admissible. There was probable cause since the
accused were acting suspiciously. They did not
seem to be acting normally which gave the police
a reason to an exclusive search. Likewise, there
were no protests and objections regarding the
search. It was an implied waiver of the right
against unreasonable searches and seizures.
ANIAG VS. COMELEC

Held:
In searches made of moving vehicle,
extensive search can only be done if there is
existence of probable cause. In this case, the SC
ruled that there was no probable cause to
conduct a more extensive search. The driver was
not acting suspiciously.
He did not become
fidgety. The guns were even hidden. They could
not be seen by mere visual search. They have to
open the trunk and the bag for the guns to be
apparent.
PEOPLE VS. CLAUDIO

TAU MU

Held: According to the court, the evidence


is admissible. There was probable cause based on
the report which also gave a description of the
suspect.

TAU MU

The NARCOM received a report that a


person is transporting marijuana. The report
described the person as a woman, 52 in height,
about 25 years of age, with short curly hair. The
agents put up check points and conducted
extensive searches on the buses. In one of the
buses, they saw a woman matching the
description sitting quietly at the rear, with a bag
on her lap. When they searched the bag, they
found marijuana.

TAU MU

PEOPLE VS. BAGISTA

COMELEC
issued
resolutions
for
implementation of the gun ban during the
election period in preparation for the coming
elections as well as the authority for the
COMELEC
to
establish
checkpoints.
Consequently, the Congress sergeant-at-arms
ordered the return of 2 firearms issued to Aniag.
In compliance, Aniag ordered his driver to pick up
the guns at his residence and deliver the same to
the sergeant-at-arms.
PNP established a
checkpoint 30 meters from the Batasan Complex,
so when the car passed by, it was flagged down.
The car was inspected and extensively searched.
The trunk was opened and a bag was found
containing guns neatly packed in their cases. The
driver was apprehended and a case was filed for
violation of the resolution and PD 1866.

TAU MU

Held: The court held that the search was legal.


1.) There was probable cause based on the report
2.) The accused acted suspiciously since he
refused to give his passport.
3.) There was a bulge on his waist.
The search was also valid since this was a
search of moving vehicles. Requiring a warrant
gives the suspect time to flee.

trying to connect something. When the bag was


opened, the police found it to contain marijuana.

TAU MU

ISSUE: Is the marijuana admissible in court?

Ateneo de Davao

TAU MU
TAU MU

The police received a report that a


foreigner from the Sagada will be riding a bus
carrying marijuana. The police immediately put
up check points and inspected the buses coming
from that direction. During the checks, they saw a
Caucasian matching the description. The police
noticed a bulge on his waist. When they asked for
his passport, he did not comply. They then
ordered him to turn of the pouch on his waist. The
police found it to contain marijuana.

The Fraternal

TAU MU
TAU MU

The government put up check points in


Manila because some problems in peace and
order. Valmonte sought to dismantle these check
points for being unconstitutional. The police
conducted arbitrary searches on the vehicles and
the passengers without a valid warrant.
Held: The court upheld the constitutionality of
the search because removing it will hamper the
police from performing some of their functions.

KITY

TAU MU TAU MU TAU MU


TAU MU TAU MU TAU MU

CONSTITUTIONAL LAW II
Order of Saint Thomas More
Atty. Philip John Pojas/Atty. Rovyne G. Jumao-as, RN
University College of Law

There was a police officer who aboard a


bus bound for Olongapo. While he was riding,
there was a woman who boarded the bus and
seated in front of the officer. However, the
woman placed her bag at the rear seat of the bus
she sat. This act of the woman raised the
suspicion of the policeman. At the next station,
when the woman went down to the bus maybe
just to relieve herself in a CR. The policeman went
to the bag and placed his hand inside the bag.
When he smelled his fingers, it smelled like
marijuana. He did not immediately arrest the
woman. At the next bus stop, the policeman
unboarded the bus at the same time he also
invited the woman to the police station. In short
he arrested the woman. In the station they
searched the bag and they found marijuana.
ISSUE: Is the marijuana admissible based on the
facts presented?

ACADCOM 2010; Contributors: Gene Geocaniga, Jarissa Guiani, Darlene Magabilen


TAU MU Page 59 of 179

Held: In search of moving vehicle, you can only


conduct extensive search if there is probable
cause. According to the S, the evidence is
admissible since the search was done in the
police station. The police had reason to do the
search and arrest because he already knew that
she is already committing a crime, she was
already carrying marijuana. The court ruled that it
was a search incident to a lawful arrest.
PEOPLE V. QUE

TAU MU
TAU MU
TAU MU
TAU MU
TAU MU

PEOPLE V. GONZALES

TAU MU

Held: Yes. The police officer, faced with an on


the spot information from the asset, had to
respond quickly to the call of duty. There was no
time to obtain a warrant. It must be noted that
the target of the pursuit was jut the thin Ilocano
person with a green bag and no other.
Evidently, there was definite information of the
identity of the person engaged in transporting
marijuana at a particular time and place. The
police officer had probable cause to stop the
buses in view of the information. He likewise had
probable cause to search accused since he fitted
the description. As the warrantless search was
valid, the marijuana taken is admissible.

TAU MU

While the policeman was waiting for a


ride, he was approached by an informer and told
that an Ilocano person was going to transport
marijuana. The informer described the suspect
as thin and carrying a green bag. At a certain
place, the policeman and the asset stopped two
buses and on the second, the policeman saw
accused who matched the description. When
searched, the bag accused carried yielded
marijuana. Is the marijuana admissible?

TAU MU

PEOPLE V. VALDEZ

TAU MU
TAU MU

Held: Yes. There was probable cause to justify


the extensive search of appellants truck even
without a warrant.
There was a reliable
information which included the plate number of
the truck. The truck the police apprehended
matched the description of the truck given by the
informant. Accused admitted to the police that
there were sawn lumber. Finally, accused could
not present the lumbers supporting documents
when asked by the police.

The Fraternal
Ateneo de Davao

TAU MU
TAU MU

The Task Force on Illegal Logging received


an information that a ten-wheeler truck bearing
plate number PAD-548 loaded with illegality cut
lumber would pass through. Two weeks later,
when policemen went on patrol, they saw the
truck which they stopped.
The truck owner
aboard admitted that the truck was carrying sawn
lumber inserted in between coconut slabs. An
examination of the cargo confirmed his
statement. Was there a valid search?

KITY

TAU MU TAU MU TAU MU


TAU MU TAU MU TAU MU

CONSTITUTIONAL LAW II
Order of Saint Thomas More
Atty. Philip John Pojas/Atty. Rovyne G. Jumao-as, RN
University College of Law

The police received an information tat a


woman with long hair, wearing maong pants
and jacket, and Ray Ban sunglasses would be
transporting marijuana along the national
highway.
According to the information, the
woman would bring a black traveling bag and
would ride a trisikad.
Based on this
information, policemen were set to the area.
They found the woman, ordered her to open the
black bag, which she refused. The police opened
it and found marijuana.
Is it admissible
evidence?
Held: Yes. The arrest was legal and the search of
her bag justified. The police had no time to
obtain a warrant since they were tipped of only
in the evening and the contraband would be
transported early in the morning of the following
day. The law enforcers had a definite target of
the arrest. There was a description of the identity
of the person engaged in transporting prohibited
drugs at a particular place and time. Accordingly,
when they saw accused who fitted the description
they had probable cause to apprehend her.
SEARCH IN THE
CUSTOM LAWS:

ENFORCEMENT

OF

PAPA VS. MAGO


Customs Authorities received information
that there were misdeclared or undervalued
goods which were to be released in the port of
Manila. This prompted to put up surveillance.
They saw two trucks presumably full of goods and
which they immediately pursued. Without the
benefit of a search warrant, they seized the items
loaded in the truck.
ISSUE: Is the search permissible? Is it legal?
Held: According to the court, it was a search
under Tarifs And Customs Code and it can be
done without a search warrant provided there is
probable cause. The court provided several
requisites where you can conduct warrantless
searches under the Tarifs and Customs Code.
HIZON V. CA
Members of the Maritime Command, in
response to reports that illegal fishing activities
were taking place in the coastal waters of
Palawan, boarded F/B Robinson, a fishing vessel.
The following day, after having placed the vessel
under guar, they obtained five fish samples from
the catch of the fishermen. A laboratory
examination of the sample showed that it was
positive of sodium cyanide, a poisonous
substance. Was the warrantless search lawful?

TAU MU

Held: Search and seizure without search warrant


of vessels for violation of custom laws have been
traditionally regarded as an exception to the
constitutional requirement of a search warrant. It
is rooted on the recognition that a vessel can be
quickly moved out of the locality or jurisdiction in

ACADCOM 2010; Contributors: Gene Geocaniga, Jarissa Guiani, Darlene Magabilen


TAU MU Page 60 of 179

which the search warrant of vessels before their


search and seizure can be validly afected. The
same exception ought to apply to seizure of
fishing vessels and boats breaching fishery laws.
These vessels are normally powered by highspeed motors that enable them to elude arresting
ships of government authorities enforcing fishery
laws.
BUREAU OF CUSTOMS V. OGARIO

TAU MU
TAU MU
TAU MU
TAU MU
TAU MU

Held: According to the court, they will allow


searches without the warrant because it was an
abnormal time and the police do not have the
opportunity to secure a search warrant. In this
case, the judge even testified that his court was
not always open because of the tension existing

TAU MU

ISSUE: Whether the evidence obtained is


admissible considering the search was without a
warrant.

TAU MU

Accused
was
charged
with
illegal
possession of firearms which was obtained from a
warrantless search. The search was conducted at
the height of the coup attempt in 1989. The
authorities were conducting searches and
surveillance as counter-measures to stop the
activities of the rebel forces in one of the
searches, the officers were able to obtain firearms
from the accused.

TAU MU

PEOPLE VS. DE GRACIA

TAU MU

SEARCH DURING EXIGENCY:

TAU MU
TAU MU

Held: No. Regional Trial Court are devoid of any


competence to pass upon the validity of seizure
and forfeiture proceedings in the Bureau of
Customs. The Collector of Customs sitting in a
seizure and forfeiture proceedings has the
exclusive jurisdiction to hear and determine all
questions touching on the seizure of dutiable
goods. The question on whether probable cause
exists for the seizure of the rice is not for the RTC
to determine. The customs authority do not have
to prove to the satisfaction of the court that the
articles on board a vessel were imported from
abroad before they may exercise the power to
efect customs searches, seizures or arrest.
Actions of the collector of customs are appealable
to the Commissioner of Customs, whose decision
in turn is subject to the exclusive appellate
jurisdiction of the Court of Tax Appeals.

The Fraternal
Ateneo de Davao

TAU MU
TAU MU

The District Collector of Customs of Cebu


issued a Warrant of Seizure and Detention of
25,000 sacks of rice on the basis of a report that
the rice had been illegally imported. Forfeiture
proceedings were subsequently started in the
customs office in Cebu.
Upon complaint,
however, the Regional Trial Court of Cebu ordered
the return of he rice to the consignee. Does the
Regional Trial Court have the power to quash the
warrant of seizure issued by the collector of
customs on the ground of lack of probable cause
in its issuance?

KITY

TAU MU TAU MU TAU MU


TAU MU TAU MU TAU MU

CONSTITUTIONAL LAW II
Order of Saint Thomas More
Atty. Philip John Pojas/Atty. Rovyne G. Jumao-as, RN
University College of Law

at that time. But this is just a very exceptional


circumstance.

SEARCH BY PRIVATE PERSONS


PEOPLE VS. MARTI
Petitioner here was trying to ship several
boxes outside of the country through a cargo
forwarder.
The wife of the cargo forwarder
received the boxes. As a matter of procedure,
she wanted to inspect the boxes.
Petitioner
informed her that the boxes contained gloves,
books and cigars so the wife did not insist on
inspecting the boxes.
Before the box were
actually shipped as a matter of procedure, the
husband conducted a final inspection. He noticed
that one of the boxes emitted an unusual smell.
So, he got something inside wrapped in a plastic
and had it examined at the NBI laboratory. It
turned out positive for prohibited drugs. The NBI
agents went with the forwarder and he opened
the boxes. Found were marijuana. Petitioner
seeks that the evidence be excluded.
Held: The search is valid. Search by private
individual is allowed provided it is without the
interference of the agents of the State. Since it
was a Standard Operating Procedure (SOP) for
the proprietor to inspect objects to be shipped,
his search for the boxes was not illegal and since
he was the one who opened the boxes and who
showed the contents to the agents, the agents
did not interfere in the search.
EXCLUSIONARY RULE
STONEHILL VS. DIOKNO
The court issued several warrants, 42 in
all, to search the houses and business addresses
of Stonehill and of the members of his board.
Stonehill was known to engage in various illegal
activities. Unfortunately, all the warrants were
declared null and void for the reason that they
were general warrants. The warrant stated that
the objects to seize were to be used as evidence
for violations of the Tarif Laws, Internal Revenue
Laws, Central Bank Laws, RPC, and etc.
Held: The court ruled that evidences obtained
from Stonehills residences cannot be held
admissible in any proceeding against him. The
court however, declared that evidences obtained
from his offices can be used against him. The
reason for this is section 2 can only be invoked by
the person whose rights have been invaded.
Evidences obtained from the corporation, being a
diferent person and entity from Stonehill, can be
held admissible in court.
Section 3. (1) The privacy of communication and
correspondence shall be inviolable except upon

ACADCOM 2010; Contributors: Gene Geocaniga, Jarissa Guiani, Darlene Magabilen


TAU MU Page 61 of 179

PRIVACY
OF
COMMUNICATION
AND
CORRESPONDENCE
rules out eavesdropping on private conversations
through the use of electronic gadgets and thus
covers w/ its protection even intangible things --so tangible and intangible things
EXCEPTIONS:
upon lawful order of the court

Prohibited acts:
1. to secretly overhear;
2. intercept
3. communicate any part thereof.
Who are prohibited: Any person not being authorized
by ALL the parties to the communication, including the
parties to the communication..

1. The law does not distinguish between a party to the


private communication or a third person. Hence, both
a party and a third person could be held liable under
R.A. 4200 if they commit any of the prohibited acts
under R.A. 4200 (Ramirez v. Ca)
2. The use of a telephone extension to overhear a
private conversation is not a violation of R.A. 4200
because it is not similar to any of the prohibited
devices under the law. Also, a telephone extension is
not purposely installed for the purpose of secretly
intercepting or recording private communication.

TAU MU
TAU MU
TAU MU
TAU MU
TAU MU

Types of communication protected: Tangible and


Intangible objects

Objects covered: private communication or spoken


word

TAU MU

EXCLUSIONARY RULE
evidence obtained in violation of this section shall
be inadmissible for any purpose in any
proceeding

Indeed the documents and papers in question are


inadmissible in evidence. The constitutional
injunction declaring the privacy of communication
and correspondence [to be] inviolable is no less
applicable simply because it is the wife (who thinks
herself aggrieved by her husbands infidelity) who
is the party against whom the constitutional
provision is to be enforced. The only exception to
the prohibition in the Constitution is if there is a
lawful order [from a] court or when public safety
or order requires otherwise, as prescribed by law.
Any violation of this provision renders the evidence
obtained inadmissible for any purpose in any
proceeding.
RA 4200 Anti-Wiretapping Act

TAU MU

Letters and sealed packages in the mails may be


examined only as to their external appearance and
weight and may not be opened except in accordance
with the constitutional requirements of lawful search
and seizures.
Listening to phone conversation via an extension
phone is not wire tapping. It is not protected and
prohibited by law.

Zulueta vs. CA This is more of an exceptional ruling


because the bill of rights is being invoked against a
private individual.

TAU MU

TAU MU
TAU MU

INTRUSION WITHOUT COURT ORDER

the intrusion may be ordered by an executive


officer, under authority of law (there must be a law w/c
justifies the act of the executive officer), if he believes
that public order or safety so requires.

PUBLIC ORDER and SAFETY concern the


security of human lives. Liberty and property against
the activities of invaders, insurrectionists and rebels.

this should be resorted to only in extreme


cases

The Fraternal
Ateneo de Davao

TAU MU
TAU MU

INTRUSION UPON LAWFUL COURT ORDER

there must be probable cause

if the subject of the intrusion is WRITTEN


CORRESPONDENCE the same must be particularly
described as to its:
a. contents,
b. the
persons
whose
communication
is
to
be
intercepted,
c. the identity of the ofense/s
sought to be prevented, and
d. the period of the authorized
intrusion

if intrusion is done through WIRE-TAPS it would


be unreasonable to require a description of the
communications contents but the following must still
be particularly described:
a. the
persons
whose
communication
is
to
be
intercepted,
b. the identity of the ofense/s
sought to be prevented, and
c. the period of the authorized
intrusion
when public safety or order requires otherwise,
as may be provided by law (Correlate this with Section
1 on whether or not the law is a valid exercise of police
power)

KITY

TAU MU TAU MU TAU MU


TAU MU TAU MU TAU MU

CONSTITUTIONAL LAW II
Order of Saint Thomas More
Atty. Philip John Pojas/Atty. Rovyne G. Jumao-as, RN
University College of Law
lawful order of the court, or when public safety
or order requires otherwise as prescribed by law.
(2) Any evidence obtained in violation of this or
the preceding section shall be inadmissible for
any purpose in any proceedings.

Notes:

The law refers to a "tap" of a wire or cable or the use of


a "device or arrangement" for the purpose of secretly
overhearing,
intercepting,
or
recording
the
communication. There must be either a physical
interruption through a wiretap or the deliberate
installation of a device or arrangement in order to
overhear, intercept, or record the spoken words.
(Gaanan v. IAC, 145 SCRA 112)
RIGHT TO PRIVACY
In the case of OPLE vs. Torres (1998); Pres. Ramos
issued Administrative Order 308, this is the adoption
of the national computerized identification. Ople
argued that it is a violation of a constitutional right
the right to privacy.
In this case the SC enlarged the scope of Section 3.

TAU MU

Even though it is not specifically stated, even


though there is no specific provision in the
Constitution which protects a person's right to
privacy, according to the SC, the Bill of Rights
has several provisions that aim to protect
privacy (indirectly) like sec. 1, 2, 3, 6, 8, 17.
These provisions indirectly protect the right to
privacy and it includes sometimes a shadow
provision over one's right to privacy in general.
So, therefore, the right to privacy is a

ACADCOM 2010; Contributors: Gene Geocaniga, Jarissa Guiani, Darlene Magabilen


TAU MU Page 62 of 179

Since the right to privacy is a fundamental right,


the Gov't has the burden of proof to show that:
(Balancing of Interest test)
(1.) the restriction to the right to privacy is
justified by a compelling government
interest, and that
(2.)the restriction must be narrowly drawn
to protect that compelling valid interest.

CASES: PRIVACY OF COMMUNICATION

TAU MU
TAU MU
TAU MU
TAU MU
TAU MU

Issue: WON the opening and reading of the detainees


letter violated their right to privacy of communication.

TAU MU

Facts: Some 321 armed soldiers, led by the now


detained junior officers (herein petitioners), entered
and took control of the Oakwood Premier Luxury
Apartments. The soldiers disarmed the security officers
of Oakwood and planted explosive devices in its
immediate surroundings. The junior officers publicly
renounced their support for the administration and
called for the resignation of PGMA and several cabinet
members. The soldiers voluntarily surrendered to the
authorities after several negotiations with government
emissaries and returned to their barracks.
Thereafter, the government prosecutors filed
Information for coup detat against the soldiers
involved in the Oakwood incident. Gen. Abaya issued a
directive to all Major Service Commanders to take into
custody the military personnel under their command
who took part in the Oakwood incident except the
detained junior officers who were to remain under the
custody of ISAFP. In this case, petitioners filed a
petition for habeas corpus with the Supreme Court. The
Court issued a Writ of Habeas Corpus directing
respondents to make a return of the writ and to appear
and produce the persons of the detainees before the
Court of Appeals. Petitioners, while being detained,
alleged that the officials of the ISAFP Detention Center
violated their right to privacy of communication when
the ISAFP officials opened and read the personal letters
of Trillanes and Capt. Milo Maestrecampo. Hence, this
case.

TAU MU
TAU MU

IN RE: PETITION FOR HABEAS CORPUS OF CAPT.


ALEJANO, et. al.
468 SCRA 185

The Fraternal
Ateneo de Davao

TAU MU
TAU MU

Exclusionary rule: Any evidence obtained shall be


inadmissible for any purpose in any proceeding.
However, in the absence of governmental interference,
the protection against unreasonable search and seizure
cannot be extended to acts committed by private
individuals. (People v. Martin)

KITY

TAU MU TAU MU TAU MU


TAU MU TAU MU TAU MU

CONSTITUTIONAL LAW II
Order of Saint Thomas More
Atty. Philip John Pojas/Atty. Rovyne G. Jumao-as, RN
University College of Law
fundamental right and it prevails over other
rights.

TAU MU

Ruling:
American
cases
recognize
that
the
unmonitored use of pre-trial detainees non-privileged
mail poses a genuine threat to jail security. Hence,
when a detainee places his letter in an envelope for
non-privileged mail, the detainee knowingly exposes
his letter to possible inspection by jail officials. A pretrial detainee has no reasonable expectation of privacy
for his incoming mail. However, incoming mail from
lawyers of inmates enjoys limited protection such that
prison officials can open and inspect the mail for
contraband but could not read the contents without
violating the inmates right to correspond with his
lawyer. The inspection of privileged mail is limited to
physical contraband and not to verbal contraband.
Basing the decision in US jurisprudence, the SC
held that the opening and reading of the detainees
letters did not violate the detainees right to privacy of
communication. The letters were not in a sealed
ACADCOM 2010; Contributors: Gene Geocaniga, Jarissa
TAU MU Page 63 of 179

envelope. The inspection of the folded letters is a valid


measure as it serves the same purpose as the opening
of sealed letters for the inspection of contraband. The
letters were not confidential letters between the
detainees and their lawyers. The petitioner who
received the letters from detainees Trillanes and
Maestrecampo was merely acting as the detainees
personal courier and not as their counsel when he
received the letters for mailing. In the present case,
since the letters were not confidential communication
between the detainees and their lawyers, the officials
of the ISAFP Detention Center could read the letters. If
the letters are marked confidential communication
between the detainees and their lawyers, the detention
officials should not read the letters but only open the
envelopes for inspection in the presence of the
detainees.
A law is required before an executive officer
could intrude on a citizens privacy rights is a
guarantee that is available only to the public at large
but not to persons who are detained or imprisoned.
The right to privacy of those detained is subject to
Section 4 of RA 7438, as well as to the limitations
inherent in lawful detention or imprisonment. By the
very fact of their detention, pre-trial detainees and
convicted prisoners have a diminished expectation of
privacy rights. The detainees in the present case are
junior officers accused of leading 300 soldiers in
committing coup detat, a crime punishable with
reclusion perpetua. The junior officers are not ordinary
detainees but visible leaders of the Oakwood incident
involving an armed takeover of a civilian building in the
heart of the financial district of the country. As
members of the military armed forces, the detainees
are subject to the Articles of War.
Moreover, the junior officers are detained with
other high-risk persons from the Abu Sayyaf and the
NPA. Thus, the military custodian must give a wider
range of deference in implementing the regulations in
the ISAFP Detention Center. The military custodian is
in a better position to know the security risks involved
in detaining the junior officers, together with the
suspected Abu Sayyaf and NPA members. Since the
appropriate regulations depend largely on the security
risks involved, the people should defer to the
regulations adopted by the military custodian in the
absence of patent arbitrariness.
The ruling in this case, however, does not
foreclose the right of detainees and convicted
prisoners from petitioning the courts for the redress of
grievances. Regulations and conditions in detention
and prison facilities that violate the Constitutional
rights of the detainees and prisoners will be reviewed
by the courts on a case-by-case basis. The courts
could aford injunctive relief or damages to the
detainees and prisoners subjected to arbitrary and
inhumane conditions. However, habeas corpus is not
the proper mode to question conditions of
confinement. The writ of habeas corpus will only lie if
what is challenged is the fact or duration of
confinement.

OPLE vs. TORRES


July 23, 1998

TAU MU
TAU MU

Facts: The petition at bar is a commendable efort on


the part of Senator Ople to prevent the shrinking of the
right to privacy, which the revered Mr. Justice Brandeis
considered as "the most comprehensive of rights and
the right most valued by civilized men." Petitioner
Ople prays the invalidation of Administrative Order No.
308 entitled "Adoption of a National Computerized
Identification Reference System" on two important
constitutional grounds, viz: one, it is a usurpation of
Guiani, Darlene Magabilen

Issue: WON AO 308 violates citizens right to privacy


of communication.

The Fraternal
Ateneo de Davao

TAU MU
TAU MU
TAU MU
TAU MU
TAU MU
TAU MU
TAU MU
TAU MU
TAU MU
TAU MU
TAU MU

Ruling: The SC held that AO 308 cannot pass


constitutional
muster
as
an
administrative
legislation because facially it violates the right to
privacy. The essence of privacy is the "right to be let
alone." The right of privacy is recognized and
enshrined in several provisions of the Constitution. The
right to privacy is a fundamental right guaranteed by
the Constitution, hence, it is the burden of government
to show that A.O. No. 308 is justified by some
compelling state interest and that it is narrowly drawn.
A.O. No. 308 is predicated on two considerations: (1)
the need to provide our citizens and foreigners with the
facility to conveniently transact business with basic
service and social security providers and other
government instrumentalities and (2) the need to
reduce, if not totally eradicate, fraudulent transactions
and misrepresentations by persons seeking basic
services. It is debatable whether these interests are
compelling enough to warrant the issuance of A.O. No.
308. The heart of A.O. No. 308 lies in its Section 4
which provides for a Population Reference Number
(PRN) as a "common reference number to establish a
linkage among concerned agencies" through the use of
"Biometrics Technology" (biological encoding is finger
scanning) and "computer application designs." But
what is not arguable is the broadness, the vagueness,
the overbreadth of A.O. No. 308 which if implemented
will put our people's right to privacy in clear and
present danger.
A.O. No. 308 falls short of assuring that
personal information which will be gathered about our
people will only be processed for unequivocally
specified purposes. The lack of proper safeguards in
this regard of A.O. No. 308 may interfere with the
individual's liberty of abode and travel by enabling
authorities to track down his movement; it may also
enable unscrupulous persons to access confidential
information and circumvent the right against selfincrimination; it may pave the way for "fishing
expeditions" by government authorities and evade the
right against unreasonable searches and seizures. The
possibilities of abuse and misuse of the PRN,
biometrics and computer technology are accentuated
when we consider that the individual lacks control over
what can be read or placed on his ID, much less verify
the correctness of the data encoded. They threaten the
very abuses that the Bill of Rights seeks to prevent.
The right to privacy is one of the most threatened
rights of man living in a mass society. The threats
emanate
from
various
sources-governments,
journalists, employers, social scientists, etc.
In the case at bar, the threat comes from the
executive branch of government which by issuing A.O.
No. 308 pressures the people to surrender their privacy
by giving information about themselves on the pretext
that it will facilitate delivery of basic services. Given
the record-keeping power of the computer, only the
indiferent will fail to perceive the danger that A.O. No.
308 gives the government the power to compile a
devastating dossier against unsuspecting citizens. It is
timely to take note of the well-worded warning of
Kalvin, Jr., "the disturbing result could be that everyone
will live burdened by an unerasable record of his past
and his limitations. In a way, the threat is that because
of its record-keeping, the society will have lost its
benign capacity to forget." Oblivious to this counsel,
the dissents still say we should not be too quick in
labelling the right to privacy as a fundamental right.

KITY

TAU MU TAU MU TAU MU


TAU MU TAU MU TAU MU

CONSTITUTIONAL LAW II
Order of Saint Thomas More
Atty. Philip John Pojas/Atty. Rovyne G. Jumao-as, RN
University College of Law
the power of Congress to legislate, and two, it
impermissibly intrudes on our citizenry's protected
zone of privacy.

We close with the statement that the right to privacy


was not engraved in our Constitution for flattery.
KMU vs. DIRECTOR-GENERAL
487 SCRA 623
Facts: Petitioners allege that, EO 420 which is
REQUIRING
ALL
GOVERNMENT
AGENCIES
AND
GOVERNMENT-OWNED
AND
CONTROLLED
CORPORATIONS TO STREAMLINE AND HARMONIZE
THEIR
IDENTIFICATION
(ID)
SYSTEMS,
AND
AUTHORIZING FOR SUCH PURPOSE THE DIRECTORGENERAL, NATIONAL ECONOMIC AND DEVELOPMENT
AUTHORITY TO IMPLEMENT THE SAME, AND FOR
OTHER PURPOSES, is unconstitutional because it
constitutes usurpation of legislative functions by the
executive branch of the government. Furthermore, they
allege that EO 420 infringes on the citizens right to
privacy because it allows access to personal
confidential data without the owners consent.
Issue: WON EO 420 infringes citizens right to privacy.
Ruling: EO 420 shows no constitutional infirmity
because it even narrowly limits the data that can be
collected, recorded and shown compared to the
existing ID systems of government entities. EO 420
further provides strict safeguards to protect the
confidentiality of the data collected, in contrast to the
prior ID systems which are bereft of strict
administrative safeguards. The right to privacy does
not bar the adoption of reasonable ID systems by
government entities. Some one hundred countries
have compulsory national ID systems. Even with EO
420, the Philippines will still fall under the countries
that do not have compulsory national ID systems but
allow only sectoral cards for social security, health
services, and other specific purposes.
Without a reliable ID system, government
entities like GSIS, SSS, Philhealth, and LTO cannot
perform efectively and efficiently their mandated
functions under existing laws. Without a reliable ID
system, GSIS, SSS, Philhealth and similar government
entities stand to sufer substantial losses arising from
false names and identities. The integrity of the LTOs
licensing system will sufer in the absence of a reliable
ID system. In the case at bar, petitioners have not
shown how EO 420 will violate their right to privacy.
Petitioners cannot show such violation by a mere facial
examination of EO 420 because EO 420 narrowly draws
the data collection, recording and exhibition while
prescribing comprehensive safeguards. EO 420 applies
only to government entities that already maintain ID
systems and issue ID cards pursuant to their regular
functions under existing laws. EO 420 does not grant
such government entities any power that they do not
already possess under existing laws. Obviously, a
national ID card system requires legislation because it
creates a new national data collection and card
issuance system where none existed before. Thus, EO
420 does not establish a national ID system but makes
the existing sectoral card systems of government
entities like GSIS, SSS, Philhealth and LTO less costly,
more efficient, reliable and user-friendly to the public.
IN RE: SABIO
504 SCRA 714

TAU MU

Facts: Senator Miriam Santiago introduced Senate


Res. No. 455, directing an inquiry in aid of legislation
on the anomalous losses incurred by the Philippines
Overseas Telecommunications Corporation (POTC),
Philippine Communications Satellite Corporation
(PHILCOMSAT), and PHILCOMSAT Holdings Corporation
(PHC) due to the alleged improprieties in their
operations
by
their
respective
Board
of
ACADCOM 2010; Contributors: Gene Geocaniga, Jarissa Guiani, Darlene Magabilen
TAU MU Page 64 of 179

TAU MU

The Fraternal
Ateneo de Davao

TAU MU
TAU MU
TAU MU
TAU MU
TAU MU

Issue: WON the petitioners right to privacy was


infringed.

KITY

TAU MU TAU MU TAU MU


TAU MU TAU MU TAU MU

CONSTITUTIONAL LAW II
Order of Saint Thomas More
Atty. Philip John Pojas/Atty. Rovyne G. Jumao-as, RN
University College of Law
Directors. Hence, a public meeting to deliberate on
Senate Res. No. 455 was conducted. Chairman Sabio of
PCGG was invited to be one of the resource persons in
the public meeting jointly conducted by Committee on
Government Corporations and Public Enterprises and
Committee on Public Services. However, he declined
the invitation because of a prior commitment. Senator
Gordon issued a Subpoena Ad Testificandum, requiring
Chairman Sabio and PCGG Commissioners to appear in
the public hearing and testify on what they know
relative to the matters specified in Senate Res. No.
455. Similar subpoena were issued against the
directors and officers of Philcomsat Holdings
Corporation. Again, Chairman Sabio refused to appear.
Hence, Senator Gordon issued an order to arrest
Chairman Sabio and PCGG Commisioners for contempt
of the Senate. He was later on arrested in his office.
With this, he filed a petition for writ of habeas corpus
against the Senate Committee on Government
Corporations and Public Enterprises and Committee on
Public Services.
PHILCOMSAT filed a petition for certiorari and
prohibition against the Senate Committees on
Government Corporations and Public Enterprises and
Public Services. PHILCOMSAT and its directors alleged
that the subpoena issued to them violated their right to
privacy and against self-incrimination.

TAU MU

Ruling: In evaluating a claim for violation of the right


to privacy, the court must determine whether a person
has exhibited a reasonable expectation of privacy and,
if so, whether that expectation has been violated by
unreasonable government intrusion.
Applying this
determination to these cases, the important inquiries
are: first, did the directors and officers of Philcomsat
Holdings Corporation exhibit a reasonable expectation
of privacy?; and second, did the government violate
such expectation?
The answers are in the negative. Petitioners
were invited in the Senates public hearing to
deliberate on Senate Res. No. 455, particularly on
the anomalous losses incurred by the Philippine
Overseas Telecommunications Corporation (POTC),
Philippine
Communications
Satellite
Corporation
(PHILCOMSAT), and Philcomsat Holdings Corporations
(PHC) due to the alleged improprieties in the
operations by their respective board of directors.
Obviously, the inquiry focuses on petitioners acts
committed in the discharge of their duties as officers
and directors of the said corporations, particularly
Philcomsat Holdings Corporation. Consequently, they
have no reasonable expectation of privacy over
matters involving their offices in a corporation where
the government has interest. Certainly, such matters
are of public concern and over which the people have
the right to information. This goes to show that the
right to privacy is not absolute where there is an
overriding compelling state interest.
In Valmonte v. Belmonte, the Court remarked
that as public figures, the Members of the former
Batasang Pambansa enjoy a more limited right to
privacy as compared to ordinary individuals, and their
actions are subject to closer scrutiny. Taking this into
consideration, the Court ruled that the right of the
people to access information on matters of public
concern prevails over the right to privacy of financial
transactions.
Under the present circumstances, the alleged
anomalies in the PHILCOMSAT, PHC and POTC, ranging
in millions of pesos, and the conspiratorial participation
of the PCGG and its officials are compelling reasons for
the Senate to exact vital information from the directors
and officers of Philcomsat Holdings Corporations, as
ACADCOM 2010; Contributors: Gene Geocaniga, Jarissa
TAU MU Page 65 of 179

well as from Chairman Sabio and his Commissioners to


aid it in crafting the necessary legislation to prevent
corruption and formulate remedial measures and policy
determination regarding PCGGs efficacy. There being
no reasonable expectation of privacy on the part of
those directors and officers over the subject covered
by Senate Res. No. 455, it follows that their right to
privacy has not been violated by respondent Senate
Committees.

SJS vs. DANGEROUS DRUGS BOARD


November 3, 2008

TAU MU

TAU MU
TAU MU
TAU MU

Facts: The constitutionality of Section 36 of Republic


Act No. (RA) 9165, otherwise known as the
Comprehensive Dangerous Drugs Act of 2002, insofar
as it requires mandatory drug testing of candidates for
public office, students of secondary and tertiary
schools, officers and employees of public and private
offices, and persons charged before the prosecutors
office with certain ofenses, among other personalities,
is put in issue. , Petitioner Social Justice Society, a
registered political party, seeks to prohibit the
Dangerous Drugs Board (DDB) and the Philippine Drug
Enforcement Agency (PDEA) from enforcing paragraphs
(c), (d), (f), and (g) of Sec. 36 of RA 9165 on the ground
that they infirm the constitution. For one, the
provisions constitute undue delegation of legislative
power when they give unbridled discretion to schools
and employers to determine the manner of drug
testing. For another, the provisions trench in the equal
protection clause inasmuch as they can be used to
harass a student or an employee deemed undesirable.
And for a third, a persons constitutional right against
unreasonable searches is also breached by said
provisions.
The drug test prescribed under Sec. 36(c), (d),
and (f) of RA 9165 for secondary and tertiary level
students and public and private employees, while
mandatory,
is
a
random
and
suspicionless
arrangement. The objective is to stamp out illegal drug
and safeguard in the process the well being of [the]
citizenry, particularly the youth, from the harmful
efects of dangerous drugs. This statutory purpose
can be achieved via the pursuit by the state of an
intensive and unrelenting campaign against the
trafficking and use of dangerous drugs through an
integrated system of planning, implementation and
enforcement of anti-drug abuse policies, programs and
projects. The primary legislative intent is not criminal
prosecution, as those found positive for illegal drug use
as a result of this random testing are not necessarily
treated as criminals. They may even be exempt from
criminal liability should the illegal drug user consent to
undergo rehabilitation.
Issue: WON the right to privacy has been violated.

TAU MU
TAU MU
TAU MU

Ruling: The right to privacy has been accorded


recognition in this jurisdiction as a facet of the right
protected by the guarantee against unreasonable
search and seizure under Sec. 2, Art. III of the
Constitution. But while the right to privacy has long
come into its own, this case appears to be the first
time that the validity of a state-decreed search or
intrusion through the medium of mandatory random
drug testing among students and employees is, in this
jurisdiction, made the focal point. Thus, the issue
tendered in these proceedings is veritably one of first
impression.
In the case at bar, the Court is of the view and
so holds that the provisions of RA 9165 requiring
mandatory, random, and suspicionless drug testing of
students are constitutional. Indeed, it is within the
Guiani, Darlene Magabilen

TAU MU
TAU MU
TAU MU

These rights are preferred rights. (Philippine


Blooming Mills case)
These rights are not absolute. They are always
subject to:

1. State regulation premised on the


need to protect society from the injurious exercise of
such freedom.
2. Police power of the State premised on
the need to promote public welfare, safety, morals and
national security.
Examples of State regulation:
1.
Seditious speeches are not covered
2.
Slanderous/libelous words can be penalized
3.
Exclusion from the mails of newspapers
containing seditious articles is valid
4.
A moviegoer has no right to yell fire
inside the movie house.
Rights included:
3. Speech and Expression
4. Assembly and petition
5. Press
6. Religion
7. To form association
Freedom of Speech and Expression is the right to
freely utter whatever the citizen may please, and to
be protected from any responsibility for doing so.
Freedom of the Press the liberty to discuss
publicly and publish truthfully any matter of public
interest without previous censorship from the
government.
Freedom of Assembly is the right to assemble
peacefully for the consultation and discussion of
matters of public interest and to petition the
government for redress of grievances.

To protect and preserve the right of


the people to information and matters of public
concern;

To enable every citizen to bring the


Government and persons in authority to the bar of
public opinion;

To encourage free and general


discussion of matters of public concern.
Scope of Protection (what are protected) : political
speeches, self or artistic expressions, symbolic
speeches. One cannot be punished for doing these
things)
Protected speech includes every form of expression,
whether oral, written, tape or disc recorded. It includes
motion pictures as well as what is known as symbolic
speech such as the wearing of an armband as a symbol
of protest. Peaceful picketing has also been included
within the meaning of speech.

TAU MU
TAU MU

freedom are not confined only to those that are


sympathetic of acceptable to the majority.
It permits the articulation of even the unorthodox
view, that it be hostile to or derided by others, or
induces a condition of unrest, creates dissatisfaction
w/ conditions as they are, or even stirs people to
anger.
The freedom to speak includes the right to be
silent. It also includes the right to an audience, in the
sense that the state cannot prohibit the people from
hearing what a person has to say, whatever be the
quality of his thoughts. The right to listen also includes
the right NOT to listen.

Purposes :

TAU MU

The ideas that may be expressed under this

TAU MU

SCOPE:

TAU MU

FREEDOM OF EXPRESSION is available only


insofar as it is exercised for the discussion of
matters afecting the public interest. Purely private
matters do not come w/i the guaranty.

TAU MU

TAU MU
TAU MU

Section 4. No law shall be passed abridging the


freedom of speech, of expression, or of the
press, or of the right of the people peaceably to
assemble and petition the government for
redress of grievances.

The Fraternal
Ateneo de Davao

TAU MU
TAU MU

Just as in the case of secondary and tertiary


level students, the mandatory but random drug test
prescribed by Sec. 36 of RA 9165 for officers and
employees of public and private offices is justifiable,
albeit not exactly for the same reason. The Court notes
in this regard that petitioner SJS, other than saying that
subjecting almost everybody to drug testing, without
probable cause, is unreasonable, an unwarranted
intrusion of the individual right to privacy, has failed
to show how the mandatory, random, and suspicionless
drug testing under Sec. 36(c) and (d) of RA 9165
violates the right to privacy and constitutes unlawful
and/or unconsented search under Art. III, Secs. 1 and 2
of the Constitution. The essence of privacy is the right
to be left alone. In context, the right to privacy means
the right to be free from unwarranted exploitation of
ones person or from intrusion into ones private
activities in such a way as to cause humiliation to a
persons
ordinary
sensibilities.
The
right
to
privacy yields to certain paramount rights of the public
and defers to the states exercise of police power.
Thus, taking into account the foregoing factors,
i.e., the reduced expectation of privacy on the part of
the employees, the compelling state concern likely to
be met by the search, and the well-defined limits set
forth in the law to properly guide authorities in the
conduct of the random testing, the SC held that the
challenged drug test requirement is, under the limited
context of the case, reasonable and, ergo,
constitutional.

KITY

TAU MU TAU MU TAU MU


TAU MU TAU MU TAU MU

CONSTITUTIONAL LAW II
Order of Saint Thomas More
Atty. Philip John Pojas/Atty. Rovyne G. Jumao-as, RN
University College of Law
prerogative of educational institutions to require, as a
condition for admission, compliance with reasonable
school rules and regulations and policies. To be sure,
the right to enroll is not absolute; it is subject to fair,
reasonable, and equitable requirements. The Court
can take judicial notice of the proliferation of prohibited
drugs in the country that threatens the well-being of
the people, particularly the youth and school children
who usually end up as victims. Accordingly, and until a
more efective method is conceptualized and put in
motion, a random drug testing of students in
secondary and tertiary schools is not only acceptable
but may even be necessary if the safety and interest of
the student population, doubtless a legitimate concern
of the government, are to be promoted and protected.

ACADCOM 2010; Contributors: Gene Geocaniga, Jarissa Guiani, Darlene Magabilen


TAU MU Page 66 of 179

CONSTITUTIONAL LAW II
Order of Saint Thomas More
Atty. Philip John Pojas/Atty. Rovyne G. Jumao-as, RN
University College of Law

1.
2.
3.
4.

ACTS NOT
COVERED
seditious
speeches
libelous speeches
obscene
speeches/express
ion
contemptu
ous
speeches

TAU MU
TAU MU
Freedom
exercise

of
be

TAU MU
TAU MU
TAU MU

whether
the
words
used
in
such
circumstances are of such nature as to create a clear
and present danger that they will bring about the
substantive evils that the State has a right to prevent

the danger created must not only be clear


and present but also traceable to the ideas expressed
--- unless this nexus is established, the individual may
not be held accountable

JUSTICE FERNANDO:

TAU MU

2. CLEAR AND PRESENT DANGER It is a working


principle that: (Before the utterance can be
punished)

TAU MU

TAU MU

This test has been abandoned by the court


already.
It is only used when there purpose to ignite
strife or sedition. You cannot use it on
anything else. It is only used when the crime
committed is sedition.
it is not necessary that some definite or
immediate
act
or
force,
violence,
or
unlawfulness be advocated in general terms
it is also not necessary that the language used
be reasonably calculated to incite persons to
acts of force, violence or unlawfulness
it is SUFFICIENT if the natural tendency and
probable efect of the utterance be to bring
about the substantive evil w/c the legislative
body seeks to prevent
a person could be punished for his ideas even
if they only tended to create the evil sought to
be prevented --- it is not necessary to actually
create the evil; mere tendency toward the evil
is enough

TAU MU

TAU MU

1. DANGEROUS TENDENCY TEST - if the word


spoken creates a dangerous tendency which the
state has the right to prevent.

TAU MU

MODES OF EXPRESSION:
1. language oral or written
2. symbolism
Tests on the Restriction of
Expression:
(When
can
the
restricted)

clear seems to point to a causal connection


w/ the danger arising from the utterance questioned
present refers to the time element --- it
must be imminent and immediate danger
the danger must not only be probable but
very likely inevitable
[Case: GONZALES vs. COMELEC (27 SCRA 835)
Notes:
a. A substantive evil must be extremely
serious, and
b. Degree of imminence extremely high
Furthermore, the danger created must
have a causal relation to the expression, and the evil
which the state has the right to suppress. (Gonzales
vs. Comelec)
Used when the suit is between a private
individual and the government.
Requisites :

TAU MU
TAU MU

1. Political speech
2. self/artistic expression
3. commercial speech
a. speech must not
be
false,
misleading
or
proposing
an
illegal activity
b. governmental
interest sought to
be served by the
regulation must be
substantial
c. the
regulation
must
directly
advance
the
governments
interest
d. regulation
must
not be overbroad
4. scientific information
5. symbolic speeches
6. picketing

The Fraternal
Ateneo de Davao

TAU MU TAU MU TAU MU


TAU MU TAU MU TAU MU

ACTS PROTECTED

KITY

a. the word used must is of such nature that would


create a danger that would bring about a substantive
evil that the state has the right to prevent;
b. the substantive evil that will be brought about by
such expression must be extremely serious and
clear. (There must be a causal relation between the
act sought to be prevented and the evil that will be
brought about by the utterance)
c. And it must be present, not mere tendency. It
probability of occurrence is inevitable and apparent.
Case: Zaldivar vs. Gonzales - there doesn't need to
be an immediate catastrophe, the evil sought to be
avoided here was the obstruction of justice, the
derogation of the justice system and the diminishing
of the standard of professional conduct. It is actually
a danger if you degrade the justice system.
3. BALANCING OF INTEREST. - Principle which
requires the court to consider the circumstances of
each particular case and thereafter it shall settle the
issue of which right demands greater protection. The
court must weigh and balance two or more
conflicting social interests. It involves many
considerations but in the end, the court will uphold
what should be considered as the most important
interest.

It is used when there are two or more rights


from the Bill of Rights being invoked by
private individuals.
When particular conduct is regulated in the
interest of public order, and the regulation
results in the indirect, conditional, partial
abridgment of speech, the duty of the courts is
to determine w/c of the 2 conflicting interests
demands
greater
protection
under
the
particular circumstances presented
if in a given situation it should appear that
there is urgent necessity for protecting the
national. security against improvident exercise
of freedom of expression, the right must yield
BUT if no special justification for its curtailment
exists, it must prevail
FLAW: it in efect allows the courts to decide
that this freedom may not be enforced unless
they believe it is reasonable to do so

Case: Lagunzad vs. Sotto (right to privacy is superior to


the freedom of expression)

ACADCOM 2010; Contributors: Gene Geocaniga, Jarissa Guiani, Darlene Magabilen


TAU MU Page 67 of 179

LAGUNSAD vs. SOTTO

authority
pre-ferred

is

BALANCE-OFINTEREST TEST

resolves the
issue in the
light of the
peculiar circumstances
obtai-ning in
each case

These tests are also applicable to other preferred


freedoms: 1) freedom of association, 2) right of
assembly and petition, and 3) freedom of religion.

TAU MU
TAU MU

official/governmental restrictions on the


press and other forms of expression in advance of
actual publication or dissemination

in cases of prior restraint the rule that all


exercise of police power bears the presumption of
constitutionality does not apply (so this is an
exception)

TAU MU

b.

When one is restrained from expressing


himself, no one knows what that
person is going to say;
The value of the presumption against
the State is that one has no evidence
for conviction.

TAU MU

a.

TAU MU

REASONS for the presumption of unconstitutionality:

TAU MU

Note: Generally, when the state exercises police


power it bears the presumption of constitutionality.
However, in cases of prior restraint, the presumption
of
constitutionality
is
not
applicable.
The
government has the burden of proof in proving the
constitutionality of the law. (Pass the Clear and
present danger test)

TAU MU

Prohibitions under Section 4/ Forms of


Curtailment of Freedom of Speech:
1. PRIOR RESTRAINT
It means official governmental restrictions
on the press or other forms of expression in advance of
actual publication or dissemination; curtailment of the
freedom of expression made through restrictions in
advance of actual publication or dissemination. This
means that the exercise of such freedom depends
upon the prior approval of the government.

TAU MU

DANGEROUS
TENDENCY TEST

TAU MU

CLEAR and
PRESENT
DANGER TEST

liberty is
pre-ferred

a question
of
proximity
and
degree

TAU MU
TAU MU

Ruling: The court held that the right to privacy of the


Padilla family should prevail. One consideration was
that the movie intruded into the private life of the
Padilla family. Another was that the movie was partly
fictionalized. Furthermore, Lagunsad also agreed to
enter into a contract with the Padillas.

The Fraternal
Ateneo de Davao

TAU MU
TAU MU

Facts: Lagunsad was a movie producer. He entered


into a contract with the heirs of Moises Padilla for the
production of the movie The MOises Padilla Story.
After the movie earned profits, Lagunsad refused to
pay the royalty to the heirs of Padilla, saying that the
contract is void because it violated the freedom of
expression. According to him, the life of Padilla is a
matter of public interest and he should be allowed to
produce the movie without the obligation to pay the
heirs. Padilla was already considered a national hero or
a folklore legend of the Ilonggos. The conflict to be
resolved here is between the right of free expression
against the right to privacy.

KITY

TAU MU TAU MU TAU MU


TAU MU TAU MU TAU MU

CONSTITUTIONAL LAW II
Order of Saint Thomas More
Atty. Philip John Pojas/Atty. Rovyne G. Jumao-as, RN
University College of Law

EXCEPTIONS (the exercise of police power


bears a presumption of constitutionality --- exception to
the exception so that the rule applies):
a. when a nation is at war
b. when the COMELEC exercises its power to
regulate the mass media for election purposes [Art. IX
C Sec. 4; NPC vs. COMELEC (207 SCRA 1)]

FORMS:
a. censorship
b. closures
c. court injunctions
d. system of issuance of permits and
licenses

Q: What is CENSORHIP?
A: CENSORSHIP conditions the exercise of freedom of
expression upon prior approval of the government. It
need not partake of total suppression; even restriction
of circulation is unconstitutional.
Examples/forms of prior restraint
1. movie censorship
2. judicial prior restraint = court injunction
against publication
3. issuances of licenses and permits, taxes
based on gross receipts for the privilege of
engaging in the business of advertising in
any newspaper
4. flat license fees for the privilege of selling
religious books
5. Closures
EXCEPTION TO THE EXCEPTION (when prior
restraint in unconstitutional and burden of proof is
on the person restrained. In other words, there is a
presumption of constitutionality):
a.) IN TIMES OF WAR -- Ex. Government
can prevent publication about the number/locations of
its troops (Near v. Minnesota, 238 US 697)
b.) ART. 9 (C), SEC. 4, -- when the
comelec exercises its power to regulate mass
media for election purposes. (take Note, for
election purposes only)
c.) OBSCENE PUBLICATIONS.
d.) CONTENT NEUTRAL- does not
restrict the content of the speech, or the content of
your publication, but the time, the manner or the
place. The restriction is not imposed on the content
of the speech and publication, but only on the time
and manner of expression. There is no presumption
of unconstitutionality. What is only required is
substantial government interest for the restriction to
be accepted. There is no need for the clear and
present danger test ex. preventing people from
writing graffiti on the wall.
As opposed to CONTENT BASED - restriction of the
content of the speech, what the person is going to
say or write ex. preventing people from writing about
GMA on the wall; there is a presumption of
unconstitutionality and the test to be applied is the
clear and present danger test)
Note: Just because it is content neutral and there is
no presumption of unconstitutionality, this doesn't
mean that okay na sya. It still has to pass a test .
What kind of test should be used if the
restriction is content neutral?
The OBrien Test (SWS vs. COMELEC
case):
A. if it (restriction) is within
constitutional powers of the government;

ACADCOM 2010; Contributors: Gene Geocaniga, Jarissa Guiani, Darlene Magabilen


TAU MU Page 68 of 179

the

TAU MU
TAU MU
TAU MU

The general rule in criticism of official conduct is


characterized by liberality. The official acts, and even
the private life of a public servant are legitimate
subjects of public comment. People have a right to
scrutinize and commend or condemn the conduct of
their chosen representatives in government.
As long as the peoples comments are made in
good faith and w/ justifiable ends, they are insulated
from prosecution or damage suits for defamation even
if such views are found to be inaccurate or erroneous.

TAU MU

TAU MU

CRITICISM OF OFFICIAL CONDUCT

TAU MU

Q: Does this right cover ideas ofensive to the public


order or decency or reputation of persons entitled to
protection by the state?
A: NO.

TAU MU

Q: Is this right absolute?


A: NO! Freedom of expression is not absolute. It is
subject to the police power and may be properly
regulated in the interest of the public.

TAU MU

Freedom of speech includes freedom after speech.

TAU MU

Coverage of valid subsequent punishment


Criminal prosecution for:

sedition

libel

obscenity

citation for contempt

damages
*Unprotected speech

TAU MU
TAU MU

2. SUBSEQUENT PUNISHMENT

a restraint that comes AFTER the


exercise of the freedom of speech, press or expression.

Punishing the expression that has


already come out. What the Constitution protects
is the right to be able to express one's ideas
without being punished for it. However, it doesn't
mean that anything that one says, one can
escape punishment for that by invoking the
freedom of expression since this freedom is not
absolute. (Correlate with unprotected speech)

Freedom from any punishment as a


consequence of, or in connection with a speech,
utterance or writing. (This is the guarantee of the
constitution)

In order to make the freedom of


expression clause more meaningful, there must be
an assurance that, after making any utterance or
publication, the author will not be subjected to any
form of punishment.

The Fraternal
Ateneo de Davao

TAU MU
TAU MU

Examples of Content neutral restrictions:


1. When the Comelec exercises its power to regulate
mass media for election purposes under Art 9-C se. 4 in
relation to Sec. 11 of RA 6646. (Osmena vs. comelec)
2. Preventing the conducting of exit polls right after
the elections. (ABS-CBN vs. Comelec here, even if it is
a content neutral provision it still violates the freedom
of expression as holding of exit polls does not really
undermine the result of the election. The comelec
resolution is overbroad. )
3. Preventing the publication of surveys before an
election. SWS vs. Comelec

KITY

TAU MU TAU MU TAU MU


TAU MU TAU MU TAU MU

CONSTITUTIONAL LAW II
Order of Saint Thomas More
Atty. Philip John Pojas/Atty. Rovyne G. Jumao-as, RN
University College of Law
B. if furthers an important or substantial
government interest;
C. if the substantial government interest
is unrelated to the suppression of the expression;
D. if the incidental restriction is no
greater than is essential to the furtherance of the
substantial interest.

A private individual may still be the subject of


public comment even if he is not a public official or at
least a public figure, as long as he is involved in a
public issue.
If a matter is a subject of general/public interest, it
cannot suddenly become less so because a private
individual did not voluntarily choose to become
involved.
The publics primary interest is in the event; the
public focus is on the conduct of the participant and
the content, efect and significance of the conduct,
NOT the participants prior anonymity or notoriety.
CONTEMPT

Public comment is proscribed pending litigation on


the ground that it would interfere w/ the administration
of justice.
Newspaper publications tending to impede,
obstruct, embarrass or influence the courts in the
administering justice in a pending suit/proceeding
constitutes criminal contempt, summarily punishable
by the courts. The rule is otherwise if the case is ended
[ROSENBLOOM vs. METROMEDIA].

This section is subordinate to the authority,


integrity and independence of the judiciary and the
proper administration of justice.
For expression to constitute contempt, the danger
must cause a serious imminent threat to the
administration of justice.
Only publication made in the pendency of a case
constituted contempt.
Where the questioned publication was alleged to
be contemptuous of the SC or its Justices, publication
was declared contemptuous. But in every case where
an inferior court or its judge was the target, absolution
followed.
The freedom of expression under Sec. 4 is
subordinate to the authority, integrity, independence of
the judiciary in the proper administration of justice.
Q: Who has the power to declare a person in
contempt?
A:
The courts this is an inherent power even
w/o any express grant in the Constitution because it is
part of the exercise of judicial power
Congress can pass a law as to which agency
has the power to cite a person in contempt.
LIBELOUS SPEECH

are by no means essential to the exposition of


ideas and are of such scant social value that any
benefit w/c may be derived from them is outweighed
by the social interest of public order

NOT protected speech

there are no tests


A. FAIR COMMENT (U.S. Rule).
These are
statements of OPINION, not of fact, and are not
considered actionable, even if the words used are
neither mild nor temperate. What is important is that
the opinion is the true and honest opinion of the
person.
The statements are not used to attack
personalities but to give ones opinion on decisions and
actions.
B. OPINIONS. With respect to public personalities
(politicians, actors, anyone with a connection to a
newsworthy event), opinions can be aired regarding
their public actuations. Comment on their private
lives, if not germane to their public personae, are not
protected.

ACADCOM 2010; Contributors: Gene Geocaniga, Jarissa Guiani, Darlene Magabilen


TAU MU Page 69 of 179

(1) He must prove that the charge is false,


(2)That the one making the charge knows that it is
false and
(3) That there was utter disregard as to whether or
not the charge is really false.

TAU MU
TAU MU
TAU MU

OBSECENTITY
TESTS of OBSCENITY:
1. whether the average person, applying
contemporary community standards, would find that
the work, taken as a whole, appeals to prurient interest
2. whether the work depicts/describes, in a
patently ofensive way, sexual conduct specifically
defined by the applicable law whether the work, taken
as a whole, lacks serious literary, artistic, political or
scientific value

TAU MU

TAU MU

if
the
statement
concerns
matters
that are essentially
of his private life are
protected by law --Sec. 4 is NOT a
defense
if
the
statement
concerns
matters
connected
w/
his
official acts, OR his
mental,
moral
or
physical fitness to be
in office --- issue is
constitutional
statements that are
purely
destructive
that
afect
his
function as an official
are restricted BUT as
a public official it is
expected that you
may be criticized

TAU MU

Libelous Statements
vs. PUBLIC
INDIVIDUALS

TAU MU
TAU MU

Libelous
Statements vs.
PRIVATE
INDIVIDUALS
almost
always
punishable
because a person
has a right to his
reputation
and
integrity
freedom
of
expression under
Sec. 4 is NOT a
defense

The Fraternal
Ateneo de Davao

TAU MU
TAU MU

Note: If the person being attacked is a public


official, and what is being said is in relation to his
public function, then the freedom of expression is
stronger even if the statement is slanderous or
libelous. Borjal case expanded the Vasquez decision
stating that not only public officials or
government employees have to prove these
things, but also someone who is a public
figure.

KITY

TAU MU TAU MU TAU MU


TAU MU TAU MU TAU MU

CONSTITUTIONAL LAW II
Order of Saint Thomas More
Atty. Philip John Pojas/Atty. Rovyne G. Jumao-as, RN
University College of Law
Note: LIBEL/SLANDER - a crime; as a general rule,
the burden of proof is on the prosecution. In libel,
generally, proof of truth is not a defense. But if it is
against the public official, proof of truth is a defense.
When the libel is against a public official, it is
incumbent on the public official to prove certain
things (3 things).

v. Proper action should be filed under Art. 201


of the RPC.
vi. Conviction is subject to appeal.
Principles:
1. Movies/Tv programs- admin officials such as
MTCRB have the power to determine what is obscene
or not (Iglesia vs. CA) Classification only, not
prohibition, ie R-18
2. Printed material it is the court who will
determine if it is obscene or not.
3. Obscene materials cannot be considered as
nuisance per se
4. Procedure to be taken by authorities when
there are alleged obscene materials:
a.Obtain a search warrant
b.It must be the judge, and not the
policemen, who will determine what is obscene and
what is not
c. The warrant can only be granted by the
judge if a clear and present danger of a substantive
evil.
CASES: RESTRICTION ON EXPRESSION
OSMENA vs. COMELEC
288 SCRA 447
Facts: This is a petition for prohibition, seeking a reexamination of the validity of Sec. 11(b) of R.A. No.
6646, the Electoral Reforms Law of 1987, which
prohibits mass media from selling or giving free of
charge print space or air time for campaign or other
political purposes, except to the Commission on
Elections. Petitioner Osmea is candidate for President
of the Philippines, while petitioner Pablo P. Garcia is
governor of Cebu Province, seeking reelection. They
contend that events after the ruling in National Press
Club v. Commission on Elections have called into
question the validity of the very premises of that
decision.
In NPC vs. COMELEC, the SC upheld upheld the
validity of Sec. 11(b) of R.A. No. 6646 against claims
that it abridged freedom of speech and of the press. In
urging a reexamination of that ruling, petitioners claim
that experience in the last five years since the decision
in that case has shown the undesirable efects of the
law because the ban on political advertising has not
only failed to level the playing field, but actually
worked to the grave disadvantage of the poor
candidates by depriving them of a medium which they
can aford to pay for while their more affluent rivals
can always resort to other means of reaching voters
like airplanes, boats, rallies, parades, and handbills.
Issue: WON freedom of expression and of the press
was abridged by Sec. 11(b) of RA 6646.

TAU MU

Ruling: The SC, in applying the OBrien test in this


case, held that Sec. 11(b) of R.A. No. 6646 is a valid
exercise of the power of the State to regulate media of
communication or information for the purpose of
ensuring equal opportunity, time and space for political
campaigns; that the regulation is unrelated to the
suppression of speech; that any restriction on freedom
In determining WON an expression is obscene,
of expression is only incidental and no more than is
the courts should apply the standard of the community
necessary to achieve the purpose of promoting
in w/c the material is being tested
equality.
Procedure for seizure of allegedly obscene
The Court is just as profoundly aware that
publications
discussion of public issues and debate on the
i. Authorities must apply for issuance of search
qualifications of candidates in an election are essential
warrant.
to the proper functioning of the government
ii. Court must be convinced that the materials
established by our Constitution. But it is precisely with
are obscene. Apply clear and present danger test.
this awareness that the SC think democratic eforts at
iii. Judge will determine whether they are in fact
reform should be seen for what they are: genuine
obscene.
eforts to enhance the political process rather than
iv. Judge will issue a search warrant.
infringements on freedom of expression. The statutory
ACADCOM 2010; Contributors: Gene Geocaniga, Jarissa Guiani, Darlene Magabilen
TAU MU Page 70 of 179

TAU MU

TAU MU

TAU MU
TAU MU
TAU MU
TAU MU
TAU MU
TAU MU
TAU MU
TAU MU

Facts: Press Secretary Bunye told reporters that the


opposition
was
planning
to
destabilize
the
administration by releasing an audiotape of a mobile
phone conversation allegedly between the PGMA and a
high-ranking official of the COMELEC. The conversation
was audiotaped allegedly through wire-tapping. Later,
in a Malacaang press briefing, Secretary Bunye
produced two versions of the tape, one supposedly the
complete version, and the other, a spliced, doctored
or altered version, which would suggest that the
President had instructed the COMELEC official to
manipulate the election results in the Presidents favor.
It seems that Secretary Bunye admitted that the voice
was that of President Arroyo, but subsequently made a
retraction.
Counsel of ERAP subsequently released an
alleged authentic tape recording of the wiretap.
Included in the tapes were purported conversations of
the President, the First Gentleman Jose Miguel Arroyo,
COMELEC Commissioner Garcillano, and the late
Senator Barbers. Respondent DOJ Secretary Raul
Gonzales warned reporters that those who had copies
of the compact disc (CD) and those broadcasting or
publishing its contents could be held liable under the
Anti-Wiretapping Act. These persons included Secretary
Bunye and Atty. Paguia, counsel of ERAP. He also
stated that persons possessing or airing said tapes
were committing a continuing ofense, subject to arrest
by anybody who had personal knowledge if the crime
was committed or was being committed in their
presence. Secretary Gonzales ordered the NBI to go
after media organizations found to have caused the
spread, the playing and the printing of the contents of
a tape of an alleged wiretapped conversation
involving the President about fixing votes in the 2004
national elections.
Thereafter, the NTC issued a press release
which states that: NTC GIVES FAIR WARNING TO RADIO
AND TELEVISION OWNERS/OPERATORS TO OBSERVE
ANTI-WIRETAPPING LAW AND PERTINENT CIRCULARS
ON PROGRAM STANDARDS. Thereafter, NTC held a
dialogue with the Board of Directors of the Kapisanan
ng mga Brodkaster sa Pilipinas (KBP). NTC allegedly
assured the KBP that the press release did not violate
the constitutional freedom of speech, of expression,
and of the press, and the right to information.
Petitioner Chavez filed a petition against
respondents Secretary Gonzales and the NTC alleging
that the acts of respondents are violations of the
freedom of expression and of the press, and the right
of the people to information on matters of public
concern. Respondents denied that the acts transgress
the Constitution, and questioned petitioners legal
standing to file the petition.

The Fraternal
Ateneo de Davao

TAU MU
TAU MU

CHAVEZ vs. GONZALES


555 SCRA 441

KITY

TAU MU TAU MU TAU MU


TAU MU TAU MU TAU MU

CONSTITUTIONAL LAW II
Order of Saint Thomas More
Atty. Philip John Pojas/Atty. Rovyne G. Jumao-as, RN
University College of Law
provision involved in this case is part of the reform
measures adopted in 1987 in the aftermath of EDSA. A
reform-minded Congress passed bills which were
consolidated into what is now R.A No. 6646 with near
unanimity. The House of Representatives, of which
petitioner Pablo P. Garcia was a distinguished member,
voted 96 to 1 (Rep. Eduardo Pilapil) in favor, while the
Senate approved it 19-0.
In the book The Irony of Free Speech by Owen
Fiss, it speaks of a truth that is full of irony and
contradiction: that the state can be both an enemy
and a friend of speech; that it can do terrible things to
undermine democracy but some wonderful things to
enhance it as well. Thus, the SC held R.A. No. 6646,
Sec. 11 (b) to be such a democracy-enhancing
measure.

Issue: WON the mere press statements of the


Secretary of Justice and of the NTC in question
constitute a form of content-based prior restraint that
has transgressed the constitutionally protected
freedoms of speech, of expression and of the press.
Ruling: The SC held that it is not decisive that the
press statements made by respondents were not
reduced in or followed up with formal orders or
circulars. It is sufficient that the press statements were
made by respondents while in the exercise of their
official functions. Undoubtedly, respondent Gonzales
made his statements as Secretary of Justice, while the
NTC issued its statement as the regulatory body of
media. Any act done, such as a speech uttered, for
and
on
behalf
of
the
government
in
an
official capacity is covered by the rule on prior
restraint. The concept of an act does not limit itself
to acts already converted to a formal order or official
circular. Otherwise, the non formalization of an act into
an official order or circular will result in the easy
circumvention of the prohibition on prior restraint. The
press statements at bar are acts that should be struck
down as they constitute impermissible forms of prior
restraints on the right to free speech and press.
There is enough evidence of chilling efect of
the complained acts on record. The warnings given to
media came from no less the NTC, a regulatory agency
that can cancel the Certificate of Authority of the radio
and broadcast media. They also came from the
Secretary of Justice, the alter ego of the Executive, who
wields the awesome power to prosecute those
perceived to be violating the laws of the land. After the
warnings, the KBP inexplicably joined the NTC in
issuing an ambivalent Joint Press Statement. After the
warnings, petitioner Chavez was left alone to fight this
battle for freedom of speech and of the press. This
silence on the sidelines on the part of some media
practitioners is too deafening to be the subject of
misinterpretation.
The constitutional imperative to strike down
unconstitutional acts should always be exercised with
care and in light of the distinct facts of each case. For
there are no hard and fast rules when it comes to
slippery constitutional questions and the limits and
construct of relative freedoms are never set in stone,
issues revolving on their construct must be decided on
a case to case basis, always based on the peculiar
shapes and shadows of each case. But in cases where
the challenged acts are patent invasions of a
constitutionally protected right, the SC must be swift in
striking them down as nullities per se. A blow too soon
struck for freedom is preferred than a blow too late.
ABS-CBN vs. COMELEC
323 SCRA 811

TAU MU

Facts: This case stems from a petition assailing


Commission on Elections (Comelec) en banc Resolution
No. 98-1419. In the said Resolution, the poll body
"RESOLVED to approve the issuance of a restraining
order to stop ABS-CBN or any other groups, its agents
or representatives from conducting such exit survey
and to authorize the Honorable Chairman to issue the
same."
The Resolution was issued by the Comelec
allegedly upon "information from a reliable source that
ABS-CBN (Lopez Group) has prepared a project, with PR
groups, to conduct radio-TV coverage of the elections
and to make an exit survey of the vote during the
elections for national officials particularly for President
and Vice President, results of which shall be broadcast
immediately." The electoral body believed that such
project might conflict with the official Comelec count,
as well as the unofficial quick count of the National
Movement for Free Elections (Namfrel). It also noted
ACADCOM 2010; Contributors: Gene Geocaniga, Jarissa Guiani, Darlene Magabilen
TAU MU Page 71 of 179

TAU MU

TAU MU

Issue: WON the act of the COMELEC in restraining


petitioner from holding exit polls and the nationwide
reporting of their results violated freedoms of speech
and of the press.

The Fraternal
Ateneo de Davao

TAU MU
TAU MU
TAU MU
TAU MU
TAU MU
TAU MU
TAU MU
TAU MU
TAU MU
TAU MU
TAU MU

Ruling: The freedom of expression is a means of


assuring individual self-fulfillment, of attaining the
truth, of securing participation by the people in social
and political decision-making, and of maintaining the
balance between stability and change. It represents a
profound commitment to the principle that debates on
public issues should be uninhibited, robust, and wide
open. It means more than the right to approve existing
political beliefs or economic arrangements, to lend
support to official measures, or to take refuge in the
existing climate of opinion on any matter of public
consequence.
The Court has always ruled in favor of the
freedom of expression, and any restriction is treated an
exemption. The power to exercise prior restraint is not
to be presumed; rather the presumption is against its
validity. And it is respondent's burden to overthrow
such presumption. Any act that restrains speech should
be greeted with furrowed brows, so it has been said. To
justify a restriction, the promotion of a substantial
government interest must be clearly shown. Thus, "A
government regulation is sufficiently justified if it is
within the constitutional power of the government, if it
furthers an important or substantial government
interest; if the governmental interest is unrelated to
the suppression of free expression; and if the incidental
restriction on alleged First Amendment freedoms is no
greater than is essential to the furtherance of that
interest."
Hence,
even
though the
government's
purposes are legitimate and substantial, they cannot
be pursued by means that broadly, stifle fundamental
personal liberties, when the end can be more narrowly
achieved.
The freedoms of speech and of the press should all the
more be upheld when what is sought to be curtailed is
the dissemination of information meant to add
meaning to the equally vital right of sufrage. The
Court cannot support any ruling or order "the efect of
which would be to nullify so vital a constitutional right
as free speech." When faced with borderline situations
in which the freedom of a candidate or a party to speak
or the freedom of the electorate to know is invoked
against actions allegedly made to assure clean and
free elections, this Court shall lean in favor of freedom.
For in the ultimate analysis, the freedom of the citizen
and the State's power to regulate should not be
antagonistic. There can be no free and honest elections
if, in the eforts to maintain them, the freedom to
speak and the right to know are unduly curtailed.
In order to justify a restriction of the people's
freedoms of speech and of the press, the state's
responsibility of ensuring orderly voting must far
outweigh them. These freedoms have additional
importance, because exit polls generate important
research data which may be used to study influencing
factors and trends in voting behavior. An absolute
prohibition would thus be unreasonably restrictive,
because it efectively prevents the use of exit poll data

KITY

TAU MU TAU MU TAU MU


TAU MU TAU MU TAU MU

CONSTITUTIONAL LAW II
Order of Saint Thomas More
Atty. Philip John Pojas/Atty. Rovyne G. Jumao-as, RN
University College of Law
that it had not authorized or deputized Petitioner ABSCBN to undertake the exit survey.
Thereafter, the Supreme Court issued the
Temporary Restraining Order prayed for by petitioner
and directed the Comelec to cease and desist, until
further orders, from implementing the assailed
Resolution or the restraining order issued pursuant
thereto, if any. In fact, the exit polls were actually
conducted and reported by media without any difficulty
or problem.

not only for election-day projections, but also for longterm research.
The Comelec's concern with the possible noncommunicative efect of exit polls -- disorder and
confusion in the voting centers -- does not justify a
total ban on them. Undoubtedly, the assailed Comelec
Resolution is too broad, since its application is without
qualification as to whether the polling is disruptive or
not. There is no showing, however, that exit polls or the
means to interview voters cause chaos in voting
centers. Neither has any evidence been presented
proving that the presence of exit poll reporters near an
election precinct tends to create disorder or confuse
the voters. Moreover, the prohibition incidentally
prevents the collection of exit poll data and their use
for any purpose. The valuable information and ideas
that could be derived from them, based on the voters'
answers to the survey questions will forever remain
unknown and unexplored. Unless the ban is restrained,
candidates, researchers, social scientists and the
electorate in general would be deprived of studies on
the impact of current events and of election-day and
other factors on voters' choices. The absolute ban
imposed by the Comelec cannot, therefore, be justified.
It does not leave open any alternative channel of
communication to gather the type of information
obtained through exit polling.
SWS INC. vs. COMELEC
357 SCRA 497
Facts: Petitioner Social Weather Stations is a private
non-stock, non-profit social research institution
conducting surveys in various fields, including
economics,
politics,
demography,
and
social
development, and thereafter processing, analyzing,
and publicly reporting the results thereof. On the other
hand, petitioner Kamahalan Publishing Corporation
publishes the Manila Standard, a newspaper of general
circulation, which features newsworthy items of
information including election surveys. Petitioners
brought this action for prohibition to enjoin the
Commission on Elections from enforcing Sec. 5.4 of
R.A. No. 9006, the Fair Elections Act.
Petitioner SWS states that it wishes to conduct
an election survey throughout the period of the
elections both at the national and local levels and
release to the media the results of such survey as well
as publish them directly. Petitioner Kamahalan
Publishing Corporation, on the other hand, states that
it intends to publish election survey results up to the
last day of the elections. Petitioners argue that the
restriction on the publication of election survey results
constitutes a prior restraint on the exercise of freedom
of speech without any clear and present danger to
justify such restraint. Respondent Commission on
Elections justifies the restrictions in Sec. 5.4 of R.A. No.
9006 as necessary to prevent the manipulation and
corruption of the electoral process by unscrupulous and
erroneous surveys just before the election.
Issue: WON Section 5.4 of RA 9006 violated the
freedoms of speech, of expression and of the press.

TAU MU

Ruling: The SC held that Sec. 5.4 of R.A. No. 9006


constitutes an unconstitutional abridgment of freedom
of speech, expression, and the press. Sec. 5.4 lays a
prior restraint on freedom of speech, expression, and
the press by prohibiting the publication of election
survey results afecting candidates within the
prescribed periods of fifteen days immediately
preceding a national election and seven days before a
local election. Because of the preferred status of the
constitutional rights of speech, expression, and the
press, such a measure is vitiated by a weighty
presumption of invalidity. Indeed, any system of prior
ACADCOM 2010; Contributors: Gene Geocaniga, Jarissa Guiani, Darlene Magabilen
TAU MU Page 72 of 179

TAU MU

TAU MU
TAU MU
TAU MU

BORJAL vs. CA
301 SCRA 1

TAU MU

Ruling: Yes. Under Art. 361 of the Revised Penal Code,


when the libelous statement is made against a public
official with respect to the discharge of his official
duties and functions, truth is a defense. For that
matter, even if the defamatory statement is false, no
liability can attach if it relates to official conduct unless
the public official proves that the statement was made
with actual malice --- that is, with the knowledge that it
was false or with reckless disregard of whether it was
false or not. In this case, the prosecution failed to
prove not only that the charges made were false, but
also that Vasquez made them with knowledge of their
falsity or with reckless disregard or whether they were
false or not. A rule placing on the accused the burden
of proving the truth of allegations of official misconduct
and/or good motives and justifiable ends in making the
allegation would infringe on the constitutionally
guaranteed freedom of expression. If the person being
attacked is a public official, and what is being said is
in relation to his public function, then the freedom of
expression is stronger even if the statement is
slanderous or libelous.

TAU MU

Issue: WON placing on the accused the burden of


proving the truth of the allegations would infringe
freedom of expression.

TAU MU
TAU MU

Facts: Vasquez was a spokesman of a group of


squatters. During an interview which was published in
a newspaper, he charged that the Barangay Captain
connived with officials of the National Housing
Authority to grab their land, and that the Barangay
Captain was involved in illegal gambling and theft of
chicken.

The Fraternal
Ateneo de Davao

TAU MU
TAU MU

VASQUEZ vs. CA
314 SCRA 460

KITY

TAU MU TAU MU TAU MU


TAU MU TAU MU TAU MU

CONSTITUTIONAL LAW II
Order of Saint Thomas More
Atty. Philip John Pojas/Atty. Rovyne G. Jumao-as, RN
University College of Law
restraints of expression comes to this Court bearing a
heavy presumption against its constitutional validity.
The Government thus carries a heavy burden of
showing justification for the enforcement of such
restraint. There is thus a reversal of the normal
presumption of validity that inheres in every
legislation.
Hence, the SC ruled that Sec. 5.4 is invalid
because (1) it imposes a prior restraint on the freedom
of expression, (2) it is a direct and total suppression of
a category of expression even though such suppression
is only for a limited period, and (3) the governmental
interest sought to be promoted can be achieved by
means other than the suppression of freedom of
expression.

TAU MU
TAU MU

Facts: Respondent, a private citizen, was Executive


Director of the First National Conference on Land
Transportation (ENOLT).
The Conference, to be
participated in by the private sector and government
agencies, was intended to find ways to solve the
transportation crises, and to draft an omnibus bill to be
presented to Congress that would embody tong-term
transportation policy. In several issues of the Philippine
Star, petitioner implicated respondent in some
anomalous activities. Hence, the latter filed a case for
libel. Is respondent a public figure so as to shift to
him the burden of proving actual malice?
Issue: WON the implications made by petitioner is
within the protection of freedoms of expresson and of
speech.
Ruling: Yes. In the crime of libel, as a general rule,
the burden of proof is on the prosecution. In libel,
generally, proof of truth is not a defense. But if it is
against the public official, proof of truth is a defense.
ACADCOM 2010; Contributors: Gene Geocaniga, Jarissa
TAU MU Page 73 of 179

When the libel is against a public official, it is


incumbent on the public official to prove certain
things such as: (1) He must prove that the charge is
false; (2)That the one making the charge knows that
it is false; and (3) That there was utter disregard as
to whether or not the charge is really false.
As held in the case of Vasquez vs. CA, if the
person being attacked is a public official, and what is
being said is in relation to his public function, then
the freedom of expression is stronger even if the
statement is slanderous or libelous. This case
expanded the decision of in Vasquez by stating that
not only public officials or government employees
have to prove these things, but also someone who is
a public figure. Respondent is deemed a public figure
within the purview of the New York Times ruling, being
a person who, by his accomplishments, fame, mode of
living, or by adopting a profession or calling which
gives the public a legitimate interest in his doings, his
afairs and his character, had become a public
personage. The FNCLT was an undertaking infused
with public interest. It was promoted as a joint project
of the government and the private sector, and
organized by top management officials and prominent
businessmen.
For this reason, it attracted media
mileage and drew public attention. As its Executive
Director
and
spokesman,
private
respondent
consequently assumed the status of a public figure.
IN RE: PUBLISHED ALLEGED THREATS AGAINST
MEMBERS OF THE COURT IN THE PLUNDER LAW
CASE HURLED BY ATTY. LEONARD DE VERA
385 SCRA 285
Facts: The court En Banc issued a Resolution directing
respondent Atty. Leonard De Vera to explain why he
should not be cited for indirect contempt of court for
uttering some allegedly contemptuous statements in
relation to the case involving the constitutionality of
the Plunder Law (Republic Act No. 7080) which was
then a pending resolution. The statements allegedly
uttered by De Vera are the following: (1) Erap camp
blamed for oust-Badoy maneuvers. De Vera asked
the Supreme Court to dispel rumors that it would vote
in favor of a petition filed by Estradas lawyers to
declare the plunder law unconstitutional for its
supposed vagueness. De Vera said he and his group
were greatly disturbed by the rumors from Supreme
Court insiders. His group said that they were afraid that
the Estrada camps efort to coerce, bribe, or influence
the justices ---considering that it has a P500 million
slush fund from the aborted power grab that May-will
most likely result in pro-Estrada decision declaring the
Plunder Law either unconstitutional or vague; (2) SC
under pressure from Erap pals, foes. He said that
people are getting dangerously passionate and
emotionally charged. He voiced his concern that a
decision by the high tribunal rendering the plunder law
unconstitutional would trigger mass actions, probably
more massive than those that led to People Power II.
De Vera warned of a crisis far worse than the jueteng
scandal that led to People Power II if the rumor turned
out to be true. He added that, people wouldnt just
swallow any Supreme Court decision that is basically
wrong. Sovereignty must prevail.
Issue: WON the statements made by De Vera is within
the protection of freedom of expression.

TAU MU
TAU MU

Ruling: Respondent cannot justify his contemptuous


statements--asking the Court to dispel rumors that it
would declare the Plunder Law unconstitutional, and
stating that a decision declaring it as such was
basically wrong and would not be accepted by the
peopleas utterances protected by his right to
freedom of speech.
Guiani, Darlene Magabilen

TAU MU
TAU MU
TAU MU
TAU MU
TAU MU

Ruling (1): The court held that the Freedom of


Expression protects not only citizens of the Philippines
but also foreigners in our country. Sec.4 also extends to
commercial media. Even if they did it for profit, they
are also protected. The reason is that most media is
privately owned and operates for profit. To prohibit
them would render Sec. 4 useless. Nobody can say
anything anymore.
Issue (2): WON it violated Enriles right to
privacy

TAU MU

Issue (1): WON Sec. 4 also protects foreigners

TAU MU
TAU MU

Facts: Hal McElroy, an Australian film-maker, wanted


to join the Peoples Power Revolution in a movie
entitled The 4-day Revolution. The movie was a
mixture of fiction and history to that in addition to a
love story, prominent personalities, like Enrile, had to
be portrayed. While the production was in progress,
Enrile obtained a court injunction to stop it. He argued
that the film violated his right to privacy. TAU MU

The Fraternal
Ateneo de Davao

TAU MU
TAU MU

AYER vs. CAPULONG

KITY

TAU MU TAU MU TAU MU


TAU MU TAU MU TAU MU

CONSTITUTIONAL LAW II
Order of Saint Thomas More
Atty. Philip John Pojas/Atty. Rovyne G. Jumao-as, RN
University College of Law
Indeed, freedom of speech includes the right to
know and discuss judicial proceedings, but such right
does not cover statements aimed at undermining the
Courts integrity and authority, and interfering with the
administration of justice. Freedom of speech is not
absolute, and must occasionally be balanced with the
requirements of equally important public interests,
such as the maintenance of the integrity of the courts
and orderly functioning of the administration of justice.
Thus, the making of contemptuous statements
directed against the Court is not an exercise of free
speech; rather, it is an abuse of such right.
Unwarranted attacks on the dignity of the courts
cannot be disguised as free speech, for the exercise of
said right cannot be used to impair the independence
and efficiency of courts or public respect therefor and
confidence therein. It is a traditional conviction of
civilized society everywhere that courts should be
immune from every extraneous influence as they
resolve
the
issues
presented
before
them.
Respondents utterances pressuring the Court to rule in
favor of the constitutionality of the Plunder Law or risk
another series of mass actions by the public cannot be
construed as falling within the ambit of constitutionallyprotected speech, because such statements are not
fair criticisms of any decision of the Court, but
obviously are threats made against it to force the Court
to decide the issue in a particular manner, or risk
earning the ire of the public. Such statements show
disrespect not only for the Court but also for the
judicial system as a whole, tend to promote distrust
and undermine public confidence in the judiciary, by
creating the impression that the Court cannot be
trusted to resolve cases impartially and violate the
right of the parties to have their case tried fairly by an
independent tribunal, uninfluenced by public clamor
and other extraneous influences.

beforehand because of the preferred character of the


Right of Freedom of Speech and of Expression. And
while production was still in progress, no one knew
whether the final outcome would pose a clear and
present danger. There should have been no prior
restraint because there was no basis yet.
NOTE: Private individuals have more rights than
public figures. The decision did not, however, define
what a public figure is. The right to privacy comes
out of the shadows of the other rights in the
Constitution. There is no textual grant of the Right of
Privacy found in the Constitution.
ASSEMBLY AND PETITION

FREEDOM OF ASSEMBLY is the right of the


people to assemble peaceably for consultation and
discussion of matters of public concern.
This right is important to freedom of expression
because public issues are better resolved after an
exchange of views among citizens meeting w/ each
other for the purpose.
Public meeting is an efective forum for the
ventilation of ideas afecting common welfare. The size
of these gatherings is often a dependable gauge of the
peoples support, or lack of it, for particular causes or
candidates, and a barometer also of the political
climate in general.
The right to assemble is NOT subject to previous
restraint or censorship.
If the assembly is intended to be held in a public
place, a permit for the use of such place, and not for
the assembly itself, may be required.
The power of local officials in this regard is only
one of regulation and not of prohibition. They cannot
altogether bar the use of public places for lawful
assemblies; the most they can do is indicate the time
and conditions for their use.
PROCEDURE to be followed before holding a RALLY:
1. Apply for a permit to hold a rally;
2. The application must contain the date, time and
place of the intended rally;
3. The permit is for the PLACE;
4. The authorities can modify the particulars if there
is CLEAR AND PRESENT DANGER;
5. If the rally is to be held on a private property, the
consent of the owner is sufficient. A permit is not
necessary.
a. in the campus of a GOCC-operated
educational institution
b. in a freedom park to be established in
every municipality and city
The permit must be filed w/ the Mayors Office at
least 5 days before the activity. The application must
be acted on by the said office w/i 2 days from the filing,
otherwise it will be deemed granted by the Mayor.
Denial of the application may be justified upon
clear and convincing evidence that the activity will
create a clear and present danger to public order,
safety, convenience, morals or health.
Action on the application shall be communicated
w/I 24 hours to the applicant who may appeal the same
to the appropriate court.
The test of the lawfulness of an assembly is the
purpose for w/c it is held, regardless of the auspices
under w/c it is organized. Untoward incidents during
the assembly do not make it unlawful.
The law prohibits law enforcers from interfering w/
a lawful assembly but permits them to detail a
contingent under a responsible commander @ least
100 meters away in case it becomes necessary to
maintain order.

TAU MU

Ruling (2): As to the contention of Enrile that it


violated his right to privacy. The SC looked at the
particular circumstances and did not apply any formula
to decide on the issue of which shall prevail: right to
privacy or the freedom of expression. The court ruled
that the events that were portrayed were of public
interest and Enrile is also a public figure. And because
of this, the SC is constrained to rule that his Right to
Privacy shall give way to Freedom of Expression. Only
the balancing of interest was used by the SC in
deciding(during this time, Enrile was a Senator). What
is protected by the Right to Privacy is unwarranted
publicity and wrongful publicizing of private afairs. The
trial judge should not have issued an injunction
ACADCOM 2010; Contributors: Gene Geocaniga, Jarissa Guiani, Darlene Magabilen
TAU MU Page 74 of 179

TAU MU

TAU MU

TAU MU

iii.
If the public authority is of the view
that there is an imminent and grave danger of a
substantive evil, the applicants must be heard on the
matter.

TAU MU

ii. The application should be filed ahead of time


to enable the public official concerned to appraise
whether there are valid objections to the grant of the
permit or to its grant, but in another public place. The
grant or refusal should be based on the application of
the Clear and Present Danger Test.

TAU MU

Rules on assembly in public places:


i. Applicant should inform the licensing
authority of the date, the public place where and the
time when the assembly will take place.

TAU MU

Note: Intelligence information is not enough to pass


the Clear and Present Danger Test.

TAU MU
TAU MU

Q: Can the authorities deny a permit to assembly?


A: YES, the freedom of assembly is not absolute. As a
general rule, No. But the authorities can deny, but it
must pass again the clear and Present Danger Test.
NOTE: The standards for allowable impairment of
speech and press also apply to the right of assembly
and petition.

The Fraternal
Ateneo de Davao

TAU MU
TAU MU

Q: Is the right of assembly and petition subject to


any prior restraint?
A: NO. You cannot prevent people if they want to
assemble. The exercise should not be made to
depend upon the issuance of any permit. It should
not be prohibited, but only regulated. It may be
subject to regulation but not prohibition by the State
as to when and where it should be held.

KITY

TAU MU TAU MU TAU MU


TAU MU TAU MU TAU MU

CONSTITUTIONAL LAW II
Order of Saint Thomas More
Atty. Philip John Pojas/Atty. Rovyne G. Jumao-as, RN
University College of Law
Q: Can there be demonstrations in the vicinity of the
courts?
A: AM 98-7-02 SC provides that:
Demonstrators, picketers, rallyists and all other
similar persons are enjoined from holding any
activity on the sidewalks and streets adjacent
to, in front of, or w/i a radius of 200 meters
from, the outer boundary of the Supreme Court
Building, any Hall of Justice, and any other
building that houses at least 1 court sala. Such
activities unquestionably interrupt and hamper
the working conditions in the salas, offices and
chambers of courts.

a concerted and unauthorized stoppage of or absence


from work carried out for essentially economic reasons.
The exercise of any constitutional liberty must be
within reasonable limits so as not to prejudice the
public welfare. In this case, by staging mass protests
or regular school days, abandoning their classes and
refusing to go back even after they had been ordered
to do so, the school teachers committed acts
prejudicial to the best interest of the service. The
holding in Philippine Blooming Mills that free
expression and assembly are superior to property
rights cannot apply. What is involved here is the
education of the youth which must, at the very least,
be equated with the freedom of assembly and petition
for redress of grievances.
BAYAN vs. ERMITA
488 SCRA 226
Facts: Petitioners come in three groups. The first
petitioners, Bayan, allege that they are citizens and
taxpayers of the Philippines and that their rights as
organizations and individuals were violated when the
rally they participated in was violently dispersed by
policemen implementing Batas Pambansa 880 (The
Public Assembly Act of 1985). The second group
consists of 26 individual petitioners, Jess del Prado,
et al., who allege that they were injured, arrested and
detained when a peaceful mass action was pre-empted
and violently dispersed by the police. They further
assert that a group they participated in marched to
Malacaang to protest issuances of the Palace which,
they claim, put the country under an undeclared
martial rule, and the protest was likewise dispersed
violently and many among them were arrested and
sufered injuries. The third group, Kilusang Mayo Uno
or KMU, allege that they conduct peaceful mass actions
and that their rights as organizations and those of their
individual members as citizens, specifically the right to
peaceful assembly, are afected by Batas Pambansa
No. 880 and the policy of Calibrated Preemptive
Response (CPR) being followed to implement it.
Issue: WON BP 880 is a curtailment of the right to
peaceable assembly and petition for redress of
grievances.

TAU MU

Ruling: The right to peaceably assemble and petition


for redress of grievances is, together with freedom of
speech, of expression, and of the press, a right that
enjoys primacy in the realm of constitutional
protection. For these rights constitute the very basis of
iv.
The decision of the public authority,
a functional democratic polity, without which all the
whether favorable or adverse, must be transmitted to
other rights would be meaningless and unprotected.
the applicants at the earliest opportunity so that they
A fair and impartial reading of B.P. No. 880 thus
may, if they so desire, have recourse to the proper
readily shows that it refers to all kinds of public
judicial authority.
assemblies that would use public places.
The
reference to lawful cause does not make it contentCASES: ASSEMBLY AND PETITION
based because assemblies really have to be for lawful
causes; otherwise they would not be peaceable and
DELA CRUZ vs. CA
entitled to protection. Neither are the words opinion,
305 SCRA 303
protesting and influencing in the definition of
public assembly content based, since they can refer to
Facts: Petitioners were public school teachers who
any subject. The words petitioning the government
participated in mass actions to dramatize their
for redress of grievances come from the wording of
demands for certain benefits granted by statutes. For
the Constitution, so its use cannot be avoided. Finally,
not attending to their classes, they were dismissed by
maximum tolerance is for the protection and benefit of
the Secretary of Education on charge of grave
all rallyists and is independent of the content of the
misconduct, gross neglect of duty and violation of the
expressions in the rally.
Civil Service Law and Rules and Regulations. TAU MU
Furthermore, the permit can only be denied on
the ground of clear and present danger to public order,
Issue: Did the dismissal violate their right to petition
public safety, public convenience, public morals or
and assembly?
public health. Not every expression of opinion is a
public
assembly.
The
law
refers
to
rally,
Ruling: No. The mass actions by the public school
demonstration, march, parade, procession or any other
teachers amounted to a strike constituting, as they did,
form of mass or concerted action held in a public
ACADCOM 2010; Contributors: Gene Geocaniga, Jarissa Guiani, Darlene Magabilen
TAU MU Page 75 of 179

TAU MU

TAU MU

TAU MU

TAU MU

TAU MU
TAU MU
TAU MU
TAU MU

RELIGION
any specific system of belief, worship, conduct,
etc., often involving a code of ethics and philosophy.
a profession of faith to an active power that binds
and elevates man to his Creator [AGLIPAY vs. RUIZ (64
Phil. 201)]
embraces matters of faith and dogma, as well as
doubt, agnosticism and atheism

TAU MU

Section 5. No law shall be made respecting an


establishment of religion, or prohibiting the free
exercise thereof.
The free exercise and
enjoyment of religious profession and worship,
without discrimination or preference, shall
forever be allowed. No religious test shall be
required for the exercise of civil or political
rights.

TAU MU

NOTE: However if the permit granted allows the rally


to be held only in a particular place, there is a
possibility that the place designated may be far and
therefore impossible for other people to heat the
groups grievances. The State is not allowed to do
such.

1. Art. VI Sec. 29;


2. Art. II Sec. 6.
MATTERS PROHIBITED:
1. The state cannot organize a church;
2. The state cannot promote one religion;
3. The sate cannot impose taxes to support
religious activity;
4. The state cannot participate in the afairs
of religious organizations.

TAU MU

NOTE: Students do not lose their constitutional rights


once they enter the school grounds. However, they still
need to obtain a permit from the school authorities.
Schools can still deny permit if there is Clear and
Present Danger.

TAU MU

Ruling: Although the case became moot and


academic, the court nevertheless decided to rule on
the merits of the constitutional issues raised and held
that the penalty imposed was out of proportion to the
misdeed. The decision emphasized that the students
did not shed their constitutional rights to freedom of
speech or expression at the schoolhouse gate,
although these rights were not unlimited.

Ateneo de Davao

TAU MU
TAU MU

Facts: Several student leaders were suspended for one


year when they held a demonstration in the premises
of a university outside the area permitted by the school
authorities which disrupted classes and disturbed the
work of the administrative personnel. The SC issued a
temporary restraining order that in efect permitted the
students to re-enroll and finish their studies.

The Fraternal

TAU MU
TAU MU

MALABANAN vs. RAMENTO

KITY

TAU MU TAU MU TAU MU


TAU MU TAU MU TAU MU

CONSTITUTIONAL LAW II
Order of Saint Thomas More
Atty. Philip John Pojas/Atty. Rovyne G. Jumao-as, RN
University College of Law
place. So it does not cover any and all kinds of
gatherings. Neither is the law overbroad. It regulates
the exercise of the right to peaceful assembly and
petition only to the extent needed to avoid a clear and
present danger of the substantive evils Congress has
the right to prevent.
There is, likewise, no prior restraint, since the
content of the speech is not relevant to the regulation.
Hence, B.P. No. 880 cannot be condemned as
unconstitutional; it does not curtail or unduly restrict
freedoms; it merely regulates the use of public places
as to the time, place and manner of assemblies.
Moreover, the Court goes even one step further in
safeguarding liberty by ordering the local governments
to designate specific freedom parks as provided under
B.P. No. 880. If no such parks are so identified in
accordance with Section 15 of the law, all public parks
and plazas of the municipality or city concerned shall in
efect be deemed freedom parks; no prior permit of
whatever kind shall be required to hold an assembly
therein. The only requirement will be written notices to
the police and the mayors office to allow proper
coordination and orderly activities.

It is not only the state that is prohibited from


interfering in purely ecclesiastical afairs; the Church is
likewise barred from meddling in purely secular
matters because a union of Church and State tends to
destroy government and to degrade religion. It
The wall between Church and State is not one of
hostility. The state recognizes the beneficent influence
of religion in the enrichment of the nations life.
NON ESTABLISHMENT CLAUSE
This clause prevents the state from passing laws w/c

aid one religion, or all religions, or prefer one religion


over another. The state cannot establish or sponsor an
official religion.
Related provisions:

The purpose of this clause is to allow each religion

to compete freely w/ each other based on their own


merit and w/o the states patronage
EXCEPTIONS to the Non-Establishment Clause:
1. Art. VI Sec. 28 (3) tax exemption of
religious properties
2. Art VI Sec. 29 (2) payment of public funds
to priests in the AFP, any penal institution, govt.
orphanage, or leprosarium
3. Art. XIV Sec. 3 (3) allows religious
instructions in public schools under certain conditions
Distinction between the clauses (School District
v. Schempp, 374 US 203)
1. The non-establishment clause does not depend
upon any showing of direct governmental compulsion.
It is violated by the enactment of laws which establish
an official religion whether those laws operate directly
to coerce non-observing individuals or not. The test of
compliance with the non-establishment clause can be
stated as follows: What are the purposes and primary
efect of the enactment? If either is the advancement
or inhibition of religion, the law violates the nonestablishment clause. Thus, in order for a law to
comply with the non-establishment clause, two
requisites must be met.
First, it has a secular
legislative purpose. Second, its primary efect neither
advances nor inhibits religion.
2. The free exercise of religion clause withdraws
from legislative power the exertion of any restraint on
the free exercise of religion. In order to show a
violation of this clause, the person afected must show
the coercive efect of the legislation as it operates
against him in the practice of his religion. While the
freedom to believe (non-establishment) is absolute, the
moment such belief flows over into action, it becomes
subject to government regulation.

TAU MU

Q: Can a statute be passed w/o infringing the nonestablishment clause?


A: YES. There is no violation if (TESTS ON
PERMISSIBLE AID TO RELIGION/LEMON TEST)
[LEMON vs. KURTZMAN (403 US 602)]:
1. the statute has a secular legislative
purpose;
2. its principal or primary efect is one that
neither advances nor inhibits religion; and

ACADCOM 2010; Contributors: Gene Geocaniga, Jarissa Guiani, Darlene Magabilen


TAU MU Page 76 of 179

Freedom of religion includes freedom FROM


religion; the right to worship includes the right NOT to
worship.

TAU MU
TAU MU
TAU MU
TAU MU
TAU MU
TAU MU

Facts: Petitioner inherited a piece of land. When the


parcel of land was ascertained to have been the birth
site of Felix Y. Manalo, the founder of INC, it passed a
resolution declaring the land to be a national historical
landmark. The resolution was approved by the Minister
of Education, Culture and Sports. The opinion of the
Secretary of Justice was asked on the legality of the
measure and he explained: According to your
guidelines, national landmarks are places or objects
that are associated with an event, achievement,
characteristic or modification that makes a turning
point or stage in Philippine history. Thus, the birth site
of the founder of the INC, who admittedly had made
contributions to Philippine history and culture, has
been declared as a national landmark. It has been held
that places invested with unusual historical interest are
a public use for which the power of eminent domain
may be authorized. Petitioners moved to dismiss the
complaint on the main thesis that the intended
expropriation was not for a public purpose and,
incidentally, that the act would constitute an

TAU MU

MANOSCA vs. CA
252 SCRA 414 (1996)

TAU MU

CASES: NON-ESTABLISHMENT CLAUSE

TAU MU

The Lemon Test applied in the case:


This does not seem to have a secular purpose
since the religious factor cannot be separated from
the social factor. (This is still considered
questionable)
Also the Bario Councils seem to be really
participating in a Religious Activity. Although it
was argued that the purpose of the fiesta was to
relieve the monotony of the people. It is not
convincing. It will have a principal efect of
validating a religion. This is a form of participation
of a government entity in a religious afair.
It
does
not
have
excessive
government
entanglement since the image is kept by the
religious leader. If it was kept by the priest, this will
be an excessive entanglement. The image was
purchased with private funds.

TAU MU
TAU MU

Payment of public funds is prohibited to


ecclesiastics only as such w/c means that they may
be paid IF they serve the govt. in a non-ecclesiastical
capacity.

The purchase of religious image by the barangay


council with private funds raised from voluntary
contributions did not violate the Constitution. The
fiesta is a socio-religious afair. It is considered part of
the Filipino tradition. Purchase of the image does not
support any religion, but a socio-religious tradition as
ruled by the Court. [GARCES vs. ESTENZO]

The Fraternal
Ateneo de Davao

TAU MU
TAU MU

Q: If a religious sect enjoys indirect benefit from


governmental action, is there a violation of Sec. 5?
A: Any benefit indirectly enjoyed by a religious
institution, as long as such benefit was only incidental
to a legitimate secular objective, would not violate the
prohibition. The use of public property for religious
purposes when the religious character of such use is
merely incidental to a temporary use w/c is available
indiscriminately to the public in general [AGLIPAY vs.
RUIZ, supra].

KITY

TAU MU TAU MU TAU MU


TAU MU TAU MU TAU MU

CONSTITUTIONAL LAW II
Order of Saint Thomas More
Atty. Philip John Pojas/Atty. Rovyne G. Jumao-as, RN
University College of Law
3. it
does
not
foster
an
excessive
government. entanglement w/ religion.

application of public funds, directly or indirectly, for the


use, benefit or support of INC, a religious entity.
Issue: WON the
members of INC.

resolution

gives

preference

to

Ruling: Eminent domain, also often referred to as


expropriation
and,
with
less
frequency,
as
condemnation, is like police power and taxation, an
inherent power of sovereignty. It need not be clothed
with any constitutional gear to exist. Instead,
provisions in our constitution on the subject are meant
more to regulate, rather than to grant, the exercise of
power. The only direct constitutional qualification is
that private property shall not be taken for public use
without just compensation, to provide a safeguard
against possible abuse and so to protect as well the
individual against those whose property the power is
sought to be enforced. The term public use, not
having been otherwise defined by the Constitution,
must be considered in its general concept of meeting a
public need or a public exigency.
The validity of the exercise of the power of
eminent domain for traditional purposes is beyond
question: it is not at all to be said, however, that public
use should thereby be restricted to such traditional
uses. The idea that public use is strictly limited to
clear cases of use by the public has long been
discarded. This attempt to give some religious
perspective to the case deserves little consideration,
for what should be significant is the principal objective
of, not the casual consequences that might follow from,
the exercise of the power. The purpose in setting up
the marker is essentially to recognize the distinctive
contribution of Felix Manalo to the culture of the
Philippines, rather than to commemorate his founding
and leadership of INC. The practical reality that greater
benefit may be derived by members of the INC than by
most others could well be true but such a peculiar
advantage still remains to be merely incidental and
secondary in nature. Indeed, that only a few actually
benefit from the expropriation of property does not
necessarily diminish the essence and character of
public use.
AGLIPAY vs. RUIZ
FACTS: The government appropriated P60T for design
of new stamps which commemorated the 33 rd
Eucharistic Congress in Manila. The design was the
Map of the Philippines. Monsignor Aglipay petitioned to
stop the making and sale of the stamps. He reasoned
that this was a violation of Section 5.
RULING: The court ruled that the purpose of the
stamp was to attract tourist to the Philippines, not for
the purpose of promoting religion. The benefiting by
the Catholic Church was only incidental to the main
purpose or the principal efect. It was not intended to
be the primary beneficiary. The stamp emphasized
Manila (as shown by the design), not the event or a
particular religion.
INTRAMURAL RELIGIOUS DISPUTES
They are disputes within the church. The
general rule when there is an intramural dispute is that
the State cannot interfere because it will be a violation
of the Free exercise clause.
When can the state interfere? (As a general rule, when
the matter is not ecclesiastical in nature as in Taruc vs.
Dela Cruz)
1. When the problem is not intramural. Even if it is
intramural, when the problem has something to do

ACADCOM 2010; Contributors: Gene Geocaniga, Jarissa Guiani, Darlene Magabilen


TAU MU Page 77 of 179

When there is a conflict between two factions in the


church, who gets the property? The property will go to
the faction which agrees to the original doctrine.
However, if there is no dispute over doctrine, but only
a typical division, the property will go to whoever is the
majority or whoever is the duly constituted authority
2. When the church authorities acts outside the
scope of his authority or contrary to its own
rules.

TAU MU
TAU MU

Q: What is an ECCLESIASTICAL AFFAIR?


A: It is one that concerns doctrine, creed or form of
worship of the church, or the adoption and
enforcement w/i a religious association of needful laws
and regulations for the government of the membership
and the power of excluding from such associations
those that are deemed unworthy of membership.

TAU MU

Intramural Disputes re: religious dogma and other


matters of faith are outside the jurisdiction of the
secular authorities.
Whatever dogma is adopted by a religious group
cannot be binding upon the state if it contravenes its
valid laws.
Where the civil right depends upon some matter
pertaining to ecclesiastical afairs, the civil tribunal
tries the civil right and nothing more [ARCHBISHOP OF
MANILA Case (51 Phil. 417)].

TAU MU

TAU MU
TAU MU

What are the matters of church which the court


cannot interfere with?
a. matters of faith;
b. matters of doctrine;
c. forms of worship, unless it is against the law;
d. law and rules of customs within the church

Q: What about those considered as PURELY SECULAR


MATTERS?
A: These are matters that have no relation whatsoever
w/ the practice of faith, worship or doctrines of the
church.

TAU MU

CASE: INTRAMURAL DISPUTE

TAU MU

AUSTRIA vs. NLRC


312 SCRA 410 (1999)

TAU MU
TAU MU

Facts: Private respondent is a religious corporation


duly organized and existing under Philippine laws.
Petitioner is a pastor of the SDA until 1991, when his
services are terminated. Due to the assistance of
Austria in collecting Pastor Rodrigos debt, the latter
harbored ill-feelings against petitioner. He also had a
quarrel with another pastor and was asked to answer
for non-remittance of church collection. Petitioner
received a letter of dismissal. NLRC ordered his
reinstatement but was vacated after appeal. TAU MU

The Fraternal
Ateneo de Davao

TAU MU
TAU MU

Austria vs. NLRC- there was an employer-employee


relationship, so the State can interfere. It was not a
purely ecclesiastical issue since he was not expelled
from the church but only released from employment as
a pastor. Accdg. to the SC, an ecclesiastical matter is
one that concerns doctrine, creed or form of worship of
the church, or adoption or enforcement within a
religious association of regulation and government of
the membership of the church.

KITY

TAU MU TAU MU TAU MU


TAU MU TAU MU TAU MU

CONSTITUTIONAL LAW II
Order of Saint Thomas More
Atty. Philip John Pojas/Atty. Rovyne G. Jumao-as, RN
University College of Law
with the law. For example, when a property right is
involved. So, when there is a dispute over a property
right within the church, the State can interfere.

Ruling: The principle of separation of church and state


finds no application in this case. This case does not
concern an ecclesiastical or purely religious afair as to
bar the state from taking cognizance of the same. An
ecclesiastical afair is one that concerns doctrine,
creed, or form of worship of the church, or the adoption
and enforcement within religious association of needful
laws and regulations for the government of the
membership, and the power of excluding from such
association those deemed unworthy of membership.
What is involved us the relationship of the church as an
employer and the minister as an employee. The SDA
cannot hide behind the mantle of protection of the
doctrine of separation of church and state. Petitioner
was terminated from service without just or lawful
cause. Having been illegally dismissed, petitioner is
entitled to reinstatement to his former position.
TARUC vs. DELA CRUZ
453 SCRA 123
Facts: Petitioners were lay members of the Philippine
Independent Church (PIC). Respondents Porfirio de la
Cruz and Rustom Florano were the bishop and parish
priest, respectively. Petitioners, led by Dominador
Taruc, clamored for the transfer of Fr. Florano to
another parish but Bishop de la Cruz denied their
request. It appears from the records that the family of
Fr. Floranos wife belonged to a political party opposed
to petitioner Tarucs, thus the animosity between the
two factions with Fr. Florano being identified with his
wifes political camp. Bishop de la Cruz, however,
found this too flimsy a reason for transferring Fr.
Florano to another parish.
Meanwhile, hostility among the members of
the PIC worsened when petitioner Taruc tried to
organize an open mass to be celebrated by a certain Fr.
Renato Z. Ambong during the town fiesta of Socorro.
When Taruc informed Bishop de la Cruz of his plan, the
Bishop tried to dissuade him from pushing through with
it because Fr. Ambong was not a member of the clergy
of the diocese of Surigao and his credentials as a
parish priest were in doubt. The Bishop also appealed
to petitioner Taruc to refrain from committing acts
inimical and prejudicial to the best interests of the PIC.
He likewise advised petitioners to air their complaints
before the higher authorities of PIC if they believed
they had valid grievances against him, the parish
priest, the laws and canons of the PIC.
Bishop de la Cruz, however, failed to stop Taruc
from carrying out his plans. Hence, Bishop de la Cruz
declared petitioners expelled/excommunicated from
the
Philippine Independent
Church.
Petitioners
appealed to the Obispo Maximo and sought
reconsideration of the above decision. In his letter to
Bishop de la Cruz, the Obispo Maximo opined that Fr.
Florano should step down voluntarily to avert the
hostility and enmity among the members of the PIC
parish in Socorro. In the meantime, Bishop de la Cruz
was reassigned to the diocese of Odmoczan and was
replaced by Bishop Timbang. Like his predecessor,
Bishop Timbang did not find a valid reason for
transferring Fr. Florano to another parish. He issued a
circular denying petitioners persistent clamor for the
transfer/re-assignment of Fr. Florano. Petitioners were
informed of such denial but they continued to celebrate
mass and hold other religious activities through Fr.
Ambong who had been restrained from performing any
priestly functions in the PIC parish of Socorro. Because
of the order of expulsion/excommunication, petitioners
filed a complaint for damages with preliminary
injunction against Bishop de la Cruz.

TAU MU

Issue: WON the courts have jurisdiction to hear a case


involving the expulsion/excommunication of members
of a religious institution
ACADCOM 2010; Contributors: Gene Geocaniga, Jarissa Guiani, Darlene Magabilen
TAU MU Page 78 of 179
Issue: WON NLRC has jurisdiction to try and decide the
complaint.

TAU MU
TAU MU
TAU MU

The reason to this is because there are no statutory,


constitutional, or legal standards apply.

TAU MU

Matters that the Court will not inquire into:


1) Matters of faith, e.g. 3 persons in one God
2) Doctrine, e.g. Virgin Mary is Mother of God
3) Forms of Worship, e.g. Priests faces people
4) Church Law or Rule, e.g. confessions, etc.
5) Customs, e.g. pouring water over the head.

TAU MU

This rule was not followed in this case. This applies


only to big churches with external leadership. In this
case, there was no external leadership involved but
only the bishops themselves. Since there is no external
leader who can decide as to who is the legitimate
authority, the basis could be to find out who was
validly elected or removed.

TAU MU

Q: What if there is no quarrel about the doctrines (so


schism), only physical division?
A: (1) Whoever constitutes the majority and (2)
Whoever is the duly constituted authority.

TAU MU

Q: In case of a division of a church, who gets the


property? (Like when a church separates as when there
are doctrinal diferences)
A: The property will go to the faction which adheres to
the original doctrines.

TAU MU

Ruling: The Court ruled that the Ordinance 2925


violated freedom of religion. The government cannot
impose some form of tax on somebody for the exercise
of his religion. American Bible was a missionary
organization which sells bibles as part of propagating
their faith. It was only making a small profit from the
sale of bibles. If they were making a lot of profit, then
that would be the time that the government should
impose a tax on them since it would not anymore be
part of propagating the faith.

TAU MU

Facts: American Bible was a religious corporation,


selling bibles and other religious articles. They were
required to obtain a license and pay the corresponding
fee for being engaged in the sale of merchandise.

TAU MU
TAU MU

AMERICAN BIBLE vs. CITY OF MANILA


101 PHIL. 386 (1957)

The Fraternal
Ateneo de Davao

TAU MU
TAU MU

Ruling: The SC held that the Church and the State are
separate and distinct from each other. The
expulsion/excommunication of members of a religious
institution/organization is a matter best left to the
discretion of the officials, and the laws and canons, of
said institution/organization. It is not for the courts to
exercise control over church authorities in the
performance of their discretionary and official
functions. Rather, it is for the members of religious
institutions/organizations to conform to just church
regulations.
In the leading case of Fonacier v. Court of
Appeals, the SC enunciated the doctrine that in
disputes
involving
religious
institutions
or
organizations, there is one area which the Court should
not touch: doctrinal and disciplinary diferences. Thus,
the amendments of the constitution, restatement of
articles of religion and abandonment of faith or
abjuration alleged by appellant, having to do with faith,
practice, doctrine, form of worship, ecclesiastical law,
custom and rule of a church and having reference to
the power of excluding from the church those allegedly
unworthy
of
membership,
are
unquestionably
ecclesiastical matters which are outside the province of
the civil courts.

KITY

TAU MU TAU MU TAU MU


TAU MU TAU MU TAU MU

CONSTITUTIONAL LAW II
Order of Saint Thomas More
Atty. Philip John Pojas/Atty. Rovyne G. Jumao-as, RN
University College of Law

The basis of this clause is the respect for the


inviolability of the human conscience.
The free exercise of religious profession/calling is
superior to contract rights. In case of conflict, the
latter, must yield to the former [VICTORIANO vs.
ELIZALDE ROPE WORKERS UNION (235 SCRA 197)].
It is only where unavoidably necessary to prevent
an immediate and grave danger to security and
welfare of the community that infringement of religious
freedom may be justified, and only to the smallest
extent necessary to avoid the danger [VICTORIANO vs.
ELIZALDE ROPE WORKERS UNION (235 SCRA 197)].
Freedom of religion implies respect for every creed.
No one is privileged to characterize the actuation of its
adherents in a derogatory sense [INK vs. Gironella (106
SCRA 1)].
The exercise of religious freedom can be regulated
by the state when it will bring about the clear and
present danger of some substantive evil w/c the state
is duty bound to prevent --- serious detriment to the
more overriding interest of public health, public morals,
or public welfare.

TWO ASPECTS OF RELIGIOUS PROFESSION AND


WORSHIP:
1.
FREEDOM TO BELIEVE

absolute as long as the belief is confined


w/i the realm of thought
2.
FREEDOM TO ACT ON ONES BELIEF

is subject to regulation where the belief


is translated into external acts that afect
the public welfare

where the individual externalizes his


beliefs in acts/ omissions that afect the
public, his freedom to do becomes
subject to state authority
a. the state cannot compel a person to do
something w/c his religion prohibits
b. the state cannot prohibit a person from
doing
something
w/c
his
religion
commands

Religious freedom can be enjoyed only w/ a proper


regard for the rights of others.
The police power can be exercised to prevent
religious practices inimical to society even if such
practices are pursued out of sincere religious
conviction and not merely for the purpose of evading
the reasonable requirements/prohibitions of law.
If the exercise of religious belief clashes w/ the
established institution of society and w/ the law, then
the former must yield and give way to the latter.
As long as it can be shown that the exercise of the
right does not impair public welfare, the attempt of the
state to regulate/ prohibit such right would be
unconstitutional.
The constitutional guaranty of FREE EXERCISE and
enjoyment of religious profession and worship carries
w/ it the right to disseminate religious information
[AMERICAN BIBLE SOCIETY vs. CITY OF MANILA (101
Phil. 386].
The test to determine w/c shall prevail between
religious freedom and the powers of the state is the
TEST OF REASONABLENESS.
The
constitutional
PROHIBITION
AGAINST
RELIGIOUS TESTS is aimed against clandestine
attempts on the part of the govt. to prevent a person
from exercising his civil or political rights because of
his religious beliefs.
To allow religious tests would have the efect of
formal or practical establishment of particular religious
faiths... w/ consequent burdens imposed on the free
exercise of the faiths of non-favored believers.

FREE EXERCISE CLAUSE


Q: Who decides whether something is religious or not?
ACADCOM 2010; Contributors: Gene Geocaniga, Jarissa Guiani, Darlene Magabilen
TAU MU Page 79 of 179

The freedom to act in accordance with one's


belief has two parts: the affirmative aspect and the
negative aspect. The affirmative aspect means the
state cannot compel the person to do something
which his religion prohibits. The negative aspect
means the state cannot prevent a person from doing
what his religion commands.

TAU MU
TAU MU
TAU MU

Facts: Respondent Iglesia ng Dios kay Cristo Jesus,


Haligi at Suhay ng Katotohanan (Church of God in Jesus
Christ, the Pillar and Ground of Truth), is a non-stock
religious society or corporation. In 1976, one Eliseo
Soriano and several other members of respondentcorporation disassociated themselves from the latter
and succeeded in registering a new non-stock religious
society or corporation named, Haligi ng Dios kay Kristo
Hess, Haligi at Saligan ng Katotohan in 1977.
Respondent-corporation filed with the SEC a petition to
compel Haligi ng Dios kay Kristo Hess, Haligi at Saligan
ng Katotohan to change its corporate name to another
name that is not similar or identical to any name
already used by a corporation, partnership or
association registered with the Commission. Petitioner
is compelled to change its corporate name and be
barred from using the same or similar name on the
ground that the same causes confusion among their
members as well as the public. SEC rendered a
decision ordering petitioner to change its name. The
CA rendered the assailed decision and affirmed the
decision of SEC En Banc.

TAU MU

ANG MGA KAANIB vs. IGLESIA


372 SCRA 172 (2001)

TAU MU

CASES: FREEDOM TO EXERCISE CLAUSE

TAU MU

NO RELIGIOUS TEST CLAUSE - one does not have to


pass a religious test to be able to exercise one's civil or
political right. Since the 1987 Constitution came out,
there is no more religious test. A priest can run for a
public office.

TAU MU
TAU MU

NOTE: Tolentino vs. Sec. case vs. American Bible


Society: The removal of the TAX exemption is
unconstitutional because this is an encroachment on
the freedom of religion. But the imposition of VAT is not
unconstitutional because the VAT here is not imposed
on sale, bartering or exchange of goods. It is not a
license tax. If it is a license tax, if you need to pay for
something in order to practice a certain thing it is
unconstitutional. But you don't need the VAT to
practice your religion, it is merely incidental.

The Fraternal
Ateneo de Davao

TAU MU
TAU MU

The application of the compelling state interest


test
could
result
to
three
situations
of
accommodation:
First,
mandatory
accommodation would result if the Court finds that
accommodation is required by the Free Exercise
Clause. Second, if the Court finds that the State may,
but is not required to, accommodate religious interests,
permissive accommodation results. Finally, if the
Court finds that that establishment concerns prevail
over potential accommodation interests, then it must
rule that the accommodation is prohibited.

KITY

TAU MU TAU MU TAU MU


TAU MU TAU MU TAU MU

CONSTITUTIONAL LAW II
Order of Saint Thomas More
Atty. Philip John Pojas/Atty. Rovyne G. Jumao-as, RN
University College of Law
A:
1. GERONA Case - court
2. EBRALINAG Case it seems that it was still
the court
3. Justice CRUZ it should not be left to the
court because it is something
w/c is intensely personal.

Ruling: The additional words Ang Mga Kaanib and


Sa Bansang Pilipinas Inc. in petitioners name are, ,
as correctly observed by the SEC, merely descriptive of
and also referring to the members or kaanib, of
respondent who are likewise residing in the Philippines.
These words can hardly serve as an efective
diferentiating medium necessary to avoid confusion or
difficulty in distinguishing petitioner from respondent.
This is especially so, since both petitioner and
respondent corporations are using the same acronyms
--- H.S.K, not to mention the fact that both are
espousing religious beliefs and operating in the same
place. The fact that there are other non-stock religious
societies or corporations using the names Church of
the Living God, Inc., Church of God Jesus Christ the Son
of God the Head, Church of God in Christ and By the
Holy Spirit and other similar names, is of no
consequence, it does not authorize the use by
petitioner of the essential and distinguishing feature of
respondents registered and protected corporate name.
Ordering petitioner to change its corporate name is not
a violation of its constitutionally guaranteed right to
religious freedom. In so doing, the SEC merely
compelled petitioner to abide by one of the SEC
guidelines in the approval of partnership and corporate
names, namely its undertaking to manifest its
willingness to change its corporate name in the event
that another person, firm or entity has a acquired a
prior right to the use of the said firm name or one
deceptively or confusingly similar to it.
IDCP, INC. vs. OFFICE
405 SCRA 497 (2003)
Facts: Petitioner, a corporation that operates under
DSWD, is a non-governmental organization that
extends voluntary services to the Filipino people,
especially to Muslim communities. Among the functions
petitioner carries out is to conduct seminars, orient
manufacturers. In 2001, respondent Office of the
Executive Secretary issued EO 46 creating the
Philippine Halal Certification Scheme and designating
respondent Office on Muslim Afairs to oversee its
implementation. Under the EO, respondent OMA has
the exclusive authority to issue halal certificates and
perform other related regulatory activities. Petitioner
contends that the subject EO violates the constitutional
provision on the separation of Church and State and
that it is unconstitutional for the government to
formulate policies and guidelines on the halal
certification scheme because said scheme is a function
only religious organizations, entity or scholars can
lawfully and validly perform for the Muslims. TAU MU
Issue: WON the EO is violates the constitutional
provision as to freedom of religion.

TAU MU

Ruling: The SC grants the petition. OMA deals with the


societal, legal, political and economic concerns of the
Muslim community as a national cultural community
and not as a religious group. Thus, bearing in mind the
constitutional barrier between the Church and State,
the latter must make sure that OMA does not intrude
into purely religious matters lest it violate the nonestablishment clause and the free exercise of religion
provision found in Article 3, section 5 of the 1987
Constitution. Freedom of religion was accorded
preferred status, well aware that it is designed to
protect the broadcast possible liberty of conscience, to
allow each man to believe as his conscience directs, to
profess his beliefs and to alive as he believes he ought
to live, consistent with the liberty of others and with
the common good. Without doubt, classifying a food
Issue: WON SECs order to petitioner to change its
product as halal is a religious function because the
corporate name violated its right to religious freedom.
standards used are drawn from the Quaran and
Islamic beliefs. By giving OMA the exclusive power to
ACADCOM 2010; Contributors: Gene Geocaniga, Jarissa Guiani, Darlene Magabilen
TAU MU Page 80 of 179

TAU MU

TAU MU

KITY

The Fraternal
Ateneo de Davao

TAU MU TAU MU TAU MU


TAU MU TAU MU TAU MU

CONSTITUTIONAL LAW II
Order of Saint Thomas More
Atty. Philip John Pojas/Atty. Rovyne G. Jumao-as, RN
University College of Law
classify food products as halal, EO 46 encroached on
the religious freedom of Muslim organizations like
herein petitioner to interpret for Filipino Muslim what
food products are fit for Muslim consumption. Also, by
arrogating to itself the task of issuing halal
certifications, the State has in efect forced Muslims to
accept its own interpretation of the Quran and Sunnah
on halal food. Only the prevention of an immediate and
grave danger to the security and welfare of the
community can justify the infringement of religious
freedom. If the government fails to show the
seriousness and immediacy of the threat, State
intrusion is constitutionally unacceptable. In a society
with a democratic framework like ours, the State must
minimize its interference with the afairs of its citizens
and instead allow them to exercise reasonable freedom
of personal and religious activity. There is no
compelling justification for the government to deprive
Muslim organizations, like herein petitioner, of their
religious right to classify a product as halal, even on
the premise that the health of Muslim Filipinos can be
efectively protected by assigning to OMA the exclusive
power to issue halal certifications. The protection and
promotion of the Muslim Filipinos right to health are
already provided for in existing laws and ministered to
by government agencies charged with ensuring that
food products relased in the market are fit for human
consumption, properly labeled and safe. Unlike EO 46,
these laws do not encroach on the religious freedom of
Muslims. With these regularity bodies given detailed
functions on how to screen and check the quality and
safety of food products, the perceived danger against
the health of Muslim and non-Muslim Filipinos alike is
totally avoided. The halal certifications issued by
petitioner and similar organizations come forward as
the official religious approval of a food product fit for
Muslim consumption.

three witnesses. At the time Escritor executed her


pledge, her husband was still alive but living with
another woman. Luciano was likewise married at that
time, but had been separated in fact from his wife.
During her testimony, Escritor volunteered to present
members of her congregation to confirm the
truthfulness of their Declarations of Pledging
Faithfulness,
but
Judge
Caoibes
deemed
it
unnecessary and considered her identification of her
signature and the signature of Quilapio sufficient
authentication of the documents.
Issue: WON respondents right to religious freedom
was abridged when she was held administratively
liable for having illicit relations.

TAU MU
TAU MU

Ruling: The Declaration of Pledging of Faithfulness


executed by the respondent and her mate greatly
afect the administrative liability of respondent.
Jehovahs Witnesses admit and recognize the
supremacy of the proper public authorities in the
marriage arrangement. Freedom of choice guarantees
the liberty of the religious conscience and prohibits any
degree of compulsion or burden, whether direct or
indirect, in the practice of ones religion. The Free
Exercise Clause principally guarantees voluntarism,
although the Establishment Clause also assures
voluntarism by placing the burden of the advancement
of religious groups on their intrinsic merits and not on
the support of the state. In interpreting the Free
Exercise Clause, the realm of belief poses no difficulty.
The realm of belief and creed is infinite and limitless
bounded only by ones imagination and thought. So is
the freedom of belief, including religious belief,
limitless and without bounds.
Court has ruled that government employees
engaged in illicit relations are guilty of disgraceful and
immoral conduct for which he/she may be held
ESTRADA vs. ESCRITOR
administratively liable. Respondent Escritor does not
408 SCRA 1 (2003)
claim that there is error in the settled jurisprudence
that an illicit relation constitutes disgraceful and
Facts: Alejandro Estrada wrote to Judge Caoibes
immoral conduct for which a government employee is
requesting for an investigation of rumors that
held liable. Nor is there an allegation that the norms of
respondent Soledad Escritor, court interpreter in said
morality with respect to illicit relations have shifted
court, is living with a man not her husband. They
towards leniency from the time these precedent cases
allegedly have a child of eighteen to twenty years old.
were decided. The Court finds that there is no such
He filed the charge against Escritor as he believes that
error or shift, thus the SC find no reason to deviate
she is committing an immoral act that tarnishes the
from these rulings that such illicit relationship
image of the court, thus she should not be allowed to
constitutes disgraceful and immoral conduct
remain employed therein as it might appear that the
punishable under the Civil Service Law. Respondent
court condones her act.
having admitted the alleged immoral conduct, she, like
Judge Caoibes referred the letter to Escritor
the respondents in the above-cited cases, could be
who stated that there is no truth as to the veracity of
held administratively liable. However, there is a
the allegation and challenged Estrada to appear in
distinguishing factor that sets the case at bar apart
the open and prove his allegation in the proper forum.
from the cited precedents, i.e., as a defense,
Judge Caoibes set a preliminary conference. Escritor
respondent invokes religious freedom since her
moved for the inhibition of Judge Caoibes from hearing
religion, the Jehovahs Witnesses, has, after thorough
her case to avoid suspicion and bias as she previously
investigation, allowed her conjugal arrangement with
filed an administrative complaint against him and said
Luciano based on the churchs religious beliefs and
case was still pending in the Office of the Court
practices.
Administrator (OCA). Escritors motion was denied.
There is no doubt that choosing between
The preliminary conference proceeded with both
keeping her employment and abandoning her religious
Estrada and Escritor in attendance. Respondent
belief and practice and family on the one hand, and
Escritor testified that when she entered the judiciary in
giving up her employment and keeping her religious
1999, she was already a widow, her husband having
practice and family on the other hand, puts a burden
died in 1998. She admitted that she has been living
on her free exercise of religion. The burden on
with Luciano without the benefit of marriage for twenty
respondent in the case at bar is even greater as the
years and that they have a son. But as a member of
price she has to pay for her employment is not only her
the religious sect known as the Jehovahs Witnesses
religious precept but also her family which, by the
and the Watch Tower and Bible Tract Society, their
Declaration Pledging Faithfulness, stands honorable
conjugal arrangement is in conformity with their
before God and men.However, even if the Court
religious beliefs. In fact, after ten years of living
deems sufficient respondents evidence on the
together, she executed a Declaration of Pledging
sincerity of her religious belief and its centrality in her
Faithfulness.
faith, the case at bar cannot still be decided using the
Escritors partner, Luciano, executed a similar
compelling state interest test. The case at bar is one
pledge. Both pledges were executed and signed by
of first impression, thus the parties were not aware
ACADCOM 2010; Contributors: Gene Geocaniga, Jarissa Guiani, Darlene Magabilen
TAU MU Page 81 of 179

TAU MU
TAU MU

TAU MU

TAU MU

TAU MU

TAU MU

TAU MU

TAU MU

TAU MU

TAU MU

TAU MU

TAU MU
TAU MU
TAU MU
TAU MU
TAU MU
TAU MU
TAU MU

Ruling: The Constitution adheres to the benevolent


neutrality
approach
that
gives
room
for
accommodation of religious exercises as required by
the Free Exercise Clause. Thus, in arguing that
respondent should be held administratively liable as
the arrangement she had was illegal per se because,
by universally recognized standards, it is inherently or
by its very nature bad, improper, immoral and contrary
to good conscience, the Solicitor General failed to
appreciate that benevolent neutrality could allow for
accommodation of morality based on religion, provided
it does not ofend compelling state interests. Finally,
even assuming that the OSG has proved a compelling
state interest, it has to further demonstrate that the
state has used the least intrusive means possible so
that the free exercise is not infringed any more than

TAU MU

Issue: WON the compelling state interest test must be


applied.

TAU MU

Facts: Respondent Soledad Escritor once again stands


before the Court invoking her religious freedom and
her Jehovah God in a bid to save her family united
without the benefit of legal marriage - and livelihood.
The State, on the other hand, seeks to wield its power
to regulate her behavior and protect its interest in
marriage and family and the integrity of the courts
where respondent is an employee.

TAU MU
TAU MU

ESTRADA vs. ESCRITOR


492 SCRA 1 (2006)

The Fraternal
Ateneo de Davao

TAU MU
TAU MU

Dispositive portion: The case is REMANDED to the


Office of the Court Administrator. The Solicitor General
is ordered to intervene in the case where it will be
given the opportunity (a) to examine the sincerity and
centrality of respondents claimed religious belief and
practice; (b) to present evidence on the states
compelling interest to override respondents religious
belief and practice; and (c) to show that the means the
state adopts in pursuing its interest is the least
restrictive to respondents religious freedom.

KITY

TAU MU TAU MU TAU MU


TAU MU TAU MU TAU MU

CONSTITUTIONAL LAW II
Order of Saint Thomas More
Atty. Philip John Pojas/Atty. Rovyne G. Jumao-as, RN
University College of Law
of the burdens of proof they should discharge in the
Courts use of the compelling state interest test. , it
is inappropriate for the complainant, a private person,
to present evidence on the compelling interest of the
state. The burden of evidence should be discharged by
the proper agency of the government which is the
Office of the Solicitor General. To properly settle the
issue in the case at bar, the government should be
given the opportunity to demonstrate the compelling
state interest it seeks to uphold in opposing the
respondents stance that her conjugal arrangement is
not immoral and punishable as it comes within the
scope of free exercise protection. Should the Court
prohibit and punish her conduct where it is protected
by the Free Exercise Clause, the Courts action would
be an unconstitutional encroachment of her right to
religious freedom.
The SC cannot therefore simply take a passing
look at respondents claim of religious freedom, but
must instead apply the compelling state interest
test. The government must be heard on the issue as it
has not been given an opportunity to discharge its
burden of demonstrating the states compelling
interest which can override respondents religious
belief and practice. To repeat, this is a case of first
impression where we are applying the compelling
state interest test in a case involving purely religious
conduct. The careful application of the test is
indispensable as how we will decide the case will make
a decisive diference in the life of the respondent who
stands not only before the Court but before her
Jehovah God.

necessary to achieve the legitimate goal of the state,


i.e., it has chosen a way to achieve its legitimate state
end that imposes as little as possible on religious
liberties. Again, the Solicitor General utterly failed to
prove this element of the test. Other than the two
documents ofered as cited above which established
the sincerity of respondents religious belief and the
fact that the agreement was an internal arrangement
within respondents congregation, no iota of evidence
was ofered. In fact, the records are bereft of even a
feeble attempt to procure any such evidence to show
that the means the state adopted in pursuing this
compelling interest is the least restrictive to
respondents religious freedom.
Thus, the SC held that in this particular case
and under these distinct circumstances, respondent
Escritors conjugal arrangement cannot be penalized as
she has made out a case for exemption from the law
based on her fundamental right to freedom of religion.
The Court recognizes that state interests must be
upheld in order that freedoms - including religious
freedom - may be enjoyed. In the area of religious
exercise as a preferred freedom, however, man stands
accountable to an authority higher than the state, and
so the state interest sought to be upheld must be so
compelling that its violation will erode the very fabric
of the state that will also protect the freedom. In the
absence of a showing that such state interest exists,
man must be allowed to subscribe to the Infinite.
RE: REQUEST OF MUSLIM EMPLOYEES IN THE
DIFFERENT COURTS IN ILIGAN CITY (RE: OFFICE
HOURS)
477 SCRA 648
Facts: Several Muslim employees in the diferent
courts in the said city requested to Judge Salazar that
they be allowed to enjoy certain privileges during the
Holy Month of Ramadhan. Judge Salazar expressed his
conformity with the first request, i.e., allowing them to
hold office from 7:30 a.m. to 3:30 p.m. without any
break during the month of Ramadan. However, he
expressed some misgivings about the second request,
i.e., excusing them from work from 10:00 a.m. to 2:00
p.m. every Friday during the entire calendar year. In
support of their requests, the Muslim employees invoke
PD 291 as amended by P.D. No. 322. The avowed
purpose of P.D. No. 291 was to reinforce national unity
by recognizing Muslim holidays and making them part
of our national holidays.
Issue: WON the freedom to exercise ones religion was
abridged in this case.
Ruling: The Court recognizes that the observance of
Ramadan and the Friday Muslim Prayer Day is integral
to the Islamic faith. However, while the observance of
Ramadan and allowing the Muslim employees in the
Judiciary to hold flexible office hours from 7:30 a.m. to
3:30 p.m. without any break during the month of
Ramadan finds support in Section 3 (a) of P.D. No. 291,
as amended by P.D. No. 322, there is no such basis to
excuse them from work from 10:00 a.m. to 2:00 p.m.
every Friday, the Muslim Prayer Day, during the entire
calendar year.
On the other hand, the need of the State to
prescribe government office hours as well as to enforce
them uniformly to all civil servants, Christians and
Muslims alike, cannot be disregarded. Underlying
Section 5, Rule XVII of the Omnibus Rules
Implementing Book V of E.O. No. 292 is the interest of
the general public to be assured of continuous
government service during office hours every Monday
through Friday. The said rule enjoins all civil servants,
of whatever religious denomination, to render public

ACADCOM 2010; Contributors: Gene Geocaniga, Jarissa Guiani, Darlene Magabilen


TAU MU Page 82 of 179

TAU MU
TAU MU
TAU MU

Ruling: The Court ruled that there was no clear and


present danger posed if they were exempted from the
requirement. There was no danger that they will
produce citizens who have no love of country if the
government allowed a religious exemption. The danger
predicted in the Gerona case has come to pass. The
Jehovahs Witnesses is a small group and it will not
shake our part of the world if we exempt them from
requirement. If your religion is against the saluting of
the flag, then you are entitled to it because of the free
exercise clause. There is no clear and present danger.
You are only required to stand and this afects only a
very small group.

TAU MU
TAU MU
TAU MU
TAU MU
TAU MU

Section 6. The liberty of abode and of changing


the same within the limits prescribed by law
shall not be impaired except upon lawful order of
the court. Neither shall the right to travel be
impaired except in the interest of national

TAU MU

Ruling: The Court ruled that the Ordinance 2925


violated freedom of religion. The government cannot
impose some form of tax on somebody for the exercise
of his religion. American Bible was a missionary
organization which sells bibles as part of propagating
their faith. It was only making a small profit from the
sale of bibles. If they were making a lot of profit, then
that would be the time that the government should
impose a tax on them since it would not anymore be
part of propagating the faith.

TAU MU

Facts: American Bible was a religious corporation,


selling bibles and other religious articles. They were
required to obtain a license and pay the corresponding
fee for being engaged in the sale of merchandise.

TAU MU

Facts: There were students, all members of the


Jehovahs Witnesses, who were expelled for refusing to
salute the flag, sing the national anthem and recite the
oath of allegiance. Their religion prohibited them from
doing so.

AMERICAN BIBLE vs. CITY OF MANILA


101 PHIL. 386 (1957)

The Fraternal
Ateneo de Davao

TAU MU
TAU MU

EBRALINAG vs. SECRETARY


219 SCAR 256 (1993)

KITY

TAU MU TAU MU TAU MU


TAU MU TAU MU TAU MU

CONSTITUTIONAL LAW II
Order of Saint Thomas More
Atty. Philip John Pojas/Atty. Rovyne G. Jumao-as, RN
University College of Law
service of no less than eight hours a day or forty (40)
hours a week.
To allow the Muslim employees in the Judiciary
to be excused from work from 10:00 a.m. to 2:00 p.m.
every Friday (Muslim Prayer Day) during the entire
calendar year would mean a diminution of the
prescribed government working hours. For then, they
would be rendering service twelve (12) hours less than
that required by the civil service rules for each month.
Further, this would encourage other religious
denominations to request for similar treatment. The
performance of religious practices, whether by the
Muslim employees or those belonging to other religious
denominations, should not prejudice the courts and the
public. Indeed, the exercise of religious freedom does
not exempt anyone from compliance with reasonable
requirements of the law, including civil service laws.
Hence, the SC GRANTED the request to allow
the Muslim employees in the Judiciary to hold office
hours from 7:30 a.m. to 3:30 p.m. without break during
the month of Ramadan pursuant to Section 3 (a) of
Presidential Decree No. 291, as amended by
Presidential Decree No. 322.

security, public safety or public health, as may


be provided by law.
RIGHTS GUARANTEED UNDER SECTION 6:
1. Freedom to choose and change ones place of
abode (Liberty of abode) - the right of a person to have
his home, or to maintain or change his home or
dwelling in whatever place he has chosen.
LIMITATION: it must be within the limits prescribed
by law. You cannot move to the public plaza.
2. Freedom to travel within the country and
outside - he right given to a person to go wherever he
pleases without interference from anyone.
FREEDOM OF MOVEMENT:
The purpose of the guaranty is to further
emphasize the individuals liberty as safeguarded in
general terms by the due process clause.
Q: What are the liberties guaranteed under this clause?
A: The liberties under this clause include the
a. right to choose ones residence,
b. to leave it whenever he pleases, and
c. to travel wherever he wills.

Right: Liberty of abode


Manner of curtailment: Lawful order of the court
and within the limits prescribed by law.

Right: Right to travel


Manner of curtailment: May be curtailed even
by administrative officers (ex. passport officers) in the
interest of national security, public safety, or public
health, as may be provided by law.
General Rule: The right to travel shall not be
impaired.
Exceptions: When can the right to travel be
restricted? Who can validly restrict this right?
1. Congress, but on the grounds of (a) national
security, (b) public safety, and (c) public health.
2. The Court.-- It is an inherent power of the court. The
limitation of the right to travel it need not be grounded
on the grounds of national security, public safety, and
public health. It is inherent for the proper
administration of justice, even if there is no express
grant from the constitution. The guarantee that the
accused will appear for trial and the issuance of the
bail bond serves as the lawful order for the refusal of
the right to travel.
3. Administrative and Executive officials i.e. Bureau of
Immigration if there is a law. (Note: Implementation of
the law enacted by the Congress, which again, must be
based on the 3 grounds in number 1)

The right to travel and the liberty of abode are


distinct from the right to return to ones country, as
shown by the fact that the Declaration of Human
Rights and the Covenant on Human Rights have
separate guarantees for these. Hence, the right to
return to ones country is not covered by the specific
right to travel and liberty of abode. [MARCOS vs.
MANGLAPUS]

The right to travel can be impaired even w/o lawful


court order BUT the executive officer must not be
armed w/ arbitrary discretion to impose limitations. He
can impose limits only on the basis of national security,
public safety, or public health and as may be provided
by law.

IMPAIRMENT may be subject to judicial review.

ACADCOM 2010; Contributors: Gene Geocaniga, Jarissa Guiani, Darlene Magabilen


TAU MU Page 83 of 179

MEASURE OF ALLOWABLE IMPAIRMENT:


a. necessity in the interest of
a.national security,
b.public safety, or
c. public health; and
d.explicit provisions of statutory law or
the Rules of Court.

TAU MU
TAU MU
TAU MU
TAU MU
TAU MU

Facts: Petitioners filed a Petition for Declaratory


Judgment with Application for Temporary Restraining
Order and Injunction which sought the declaration of
nullity of the administrative issuances for being
inconsistent with the provisions of Republic Act 2000,
entitled "Limited Access Highway Act" enacted in 1957
such as (a) DPWH Administrative Order No. 1, Series of
1968; (b) DPWH Department Order No. 74, Series of
1993; and, (c) Art. II, Sec. 3(a) of the Revised Rules on
Limited Access Facilities promulgated in 199[8] by the
DPWH thru the Toll Regulatory Board (TRB). Previously,
pursuant to its mandate under R.A. 2000, DPWH issued
DO No. 215 declaring the Manila-Cavite (Coastal Road)
Toll Expressway as limited access facilities.
Accordingly, petitioners filed an Amended
Petition wherein petitioners sought the declaration of
nullity of the aforesaid administrative issuances.
Moreover, petitioners prayed for the issuance of a
temporary restraining order and/or preliminary
injunction to prevent the enforcement of the total ban
on motorcycles along the entire breadth of North and
South Luzon Expressways and the Manila-Cavite
(Coastal Road) Toll Expressway under DO 215.
Thereafter, the trial court, after due hearing, issued an
order granting petitioners application for preliminary
injunction. The DPWH acting thru the TRB, issued
Department Order No. 123 allowing motorcycles with
engine displacement of 400 cubic centimeters inside
limited access facilities (toll ways).

TAU MU

MIRASOL vs. DPWH


490 SCRA 318 (2006)

TAU MU

CASES: RIGHT TO TRAVEL/LIBERTY OF ABODE

TAU MU
TAU MU

Q: Can the accused be granted the right to travel?


A: YES if
1. he can show to the court the URGENCY to
travel
2. in the application, he must state the
DURATION of his travel
3. he must obtain the CONSENT OF HIS
SURETY.

The Fraternal
Ateneo de Davao

TAU MU
TAU MU

This right should not be construed as delimiting the


inherent power of the Courts to use all means
necessary to carry their orders into efect in criminal
cases pending before them.

While this right covers both the right to exit from


and entry into the country, aliens cannot claim the
same right because every sovereign nation has the
inherent power essential to self-preservation to forbid
the entrance of foreigners w/i its dominion, or to admit
them only in such cases and upon such conditions as it
may see fit to prescribe [EKIU vs. US (42 US 651)].

Norms for admission of aliens into the country are


political matters virtually beyond the reach of judicial
review.

KITY

TAU MU TAU MU TAU MU


TAU MU TAU MU TAU MU

CONSTITUTIONAL LAW II
Order of Saint Thomas More
Atty. Philip John Pojas/Atty. Rovyne G. Jumao-as, RN
University College of Law

Thus a person facing criminal charges may be


restrained from leaving the country or, if abroad,
compelled to return [ZEMEL vs. RUSK (381 US 1)].

Ruling: A toll way is not an ordinary road. As a facility


designed to promote the fastest access to certain
destinations, its use, operation, and maintenance
require close regulation. Public interest and safety
require the imposition of certain restrictions on toll
ways that do not apply to ordinary roads. As a special
kind of road, it is but reasonable that not all forms of
transport could use it. The right to travel does not
mean the right to choose any vehicle in traversing a
toll way. The right to travel refers to the right to move
from one place to another. Petitioners can traverse the
toll way any time they choose using private or public
four-wheeled vehicles. Petitioners are not denied the
right to move from Point A to Point B along the toll way.
Petitioners are free to access the toll way, much as the
rest of the public can. The mode by which petitioners
wish to travel pertains to the manner of using the toll
way, a subject that can be validly limited by regulation.
Petitioners themselves admit that alternative
routes are available to them. Their complaint is that
these routes are not the safest and most convenient.
Even if their claim is true, it hardly qualifies as an
undue curtailment of their freedom of movement and
travel. The right to travel does not entitle a person to
the best form of transport or to the most convenient
route to his destination. The obstructions found in
normal streets, which petitioners complain of (i.e.,
potholes, manholes, construction barriers, etc.), are not
sufered by them alone. Moreover, contrary to what the
petitioners claim that their possession of a drivers
license from the LTO and the fact that their vehicles are
registered with that office entitle them to use all kinds
of roads in the country, the SC held that there exists no
absolute right to drive because this privilege is heavily
regulated. Only a qualified group is allowed to drive
motor vehicles: those who pass the tests administered
by the LTO. A drivers license issued by the LTO merely
allows one to drive a particular mode of transport. It is
not a license to drive or operate any form of
transportation on any type of road. Vehicle registration
in the LTO on the other hand merely signifies the
roadworthiness of a vehicle. This does not preclude the
government from prescribing which roads are
accessible to certain vehicles.
YAP Jr. vs. CA, PEOPLE OF THE PHILIPPINES
358 SCRA 564 (2001)
Facts: For misappropriating amounts, petitioner was
convicted of estafa. The RTC denied the motion to fix
bail for his provisional liberty under the cash bond he
had filed earlier in the proceedings. CA granted the
said
motion
but
with
conditions
that
a
certification/guaranty from the Mayor of the place of
his residence that he is a resident of the area and that
he will remain to be a resident therein until final
judgment is rendered or in case he transfers residence,
it must be prior notice to the court, and that a hold
departure order be issued against and that he
surrender his passport.
Issue: WON petitioners constitutional liberty of abode
and travel in imposing the other conditions for the
grant of bail was unduly restricted.

TAU MU

Ruling: Petitioner contests the condition that he


secures such certification or guaranty from the Mayor,
claiming that the same violates his liberty of abode and
travel. Notably, petitioner does not question the holddeparture order which prevents him from leaving the
Philippines unless expressly permitted by the court
which issued the order. The right to change abode and
Issue: WON the prohibition on the use of motorcycles
travel within the Philippines are not absolute rights.
in toll ways unduly deprive the petitioners of their right
The order of the Ca releasing petitioner on bail
to travel.
constitutes such lawful order as contemplated by the
provision in the Constitution. It is simply consistent
ACADCOM 2010; Contributors: Gene Geocaniga, Jarissa Guiani, Darlene Magabilen
TAU MU Page 84 of 179

TAU MU

MANOTOC vs. CA
142 SCRA 149 (1987)

TAU MU
TAU MU
TAU MU

This right to return is part of those generally accepted


principles.

TAU MU

Ruling 1: The Court held that the right to travel means


only the right to leave. It does not include the right to
return to the country. These are two distinct rights. The
right to return is not protected by the Bill of Rights but
is protected by the Philippine Law by virtue of the
Principle of Incorporation. The principle of incorporation
states
that
generally
accepted
principles
of
international law is part of the law of the land.

TAU MU

Facts: Former President Marcos sought to compel the


Secretary of Foreign Afairs to issue him travel
documents so that he could return to the Philippines
from his exile in Hawaii. He argued that the decision of
then Pres. Cory Aquino to bar his return violates his
right to travel as guaranteed by the Bill of Rights.
Note: There was no court order, nor was there a law
limiting his right to travel. TAU MU

TAU MU

MARCOS vs. MANGLAPUS


177 SCRA 668 (1989)

TAU MU

Ruling: The Court held that a court has the power to


restrict the right to travel, with or without an express
grant from the Constitution if there is a criminal case
before it. This is necessary for the efficient
administration of justice. The power is inherent in the
courts. This is an implied power in the administration of
justice. If the courts would have no right to limit the
right to travel, the administration of justice would not
be efective.

TAU MU

Facts: Silverio was charged with a criminal case and


he posted bail. Two years later, the government filed a
motion to cancel his passport because he could not be
arraigned as he was abroad every time the
arraignment was set. The trial court granted the
motion. He now contends that a court can only restrict
the right to travel but only on the ground of national
security, public safety or public health.

TAU MU

SILVERIO vs. CA
195 SCRA 760 (1991)

TAU MU
TAU MU

Ruling: The Court held that since he was on bail, the


Court can validly restrict his right to travel. The bail is
supposed to be used as guarantee that he will appear
on trial. If he leaved for the U.S., the Court can no
longer have jurisdiction over him. The conditions of the
bail bond serves as the lawful order for the refusal of
the right to travel. However, this is not absolute. The
Court may allow those out on bail to travel according to
the courts discretion.

The Fraternal
Ateneo de Davao

TAU MU
TAU MU

Facts: Manotoc was facing several charges of estafa


and he was out on bail. During the pendency of the
case, he filed a motion in court to allow him to travel to
the US in connection with some business transactions.
The prosecutor opposed his motion. He went to the SC
and contended that the denial violated his right to
travel.

KITY

TAU MU TAU MU TAU MU


TAU MU TAU MU TAU MU

CONSTITUTIONAL LAW II
Order of Saint Thomas More
Atty. Philip John Pojas/Atty. Rovyne G. Jumao-as, RN
University College of Law
with the nature and function of a bail bond, which is to
ensure that petitioner will make himself available at all
times whenever the Court requires his presence.
Besides, a closer look at the questioned condition will
show that petitioner is not prevented from changing
abode; he is merely required to inform the court in
case he does so.

If the right to return is protected by the


Constitution, it can only be restricted by law. In
this case, there was no law.
If the right to return is not protected by the Bill
of Rights but is protected by the Bill of Rights
but is protected by the Philippine Law, who can
restrict it?

Ruling 2: The Court ruled that there is no more need


of a law. the executive can limit it as part of the
Presidents implied powers (Doctrine of Residual
Powers). This was how the SC justified the power of
Pres. Aquino in barring the return of former Pres.
Marcos.
Note: Inherent power of the executive: (1) The power
to do anything not prohibited by the Constitution; and
(2) The power to do anything not prohibited by law.
Section 7. The right of the people to information on
matters of public concern shall be recognized. Access
to official records, and to documents and papers
pertaining to official acts, transactions, or decisions, as
well as to government research data used as basis for
policy development, shall be aforded the citizen,
subject to such limitations as may be provided by law.
Pricnciple of Transparency and full disclosure in
Government:
Article. 2, Section 28: Subject to reasonable
conditions prescribed by law, the State adopts and
implements a policy of full public disclosure of all its
transactions involving public interest.
Rights guaranteed under Section 7:
1. Right to information on matters of public concern The right only involves matters of public concern. So,
when one invokes this right, you have to consider first
whether what the person wants to find out is a matter
of public concern.
2. Right of access to official records and to documents
and papers pertaining to official acts, transactions or
decisions, as well also government research data used
as basis for policy development
Q: What may be the subject of the right of access?
A:
1. official records
2. documents and papers pertaining to official
acts, transactions, or decisions
3. government research data used as basis for
policy development
Q: Can the right to access be limited?
A: Yes, as may be provided by law.
Limitations: As may be provided by law; the
custodians of the record can regulate the manner of
access to the documents, such as time and place. But
the custodian cannot deny one access because they
are public documents.
Persons entitled to the above rights:
The first right is guaranteed to anyone, but the
second right is reserved only to citizens of the
Philippines. But if the alien wants to access records,
say there is a criminal record involving him, he can
access such records.
Discretion of government:
The government has discretion with respect to
the authority to determine what matters are of public
concern and the authority to determine the manner of
access to them.

ACADCOM 2010; Contributors: Gene Geocaniga, Jarissa Guiani, Darlene Magabilen


TAU MU Page 85 of 179

CONSTITUTIONAL LAW II
Order of Saint Thomas More
Atty. Philip John Pojas/Atty. Rovyne G. Jumao-as, RN
University College of Law

KITY

Ateneo de Davao

Matters of public concern:


It is a broad spectrum of subjects w/c the public
may want to know, either because these directly afect
their lives or simply because such matters arouse the
interest of an ordinary citizen.

of

the

TAU MU
TAU MU
TAU MU
TAU MU
TAU MU

By government research data what is meant are


findings of government-funded research.

TAU MU

TAU MU

1. NATIONAL
SECURITY
MATTERS
and
INTELLIGENCE INFORMATION
These include state secrets re: military,
diplomatic and other natl. security, and information on
inter-govt. exchanges prior to the conclusion of
treaties and executive agreements.
Where there is no need to protect state
secrets, the privilege to withhold documents and other
information may not be invoked, provided that they are
examined in strict confidence and given scrupulous
protection.
2. TRADE
SECRETS
and
BANKING
TRANSACTIONS
This is in pursuance to the Intellectual
Property Code (RA 8293) and the Secrecy of Bank
Deposits Act (RA 1405).
Exception: if there is a case filed
against a person and there is a lawful order of
the court)
3. INTER-GOVERNMENT EXCHANGES PRIOR
TO THE CONCLUSION OF TREATIES AND
EXECUTIVE AGREEMENTS
4. DIPLOMATIC CORRESPONDENCE
5. CLOSED DOOR CABINET MEETINGS
6. EXECUTIVE SESSIONS OF EITHER HOUSE
OF CONGRESS
7. INTERNAL
DELIBERATIONS
OF
THE
SUPREME COURT
8. CRIMINAL MATTERS
9. ETHICAL STANDARDS ACT prohibits public
officials and employees from divulging confidential or
classified information officially known to them by
reason of their office which is NOT made available to
the public

TAU MU

the right

TAU MU

on

TAU MU
TAU MU

Note: While access to official records may NOT be


prohibited, it may be regulated by statutory law or by
the inherent power of an officer to control his office and
the records under his custody... to exercise some
discretion as to the manner in w/c persons desiring to
inspect, examine, or copy the record may exercise their
rights

TAU MU
TAU MU

This is also related to Art. II Sec. 28 (Policy of


Disclosure) and the Principle of Transparency under PIL.
Access to information on matters of public concern
is essential to the proper exercise of freedom of
expression on such matters.
The citizen has a right to know what is going on in
the country and in his government so he can express
his views thereon knowledgeably and intelligently.
The purpose of these rights is to prevent abuse of
government power.

TAU MU TAU MU TAU MU


TAU MU TAU MU TAU MU

Note: This right is recognized but this does not mean


that the government agency who is obliged to supply
the information should supply the documents. The
constitution only guarantees access to the documents,
and not to compel the government to provide the
copies to any person exercising such right.

Recognized restrictions
people to information:

The Fraternal

In the exercise of the courts discretion that access


to court records may be permitted subject to the
supervisory and protective powers of the court,, the
following issues may be relevant:
(1) whether parties have interest in privacy;
(2) whether information is being sought for
legitimate purpose or for improper purpose;
(3) whether there is threat of particularly serious
embarrassment to party,
(4) whether information is important to public
health and safety;
(5) whether sharing of information among litigants
would promote fairness and efficiency;
(6) whether party benefiting from confidentiality
order is public entity or official; and
(7) whether case involves issues important to the
public.
[Case: HILADO vs. REYES 496 SCRA 282]
Remedies of persons unlawfully denied of the
right to information:
a. Mandamus
b. Damages under Articles 19 and 27 of
the Civil Code
Note: [CHAVEZ vs. PCGG] - Does the right to
information include the right to access to the terms of
Government negotiations prior to the conclusion of the
negotiation?
A: The public has the right to access to this
information, even the negotiation proper the public has
the right to that kind of information -- not only the
contract or the finished product. However, there should
be a distinction between proposal and inter-agency
communication. Inter-agency communications or
the process of formulating proposals is not
included in this right.
Note:
Just
distinguish
between
inter-agency
communications if the discussion is still within the
office, this is still not covered by the right to
information; but once they have agreed that this
(option) will be our proposal, then that is already
covered by the right to information (Chavez vs. PEA)
Note: The right to information, however, does not
extend to matters recognized as privileged information
under the separation of powers. The right does not also
apply to information on military and diplomatic secrets,
information afecting national security, and information
on investigations of crimes by law enforcement
agencies before the prosecution of the accused, which
courts have long recognized as confidential. The right
may also be subject to other limitations that Congress
may impose by law. (Chavez vs. PEA, 384 SCRA 152)
Valmonte v. Belmonte, Jr.: An essential element of
these freedoms is to keep open a continuing dialogue
or process of communication between the government
and the people. It is in the interest of the State that
the channels for free political discussion be maintained
to the end that the government may perceive and be
responsive to the peoples will. Yet, this open dialogue
can be efective only to the extent that the citizenry is
informed and thus able to formulate its will
intelligently. Only when the participants in the
discussion are aware of the issues and have access to
information relating thereto can such bear fruit.
CASES: RIGHT TO INFORMATION
ECHEGARAY vs. SECRETARY
297 SCRA 754 (October 12, 1998)

ACADCOM 2010; Contributors: Gene Geocaniga, Jarissa Guiani, Darlene Magabilen


TAU MU Page 86 of 179

The Fraternal
Ateneo de Davao

TAU MU
TAU MU
TAU MU
TAU MU
TAU MU
TAU MU

Issue: WON the petitioner was deprived of his right to


information of the execution procedure by a convict
through lethal injection.

KITY

TAU MU TAU MU TAU MU


TAU MU TAU MU TAU MU

CONSTITUTIONAL LAW II
Order of Saint Thomas More
Atty. Philip John Pojas/Atty. Rovyne G. Jumao-as, RN
University College of Law
Facts: The SC affirmed the conviction of petitioner
Echegaray for the crime of rape of the 10 year-old
daughter of his common-law spouse and the imposition
upon him of the death penalty for the said crime.
Petitioner
subsequently
filed
a
Motion
for
Reconsideration raising mainly factual issues, and on
its heels, a Supplemental Motion for Reconsideration
raising for the first time the issue of the
constitutionality of Republic Act No. 7659 (the death
penalty law) and the imposition of the death penalty
for the crime of rape. In the meantime, Congress had
seen it fit to change the mode of execution of the
death penalty from electrocution to lethal injection,
and passed Republic Act No. 8177, An Act Designating
Death By Lethal Injection As The Method Of carrying
Out Capital Punishment, Amending For The Purpose
Article 81 Of The Revised Penal Code, As Amended By
Section 24 Of Republic Act 7659. Pursuant to the
provisions of said law, the Secretary of Justice
promulgated the Rules and Regulations to Implement
Republic Act No. 8177 (Implementing Rules) and
directed the Director of the Bureau of Corrections to
prepare the Lethal Injection Manual. Petitioner filed a
Petition for Prohibition, Injunction and/or Temporary
Restraining Order to enjoin respondents Secretary of
Justice and Director of the Bureau of Prisons from
carrying out the execution by lethal injection of
petitioner under R.A. No. 8177 and its implementing
rules as these are unconstitutional and void.
Thereafter, petitioner filed a Motion for Leave
of Court to Amend and Supplement Petition with the
Amended and Supplemental Petition attached thereto,
invoking the additional ground of violation of equal
protection.

TAU MU

Ruling: Yes. The SC held that Section 19 fails to


provide for review and approval of the Lethal Injection
Manual by the Secretary of Justice, and unjustifiably
makes the manual confidential, hence unavailable to
interested parties including the accused/convict and
counsel. Section 19 of the Rules and Regulations to
Implement Republic Act No. 8177 provides that: SEC.
19. EXECUTION PROCEDURE. - Details of the
procedure prior to, during and after administering the
lethal injection shall be set forth in a manual to be
prepared by the Director. The manual shall contain
details of, among others, the sequence of events
before and after execution; procedures in setting up
the intravenous line; the administration of the lethal
drugs; the pronouncement of death; and the removal
of the intravenous system. xxx Said manual shall be
confidential and its distribution shall be limited to
authorized prison personnel.
Thus, the Supreme Court finds in the first
paragraph of Section 19 of the implementing rules a
veritable vacuum. The Secretary of Justice has
practically abdicated the power to promulgate the
manual on the execution procedure to the Director of
the Bureau of Corrections, by not providing for a mode
of review and approval thereof. Being a mere
constituent unit of the Department of Justice, the
Bureau of Corrections could not promulgate a manual
that would not bear the imprimatur of the
administrative superior, the Secretary of Justice as the
rule-making authority under R.A. No. 8177. Such
apparent abdication of departmental responsibility
renders the said paragraph invalid.
As to the second paragraph of section 19, the
Court finds the requirement of confidentiality of the
contents of the manual even with respect to the
convict unduly suppressive.
It sees no legal
impediment for the convict, should he so desire, to
ACADCOM 2010; Contributors: Gene Geocaniga, Jarissa
TAU MU Page 87 of 179

obtain a copy of the manual. The contents of the


manual are matters of public concern "which the public
may want to know, either because these directly afect
their lives, or simply because such matters naturally
arouse the interest of an ordinary citizen." The
incorporation in the Constitution of a guarantee of
access to information of public concern is recognition
of the essentiality of the free flow of ideas and
information in a democracy. In the same way that free
discussion enables members of society to cope with
the exigencies of their time, access to information of
general interest aids the people in democratic decisionmaking by giving them a better perspective of the vital
issues confronting the nation.
CHAVEZ vs. PCGG
299 SCRA 744 (1998)
Facts: Petitioner, invoking his constitutional right to
information and the correlative duty of the state to
disclose publicly all its transactions involving the
national interest, demands that respondents make
public any and all negotiations and agreements
pertaining to PCGGs task of recovering the Marcoses
ill-gotten wealth. He claims that any compromise on
the alleged billions of ill-gotten wealth involves an
issue of paramount public interest, since it has a
debilitating efect on the countrys economy that
would be greatly prejudicial to the national interest of
the Filipino people. Hence, the people in general have a
right to know the transactions or deals being contrived
and efected by the government. Respondents, on the
other hand, do not deny forging a compromise
agreement with the Marcos heirs. They claim that
petitioners action is premature because there is no
showing that he asked the PCGG to disclose the
negotiations and the Agreements. And even if he has,
PCGG may not yet be compelled to make any
disclosure since the proposed terms and conditions of
the agreements have not become efective and
binding.
Issue: WON the right to information includes access to
the terms of government negotiations prior to their
consummation or conclusion.

TAU MU
TAU MU
TAU MU
TAU MU
TAU MU
TAU MU

Ruling: The petition is imbued with merit. Access to


public documents and records is a public right and the
real parties in interest are the people themselves. The
instant petition is anchored on the right of the people
to information and access to official records,
documents and papers a right guaranteed under
Section 7, Article 3 of the 1987 Constitution. Because
of the satisfaction of the two basis requisites laid down
by decisional law to sustain petitioners legal standing,
i.e. (1) the enforcement of a public right, (2) espoused
by a Filipino citizen, the Court rules that the petition at
bar, should be allowed. There are no specific laws
prescribing the exact limitations within which the right
may be exercised or the correlative state duty may be
obliged. The following are some of the recognized
restrictions: (1) national security matters and
intelligence information, (2) trade secrets and banking
transactions, (3) criminal matters, and (4) other
confidential information. It is for the courts to
determine on a case by case basis whether the matter
at issue is of interest or importance, as it relates to or
afects the public. The recovery of the Marcoses
alleged ill-gotten wealth is a matter of public concern
and imbued with public interest. It is incumbent upon
the PCGG and its officers, as well as other government
representatives to disclose sufficient public information
on any proposed settlement they have decided to take
up with the ostensible owners and holders of ill-gotten
wealth.
Guiani, Darlene Magabilen

CHAVEZ vs. PEA


384 SCRA 152

KITY

The Fraternal
Ateneo de Davao

TAU MU TAU MU TAU MU


TAU MU TAU MU TAU MU

CONSTITUTIONAL LAW II
Order of Saint Thomas More
Atty. Philip John Pojas/Atty. Rovyne G. Jumao-as, RN
University College of Law

Issue: WON the constitutional right to information


includes official information on on-going negotiations
before a final agreement.

TAU MU
TAU MU

Ruling: Before the consummation of the contract, PEA


must, on its own and without demand from anyone,
Facts: The government, through the Commissioner of
disclose to the public matters relating to the disposition
Public Highways, signed a contract with the
of its property. These include the size, location,
Construction and Development Corporation of the
technical description and nature of the property being
Philippines to reclaim certain foreshore and ofshore
disposed of, the terms and conditions of the
areas of Manila Bay. The contract also included the
disposition, the parties qualified to bid, the minimum
construction of Phases I and II of the Manila-Cavite
price and similar information. PEA must prepare all
Coastal Road. CDCP obligated itself to carry out all the
these data and disclose them to the public at the start
works in consideration of fifty percent of the total
of the disposition process, long before the
reclaimed land. In 1977, then President Marcos issued
consummation
of
the
contract,
because
the
Presidential Decree No. 1084 creating PEA. PD No.
Government Auditing Code requires public bidding. If
1084 tasked PEA to reclaim land, including foreshore
PEA fails to make this disclosure, any citizen can
and submerged areas, and to develop, improve,
demand from PEA this information at any time during
acquire, x x x lease and sell any and all kinds of lands.
the bidding process.
On the same date, then President Marcos issued
Information, however, on on-going evaluation
Presidential Decree No. 1085 transferring to PEA the
or review of bids or proposals being undertaken by the
lands reclaimed in the foreshore and ofshore of the
bidding or review committee is not immediately
Manila Bay under the Manila-Cavite Coastal Road and
accessible under the right to information. While the
Reclamation Project.
evaluation or review is still on-going, there are no
In 1981, then President Marcos issued a
official acts, transactions, or decisions on the bids or
memorandum directing PEA to amend its contract with
proposals. However, once the committee makes its
CDCP. In 1988, President Aquino issued Special Patent
official recommendation, there arises a definite
No. 3517, granting and transferring to PEA the parcels
proposition on the part of the government. From this
of land so reclaimed under the Manila-Cavite Coastal
moment, the publics right to information attaches, and
Road and Reclamation Project containing a total area of
any citizen can access all the non-proprietary
1,915,894 square meters. Subsequently, the Register
information leading to such definite proposition.
of Deeds of the Municipality of Paraaque issued
A consummated contract is not a requirement
Transfer Certificates of Title in the name of PEA,
for the exercise of the right to information. Otherwise,
covering the three reclaimed islands known as the
the people can never exercise the right if no contract is
Freedom Islands. In 1995, PEA entered into a Joint
consummated, and if one is consummated, it may be
Venture Agreement with AMARI, a private corporation,
too late for the public to expose its defects. Requiring a
to develop the Freedom Islands. The JVA also required
consummated contract will keep the public in the dark
the reclamation of an additional 250 hectares of
until
the
contract,
which
may
be
grossly
submerged areas surrounding these islands to
disadvantageous to the government or even illegal,
complete the configuration in the Master Development
becomes a fait accompli. This negates the State
Plan of the Southern Reclamation Project-MCCRRP. PEA
policy of full transparency on matters of public
and AMARI entered into the JVA through negotiation
concern, a situation which the framers of the
without public bidding. The Board of Directors of PEA,
Constitution could not have intended.
Such a
in its Resolution No. 1245, confirmed the JVA.
requirement
will
prevent
the
citizenry
from
Thereafter, JVA was approved.
participating in the public discussion of any proposed
In 1996, then Senate President Maceda
contract, efectively truncating a basic right enshrined
delivered a privilege speech in the Senate and
in the Bill of Rights. The Court can allow neither an
denounced the JVA as the grandmother of all scams.
emasculation of a constitutional right, nor a retreat by
The Senate Committees reported the results of their
the State of its avowed policy of full disclosure of all
investigation in Senate Committee Report No. 560 and
its transactions involving public interest.
among the conclusions of their report are: (1) the
The information that petitioner may access on
reclaimed lands PEA seeks to transfer to AMARI under
the renegotiation of the JVA includes evaluation
the JVA are lands of the public domain which the
reports, recommendations, legal and expert opinions,
government has not classified as alienable lands and
minutes of meetings, terms of reference and other
therefore PEA cannot alienate these lands; (2) the
documents attached to such reports or minutes, all
certificates of title covering the Freedom Islands are
relating to the JVA. However, the right to information
thus void, and (3) the JVA itself is illegal.
does not compel PEA to prepare lists, abstracts,
Hence, petitioner, as a taxpayer, filed the
summaries and the like relating to the renegotiation of
instant Petition for Mandamus with Prayer for the
the JVA. The right only afords access to records,
Issuance of a Writ of Preliminary Injunction and
documents and papers, which means the opportunity
Temporary Restraining Order. Petitioner contends the
to inspect and copy them. One who exercises the right
government stands to lose billions of pesos in the sale
must copy the records, documents and papers at his
by PEA of the reclaimed lands to AMARI. Petitioner
expense. The exercise of the right is also subject to
prays that PEA publicly disclose the terms of any
reasonable regulations to protect the integrity of the
renegotiation of the JVA, invoking Section 28, Article II,
public records and to minimize disruption to
and Section 7, Article III, of the 1987 Constitution on
government operations, like rules specifying when and
the right of the people to information on matters of
how to conduct the inspection and copying.
public concern. Petitioner assails the sale to AMARI of
There is no claim by PEA that the information
lands of the public domain as a blatant violation of
demanded by petitioner is privileged information
Section 3, Article XII of the 1987 Constitution
rooted in the separation of powers. The information
prohibiting the sale of alienable lands of the public
does
not
cover
Presidential
conversations,
domain to private corporations. Finally, petitioner
correspondences, or discussions during closed-door
asserts that he seeks to enjoin the loss of billions of
Cabinet meetings which, like internal deliberations of
pesos in properties of the State that are of public
the Supreme Court and other collegiate courts, or
dominion.
executive sessions of either house of Congress, are
recognized as confidential. This kind of information
ACADCOM 2010; Contributors: Gene Geocaniga, Jarissa Guiani, Darlene Magabilen
TAU MU Page 88 of 179

TAU MU
TAU MU

TAU MU

TAU MU

TAU MU

TAU MU

TAU MU

TAU MU

TAU MU

TAU MU

TAU MU

Note: The right covers three categories of information


which are matters of public concern, namely: (1)
official records; (2) documents and papers pertaining to
official acts, transactions and decisions; and (3)
government research data used in formulating
policies. The first category refers to any document that
is part of the public records in the custody of
government agencies or officials. The second category
refers to documents and papers recording, evidencing,
establishing, confirming, supporting, justifying or
explaining official acts, transactions or decisions of
government agencies or officials. The third category
refers to research data, whether raw, collated or
processed, owned by the government and used in
formulating government policies.
The State policy of full transparency in all
transactions involving public interest reinforces the
peoples right to information on matters of public
concern. This State policy is expressed in Section 28,
Article II of the Constitution, thus: Subject to
reasonable conditions prescribed by law, the State
adopts and implements a policy of full public disclosure
of all its transactions involving public interest.
These twin provisions of the Constitution seek
to promote transparency in policy-making and in the
operations of the government, as well as provide the
people sufficient information to exercise efectively
other constitutional rights. These twin provisions are
essential to the exercise of freedom of expression. If
the government does not disclose its official acts,
transactions and decisions to citizens, whatever
citizens say, even if expressed without any restraint,
will be speculative and amount to nothing. These twin
provisions are also essential to hold public officials at
all times accountable to the people, for unless citizens
have the proper information, they cannot hold public
officials accountable for anything. Armed with the
right information, citizens can participate in public
discussions leading to the formulation of government
policies and their efective implementation.
An
informed citizenry is essential to the existence and
proper functioning of any democracy.

KITY

The Fraternal
Ateneo de Davao

TAU MU TAU MU TAU MU


TAU MU TAU MU TAU MU

CONSTITUTIONAL LAW II
Order of Saint Thomas More
Atty. Philip John Pojas/Atty. Rovyne G. Jumao-as, RN
University College of Law
cannot be pried open by a co-equal branch of
government.
Hence, the SC ruled that the constitutional
right to information includes official information on ongoing negotiations before a final contract. The
information, however,
must constitute
definite
propositions by the government and should not cover
recognized exceptions like privileged information,
military and diplomatic secrets and similar matters
afecting national security and public order.

Executive Order No. 213 (Creation of a Philippine


Coordinating Committee to study the feasibility of the
Japan-Philippines Economic Partnership Agreement) to
study and negotiate the proposed JPEPA, and to furnish
the Committee with a copy of the latest draft of the
JPEPA. Usec. Aquino did not heed the request,
however. Congressman Aguja later requested for the
same document, but Usec. Aquino replied that the
Congressman shall be provided with a copy thereof
once the negotiations are completed and as soon as a
thorough legal review of the proposed agreement has
been conducted.
Issue: WON the right to information includes an ongoing negotiation before a final agreement.

TAU MU
TAU MU

Ruling: Public disclosure of the text of the JPEPA after


its signing by the President, during the pendency of the
present petition, has been largely rendered moot and
academic. With the Senate deliberations on the JPEPA
still pending, the agreement as it now stands cannot
yet be considered as final and binding between the two
States.
To be covered by the right to information, the
information sought must meet the threshold
requirement that it be a matter of public concern. In
determining whether or not a particular information is
of public concern there is no rigid test which can be
applied. Public concern like public interest is a term
that eludes exact definition. Both terms embrace a
broad spectrum of subjects which the public may want
to know, either because these directly afect their lives,
or simply because such matters naturally arouse the
interest of an ordinary citizen. In the final analysis, it is
for the courts to determine on a case by case basis
whether the matter at issue is of interest or
importance, as it relates to or afects the public. From
the nature of the JPEPA as an international trade
agreement, it is evident that the Philippine and
Japanese ofers submitted during the negotiations
towards its execution are matters of public concern.
This, respondents do not dispute. They only claim that
diplomatic negotiations are covered by the doctrine of
executive privilege, thus constituting an exception to
the right to information and the policy of full public
disclosure.
It is well-established in jurisprudence that
neither the right to information nor the policy of full
public disclosure is absolute, there being matters
which, albeit of public concern or public interest, are
recognized as privileged in nature. Whether a claim of
executive privilege is valid depends on the ground
invoked to justify it and the context in which it is made.
In the present case, the ground for respondents claim
AKBAYAN vs. AQUINO
of privilege is that the categories of information that
558 SCRA 468
may be considered privileged includes matters of
diplomatic character and under negotiation and
Facts: Petitioners - non-government organizations,
review. In this case, the privileged character of the
Congresspersons, citizens and taxpayers seek via
diplomatic negotiations has been categorically invoked
petition for mandamus and prohibition to obtain from
and clearly explained by respondents particularly
respondents the full text of the Japan-Philippines
respondent DTI Senior Undersecretary. The ground
Economic Partnership Agreement (JPEPA) including the
relied upon by respondents is thus not simply that the
Philippine and Japanese ofers submitted during the
information sought involves a diplomatic matter, but
negotiation process and all pertinent attachments and
that it pertains to diplomatic negotiations then in
annexes thereto.
progress.
Petitioners Congressmen. Taada and Aguja
It is clear that while the final text of the JPEPA
filed House Resolution No. 551 calling for an inquiry
may not be kept perpetually confidential since there
into the bilateral trade agreements then being
should be ample opportunity for discussion before a
negotiated by the Philippine government, particularly
treaty is approved the ofers exchanged by the
the JPEPA. The Resolution became the basis of an
parties during the negotiations continue to be
inquiry subsequently conducted by the House Special
privileged even after the JPEPA is published. It is
Committee on Globalization (the House Committee)
reasonable
to
conclude
that
the
Japanese
into the negotiations of the JPEPA.
representatives submitted their ofers with the
In the course of its inquiry, the House
understanding that historic confidentiality would
Committee requested Usec. Aquino, Chairman of the
govern the same. Disclosing these ofers could impair
Philippine Coordinating Committee created under
the ability of the Philippines to deal not only with Japan
ACADCOM 2010; Contributors: Gene Geocaniga, Jarissa Guiani, Darlene Magabilen
TAU MU Page 89 of 179

TAU MU
TAU MU

TAU MU

TAU MU

TAU MU

TAU MU

TAU MU

TAU MU

TAU MU

TAU MU

TAU MU

TAU MU
TAU MU
TAU MU

Facts: Petitioner Gonzales, in his capacity as a citizen


and taxpayer, prays for an order compelling
respondent Zamora to furnish with information for the
names of executive officials holding multiple positions
in the government, copied of their appointments and a
list of the recipients of luxury vehicles seized by
Bureau of Customs and turned over to Malacanang.
Issue: WON the petitioner has the right to information.

TAU MU
TAU MU
TAU MU
TAU MU
TAU MU

Ruling: Section 7 of the Bill of Rights is a selfexecutory provision which can be invoked by any
citizen before the courts. The right to information as a
public right and when a mandamus proceeding
involves the assertion of a public right, the requirement
of personal interest is satisfied by the mere fact that
the petitioner is a citizen and therefore, part of the
general public which possesses the right. However,
Congress may provide for reasonable conditions upon
the access to information. Such limitations were
embodied in RA 6713, otherwise known as the Code of
Conduct and Ethical Standards for Public Officials and
Employees. This law provides that in the performance
of their duties, all public officials and employees are
obliged to respond to letters sent by the public within
15 working days from receipt thereof and to ensure the
accessibility of all public documents for inspection by
the public within reasonable working hours, subject to
the
reasonable claims
of
confidentiality.
The
incorporation of this right in the Constitution is a
recognition of the fundamental role of the free
exchange of information in a democracy. The
information to which the public is entitled to those
concerning matters of public concern, a term which
embraces a broad spectrum of subjects which the
public may want to know, either because these directly
afect their lives, or simply because such matters
naturally arouse the interest of an ordinary citizen. In
the final analysis, it is for the courts to determine in a
case to case basis whether the matter at issue is of
interest or importance, as it relates to or afects the
public. Respondent Zamora, in his official capacity as
executive secretary, has a constitutional and statutory
duty to answer petitioners letter dealing with matters
which are unquestionably of public concern. With
regard to petitioners request for copies of the
appointment papers of certain officials, respondent
Zamora is obliged to allow the inspection and copying
of the same subject to the reasonable limitations
required for the orderly conduct of official business.

The Fraternal
Ateneo de Davao

TAU MU
TAU MU

GONZALES vs. NARVASA, et. al


337 SCRA 733 (2000)

KITY

TAU MU TAU MU TAU MU


TAU MU TAU MU TAU MU

CONSTITUTIONAL LAW II
Order of Saint Thomas More
Atty. Philip John Pojas/Atty. Rovyne G. Jumao-as, RN
University College of Law
but with other foreign governments in future
negotiations.
Diplomatic
negotiations,
therefore,
are
recognized as privileged in this jurisdiction, the JPEPA
negotiations constituting no exception. It bears
emphasis, however, that such privilege is only
presumptive. Recognizing a type of information as
privileged does not mean that it will be considered
privileged in all instances. Only after a consideration
of the context in which the claim is made may it be
determined if there is a public interest that calls for the
disclosure of the desired information, strong enough to
overcome its traditionally privileged status.
Hence, the SC held that petitioners demand to
be furnished with a copy of the full text of the JPEPA
has become moot and academic, it having been made
accessible to the public since September 11, 2006. As
for their demand for copies of the Philippine and
Japanese
ofers
submitted
during
the
JPEPA
negotiations, the same must be denied, respondents
claim of executive privilege being valid.

SENATE vs. ERMITA


488 SCRA 1
Facts: Senate President Drilon received from Executive
Secretary Ermita a letter respectfully requesting for
the postponement of the hearing regarding the
NorthRail project to which various officials of the
Executive Department have been invited in order to
aford said officials ample time and opportunity to
study and prepare for the various issues so that they
may better enlighten the Senate Committee on its
investigation. Senate President Drilon, however,
wrote Executive Secretary Ermita that the Senators
are unable to accede to his request as it was sent
belatedly and all preparations and arrangements as
well as notices to all resource persons were completed
the previous week.
Senate President Drilon likewise received a letter
from the President of the North Luzon Railways
Corporation Cortes, Jr. requesting that the hearing on
the NorthRail project be postponed or cancelled until a
copy of the report of the UP Law Center on the contract
agreements relative to the project had been secured.
In 2005, the President issued E.O. 464, ENSURING
OBSERVANCE OF THE PRINCIPLE OF SEPARATION OF
POWERS, ADHERENCE TO THE RULE ON EXECUTIVE
PRIVILEGE AND RESPECT FOR THE RIGHTS OF PUBLIC
OFFICIALS APPEARING IN LEGISLATIVE INQUIRIES IN AID
OF LEGISLATION UNDER THE CONSTITUTION, AND FOR
OTHER PURPOSES.
Senate
President
Drilon
received
from
Executive Secretary Ermita a copy of E.O. 464, and
another letter informing him that officials of the
Executive Department invited to appear at the meeting
regarding the NorthRail project will not be able to
attend the same without the consent of the President,
pursuant to E.O. 464 and that said officials have not
secured the required consent from the President. On
even date which was also the scheduled date of the
hearing on the alleged wiretapping, Gen. Senga sent a
letter to Senator Biazon, Chairperson of the Committee
on National Defense and Security, informing him that
per instruction of President Arroyo, thru the Secretary
of National Defense, no officer of the AFP is authorized
to appear before any Senate or Congressional hearings
without seeking a written approval from the President
and that no approval has been granted by the
President to any AFP officer to appear before the public
hearing of the Senate Committee on National Defense
and Security.Despite the communications received
from Executive Secretary Ermita and Gen. Senga, the
investigation scheduled by the Committee on National
Defense and Security pushed through, with only Col.
Balutan and Brig. Gen. Gudani among all the AFP
officials invited attending.
Issue: WON the withholding of the information violated
petitioners right to information.

TAU MU

Ruling: E.O 464 is concerned only with the demands


of Congress for the appearance of executive officials in
the hearings conducted by it, and not with the
demands of citizens for information pursuant to their
right to information on matters of public concern.
Petitioners are not amiss in claiming, however, that
what is involved in the present controversy is not
merely the legislative power of inquiry, but the right of
the people to information.
There are clear distinctions between the right
of Congress to information which underlies the power
of inquiry and the right of the people to information on
matters of public concern. For one, the demand of a
citizen for the production of documents pursuant to his
right to information does not have the same obligatory
force as a subpoena duces tecum issued by Congress.
Neither does the right to information grant a citizen the
ACADCOM 2010; Contributors: Gene Geocaniga, Jarissa Guiani, Darlene Magabilen
TAU MU Page 90 of 179

TAU MU

TAU MU

TAU MU
TAU MU
TAU MU
TAU MU
TAU MU
TAU MU

Facts: Petitioners mortgaged their property to the


respondent GSIS to secure a housing loan. As
petitioners failed to pay their loan when it fell due,
GSIS foreclosed the mortgage. Petitioner wrote GSIS
Board of Trustees to inform them of their desire to
redeem the subject property and for advice of the
procedure of redemption. GSIS respondent advised
them to pay the total redemption price of Php154, 896
on or before the expiry date of redemption. Unable to
find financing to repurchase the subject property,
petitioner
requested
for
re-mortgage
through
repurchase of the subject property. Respondent dela
Cruz commenced negotiations with respondent GSIS
for her purchase of the subject property. A deed of
absolute sale over the subject property was executed
between GSIS and respondent dela Cruz. Having
learned about the sale of the subject property,
petitioner wrote to the GSIS protesting the said sale
and requesting its consideration and recall. Petitioners
filed the instant case before the RTC of QC which was
dismissed and affirmed by the CA.

The Fraternal
Ateneo de Davao

TAU MU
TAU MU

VDA. DE URBANO, et. al. vs. GSIS, et. al


367 SCRA 672 (2001)

KITY

TAU MU TAU MU TAU MU


TAU MU TAU MU TAU MU

CONSTITUTIONAL LAW II
Order of Saint Thomas More
Atty. Philip John Pojas/Atty. Rovyne G. Jumao-as, RN
University College of Law
power to exact testimony from government officials.
These powers belong only to Congress and not to an
individual citizen. Thus, while Congress is composed of
representatives elected by the people, it does not
follow, except in a highly qualified sense, that in every
exercise of its power of inquiry, the people are
exercising their right to information.
To the extent that investigations in aid of
legislation are generally conducted in public, however,
any executive issuance tending to unduly limit
disclosures of information in such investigations
necessarily deprives the people of information which,
being presumed to be in aid of legislation, is presumed
to be a matter of public concern. The citizens are
thereby denied access to information which they can
use in formulating their own opinions on the matter
before Congress opinions which they can then
communicate to their representatives and other
government officials through the various legal means
allowed by their freedom of expression. Thus holds
Valmonte v. Belmonte: It is in the interest of the State
that the channels for free political discussion be
maintained to the end that the government may
perceive and be responsive to the peoples will. Yet,
this open dialogue can be efective only to the extent
that the citizenry is informed and thus able to
formulate its will intelligently.
Only when the
participants in the discussion are aware of the issues
and have access to information relating thereto can
such bear fruit.
The impairment of the right of the people to
information as a consequence of E.O. 464 is, therefore
just as direct as its violation of the legislatures power
of inquiry.

involving public interest. The negotiation and


subsequent sale of the subject property by the GSIS to
dela Cruz was by no stretch of the imagination imbued
with public interest as it was a purely private
transaction.
HILADO vs. REYES
496 SCRA 282
Facts: PR Benedicto, the surviving spouse of the
deceased Roberto S. Benedicto, filed a petition for
issuance of letters of administration, Intestate Estate
of Roberto S. Benedicto. The case was raffled to
Branch 21 presided by Public Respondent Judge Reyes.
Private respondent was appointed Administratrix of the
estate of Benedicto and letters of administration were
thereafter issued in her favor.
Petitioners, Hilado, Lacson, Tuvilla, Limjap,
Lopez Sugar Corporation and First Farmers Holding
Corporation had, during the lifetime of Benedicto, filed
two complaints for damages or collection of sums of
money against Roberto Benedicto et al. The Branch
Clerk of Court of Branch 21 of the Manila RTC allowed
petitioners through counsel Sedigo and Associates to
regularly and periodically examine the records of the
case and to secure certified true copies thereof.
However, Atty. Paredes, an associate of petitioners
counsel, was denied access to the last folder-record of
the case which, according to the courts clerical staf,
could not be located and was probably inside the
chambers of public respondent for safekeeping.
Petitioners counsel thus requested public
respondent to allow Atty. Paredes to personally check
the records of the case. The Officer-In- Charge/Legal
Researcher of Branch 21 advised petitioners counsel in
writing that per instruction of the Hon. Presiding
Judge, only parties or those with authority from the
parties are allowed to inquire or verify the status of the
case pending in this Court, and that they may be
allowed to go over the records of the case upon
presentation
of
written
authority
from
the
administratrix. Hence, petitioners filed petition for
mandamus and prohibition to compel public
respondent to allow them to access, examine, and
obtain copies of any and all documents forming part of
the records of the case and disqualify public
respondent from further presiding
Issue: WON petitioners right to information was
violated.

TAU MU

Ruling: Insofar as the right to information relates to


judicial records, an understanding of the term judicial
record or court record is in order. The term judicial
record or court record does not only refer to the
orders, judgment or verdict of the courts. It comprises
the official collection of all papers, exhibits and
pleadings filed by the parties, all processes issued and
returns made thereon, appearances, and word-for-word
testimony which took place during the trial and which
Issue: WON petitioners were deprived of their right to
are in the possession, custody, or control of the
information.
judiciary or of the courts for purposes of rendering
court decisions. It has also been described to include
Ruling: The petitioner is devoid of merit. GSIS sold the
any paper, letter, map, book, other document, tape,
subject property to dela Cruz only after giving
photograph, film, audio or video recording, court
petitioners an almost one year opportunity to
reporters notes, transcript, data compilation, or other
repurchase the property and only after ascertaining
materials, whether in physical or electronic form, made
that the purchase price proposed by dela Cruz in
or received pursuant to law or in connection with the
payment of the subject property would benefit the
transaction of any official business by the court, and
GSIS. Nor can petitioners, on the strength of Valmonte
includes all evidence it has received in a case.
vs. Belmonte, impute bad faith on the part of GSIS
In determining whether particular information
when the latter did not disclose to petitioners that it
is of public concern, there is no right test. In the final
was negotiating with dela Cruz for the sale of the
analysis, it is for the courts to determine on a case to
subject property as soon as it started the negotiations.
case basis whether the matter at issue is of interest or
The constitutional right to information is limited to
importance as it relates to or afect the public. It bears
maters of public concern, and to transaction
emphasis that the interest of the public hinges on its
ACADCOM 2010; Contributors: Gene Geocaniga, Jarissa Guiani, Darlene Magabilen
TAU MU Page 91 of 179

TAU MU

TAU MU

TAU MU

TAU MU

KITY

The Fraternal
Ateneo de Davao

TAU MU TAU MU TAU MU


TAU MU TAU MU TAU MU
TAU MU
TAU MU

CONSTITUTIONAL LAW II
Order of Saint Thomas More
Atty. Philip John Pojas/Atty. Rovyne G. Jumao-as, RN
University College of Law
right to transparency in the administration of justice, to
the end that it will serve to enhance the basic fairness
of the judicial proceedings, safeguard the integrity of
the fact-finding process, and foster an informed public
discussion of governmental afairs.
Decisions and opinions of a court are of course
matters of public concern or interest for these are the
authorized expositions and interpretations of the laws,
binding upon all citizens, of which every citizen is
charged with knowledge. Justice thus requires that all
should have free access to the opinions of judges and
justices, and it would be against sound public policy to
prevent, suppress or keep the earliest knowledge of
these from the public. Unlike court orders and
decisions, however, pleadings and other documents
filed by parties to a case need not be matters of public
concern or interest. For they are filed for the purpose
of establishing the basis upon which the court may
issue an order or a judgment afecting their rights and
interests. In thus determining which part or all of the
records of a case may be accessed to, the purpose for
which the parties filed them is to be considered.
Information regarding the financial standing of
a person at the time of his death and the manner by
which his private estate may ultimately be settled is
not a matter of general, public concern or one in which
a citizen or the public has an interest by which its legal
rights or liabilities maybe afected.
Granting
unrestricted public access and publicity to personal
financial information may constitute an unwarranted
invasion of privacy to which an individual may have an
interest in limiting its disclosure or dissemination.
If the information sought then is not a matter
of public concern or interest, denial of access thereto
does not violate a citizens constitutional right to
information. Once particular information has been
determined to be of public concern, the accessory right
of access to official records, including judicial records,
are open to the public. The accessory right to access
public records may, however, be restricted on a
showing of good cause.
Thus, access to court records may be permitted
at the discretion and subject to the supervisory and
protective powers of the court, after considering the
actual use or purpose for which the request for access
is based and the obvious prejudice to any of the
parties. In this case, the SC granted the petition for
mandamus filed by Hilado. Public respondent Reyes
was ordered to allow petitioners to access, examine
and obtain copies of any and all documents-part of the
records of the special proceedings.

seeking to disqualify the nominees of certain party-list


organizations. Both petitioners appear not to have the
names of the nominees sought to be disqualified since
they still asked for a copy of the list of nominees.
Meanwhile, reacting to the emerging public perception
that the individuals behind the aforementioned 14
party-list groups do not actually represent the poor and
marginalized sectors, petitioner Rosales addressed a
letter to Director Dalaig of the Comelecs Law
Department requesting a list of that groups nominees.
However, neither the Comelec Proper nor its Law
Department
responded to petitioner Rosales
requests.
Issue: WON there was a violation of the constitutional
right to information when COMELEC refused to disclose
the names of the party-lists.

TAU MU
TAU MU

Ruling: Any citizen can challenge any attempt to


obstruct the exercise of his right to information and
may seek its enforcement by mandamus. And since
every citizen by the simple fact of his citizenship
possesses the right to be informed, objections on
ground of locus standi are ordinarily unavailing.
However, the right to information and its companion
right of access to official records are not absolute. The
peoples right to know is limited to matters of public
concern and is further subject to such limitation as
may be provided by law. Similarly, the policy of full
disclosure is confined to transactions involving public
interest and is subject to reasonable conditions
prescribed by law. There is also the need of preserving
a measure of confidentiality on some matters, such as
military, trade, banking and diplomatic secrets or those
afecting national security.
In the case at bar, no national security or like
concerns is involved in the disclosure of the names of
the nominees of the party-list groups in question.
Doubtless, the Comelec committed grave abuse of
discretion in refusing the legitimate demands of the
petitioners for a list of the nominees of the party-list
groups subject of their respective petitions. Mandamus,
therefore, lies. The last sentence of Section 7 of RA
7941 reading: The names of the party-list nominees
shall not be shown on the certified list is certainly not
a justifying card for the Comelec to deny the
requested disclosure. The prohibition imposed on the
Comelec under said Section 7 is limited in scope and
duration, meaning, that it extends only to the certified
list which the same provision requires to be posted in
the polling places on election day. To stretch the
coverage of the prohibition to the absolute is to read
into the law something that is not intended. As it were,
BANTAY REPUBLIC ACT 7941 vs. COMELEC
there is absolutely nothing in RA 7941 that prohibits
523 SCRA 1 (2007)
the Comelec from disclosing or even publishing
Facts: The case filed before the SC are two
through mediums other than the Certified List the
consolidated petitions for certiorari and mandamus to
names of the party-list nominees.
nullify and set aside certain issuances of the COMELEC
The Comelec obviously misread the limited
respecting party list groups which have manifested
non-disclosure aspect of the provision as an absolute
their intention to participate in the party-list elections
bar to public disclosure before the May 2007 elections.
in May 2007. Both petitions seek to compel COMELEC
The interpretation thus given by the Comelec virtually
to disclose or publish the names of the nominees of the
tacks an unconstitutional dimension on the last
various party list groups.
sentence of Section 7 of RA7941. The Comelecs
In January 2007, the COMELEC issued a
reasoning that a party-list election is not an election of
resolution prescribing rules and regulations to govern
personalities is valid to a point. It cannot be taken,
the filing of manifestation of intent to participate and
however, to justify its assailed non-disclosure stance
submission of names of nominees under the party-list
which comes, as it were, with a weighty presumption of
system of representation in connection with the May
invalidity, impinging, as it does, on a fundamental right
2007 elections. Pursuant thereto, a number of
to information. While the vote cast in a party-list
organized groups filed the necessary manifestations.
elections is a vote for a party, such vote, in the end,
Among these and ostensibly subsequently accredited
would be a vote for its nominees, who, in appropriate
by the Comelec to participate in the 2007 elections cases, would eventually sit in the House of
are 14 party-list groups. Petitioners BA-RA 7941 and
Representatives.
UP-LR presented a longer, albeit an overlapping, list.
Thus, the SC orderd the Comelec to
Subsequent events saw BA-RA 7941 and UP-LR
immediately disclose and release the names of the
filing with the Comelec an Urgent Petition to Disqualify,
nominees of the party-list groups, sectors or
ACADCOM 2010; Contributors: Gene Geocaniga, Jarissa Guiani, Darlene Magabilen
TAU MU Page 92 of 179

TAU MU

TAU MU

TAU MU

TAU MU

TAU MU

TAU MU

TAU MU

TAU MU

TAU MU

AQUINO vs. MORATO


203 SCRA 515 (1991)
Facts: The petitioner, a member of the MTRCB,
requested that she be allowed to examine the voting
slips of other members. It is on the basis of these slips
that the films are banned, classified or cut by the
Board. Petitioners request was refused on the ground
that the votes, which are purely personal, are private
and confidential.

VALMONTE vs. BELMONTE


170 SCRA 256 (1989)

TAU MU
TAU MU
TAU MU
TAU MU

Section 8. The right of the people, including


those employed in the public and private
sectors, to form unions, associations, or
societies for purposes not contrary to law, shall
not be abridged.

TAU MU

Ruling 2: The Court held that there is no law which


grants GSISs confidentiality. Also, the right to
confidentiality involves privacy which cannot be
invoked by GSIS because matters about privacy are not
available to an artificial person (such as corporations or
government entities). The right of confidentiality
pertains to borrowers. Privacy is granted only to
natural beings.

TAU MU

Issue 2: Is the information excluded by law on the


ground of confidentiality?

TAU MU

Ruling 1: Only information which are matters of public


concern are covered by Section 7. The OCUrt held that
the information sought is a matter of public concern.
Since the GSIS is the custodian of public funds, it shall
be the interest of the citizen which will determines as
to how the money must be disturbed (NATURE OF THE
FUNDS). The Court defines a MATTER OF PUBLIC
CONCERN as any subject of which the public may want
to know because it directly afects their lives or they
arouse the interest of a citizen. And the Court also
stated that the members of the Batasan are public
figures, and this contributes to the idea that this is a
matter of public concern (CHARACTER OF THE
BORROWER).

TAU MU

Issue 1: Is the information sought a matter of public


concern?

TAU MU
TAU MU

Facts: Special civil action for mandamus with


preliminary injunction. Valmonte wrote to Belmonte,
GSIS General Manager, requesting the list of names of
the opposition members of the Batasang Pambansa
who were able to secure a clean loan on guaranty of
Mrs. Imelda Marcos, certified true copies of the
documents evidencing their loan or access to such
documents. It was denied on the ground that a
confidential relationship exists between GSIS and all
those who borrow from it, whoever they may be.

The Fraternal
Ateneo de Davao

TAU MU
TAU MU

Ruling: The Court held that they are not private


because the members of the Board occupy a public
position. These are actually public records and are
therefore not private or personal.

KITY

TAU MU TAU MU TAU MU


TAU MU TAU MU TAU MU

CONSTITUTIONAL LAW II
Order of Saint Thomas More
Atty. Philip John Pojas/Atty. Rovyne G. Jumao-as, RN
University College of Law
organizations accredited to participate in the May 2007
party-list elections.

freedom of speech and expression, subject to the same


limitation.
Even without this constitutional provision, the right of
association is part and parcel of the freedom of
expression and assembly. It is also an aspect of liberty,
which is covered by the freedom clause. It is also an
aspect of contracts, which is also related to freedom of
religion because normally people form associations to
advance a specific belief.
Limitations:
1. Imposed by law
2. Dominant police power of the State
Rights included/ 2 aspects:
1. The positive aspect is the right to join or be a
member of an association.
2. The negative aspect is the right not to be
compelled to be a member of any association.
Exception to Negative Aspect: When there is a
closed shop agreement in the CBA. An employee can
be compelled to be member of the union. Because a
closed shop agreement is a valid exercise of the police
power, and the state has an interest in promoting
unionism to protect labor. So, in this case, a person can
be compelled to join an association.
Exception to the exception: In the case of Victoriano
vs. Elizalde: a person cannot be compelled to be a
member of a union, even under a legal closed shop
agreement if his religion prohibits him to join
associations because the freedom of religion is superior
to the freedom of association.
Note: But in Hernandez, mere membership in Huk
Balahap is punishable.
Tests used in limiting the right of association:
1. Test for valid exercise of police power - the
right to association is limited when the state is invoking
or exercising this police power. (Remember the tests
for valid exercise of police power). This test is used
when the purpose of association is to promote property
rights.
2. Clear and Present Danger Test - used when the
purpose of association is to promote advance basic
human rights or political ideas. So if you're joining a
political group, this test is used.

Under section 8, it says that even those employed


in the public sector can join a strike. But there are
limitations because the right of public employees are
limited to the right to form associations. Actually, they
have a right to form unions, but they do not have the
constitutional right to strike. Private employees have
the constitutional right to strike, but not the public
employees.

PAFLU case- The purpose of the registration was for


the acquisition of the legal personality. Here, nobody
was being prevented from joining a union. The union
exists. But without the registration, the union cannot
enter into CBA, etc. Its just the activities that they can't
enter into if they don't enter, but there is no violation
of the right of association.

The right to form an association is superior to the


freedom of assembly, because in association the
people not only assemble but start to form an
organization. And to form an association already
involves a purpose, assembly is just assemble, get
together

TAU MU

The right to form associations shall not be impaired


without due process of law and is thus an aspect of the
right of liberty. It is also an aspect of the freedom of
contract. In addition, insofar as the associations may
have for their object the advancement of beliefs and
ideas, the freedom of association is an aspect of the
ACADCOM 2010; Contributors: Gene Geocaniga, Jarissa Guiani, Darlene Magabilen
TAU MU Page 93 of 179


The right to form association DOES NOT INCLUDE
the following:
a. automatic acquisition of legal personality;
b. possession of rights and privileges granted by
law
to
legitimate
labor
organizations/associations;
c. automatic registration of the association with
any legal entity.

TAU MU
TAU MU
TAU MU
TAU MU

Q: Can this right be exercised by the unemployed?


Where is the seat of this right?
A: YES. The right is recognized as belonging to the
PEOPLE whether employed or not, and whether
employed in government or the private sector.

TAU MU

Strictly
speaking,
this
right
is
already
comprehended in due process, particularly as it
protects the persons liberty. It was nevertheless
deemed advisable to provide for this separate section
by way of emphasis upon this specific right.
This right is also deemed embraced in the freedom
of expression because the organization can be used as
a vehicle for the expression of views that have a
bearing on the public welfare. These views would be
more efectively disseminated and enjoy a more
respectful audience if articulated through and
organization to w/c a person belongs than if he were to
ventilate them as a mere individual.
The instinct to organize is a very basic human
drive.
The provision guarantees the right to form
association. IT does not include the right to compel
others to form an association. BUT there may be
situations w/c, by entering into a contract, one may
also be agreeing to join an association [BEL-AIR
VILLAGE ASSOCIATION vs. DIONISIO (174 SCRA 589)].

TAU MU
TAU MU

The Fraternal
Ateneo de Davao

TAU MU
TAU MU

Q: Can a lawyers name be cancelled in the roll of


attorneys if he fails to pay IBP dues, or stated
otherwise, can a lawyer be forced to join the IBP?
A: Yes. (See: In Re: Edillion) Membership in IBP is not a
violation of a persons freedom of association Bar
association does not force lawyers to associate with
anyone. It is the lawyers choice to attend or not to
attend the meeting. The only compulsion is the
payment of fees which is a valid exercise of police
power.

KITY

TAU MU TAU MU TAU MU


TAU MU TAU MU TAU MU

CONSTITUTIONAL LAW II
Order of Saint Thomas More
Atty. Philip John Pojas/Atty. Rovyne G. Jumao-as, RN
University College of Law
Q: Is membership in the communist party punishable
as a criminal ofense?
A: Mere membership and nothing more merely implies
advocacy of abstract theory or principle, remember,
the freedom to believe is absolute. It becomes a
criminal ofense only if it is coupled with action or
advocacy of action.

To compel a lawyer to become a member of the

integrated bar is not violative of his freedom to


associate [IN RE: EDILLON (127 SCRA 404)].
The right to form association does not necessarily
include the right to be given legal personality --- must
register!
REGISTRATION is merely a condition sine qua non
for the acquisition of LEGAL PERSONALITY and the
possession of the rights and privileges granted by law.
CASES: RIGHT TO ASSOCIATE
STA. CLARA HOMEOWNERS ASSOCIATION vs.
SPOUSES GASTON
374 SCRA 399 (2002)
Facts: Sps. Gaston purchased their lots at Sta. Clara
Subdivision and at the same time purchase; there was
no mention or requirement of membership in any
homeowners association. Only member in good
standing were issued stickers for their vehicles.
Respondents son was prevented entry and demanded
from him his drivers license for identification. Also, the
SCHA by-laws contain a provision that all real estate
owners in Sta. Clara Subdivision automatically become
members of the association. Spouses Gaston alleged
that they had never joined the association.
Issue: WON Sps. Gaston are SCHA member.
Ruling: The constitutionally guaranteed freedom of
association includes the freedom not to associate. The
right to choose with whom one will associate oneself is
the very foundation and essence of that partnership. It
does not include the right to compel others to form or
join one. Sps. Gaston cannot be compelled to become
members of the SCHA by including them in the Article
of Incorporation and by-laws without their express or
implied consent. it may be to the mutual advantage of
lot owners to band themselves together to promote
their common welfare. But that is impossible only if the
owners voluntarily agree, directly or indirectly, to
become members of the association. Such membership
may be acquired in various ways like deed of sale,
torrens certificate or other forms of evidence of
property ownership. Other that the Article of
Incorporation and by-laws, there is no showing that
Sps. Gaston have agreed to be SCHA members.
PADCOM CONDOMINIUM CORPORATION vs.
ORTIGAS CENTER ASSOCIATION
382 SCRA 222

TAU MU

Facts: PADCOM owns and manages the Padilla Office


Condominium Building (PADCOM Building). The land on
which the building stands was originally acquired from
the Ortigas & Company, Limited Partnership (OCLP), by
...for purposes not contrary to law is a built-in
Tierra Development Corporation (TDC) under a Deed of
limitation of the right. All rights are subject to the
Sale. Among the terms and conditions in the deed of
inherent police power.
sale was the requirement that the transferee and its
The right to associate includes the right NOT to
successor-in-interest must become members of an
associate. This right is superior to the right to assemble
association for realty owners and long-term lessees in
BUT must yield to the right to freedom of religion [INK
the area later known as the Ortigas Center.
case].
Subsequently, the said lot, together with improvements
The right does not include the right to strike.
thereon, was conveyed by TDC in favor of PADCOM in a
Q: Can civil servants unionize?
Deed of Transfer.
A: YES under Art. IX B Sec. 2 (5) The right to selfIn 1982, respondent Ortigas was organized to
organization shall not be denied to govt. employees
advance the interests and promote the general welfare
--- whether performing proprietary or govt. function.
of the real estate owners and long-term lessees of lots
in the Ortigas Center. It sought the collection of
The closed-shop is a valid form of union security and
membership dues from PADCOM. The corporate books
a provisions therefor in a CBA is not considered a
showed that PADCOM owed Ortigas which represents
restriction of the right of association [VILLAR vs.
membership dues, interests and penalty charges from
INCIONG (121 SCRA 444)].
April 1983 to June 1993. The letters exchanged
between the parties through the years showed
repeated demands for payment, requests for
ACADCOM 2010; Contributors: Gene Geocaniga, Jarissa Guiani, Darlene Magabilen
TAU MU Page 94 of 179

TAU MU

TAU MU

TAU MU

TAU MU
TAU MU
TAU MU
TAU MU
TAU MU
TAU MU
TAU MU
TAU MU

Issue: WON this is a restriction to Sec. 8 of the Bill of


Rights.

TAU MU

Facts: There was a law passed requiring unions to be


registered in order for them to acquire legal
personality. Acquiring legal personality is important
because if there is none, the union virtually cannot do
anything. It cannot bargain with the management, be
elected as bargaining representative and it cannot
serve as agent to the employees. The efect is they
exist as entities but they have no rights or privileges
under the law. Their existence will have no relevance to
the workers. This law was challenged as violative of the
right to association.

TAU MU
TAU MU

PAFLU vs. SECRETARY


27 SCRA 41 (1969)

The Fraternal
Ateneo de Davao

TAU MU
TAU MU

Issue: WON the automatic membership clause is a


violation of PADCOMS freedom of association.
Ruling: It is undisputed that when the land in question
was bought by PADCOMs predecessor-in-interest, TDC,
from OCLP, the sale bound TDC to comply with
paragraph (G) of the covenants, conditions and
restrictions of the Deed of Sale, which reads as follows:
G. AUTOMATIC MEMBERSHIP WITH THE ASSOCIATION:
The owner of this lot, its successor-in-interest hereby
binds himself to become a member of the
ASSOCIATION which will be formed by and among
purchasers, fully paid up Lot BUYERS, Building Owners
and the COMPANY in respect to COMPANY OWNED
LOTS. The OWNER of this lot shall abide by such rules
and regulations that shall be laid down by the
ASSOCIATION in the interest of security, maintenance,
beautification and general welfare of the OFFICE
BUILDING zone. The ASSOCIATION when organized
shall also, among others, provide for and collect
assessments which shall constitute a lien on the
property, junior only to liens of the Government for
taxes.
Evidently, it was agreed by the parties that
dues shall be collected from an automatic member and
such fees or assessments shall be a lien on the
property.
Hence, the SC ruled in the negative. PADCOM
was never forced to join the association. It could have
avoided such membership by not buying the land from
TDC. Nobody forced it to buy the land when it bought
the building with the annotation of the condition or lien
on the Certificate of Title thereof and accepted the
Deed. PADCOM voluntarily agreed to be bound by and
respect the condition, and thus to join the Association.

KITY

TAU MU TAU MU TAU MU


TAU MU TAU MU TAU MU

CONSTITUTIONAL LAW II
Order of Saint Thomas More
Atty. Philip John Pojas/Atty. Rovyne G. Jumao-as, RN
University College of Law
extensions of payment, and even a settlement scheme
proposed by PADCOM
In view of PADCOMs failure and refusal to pay
its arrears in monthly dues, including interests and
penalties thereon, Ortigas filed a complaint for
collection of sum of money. Ortigas averred that
purchasers of lands within the Ortigas Center complex
from OCLP are obligated under their contracts of sale
to become members of the Association. This obligation
was allegedly passed on to PADCOM when it bought
the lot from TDC, its predecessor-in-interest. In its
answer, PADCOM contended that it is a non-stock, nonprofit association, and for it to become a special
member of the Association, it should first apply for and
be accepted for membership by the latters Board of
Directors. No automatic membership was apparently
contemplated in the Associations By-laws. PADCOM
added that it could not be compelled to become a
member without violating its right to freedom of
association. And since it was not a member of the
Association, it was not liable for membership dues,
interests and penalties.

Ruling: The Court held that the right guaranteed by


the Constitution is the right to form associations and
the law does not violate this right. The employees can
still form associations. However, in order to be able to
do something, they must register first to be
recognized. The Constitution does not automatically
guarantee legal personalities to associations. The
reason for the requirement of registration is a valid
exercise of police power because the issue of unionism
is impressed with public interest. This requirement of
registration would protect labor and the public against
fraud, abuses and impostors who pose as organizers.
BEL AIR vs. DIONISIO
174 SCRA 589 (1989)
Facts: Plaintif filed a complaint against the defendant
for the collection of the association dues assessed on
the lot owned by the defendant as member of the
plaintif association. Without applying for membership
in plaintif association, defendant in this case, like the
other members, automatically became a member
because he is the registered owner of a lot located
inside the Bel Air Village. The inferior court rendered its
decision in favor of the plantif.
Issue: WON such membership collides with the
constitutional guarantee of freedom of association.
Ruling: There is no dispute that the title covering the
subject parcel of land issued in the name of the
petitioner contains an annotation to the efect that the
lot owner becomes an automatic member of the
respondent Bel Air Association and must abide by such
rules and regulations laid down by the Association.
When the petitioner voluntarily bought the subject
parcel of land, it was understood that he took the same
free of all encumbrances except notations at the back
of the certificate of title, among them, that he
automatically becomes a member of the respondent
association. One of the obligations of a member of the
respondent association is to pay certain amounts for
the operation and activities of the association which is
being collected by the Board of Governors. The dues
collected are intended for garbage collection, salary of
security guards, cleaning and maintenance of streets
and street lights and establishments of parks. The
amount to be paid by each lot owner is computed on
the basis of the area per square meter of the lot owned
by every member. The transaction between the
defendants and the original seller (defendants
immediate predecessor) is a sale and the conditions
have been validly imposed by the said vendor and the
same not being contrary to law, morals, good customs
and public policy. The fact that it has been approved by
the Land Registration Commission did not make it a
governmental act subject to the constitutional
restriction against infringement of the right of
association. The Constitutional proscription that no
person can be compelled to be a member of an
association against his will applies only to government
acts and not to private transactions like the one in
question. If he does not desire to comply with the
annotation or lien in question he can at any time
exercise his inviolable freedom of disposing of the
property and free himself from the burden of becoming
a member of the plaintif association.
OCCENA vs. COMELEC
127 SCRA 404 (1984)
Facts: Occena challenged as violative of Section 8 of
the Bill of Rights a law which prohibits a candidate of
the brgy. election from representing himself as a
member of the political party and also prohibited the

ACADCOM 2010; Contributors: Gene Geocaniga, Jarissa Guiani, Darlene Magabilen


TAU MU Page 95 of 179

Ruling: The Court held that the provision is not


absolute. It is subject to the police power of the State.
The law is a valid exercise of such power.
Note: Whether there is a permissible exercise of police
power by the State, the Court did not really aver what
should be the test to be applied in limiting this right. In
this case, the Court referred to the Gonzales vs.
COMELEC case and the implication that there is to
apply the clear and present danger rule. The Court held
that the right to organize is not violated by the law
because the political parties can still organize, but only
the activities are restricted.

TAU MU
TAU MU
TAU MU
TAU MU
TAU MU
TAU MU
TAU MU

Ruling: Petition is meritorious. The SC held that the


closed-shop agreement was a valid form of union
security, and such provision in a CBA is not a restriction
of the right of freedom of association guaranteed by
the Constitution. It was grave abuse of discretion for
respondent court to order the reinstatement and back
wages of Blanco who was found to be in an identical
position as the 5 other employees whose separation
from the service was upheld by the said Court. Blanco
could be dismissed on the ground of willful
disobedience to rules, orders and instructions of the
employer which were reasonable and lawful, known to
the employee and pertaining to duties which he
discharged. If his name was not included in the list of
18 employees recommended for dismissal, it was
because he had been dismissed three days before by
the company. And if he had not been dismissed by the
company, his dismissal would have been demanded by
LEA considering that he was one of those investigated
by LEAs grievance committee which had approved the
recommendation to dismiss them on the charge of
being members of another union.

TAU MU

Issue: WON the CBA is a restriction of the right of


freedom of association.

TAU MU

Facts: Employed in 1959 by LITEX, Blanco joined the


then existing labor union, the Litex Employees
Association (LEA). In 1960, LEA entered into a
collective bargaining agreement with LITEX which was
to expire in 1965. The CBA contained a closed shop
provision. LEAs constitution and by-laws also provided
for the expulsion of members who are associated with
other labor unions. In 1964, Blanco and several Litex
members organized the Confederation of Industrial and
Allied Labor Organization (CIAOLO). The Court of
Industrial Relations (CIR) certified LEA as the sole
bargaining representative of the rank and file
employees of LITEX. LITEX dismissed 18 of its
employees found to have violated the union
constitution and by-laws and the CBA for having joined
the CIALO. 6 of the dismissed employees and Blanco
filed a case for illegal dismissal. The CIR dismissed the
case as regards the 6 employees but ordered the
reinstatement with back wages of Blanco. Failing to
obtain reconsideration of the ruling, LITEX sought
review.

TAU MU
TAU MU

LIRAG vs. BLANCO and CIR


109 SCRA 87 (1981)

The Fraternal
Ateneo de Davao

TAU MU
TAU MU

Reason or policy arguments of the Court: It is


necessary to insulate the barangay from partisan
political activities. The barangay is a basis political
structure in our society. Because of this important
public interest, the Court considered resctricting the
activities of political parties as valid.

KITY

TAU MU TAU MU TAU MU


TAU MU TAU MU TAU MU

CONSTITUTIONAL LAW II
Order of Saint Thomas More
Atty. Philip John Pojas/Atty. Rovyne G. Jumao-as, RN
University College of Law
political parties from supporting a particular candidate
in the elections.

Section 9. Private property shall not be


taken
for
public
use
without
just
compensation.
Definition of Eminent Domain
It is the right of the State to acquire private
property for public use upon payment of just
compensation. This is based on the Regalian
doctrine that all property belongs to the State.
This power is inseparable in sovereignty, being
essential to the existence of the State and
inherent in government, even in its most
primitive forms. No laws therefore are ever
necessary to confer this right upon the
government.
It is the ultimate right of sovereign power to
appropriate, not only the public, but [even] the
private property of all citizens within the
territorial sovereignty, to public purposes
[Bernas, The 1987 Philippine Constitution: A
Comprehensive Reviewer, p. 101].
Eminent domain is described as the highest and
most exact idea of property remaining in the
government that may be acquired for some
public purpose through a method in the nature
of a compulsory sale to the State.
Being inherent, the power of eminent domain
does not need to be specifically conferred on the
government by the Constitution. As it happens,
however, it is expressly provided in Article III,
Section 9, that private property shall not be
taken for public use without just compensation.
This provision is not a grant but indeed a
limitation of the power as its negative and
restrictive language clearly suggests [Cruz,
Constitutional Law 2007, p.62].
The procedure of taking is called expropriation.
This provision, sec. 9, is included in the Bill of
Rights is not a grant of powers, but a limitation of
the power of eminent domain.
Telecommunications vs. COMELEC
289 SCRA 337 (1998)
Facts: GMA Network, Inc., is challenging Sec. 52
of BP Blg. 881, which provides:
The Commission shall procure radio and
television time to be known as Comelec Time
which shall be allocated equally and impartially
among candidates For this purpose, the
franchise of all radio broadcasting and television
stations are hereby amended so as to provide
radio or television time, free of charge, during the
election period.
Petitioner claims that this is a form of taking or
property without compensation.
Held:
The argument has no merit.
All
broadcasting, whether by radio or television
stations, is licensed by the government. Airwave
frequencies have to be allocated as there are
more individuals who want to broadcast than
there are frequencies assign. A franchise is thus
a privilege subject, among other things, to

ACADCOM 2010; Contributors: Gene Geocaniga, Jarissa Guiani, Darlene Magabilen


TAU MU Page 96 of 179

TAU MU
TAU MU
TAU MU
TAU MU

3. The airspace is not a property of the TV


stations/networks. It is just a franchise given to

TAU MU

2. PPI vs. COMELEC -- there is unlawful taking


here because there was not a need to
expropriate. It wasn't shown that PPI was not
willing to sell the newspaper space. 2nd, there
was no just compensation. 3rd, the Comelec is
not one of the entities empowered under the
constitution to exercise the power of eminent
domain. It was merely a resolution empowering
itself which is very arbitrary on the part of
COMELEC.

TAU MU

Note:
1. It is only necessary when the owner does not
want or opposes the sale of his property. Thus, if
a valid contract exists between the government
and the owner, the government cannot exercise
the power of eminent domain as a substitute to
the enforcement of the contract.

TAU MU

1. When property is needed for public use, and


the property owner does not want to sell his
property.
2. When the owner wants/willing to sell, but there
is failure to reach an agreement as to the price.

TAU MU

Situations where the Government is forced


to expropriate or to exercise the power of
eminent domain:

TAU MU

Who May Exercise the Power


Congress, and by delegation, the President,
administrative bodies, LGUs, and even private
enterprises performing public services [Nachura,
Outline Reviewer in Political Law 2006, p. 48].

TAU MU
TAU MU

The exercise of the right of eminent domain,


whether directly by the State, or by its authorized
agents, is necessarily in derogation of private
rights, and the rule in that case is that the
authority must be strictly construed. [Cruz,
Constitutional Law 2007, p.63]

The Fraternal
Ateneo de Davao

TAU MU
TAU MU

amendment by Congress in accordance with the


Constitution if the common good so requires. In
truth,
radio
and
television
broadcasting
companies, which are given franchises, do not
own the airwaves and frequencies though which
they transmit broadcast signal and images.
Airtime is not a finished product which become
the property of the company, like oil produced
form refining. Thus, no private property is taken.
As the government spends public funds in
granting broadcasting companies the privilege
and in supervising them, it would be strange if
they cannot be required to render public service
by giving free airtime.
Strict Construction against Expropriator
The limiting function imposed by the Bill is in
keeping with the philosophy of the Bill of Rights
against the arbitrary exercise of governmental
powers to the detriment of individual rights.
Given this function, the provision of Article III,
Section 9 should therefore be strictly interpreted
against the expropriator and liberally in favor of
the property owner.

KITY

TAU MU TAU MU TAU MU


TAU MU TAU MU TAU MU

CONSTITUTIONAL LAW II
Order of Saint Thomas More
Atty. Philip John Pojas/Atty. Rovyne G. Jumao-as, RN
University College of Law

them by the government, it is a privilege and not


a right. And being so, the Congress can take it
away anytime. In other words, there is no taking
of private property.
Under existing laws, the following may exercise
the power of expropriation:
f. The National Government
i.
Congress, through a law, and
the law shall provide for the procedure of the
expropriation
ii.
Executive,
pursuant
to
legislation enacted by Congress
iii.
Local legislative bodies, like the
Sanggunian, provided that it is granted by the
local government code
b. Local government units, pursuant to an
ordinance enacted by their respective legislative
bodies (under LGC)
c. Public utilities, as may be delegated by
law.
Certain public corporations, like the Land
Authority and the National Housing Authority;
d. Quasi-public corporations like Philippine
National Railways, PLDT and the MERALCO.
Eminent Domain vs. Destruction from
Necessity
EMINENT DOMAIN
DESTRUCTION FROM
NECESSITY
The right of eminent The right of necessity
domain is a public arises under the laws of
right; it arises from the society or society itself.
laws of society and is It is the right of selfvested in the state or defense,
of
selfits
grantee,
acting preservation, whether
under the right and applied to persons or to
power of the state, or property. It is a private
benefit of the state, or right vested in every
those acting under it.
individual, and with
which the right of the
state or state necessity
has nothing to do.
Requires payment of Does
not
require
just compensation
payment
of
just
compensation
Property must be for Cannot
require
the
public use
conversion
of
the
property taken to public
use
Scope of the Power of Eminent Domain
In the hands of Congress the scope of the power
is, like the scope of legislative power itself,
plenary. It is as broad as the scope of police
power itself. It can thus reach every form of
property which the State might need for public
use. It can reach even private property already
dedicated to public use or even property devoted
to religious worship [Bernas, The 1987 Philippine
Constitution: A Comprehensive Reviewer, p. 22].
Foundation of Right to Exercise the Power
of Eminent Domain
The foundation of the right to exercise eminent
domain is genuine necessity and that necessity
must be of public character. Government may not

ACADCOM 2010; Contributors: Gene Geocaniga, Jarissa Guiani, Darlene Magabilen


TAU MU Page 97 of 179

capriciously or arbitrarily choose which private


property should be expropriated [Nachura,
Outline Reviewer in Political Law 2006, p. 49].
Private property may be expropriated for public
use and upon payment of just compensation;
condemnation of property is justified only if it is
for the public good and there is genuine necessity
therefor of a public character. Consequently, the
courts have the power to inquire into the legality
of the right of eminent domain and to determine
whether or not there is a genuine necessity
therefor [Cruz, Constitutional Law 2007, p.66].

TAU MU
TAU MU
TAU MU
TAU MU
TAU MU

Taking, in the constitutional sense, may include


trespass without actual eviction of the owner,
material impairment of the value of the property
for which the property was intended. In People
vs. Fajardo (104 Phil 44), a municipal ordinance
prohibiting a building which would impair the
view of the plaza from the highway was
considered taking. The property owner was
entitled to payment of just compensation
[Nachura, Outline Reviewer in Political Law 2006,
p. 50].

TAU MU

Two concepts of TAKING:


1. Physical taking or the transfer of title
2. Taking as prevention or impairment of
ordinary use - This is any act which necessarily
afects the useful or beneficial use of the
property. (No actual transfer)

TAU MU

TAU MU

Requisites of Taking (Republic vs. Castellvi):


6. The expropriator must enter a private
property;
7. The entry must be for more than a
momentary period;
8. The entry must be under warrant or color
of legal authority;
9. The property must be devoted to public
use or otherwise informally appropriated
or injuriously afected;
10. The utilization of the property for public
use must be in such a way as to oust the
owner and deprive him of the beneficial
enjoyment of the property.

TAU MU
TAU MU
TAU MU

Note:
1. Compensable taking does not need to involve
all the property interests which form part of the
right of ownership. When one or more of the
property rights are appropriated and applied to a

The Fraternal
Ateneo de Davao

TAU MU
TAU MU

Requisites for the Exercise of the Power of


Eminent Domain
f.
TAKING
Taking, as the term is commonly understood,
imports a physical dispossession of the owner, as
when he is ousted from his land or relieved of his
watch or his car and is thus deprived of all
beneficial use and enjoyment of his property. In
law, however, the term has a broader
connotation. Taking may include trespass without
actual eviction of the owner, material impairment
of the value of the property or prevention of the
ordinary uses for which the property was
intended [Cruz, Constitutional Law 2007, p.70].

KITY

TAU MU TAU MU TAU MU


TAU MU TAU MU TAU MU

CONSTITUTIONAL LAW II
Order of Saint Thomas More
Atty. Philip John Pojas/Atty. Rovyne G. Jumao-as, RN
University College of Law

public purpose, there is already a compensable


taking, even if bare title still remains with the
owner.
2. All kinds of property can be expropriated,
private property, personal property, real property.
But what cannot be expropriated? MONEY. So, if
expropriations
proceedings
were
instituted
against a person's bank account, that is not
allowed.
When municipal property is taken by the
State:
There is compensable taking depending on the
nature of the property. If it is patrimonial property
of the municipality, that is, property acquired by
the municipality with its private funds in its
corporate or private capacity, compensation is
required. However, if it is any other property such
as public buildings or legua communal held by
the municipality for the State in trust for the
inhabitants, the State is free to dispose of it at
will [Bernas, The 1987 Philippine Constitution: A
Comprehensive Reviewer, p. 104].
Effect of Police Power and Eminent Domain
on Private Property*
EMINENT DOMAIN
POLICE POWER
Property is taken
Property is regulated
Transfer of ownership
No
transfer
of
ownership
The term taking must The term regulation is
be compensated.
not compensable
*[Bernas, The 1987 Philippine Constitution: A
Comprehensive Reviewer, p. 104]

CASES: TAKING
REPUBLIC vs. CASTELLVI
58 SCRA 336 (1974)
Facts: In 1947, The PAF leased the property of
Castellvi renewable year to year. The lease was
renewed yearly until 1956, Castellvi refused to
renew the contract with PAF any longer. In 1959,
the government instituted an action to
expropriate the property previously leased. Also,
in the same year, the property was placed under
the possession of the government by virtue of a
court order. TAU MU
Issues:
1. Castellvi and the government cannot agree on
the price
Government P 0.20 per sq.meter
Castellvi P15.00 per sq.meter
2. They do not also agree on the date which will
be the basis for the determination of the value of
the land.
Governement value in 1947
Castellvi value in 1959
Held: The court held that the value should be
determined at the time of the taking. The taking
here would mean 1959, where all the elements of
taking were present. It cannot be 1947 since it
did not satisfy the 2nd and 5th elements.
DIDIPIO EARTH SAVERS vs. GUZON

ACADCOM 2010; Contributors: Gene Geocaniga, Jarissa Guiani, Darlene Magabilen


TAU MU Page 98 of 179

485 SCRA 586 (2006)


Facts: Sec. 76 of RA 7942 otherwise known as
the Philippine Mining Act of 1995, provides:
Entry into private lands and concession areas
Subject to prior notification, holders of mining
rights shall not be prevented from entry into
private lands and concession areas by surface
owners, occupants, or concessionaires when
conducting mining operations therein.
Issue:Is the provision a form of taking which is
subject to just compensation?

TAU MU
TAU MU
TAU MU
TAU MU
TAU MU
TAU MU
TAU MU

Held: Yes, there was taking. The ownership of


land extends to the surface as well as to the
subsoil under it.
First, the argument that the landowners
right extends to the subsoil insofar as necessary
for their practical interests serves only to further
weaken the NPCs case. The theory would limit
the right to the subsoil upon the economic utility
which such area ofers to the surface owners.

TAU MU

Issue: Was there taking as to entitle the owners


to compensation?

TAU MU

Facts: In 1978, NAPOCOR, without the


landowners knowledge and prior consent, took
possession of the sub-terrain area of their lands
and constructed therein underground tunnels.
When discovered by the owners in 1992, they
asked for compensation. NAPOCOR denied
arguing that:
3. The owners rights extend to the subsoil
only insofar as necessary for their
practical interests, and
4. The tunnels merely constitute and
easement upon their property, which does
not involve any loss or title or possession.
Hence, they were not denied the beneficial
use of their subject properties as to entitle
them to just compensation by way of
damages.

TAU MU
TAU MU

NPC vs. IBRAHIM


526 SCRA 149 (2007)

The Fraternal
Ateneo de Davao

TAU MU
TAU MU

Held: Yes. The entry referred to in Sec. 76 is not


just a simple right-of-way which is ordinarily
allowed under the provisions of the Civil Code.
Here, the holders of mining rights enter private
lands for purposes of conducting mining activities
such as exploration, extraction and processing of
minerals. Mining right holders build mine
infrastructure, dig mine shafts and connecting
tunnels, prepare tailing ponds, storage areas and
vehicle
depots,
install
their
machinery,
equipment and server systems. On top of this,
under Sec 75, easement rights are accorded to
them where they may build warehouses, port
facilities, electric transmission, railroads and
other infrastructures necessary for mining
operations. All these will definitely oust the
owners or occupants of the afected areas the
beneficial ownership of their lands. Without a
doubt taking occurs once mining operations
commence.

KITY

TAU MU TAU MU TAU MU


TAU MU TAU MU TAU MU

CONSTITUTIONAL LAW II
Order of Saint Thomas More
Atty. Philip John Pojas/Atty. Rovyne G. Jumao-as, RN
University College of Law

Presumably, the landowners right extends to


such height or depth where it is possible for them
to obtain some benefit or enjoyment, and it is
extinguished beyond such limit as there would be
no more interest protected by law.
The owners could have dug upon their
property motorized deep wells but were
prevented from doing so by NPC precisely
because of the construction and existence of the
tunnels underneath the surface of their property.
They, therefore, still had a legal interest in the
sub-terrain portion insofar as they could have
excavated the same for the construction of the
deep well.
Second, the manner in which the easement was
created by NPC, however, violates the due
process rights of the owners as it was, without
notice and indemnity to them and did not go
through proper expropriation proceedings.
NPC could have, at any time, validly
exercised the power of eminent domain to
acquire the easement over the property as this
power encompasses not only the taking or
appropriation of title to and possession of the
expropriated property but likewise covers even
the imposition of a mere burden upon the owner
of the condemned property.
The
underground
tunnels
impose
limitations on the use of the property for an
indefinite period and deprives them of its
ordinary use. Based upon the foregoing, the
owners are clearly entitled to the payment of just
compensation.
REPUBLIC vs. ANDAYA
524 SCRA 671 (2007)
Facts: Ismael Andaya is the owner of two parcels
of land in Butuan City, which were subject to a
60-meter wide perpetual easement for public
highways, irrigation ditches, aqueducts, and other
similar works of the government or public
enterprise. Out of the 10,380 sq. m. property, 701
sq. m. was subject of the easement. During the
expropriation proceedings, Andaya demanded
that the consequential damages he must be paid
should be passed on the remaining area of 9,679
sq. m. which is the entire property minus the
easement. Andaya alleged that the easement
would render his entire property unusable and
unhabitable. Is he correct?
Held: Yes. Taking, in the exercise of the power of
eminent domain, occurs not only when the
government actually deprives or dispossesses the
property owner of his property or of its ordinary
use, but also when there is a practical destruction
or material impairment of the value of his
property. Using this standard, there was
undoubtedly a taking of the remaining area of
Andayas property. True, no burden was imposed
thereon and Andaya still retained title and
possession of the property. But the nature and
the efect of the floodwalls would deprive Andaya
of the normal use of the remaining areas. It would
prevent ingress and egress to the property and
turn it into a catch basin for the floodwaters
coming from the Agusan River. Hence, Andaya is

ACADCOM 2010; Contributors: Gene Geocaniga, Jarissa Guiani, Darlene Magabilen


TAU MU Page 99 of 179

CARLOS SUPERDRUG vs. DSWD


526 SCRA 130 (2007)
Facts:
Carlos
Superdrug
questioned
the
constitutionality of the Expanded Senior Citizens
Act of 2003, which grants to senior citizens 20%
discount on their purchase of medicines.
According to CS, the law is confiscatory because
it compels drugstore owners to grant the discount
and will result in a loss of profit and capital since
it tailed to provide a scheme whereby drugstores
will be justly compensated for the discount.
Issue: Was there a valid taking?

TAU MU
TAU MU
TAU MU
TAU MU
TAU MU
TAU MU
TAU MU

As a requirement of eminent domain, public use


is the general concept of meeting public need or
public exigency. It is not confined to actual use by
the public in its traditional sense. The idea that
public use is strictly limited to clear cases of
use by the public has been abandoned. The

The
predominant
precept
is
that
upon
abandonment of real property condemned for
public purpose, the party who originally
condemned the property recovers control of the
land if the condemning party continues to use the
property for public purpose; however, if the
condemning authority ceases to use the property
for a public purpose, property reverts to the
owner in fee simple. The governments taking of
private property, and then transferring it to
private persons under the guise of public use or
purpose is the despotism found in the immense
power of eminent domain. Moreover, the direct
and unconstitutional states power to oblige a
landowner to renounce his productive and
invaluable possession to another citizen, who will
use it predominantly for his own private gain, is
ofensive to our laws [Bernas, The 1987
Philippine
Constitution:
A
Comprehensive
Reviewer, p. 104].

TAU MU

Note:
1. Public use, for purposes of expropriation, is
synonymous with public welfare as the latter
term is used in the concept of police power.
2. Examples of public use include land reform and
socialized housing.
3. As a general rule, when private property is
expropriated for public use and it is not used, it
cannot be returned to the owner. But if there is an
implication that the property has to be returned
to the owner if the property is abandoned, then
the property should be returned. But the original
owner must return the compensation. (Heirs of
Moreno case)

term public use has not been held to be


synonymous with public interest, public benefit,
public welfare, and public convenience [Nachura,
Outline Reviewer in Political Law 2006, p. 51].

TAU MU

g. PUBLIC USE
Two concepts of public use:
1. The traditional concept: those which are to be
used by the public whether for a fee or for free.
i.e. the roads, public plaza, etc.
2. The expanded concept: one which will satisfy
public use even if the expropriated property is not
made directly available to the public for as long
as it results to an indirect advantage or benefit.
The advantage doesn't have to be direct.

Ateneo de Davao

TAU MU
TAU MU

Held: Yes. The Senior Citizens Act was enacted


primarily to maximize the contribution of senior
citizens to nation-building, and to grant benefits
and privileges to them for their improvement and
well-being as the State considers them an
integral part of our society.
The law is a legitimate exercise of police
power which, similar to the power of eminent
domain, has general welfare for its object.
Police power is not capable of an exact
definition but has been purposely veiled in
general
terms
to
underscore
its
comprehensiveness to meet all exigencies and
provide enough room for an efficient and flexible
response to conditions and circumstances, thus
assuring the greatest benefits.

The Fraternal

TAU MU
TAU MU

entitled to payment of just compensation, which


must neither more nor less than the monetary
equivalent of the land.

KITY

TAU MU TAU MU TAU MU


TAU MU TAU MU TAU MU

CONSTITUTIONAL LAW II
Order of Saint Thomas More
Atty. Philip John Pojas/Atty. Rovyne G. Jumao-as, RN
University College of Law

This original meaning of the phrase has now been


broadened to cover uses which, while not directly
available to the public, redound to their indirect
advantage or benefit [Cruz, Constitutional Law
2007, p.76].
Private property condemned for public use
sold to private user effect (Heirs of Moreno
vs. Mactan-Cebu, 2005):

CASES: PUBLIC USE


MASIKIP vs. CITY OF PASIG
479 SCRA (2006)
Facts: The City of Pasig, notified Masikip of its
intention to expropriate a portion of her property
to be used for the sports development and
recreational activities. She replied that the
intended expropriation of her property is
unconstitutional, invalid, and oppressive, as the
area of her lot is neither sufficient nor suitable to
provide land opportunities to deserving poor
sectors of our community.
She contended that the City failed to
establish a genuine necessity which justifies the
condemnation of her property. While she does not
dispute the intended public purpose, nonetheless,
she insists that there must be a genuine
necessity for the proposed use and purposes.
Held: LGUs have no inherent power of eminent
domain and may exercise it only when expressly
authorized by statute. Section 19 of the LGC
prescribes the delegation by Congress of the
power of eminent domain to LGUs and lays down
the parameters for its exercise.
SEC. 19. Eminent Domain. - A local government
unit may, through its chief executive and acting
pursuant to an ordinance, exercise the power of
eminent domain for public use, or purpose, or
welfare for the benefit of the poor and the
landless, upon payment of just compensation,

ACADCOM 2010; Contributors: Gene Geocaniga, Jarissa Guiani, Darlene Magabilen


TAU MU Page 100 of 179

TAU MU
TAU MU
TAU MU
TAU MU
TAU MU
TAU MU
TAU MU
TAU MU

Held: The act of NHA of entering into a contract


with a real estate developer for the construction
of low cost housing on the expropriated lots
cannot be taken to mean as a deviation from the
stated
public
purpose
of
their
taking.
Jurisprudence has it that the expropriation of
private land for slum clearance and urban
development is for a public purpose even if the
developed area is later sold to homeowners,
commercial firms, service companies and other
private concerns. Moreover, the Constitution itself
allows the State to undertake, for the common
good, and in cooperation with the private sector,

TAU MU

Facts: In 1977, the National Housing Authority


(NHA) field complaints to expropriate sugarcane
lands in Cavite. The purpose of the expropriation
was
the
expansion
of
the
Damarias
Resettlement Project to accommodate the
squatters who were relocated from the
Metropolitan Manila Area. The trial court rendered
judgment ordering the expropriation, after
payment of just compensation, of the subject lots
which decision was affirmed by the SC in 1987. In
1992, the previous owners of the expropriated
properties filed a case against NHA for their
return on the ground that NHA deviated from the
public purpose of the taking when it entered into
a contract for the construction of low cost
housing units to be sold to low-income
beneficiaries.

TAU MU
TAU MU

REYES vs. NHA


395 SCRA 495 (2003)

The Fraternal
Ateneo de Davao

TAU MU
TAU MU

pursuant to the provisions of the Constitution and


pertinent laws: Provided, however, That the
power of eminent domain may not be exercised
unless a valid and definite offer has been
previously made to the owner, and such offer was
not accepted: Provided, further, That the local
government
unit
may
immediately
take
possession of the property upon the filing of the
expropriation proceedings and upon making a
deposit with the proper court of at least fifteen
percent (15%) of the fair market value of the
property based on the current tax declaration of
the property to be expropriated: Provided, finally,
That, the amount to be paid for the expropriated
property shall be determined by the proper court,
based on the fair market value at the time of the
taking of the property.
According to Masikip, there is already an
established sports development and recreational
activity center. The City does not dispute this.
Evidently, there is no genuine necessity to
justify the expropriation.
Where the taking by the State of private property
is done for the benefit of a small community
which seeks to have its own sports and
recreational facility, notwithstanding that there is
such a recreational facility only a short distance
away, such taking cannot be considered to be for
public use. Its expropriation is not valid.
The right to take private property for public
purposes necessarily originates from the
necessity and the taking must be limited to such
necessity.

KITY

TAU MU TAU MU TAU MU


TAU MU TAU MU TAU MU

CONSTITUTIONAL LAW II
Order of Saint Thomas More
Atty. Philip John Pojas/Atty. Rovyne G. Jumao-as, RN
University College of Law

a continuing program of urban land reform and


housing which will make at afordable cost decent
housing to homeless citizens. It follows that the
low cost housing of the NHA on the expropriated
lots is consistent with the public use requirement.
ESTATE vs. PHILIPPINE EXPORT
349 SCRA 240 (2000)
Facts: Philippine Export Processing Zone (PEZA)
initiated
before
the
RTC
expropriation
proceedings on 3 parcels of irrigated rice land.
The power of eminent domain of PEZA is
contained in its original charter, PD No. 66, which
allows it to expropriate any property for export
processing zones, low-cost housing, or for the
construction
of
dams,
terminal
facilities,
structures and approaches thereto. Considering
that portions of the expropriated property would
be leased to banks, is it still for a public
purpose?
Held: Yes. The term public use has acquired a
more comprehensive coverage. To the literal
import of the term signifying strict use by the
public has been added the broader notion of
indirect public benefit or advantage. It should be
pointed out that PEZA was created to be a viable
commercial, industrial and investment area. The
expropriation of the lot for the purpose of being
leased to banks and for the construction of a
terminal has the purpose of making banking and
transportation facilities easily accessible to
persons working at industries located at PEZA. It
therefore comes as a matter of necessity to bring
life to the purpose of the law, as reaffirmed by
RA No. 9716. Moreover, Congress can determine
the necessity of expropriating private property. In
such a case, it is a question exclusively for the
legislature to decide.
MANAPAT vs. CA and NHA
536 SCRA 32 (2007)
Facts: The subject matter of this case were the
parcels of land which form part of what was
originally known as the Grace Park Subdivision
and formerly owned by the Roman Catholic
Archbishop of Manila (RCAM) and/or the Philippine
Realty Corporation (PRC). In the 1960s, RCAM
allowed a number of individuals to occupy the
Grace Park property on condition that they would
vacate the premises should the former push
through with the plan to construct a school in the
area. The plan, however, did not materialize,
thus, the occupants ofered to purchase the
portions they occupied. Later, as they could not
aford RCAMs proposed price, the occupants
petitioned the Government for the acquisition of
the said property, its subdivision into home lots,
and the resale of the subdivided lots to them at a
low price.
The Government, through the Peoples
Homesite and Housing Corporation (PHHC),
negotiated for the acquisition of the property
from RCAM/PRC. But because of the high asking
price of RCAM and the budgetary constraints of
the Government, the latters efort to purchase
and/or to expropriate the property was

ACADCOM 2010; Contributors: Gene Geocaniga, Jarissa Guiani, Darlene Magabilen


TAU MU Page 101 of 179

TAU MU
TAU MU
TAU MU
TAU MU
TAU MU
TAU MU
TAU MU
TAU MU
TAU MU

Ruling: Albeit the power of eminent domain


partakes of a sovereign character, it is by no
means absolute. Its exercise is subject to
limitations, one of which is, precisely, Section 9,
Article III of the Constitution. The SC has
recognized the following requisites for the valid
exercise of the power of eminent domain: (1) the
property taken must be private property; (2)
there must be genuine necessity to take the
private property; (3) the taking must be for public
use; (4) there must be payment of just
compensation; and (5) the taking must comply
with due process of law.
It is incontrovertible that the parcels of
land subject of these consolidated petitions are
private property. Thus, the first requisite is
satisfied. With respect to the second, the
foundation of the right to exercise eminent
domain is genuine necessity, and that necessity
must be of a public character. As a rule, the
determination of whether there is genuine
necessity for the exercise is a justiciable question.
However, when the power is exercised by the
Legislature, the question of necessity is
essentially a political question.
In the case at
bar, the authority to expropriate came from PD
1072. At that time, and as explicitly recognized
under the 1973 Constitution, President Marcos
had
legislative
powers.
Perforce,
the
expropriation of the subject properties identified
with specificity in the P.D. --- was directed by
legislation. The issue of necessity then assumed
the nature of a political question.
As to the third requisite of public use, the
SC examined the purpose for which the
expropriation was undertaken by NHA. As set
forth in its petition, NHA justifies the taking of the
subject property for the purpose of improving and
upgrading the area by constructing roads and
installing
facilities
thereon
under
the
Governments zonal improvement program and
subdividing them into much smaller lots for
distribution and sale at a low cost to qualified
beneficiaries, mostly underprivileged long-time
occupants of Grace Park. Around 510 families
with approximately 5 members each will be

TAU MU
TAU MU

Issue: WON NHA has a lawful right to take the lot


of Manapat for supposed public use.

The Fraternal
Ateneo de Davao

TAU MU
TAU MU

discontinued. RCAM then decided to efect, on its


own, the subdivision of the property and the sale
of the individual subdivided lots to the public. The
parties in these consolidated cases were among
those who purchased individual subdivided lots of
Grace Park directly from RCAM and/or PRC.
A significant turn of events however
happened when the President Marcos issued PD
No. 1072, appropriating Php1.2M out of the
Presidents Special Operations Funds to cover the
additional amount needed for the expropriation of
Grace Park. The NHA, PHHCs successor, then
filed several expropriation proceedings over the
already subdivided lots for the purpose of
developing Grace Park under the Zonal
Improvement Program (ZIP) and subdividing it
into small lots for distribution and resale at a low
cost to the residents of the area.

KITY

TAU MU TAU MU TAU MU


TAU MU TAU MU TAU MU

CONSTITUTIONAL LAW II
Order of Saint Thomas More
Atty. Philip John Pojas/Atty. Rovyne G. Jumao-as, RN
University College of Law

benefited by the project. The only remaining


obstacle in the completion of this project is the
lots subject of these consolidated petitions as the
other lots in Grace Park have already been
expropriated.
The Zonal Improvement Program (ZIP),
being implemented for government by NHA,
draws breath from policy mandates found in the
1987 Constitution. It is an integral part of the
governments socialized housing program
which the SC deemed compliant with the public
use requirement, it being a program clearly
devoted to a public purpose.
It need only be added that the public
use requisite for the valid exercise of the power
of eminent domain is a flexible and evolving
concept influenced by changing conditions. It
may not be amiss to state that whatever is
beneficially employed for the general welfare
satisfies the requirement of public use. Still,
Manapat insists that, being himself a beneficiary
of the expropriation (because he has been a longtime resident of Grace Park), it would be
incongruous for government to take his land
away from him only to give it back to him. This
contention sadly fails to comprehend the public
purpose for the taking under the socialized
housing program. The parcels of land subject of
the expropriation are, precisely, being taken so
that they can be subdivided into much smaller
lots --- at an average of 66.5 square meters per
lot --- for distribution to deserving dwellers in the
area. Upon the completion of the project,
Manapat, and those similarly situated as he,
cannot assert any right to be awarded the very
same lots they currently occupy, nor be entitled
to the same area of the land they now have.
Note: The power of eminent domain is an
inherent and indispensable power of the State.
Also called the power of expropriation, it is
described as the highest and most exact idea of
property remaining in the government that may
be acquired for some public purpose through a
method in the nature of a compulsory sale to the
State. By virtue of its sovereign character, the
exercise of the power prevails over the nonimpairment clause, and is clearly superior to the
final and executory judgment rendered by a court
in an ejectment case.
Being inherent, the power need not be
specifically conferred on the government by the
Constitution.
Section 9, Article III of the
Constitution, which mandates that private
property shall not be taken for a public use
without just compensation, merely imposes a
limit on the governments exercise of the power
and provides a measure of protection to the
individuals right to property. Just like its two
companion fundamental powers of the State, the
power of eminent domain is exercised by the
Legislature. However, it may be delegated by
Congress to the President, administrative bodies,
local government units, and even to private
enterprises performing public services.
HEIRS OF MORENO vs. MACTAN CEBU
413 SCRA 502 (2003)

ACADCOM 2010; Contributors: Gene Geocaniga, Jarissa Guiani, Darlene Magabilen


TAU MU Page 102 of 179

TAU MU
TAU MU
TAU MU
TAU MU
TAU MU
TAU MU
TAU MU
TAU MU
TAU MU
TAU MU
TAU MU

Held: In Fery, the SC declared that the


government acquires only such rights in the
expropriated parcels of land as may be allowed
by the character of its title over the properties:
If x x x
land is expropriated for a
particular purpose, with the condition that when
that purpose is ended or abandoned, the
property shall return to its former owner, then or
course, when the purpose is terminated or
abandoned the former owner reacquires the
property so expropriated. If x x x land is
expropriated for a public street and the
expropriation is granted upon condition that the
city can only use it for a public street, it returns
to the former owner, unless there is a statutory
provision to the contrary x x x if, upon the
contrary, however, the decree if expropriation
gives to the entity a fee simple title, then, of
course, the land becomes absolute property of
the expropriator, whether it be a State, province,
or municipality, and in that case the non-use does
not have the effect of defeating the title acquired
by the expropriation proceedings x x x
As for the public purpose of the
expropriation proceeding, it cannot now be
doubted. Although Mactan Airport is being
constructed, it does not take away the actual
usefulness and importance of the Lahug Airport:
it is handling the air traffic both civilian and
military. From it aircrafts fly to Mindanao and

The Fraternal
Ateneo de Davao

TAU MU
TAU MU

Facts: In 1949, the National Airport Corporation


as the predecessor agency of respondent MactanCebu International Airport Authority (MCIAA)
wanted to acquire Lot Nos. 916 and 920 among
other parcels of land for the proposed expansion
of Lahug Airport. To entice the landowners to
cede their properties, the government assured
them that they could repurchase their lands once
Lahug Airport was closed or its operations
transferred to Mactan Airport. TAU MU
At the end of 1991, Lahug Airport ceased
operations as the Mactan Airport was opened for
incoming and outgoing flights. Lot Nos. 916 and
920 which had been expropriated for the
extension of Lahug Airport were not utilized.
Hence, petitioners filed a complaint for
reconveyance and damages to compel the
repurchase of Lot Nos. 916 and 920.
Petitioners argue that Fery vs. Municipality of
Cabanatuan does not apply to the case at bar
since what was involved therein was the right of
reversion and not the right of repurchase
which they are invoking. They also diferentiate
Mactan-Cebu International Airport Authority vs.
CA from the instant case in that the landowners
in the MCIAA case ofered inadmissible evidence
to show their entitlement to a right of repurchase,
while petitioners herein ofered evidence based
on personal knowledge for which reason MCIAA
did not object and thus waived whatever
objection it might have had to the admissibility
thereof. Finally, petitioners allege that their right
to equal protection of the laws would be infringed
if some landowners are given the right to
repurchase their former properties even as they
are denied the exercise of such prerogative.

KITY

TAU MU TAU MU TAU MU


TAU MU TAU MU TAU MU

CONSTITUTIONAL LAW II
Order of Saint Thomas More
Atty. Philip John Pojas/Atty. Rovyne G. Jumao-as, RN
University College of Law

Visayas and past through it or their flights to the


North and Manila. Then, no evidence was
adduced to show how soon is the Mactan Airport
to be placed in operation and whether the Lahug
Airport will be closed immediately thereafter. It is
up to the other departments of the Government
to determine said matters. The Court cannot
substitute its judgment for those of the said
departments or agencies. In the absence of such
showing, the Court will presume that the Lahug
Airport will continue to be in operation.
The predicament of petitioners involves a
constructive trust, one that is akin to the implied
trust referred to in Art. 1454 of the NCC.
If an absolute conveyance of property is made in
order to secure the performance of an obligation
of the grantor toward the grantee, a trust by
virtue of law is established. If the fulfillment of
the obligation is offered by the grantor when it
becomes due, he may demand the reconveyance
of the property to him.
In the case at bar, petitioners conveyed Lot Nos.
916 and 920 to the government with the latter
obliging itself to use the realties for the
expansion of Lahug Airport; toiling to keep its
bargain, the government can be compelled by
petitioners to reconvey the parcels of land to
them, otherwise, petitioners would be denied the
use of their properties upon a state of afairs that
was not conceived nor contemplated when the
expropriation was authorized.
Hence,
respondent
MCIAA
as
representative of the State is obliged to reconvey
Lot Nos. 916 and 920 to petitioners who shall
hold the same subject to existing liens thereon, ie
leasehold right of DPWH.
HEIRS OF MORENO vs. MACTAN-CEBU
466 SCRA 288 (2005)
Facts: On MFR by Mactan International Airport
Authority, it was argued that the decision of the
Court which granted the owners right of
repurchase efectively overturns the ruling in
Fery, which requires that for an expropriation
judgment to be conditional, it must clearly spell
out said condition.
Held: Nothing in the Fery case bespeaks that
there should be an express condition in the
dispositive portion of the decision before the
condemned property can be returned to its
former owner after the purpose for its taking has
been abandoned or ended.
The indisputable certainty in the present
case is that there was a prior promise by the
predecessor of Mactan that the expropriated
properties may be recovered by the former
owners since the airport is transferred to Mactan,
Cebu. In fact, the witness for Mactan testified that
15 lots were already reconveyed to their previous
owners.
SUMULONG vs. GUERRERRO
154 SCRA 461 (1987)

ACADCOM 2010; Contributors: Gene Geocaniga, Jarissa Guiani, Darlene Magabilen


TAU MU Page 103 of 179

Facts: The NHA wanted to use Sumulongs


property for socialized housing for the lower and
middle class. The owner contended that
socialized housing is not public use because not
everyone can benefit from this, only the handful
of people who to be given the houses.

TAU MU
TAU MU
TAU MU
TAU MU
TAU MU
TAU MU
TAU MU

Held: The power of eminent domain is inherent


in every state and the provisions in the
Constitution pertaining to such power only serve
to limit its exercise in order to protect the
individual against whose property the power is
sought to be enforced. The State is not subject to
any limitation other than those imposed by the
Constitution which are: First, the taking must be
for a public use; Second, the payment of just
compensation must be made and;Third, due
process must be observed in the taking.
The due process clause cannot be
rendered nugatory everytime a specific decree or
law orders the expropriation of somebodys
property and provides its own peculiar manner of
taking the same. Neither should the courts adopt
a hands-of policy just because the public use has
been ordained as existing by the decree of the
just compensation has been fiexed and
determined beforehand by a statute.
Although due process does not always
necessarily demand that a proceeding be had

TAU MU

Issue: W/N the government had the right to


expropriate the said areas.

TAU MU

Facts: The President of the Philippines issued a


Letter of Instruction instituting an nationwide
slum improvement and resettlement program and
to adopt slum improvement as a national housing
policy. The Metro Manila Zonal Improvement
Program included the properties known as the
Tambunting Estate and the Sunog-Apog area in its
priority list for a zonal improvement program. The
President also designated the NHA to negotiate
with the owners of the property for the
acquisition of the same. This, however, did not
materialize as the negotiations for the purchase
of the property failed. The President issued Proc.
No. 1810 declaring all sites identified by the
Metro Manila local governments and approved by
the Ministry of Human Settlements to be included
in the ZIP upon proclamation of the President.
The Tambunting Estate and the Sunog-Apog area
were among the sites included.

TAU MU
TAU MU

MANOTOK vs. CA
150 SCRA 87 (May 21, 1987)

The Fraternal
Ateneo de Davao

TAU MU
TAU MU

Held: The court held that the socialized housing


is within the context of public use. Public use has
acquired a more comprehensive meaning. That is
whatever would result to indirect public benefit or
welfare is also public use. It also ruled that it will
benefit everyone in the sense that it will afect
the safety, health and environment. Providing
housing to these people will help in lessening the
incidence of violence and problems concerning
health. In the end, it will benefit everybody in a
way. In short, socialize housing falls within the
meaning of public use.

KITY

TAU MU TAU MU TAU MU


TAU MU TAU MU TAU MU

CONSTITUTIONAL LAW II
Order of Saint Thomas More
Atty. Philip John Pojas/Atty. Rovyne G. Jumao-as, RN
University College of Law

before a court of law, it still mandates some form


of proceeding wherein notice and reasonable
opportunity to be heard are given to the owner to
protect his property rights. There are exceptional
situations when, in the exercise of the power of
eminent domain, the requirement of due process
may not necessarily entail judicial process. But
where it is alleged that in the taking of a persons
property, his right to due process of law has been
violated, the courts will have to step in and probe
into such an alleged violation. A necessity must
exist for the taking of private property for the
proposed uses and purposes but accepted the
fact that modern decisions do not call for
absolute necessity. It is enough if the condemnor
can show a reasonable or practical necessity,
which of course, varies with the time and peculiar
circumstances of each case.
In the instant petitions, there is no showing
whatsoever as to why the properties involved
were singled out for expropriation through
decrees or what necessity impelled the particular
choices or selections. The provision of P.D. 1669
which allows NHA, at its sole option, to put
portions of the expropriated area to commercial
use in order to defray the development costs of
its housing projects cannot stand constitutional
scrutiny. P.D. Nos. 1669 and 1670 is violative of
the petitioners' right to due process of law and,
therefore,
they
must
fail
the
test
of
constitutionality. The decrees, do not by
themselves, provide for any form of hearing or
procedure by which the petitioners can question
the propriety of the expropriation of their
properties or the reasonableness of the just
compensation. The fixing of the maximum
amounts of compensation and the bases thereof
which are the assessed values of the properties in
1978 deprive the petitioner of the opportunity to
prove a higher value because, the actual or
symbolic taking of such properties occurred only
in 1980 when the questioned decrees were
promulgated. The decision of the government to
acquire a property through eminent domain
should be made known to the property owner
through a formal notice wherein a hearing or a
judicial proceeding is contemplated. This shall be
the time of reckoning the value of the property
for the purpose of just compensation. The
questioned decrees transgress the petitioners'
right to just compensation. Having violated the
due process and just compensation guarantees, P.
D. Nos. 1669 and 1670 are unconstitutional and
void.
MANOSCA vs. CA
252 SCRA 412 (1996)
Facts: Petitioners own a piece of land consisting
of 492 square meters, which was later
ascertained by the National Historical Institute to
be the birth site of Felix Y. Manalo, the founder of
the Iglesia Ni Cristo.
Consequently, it was
declared a national historical landmark, and the
Republic instituted expropriation proceedings.
Issue: Would the expropriation constitute an
application of funds for the use, benefit or
support of a religious entity?

ACADCOM 2010; Contributors: Gene Geocaniga, Jarissa Guiani, Darlene Magabilen


TAU MU Page 104 of 179

TAU MU
TAU MU
TAU MU
TAU MU
TAU MU
TAU MU
TAU MU
TAU MU

Cross-reference
with
PubCorp.:
Local
legislative body can delegate its power to the
Mayor.
Under the Local Government Code the f.
requisites must be present:

TAU MU

Note:
1. The medium of payment is money. It is always
money. But we know that there is an exception to
this, and that is under CARP, because the CARP is
a revolutionary undertaking. (Association of Small
Land Owners vs. Secretary)
2.
The determination of the amount of just
compensation is a judicial function.
3. Compensation is just if the owner receives a
sum equivalent to the market value of his
property. Market value is generally defined as the
fair value of the property as between one who
desires to purchase and one who desires to sell.
4. The point of reference use in determining fair
value is the value at the time the property was
taken. Thus, future potential use of the land is
not considered in computing just compensation.

TAU MU
TAU MU

h.
JUST COMPENSATION
Just compensation is described as a full and fair
equivalent of the property taken from the private
owner by the expropriator. This is intended to
indemnify the owner fully for the loss he has
sustained as a result of the expropriation. The
measure of this compensation is not just the
takers gain but the owners loss. The word just
is used to intensify the meaning of the word
compensation, to convey the idea that the
equivalent to be rendered for the property taken
shall be real, substantial, full, ample [Cruz,
Constitutional Law 2007, p.76].
Expressed diferently, the compensation given to
the owner is just if he receives for his property a
sum equivalent to its market value [Bernas,
The
1987
Philippine
Constitution:
A
Comprehensive Reviewer, p. 104].
It is the just and complete equivalent of the loss
which the owner of the thing appropriated has to
sufer by reason of the expropriation. It is the
market value to which can be added the
consequential damages, if any and from which
must be deducted consequential benefits, if any.

The Fraternal
Ateneo de Davao

TAU MU
TAU MU

Held:
The attempt to give some religious
perspective
to
the
case
deserves
little
consideration, for what should be significant is
the principal objective of, not the casual
consequences that might follow from, the
exercise of power. The purpose in setting up a
marker is essentially to recognize the distinctive
contribution of the late Felix Manalo to the culture
of the Philippines, rather than to commemorate
his founding and leadership of the Iglesia Ni
Cristo.
The practical reality that the greater
benefit may be derived by its members than by
most others could well be true but such a peculiar
advantage still remains to be merely incidental
and secondary in nature. Indeed that only a few
would actually benefit from the expropriation of
the property does not necessarily diminish the
essence and character of public use.

KITY

TAU MU TAU MU TAU MU


TAU MU TAU MU TAU MU

CONSTITUTIONAL LAW II
Order of Saint Thomas More
Atty. Philip John Pojas/Atty. Rovyne G. Jumao-as, RN
University College of Law

1. There must be a definite ofer which was


previously made upon the owner;
2. An ordinance enacted by the council
authorizing the Mayor to initiate the proceedings;
3. Exercised to expropriate the property for public
use;
4. Just compensation
Section 9, Article III of the Constitution
specifically mandates that Private property shall
not be taken for public use without just
compensation.
In City of Manila vs. Estrada, just
compensation means an equivalent for the value
of land (property) taken. The use of the word
just is to convey the idea that the equivalent to
be rendered for the property taken shall be real,
substantial,
full,
and
ample.
Thus,
just
compensation is defined as a fair and full
equivalent for the loss sustained.
In Manila Railroad Co. v. Caligsahan, the
SC held that to be exactly just, the
compensation should be estimated at the time of
the taking.
In Republic vs. Vda. De Castellvi, it was
ruled that just compensation is determined as of
the date of the taking of the property or the filing
of the complaint, whichever came first.

CASES: JUST COMPENSATION


SANTOS vs. LAND BANK
340 SCRA 59 (2000)
Facts: An RTC ordered Land Bank to pay P49M to
Santos for the taking of some 77 hectares of
agricultural land. It was also ordered that the
compensation shall be in the manner provided
under the CARL, which allows compensation in
cash and in bonds.
Issue: Is the mode of payment under the CARL
constitutional?
Held: Yes. Traditionally, the medium of payment
if just compensation is money and no other.
However, in land reform, we do not deal with the
traditional exercise of the power of eminent
domain.
This
is
a
revolutionary
kind
of
expropriation that does not afect only a specific
property or a limited area. Accepting the theory
that payment of just compensation need not
always be made fully in money, we find that the
proportion of cash payment is not unduly
oppressive. No less important, the government
financial instruments in making up the balance of
the payment are negotiable at any time.
-BASIS OF DETERMINATIONESLABAN vs. VDA DE ONORIO
360 SCRA 230 (2001)

ACADCOM 2010; Contributors: Gene Geocaniga, Jarissa Guiani, Darlene Magabilen


TAU MU Page 105 of 179

KITY

Held: Sec. 4, of Rule 67 of the Rules of Court


states that just compensation should be
determined as of the date of the taking of the
property or the filing of the complaint, whichever
comes first. The price of the land at the time of
the taking, not its value after the passage of
time, represents the true value to be paid as just
compensation. Thus, the just compensation to be
paid respondent should be determined as of its
taking by the NIA in 1981, not the filing of the
complaint in 1990.

TAU MU
TAU MU
TAU MU
TAU MU
TAU MU
TAU MU
TAU MU
TAU MU
TAU MU
TAU MU
TAU MU

Facts: Ibrahim and his co-heirs claimed that they


were owners of several parcels of land described
in Survey Plan FP (VII-5) 2278 consisting of
70,000 square meters, divided into three (3) lots,
i.e. Lots 1, 2, and 3 consisting of 31,894, 14,915,
and 23,191 square meters each respectively. In
1978, NAPOCOR, through alleged stealth and
without respondents knowledge and prior
consent, took possession of the sub-terrain area
of
their
lands
and
constructed
therein
underground tunnels. The existence of the
tunnels was only discovered sometime in July
1992 by respondents and then later confirmed on
November 13, 1992 by NAPOCOR itself through a
memorandum issued by the latters Acting
Assistant Project Manager. The tunnels were
apparently being used by NAPOCOR in siphoning
the water of Lake Lanao and in the operation of
NAPOCORs projects.
In 1992, respondent Maruhom, Ibrahims
co-heirs, requested the Marawi City Water District
for a permit to construct and/or install a
motorized deep well in Lot 3 but his request was
turned down because the construction of the
deep well would cause danger to lives and
property. Thereafter, respondents demanded that
NAPOCOR pay damages and vacate the subterrain portion of their lands but the latter refused
to vacate much less pay damages. Respondents
further averred that the construction of the
underground tunnels has endangered their lives
and properties as Marawi City lies in an area of
local volcanic and tectonic activity. Further, these
illegally constructed tunnels caused them
sleepless nights, serious anxiety and shock
thereby entitling them to recover moral damages
and that by way of example for the public good,

TAU MU
TAU MU

Facts: Respondent owns a piece of land with an


area of 39,512 square meters. On October 6,
1981, with the consent of respondent provided
they would be paid for the area taken after the
processing of documents by the Commission on
Audit, the National Irrigation Administration
constructed an irrigation canal.
The canal
afected 24,660 square meters. Since NIA failed
to pay for the property, on Dec. 10,1990,
respondent filed a complaint praying that NIA be
ordered to pay compensation for the property
used in the canal constructed. What year should
be used as basis for determining just
compensation?

NPC vs. IBRAHIM


526 SCRA 149 (2007)

The Fraternal
Ateneo de Davao

TAU MU TAU MU TAU MU


TAU MU TAU MU TAU MU

CONSTITUTIONAL LAW II
Order of Saint Thomas More
Atty. Philip John Pojas/Atty. Rovyne G. Jumao-as, RN
University College of Law

NAPOCOR must be held liable for exemplary


damages.
Issue: WON the respondents are entitled to just
compensation.
Ruling: The Court has held that if the
government takes property without expropriation
and devotes the property to public use, after
many years, the property owner may demand
payment of just compensation in the event
restoration of possession is neither convenient
nor feasible. This is in accordance with the
principle that persons shall not be deprived of
their property except by competent authority and
for public use and always upon payment of just
compensation.
Just compensation has been understood to
be the just and complete equivalent of the loss
and is ordinarily determined by referring to the
value of the land and its character at the time it
was taken by the expropriating authority. There is
a taking in this sense when the owners are
actually deprived or dispossessed of their
property, where there is a practical destruction or
a material impairment of the value of their
property, or when they are deprived of the
ordinary use thereof. There is a taking in this
context when the expropriator enters private
property not only for a momentary period but for
more permanent duration, for the purpose of
devoting the property to a public use in such a
manner as to oust the owner and deprive him of
all beneficial enjoyment thereof. Moreover,
taking of the property for purposes of eminent
domain entails that the entry into the property
must be under warrant or color of legal authority.
Under the factual backdrop of this case, the last
element of taking mentioned is patently lacking.
Hence, the taking was not made properly.
The manner in which the easement was
created by petitioner, however, violates the due
process rights of respondents as it was without
notice and indemnity to them and did not go
through proper expropriation proceedings. Hence,
respondents are entitled to just compensation.
Petitioner could have validly exercised the power
of eminent domain to acquire the easement over
respondents
property
as
this
power
encompasses not only the taking or appropriation
of title to and possession of the expropriated
property but likewise covers even the imposition
of a mere burden upon the owner of the
condemned property.
In the case at bar, the SC made reference
to the case Mangondato in computing the amount
of compensation to be given to the respondents.
In the Mangondato case, the general rule in
determining
just
compensation
in
eminent domain is the value of the property
as of the date of the filing of the complaint.
The general rule, however, admits of an
exception: where this Court fixed the value of the
property as of the date it was taken and not the
date of the commencement of the expropriation
proceedings. Thus, to allow petitioner to use the
date it constructed the tunnels as the date of
valuation would be grossly unfair. First, it did not

ACADCOM 2010; Contributors: Gene Geocaniga, Jarissa Guiani, Darlene Magabilen


TAU MU Page 106 of 179

enter the land under warrant or color of legal


authority or with intent to expropriate the same.
In fact, it did not bother to notify the owners and
wrongly assumed it had the right to dig those
tunnels under their property. Secondly, the
improvements
introduced
by
petitioner,
namely, the tunnels, in no way contributed to an
increase in the value of the land. The trial court,
therefore, as affirmed by the CA, rightly
computed the valuation of the property as of
1992,
when
respondents
discovered
the
construction of the huge underground tunnels
beneath their lands and petitioner confirmed the
same and started negotiations for their purchase
but no agreement could be reached.

TAU MU
TAU MU
TAU MU
TAU MU
TAU MU
TAU MU
TAU MU
TAU MU
TAU MU

Issue: WON the just compensation for


petitioners property should be based on the BIR

TAU MU
TAU MU

Facts: Prior to the transfer of the property to


petitioner
by
an
Antonio
Development
Corporation (SADC), Public Estates Authority
(PEA) wrote SADC requesting permission to enter
the latters property, then covered by TCT No.
439101, for the purpose of constructing thereon
the southern abutment of the Zapote Bridge at
the Coastal Road. PEA also proposed to SADC to
start their negotiation for its acquisition of the
latters property. SADC replied authorizing PEA to
enter the property, subject to the condition that
the latter should pay a monthly rental of Php10k.
PEA then directed its contractor, the Philippine
National Construction Corporation, to enter the
property and begin the necessary engineering
works on the Coastal Road.
Thereafter, PEA requested SADC either to
donate or sell the property to the government.
SADC replied by ofering to sell the property to
PEA. PEA informed SADC it has no plan to buy
the whole lot, but only the 1,131 square meter
portion above sea level. PEA then asked SADC to
submit proofs of ownership and costs of the
improvements which were demolished.
Negotiations then ensued between the parties.
However, for the past twenty (20) years, they
failed to reach an agreement. In 2000, SADC
asked PEA to pay compensation equivalent to the
current zonal value plus interest of 10% per
annum and a monthly rental of Php10k, also with
the same interest. These sums, according to
SADC, could be considered just compensation for
the governments use of the property since 1985
until September 2000 and thereafter. The
following month, PEA inquired from the BIR
District 53, the zonal value of the SADC
property. It submitted to the BIR the appraisal
reports prepared by two (2) independent licensed
appraisers. On April 2001, petitioner acquired the
property from SADC. The BIR sent a letter to PEA
stating that the zonal value of the property is
P2,900.00 per square meter, with the caveat that
the said assessment is subject to review and
approval by higher tax authorities. Thereafter, the
BIR informed PEA that the current zonal value of
the property is Php20k per square meter.

The Fraternal
Ateneo de Davao

TAU MU
TAU MU

TAN vs. REPUBLIC


May 25, 2007

KITY

TAU MU TAU MU TAU MU


TAU MU TAU MU TAU MU

CONSTITUTIONAL LAW II
Order of Saint Thomas More
Atty. Philip John Pojas/Atty. Rovyne G. Jumao-as, RN
University College of Law

zonal valuation in 1985 when petitioner entered


the subject property.
Ruling:
PEAs entry into the property with
the permission of SADC, its previous owner, was
not for the purpose of expropriating the property.
SADC allowed PEA to enter the land on condition
that it should pay a monthly rental ofPhp10k.
Thereafter, PEA requested SADC to donate or sell
the land to the government. Thereafter, SADC
ofered
to
sell
the
land
to
PEA
for
Php1,288,980.00,
plus
Php400,000.00
representing the value of the improvements
destroyed by PEA when it entered the property.
However, since 1985 up to the present, no
agreement has been reached between PEA and
SADC or herein petitioner who acquired the
property from the latter. While PEA has been
earning huge toll fees, it has refused to pay
petitioner any compensation for the use of her
property in violation of her right as an owner.
The above circumstances clearly show
that when PEA entered petitioners land in 1985,
it was not for the purpose of expropriating it.
After its entry, PEA wrote SADC requesting to
donate or sell the land to the government.
Indeed, there was no intention on the part of PEA
to expropriate the subject property. Section 2,
Rule 67 (on Expropriation) of the same Rules
provides, among others, that upon the filing of
the complaint or at any time thereafter and after
due notice to the defendant, the plaintif shall
have the right to take or enter upon the
possession of the real property involved if he
deposits with the authorized government
depositary an amount equivalent to the assessed
value of the property. There was no taking of the
property in 1985 by PEA for purposes of
expropriation. As shown by the records, PEA filed
with the RTC its petition for expropriation only in
September 2003. The trial court, therefore, was
correct in ordering respondent, through PEA,
upon the filing of its complaint for expropriation,
to pay petitioner just compensation on the basis
of the BIR zonal valuation of the subject property
at Php20k per square meter.
HEIRS of PIDACAN vs. AIR TRANSPORTATION
OFFICE
July 15, 2007
Facts: In 1935, spouses Mateo Pidacan and
Romana Eigo acquired under the homestead
provision of Act 2874 a parcel of land consisting
of about 22 hectares. Patent No. 33883 and OCT
No. 2204 were issued on the land, in the names
of the Pidacan spouses. In 1948, the Civil
Aeronautics Administration (now ATO) used a
portion of the said property as an airport. Upon
the death of the Pidacan spouses, the ATO
constructed a perimeter fence and a new
terminal building on the property. The ATO also
lengthened, widened, and cemented the airports
runway. The spouses heirs demanded from ATO
the payment of the value of the property as well
as rentals for the use of the occupied premises.
However, they were told that payment could not
be made because the property was still in their

ACADCOM 2010; Contributors: Gene Geocaniga, Jarissa Guiani, Darlene Magabilen


TAU MU Page 107 of 179

right of eminent domain to take advantage of the


ridiculously low value of the property at the time
of taking that it arbitrarily chooses to the
prejudice of petitioners.
In this particular case, justice and fairness
dictate that the appropriate reckoning point for
the valuation of petitioners property is when the
trial court made its order of expropriation in
2001. As for the fair value of the subject property,
the SC believe that the amount arrived at by the
commissioners appointed by the trial court,
Php304.39 per square meter, constitutes just
compensation to petitioners.
TIONGSON vs. NHA
558 SCRA 56 (2008)

TAU MU

Facts: NHA took possession in 1978 of properties


belonging to petitioners pursuant to P.D. No. 1669
(An Act Providing for the Expropriation of the
Property known as the Tambunting Estate) and
P.D. No. 1670 (An Act Providing for the
Expropriation of the Property along the Estero de
Sunog-Apog). The two presidential decrees were
held to be unconstitutional in 1987 for being
violative of right to due process.
Petitioners argue that since P.D. No. 1669
pursuant to which NHA took possession of their
properties in 1978 was declared unconstitutional,
necessarily, in thereafter resurrecting the filing
of another complaint for expropriation of the
same properties, it would be unlawful to fix the
reckoning period for purposes of computing the
just compensation based on NHAs previous
unlawful taking of said properties in 1978. They
thus maintain that the trial courts in 1997
holding that the determination of the just
compensation of their properties should be
reckoned from the date NHA filed the petition
before the RTC in 1987 is in order.

TAU MU

Issue: WON the subject properties sought to be


expropriated should be reckoned from the date of
taking of the property or from the date of the
filing of the complaint.

TAU MU

Ruling: The determination of just compensation


should be reckoned from NHAs taking of the
properties in 1978. The parties admitted that NHA
took possession of the subject properties as early
as 1978. In NHAs petition for expropriation filed
before the RTC constitute judicial admission of
NHA that it possessed the subject properties until
this Courts declaration promulgated on May 21,
1987, that P.D. No. 1669 pursuant to which NHA
took possession of the properties of petitioners in
1978 was unconstitutional and, therefore, null
and void. Hence, in the case at bar, just
compensation of petitioners properties must be
determined as of the date of the filing of NHAs
complaint on September 14, 1987.

TAU MU
TAU MU
TAU MU
TAU MU
TAU MU
TAU MU
TAU MU
TAU MU

Issue: WON petitioners are entitled to just


compensation.
Ruling: Eminent domain or expropriation is the
inherent right of the state to condemn private
property to public use upon payment of just
compensation. A number of circumstances must
be present in the taking of property for purposes
of eminent domain: (1) the expropriator must
enter a private property; (2) the entrance into
private property must be for more than a
momentary period; (3) the entry into the property
should be under warrant or color of legal
authority; (4) the property must be devoted to a
public use or otherwise informally appropriated or
injuriously afected; and (5) the utilization of the
property for public use must be in such a way as
to oust the owner and deprive him of all
beneficial enjoyment of the property.
When private property is rendered
uninhabitable by an entity with the power to
exercise eminent domain, the taking is deemed
complete. Taking occurs not only when the
government actually deprives or dispossesses the
property owner of his property or of its ordinary
use, but also when there is a practical destruction
or material impairment of the value of his
property.
In this case, it is undisputed that
petitioners private property was converted into
an airport by respondent ATO. As a consequence,
petitioners were completely deprived of beneficial
use and enjoyment of their property. Clearly,
there was taking in the concept of expropriation
as early as 1948 when the airport was
constructed on petitioners private land. As a rule,
the determination of just compensation in
eminent domain cases is reckoned from the time
of taking. In this case, however, application of the
said rule would lead to grave injustice. Note that
the ATO had been using petitioners property as
airport since 1948 without having instituted the
proper expropriation proceedings. To peg the
value of the property at the time of taking in
1948, despite the exponential increase in its
value considering the lapse of over half a
century, would be iniquitous. Hence, the SC
cannot allow the ATO to conveniently invoke the

The Fraternal
Ateneo de Davao

TAU MU
TAU MU

parents name. The heirs presented TCT No. T7160 and the death certificates of their parents to
the ATO, but the latter still refused to pay them.
The heirs claimed that they were entitled
to payment of rentals plus the value of the
property. The ATO countered that the heirs were
not entitled to any payment, either of the value of
the land or of the rentals because the property
had been sold to its predecessor, the defunct Civil
Aeronautics Administration for P0.70 per square
meter. The ATO claimed that even if it failed to
obtain title in its name, it had been declaring the
property for taxation purposes. The heirs
subsequently filed with the RTC a complaint
against the ATO for payment of the value of the
property as well as rentals for its use and
occupation. The ATO, in turn, filed a complaint for
expropriation, which was dismissed on the ground
that it would be absurd for the ATO to expropriate
a parcel of land it considered its own.

KITY

TAU MU TAU MU TAU MU


TAU MU TAU MU TAU MU

CONSTITUTIONAL LAW II
Order of Saint Thomas More
Atty. Philip John Pojas/Atty. Rovyne G. Jumao-as, RN
University College of Law

NEPUMUCENO vs. CITY of SURIGAO


560 SCRA 41 (2008)
Facts:
Petitioner
alleged
that
the
city
government neither asked her permission to use
the land nor instituted expropriation proceedings

ACADCOM 2010; Contributors: Gene Geocaniga, Jarissa Guiani, Darlene Magabilen


TAU MU Page 108 of 179

TAU MU
TAU MU
TAU MU
TAU MU
TAU MU
TAU MU

Facts: In 1969, the government instituted


expropriation proceedings against tracts of land
in Malolos, Bulacan, to be utilized for broadcast
operation and use of radio transmitter facilities of
the Voice of the Philippines. In 1979, the trial
court rendered judgment condemning the

TAU MU

REPUBLIC vs. CA
383 SCRA 611 (2002)

TAU MU

-EFFECT OF NON-PAYMENT-

TAU MU

Ruling: Where actual taking is made without the


benefit of expropriation proceedings and the
owner seeks recovery of the possession of the
property prior to the filing of expropriation
proceedings, it is the value of the property at the
time of taking that is controlling for purposes of
compensation. As held in the case of Republic vs.
Lara, the SC ruled that The owner of private
property should be compensated only for what he
actually loses; it is not intended that his
compensation shall extend beyond his loss or
injury. And what he loses is only the actual value
of his property at the time it is taken. This is the
only way the compensation to be paid can be
truly just; i.e., just not only to the individual
whose property is taken, but to the public, which
is to pay for it. Thus, the value of petitioners
property must be ascertained as of 1960 when it
was actually taken. It is as of that time that the
real measure of their loss may fairly be adjudged.
The value, once fixed, shall earn interest at the
legal rate until full payment is efected,
conformably with other principles laid down by
case law. Moreover, the damages prayed for by
the petitioners should be denied there being no
contractual obligations entered into between the
parties.

TAU MU
TAU MU

Issue: WON petitioners are entitled to just


compensation.

The Fraternal
Ateneo de Davao

TAU MU
TAU MU

for its acquisition. She and her husband wrote


respondent (then Surigao City Mayor) Salvador
Sering a letter proposing an amicable settlement
for the payment of the portion taken over by the
city. They subsequently met with Mayor Sering to
discuss their proposal but the mayor rebufed
them in public and refused to pay them anything.
Petitioners sought reconsideration of the mayors
stand. But again, the city mayor turned this down
in his reply. As a consequence, petitioners
claimed that they sufered mental anguish,
embarrassment, disappointment and emotional
distress which entitled them to moral damages.
Respondents admitted the existence of
the road in question but alleged that it was
constructed way back in the 1960s during the
administration of former Mayor Pedro Espina. At
that time, the lot was owned by the spouses
Fernandez who signed a road right-of-way
agreement in favor of the municipal government.
However, a copy of the agreement could no
longer be found because the records were
completely destroyed and lost when the Office of
the City Engineer was demolished by typhoon
Nitang in 1994.

KITY

TAU MU TAU MU TAU MU


TAU MU TAU MU TAU MU

CONSTITUTIONAL LAW II
Order of Saint Thomas More
Atty. Philip John Pojas/Atty. Rovyne G. Jumao-as, RN
University College of Law

property and ordering the government to pay as


just compensation P6.00 per square meter, which
included the 76,589 square meter subject of the
dispute. Eventually, portions of the land were
used for the expansion of Bulacan State
University. Despite the judgment however, and
despite utilization for public use by the
government, as of Sept. 16, 1999, the owners of
the property were not compensated. May the
owners demand the return of the property? If
not, may they demand compensation based on
the current value of the property?
Held: By final and executory judgment in the
expropriation proceedings, the previous owners
are not entitled to recover possession of their
expropriated land which are still devoted to
public use but only to demand the fair market
value.
It is of no moment that the present
property use difers from the original purpose of
the expropriation. Surely, the government, as
condemnor and owner of the property, is well
within its rights to alter and decide the use, the
only, limitation being that it is for public use. The
compensation for the property should be
computed at its market value at the time it was
taken and appropriated by the State. However,
the owners are entitled to an interest of 12% per
annum computed from the date of the taking of
the property, i.e. September 1969, until the due
amount shall have been fully paid.
REYES vs. NHA
395 SCRA 495
Ruling: The refusal of respondent NHA to pay
just compensation, allegedly for failure of
petitioners to pay capital gains tax and surrender
the owners duplicate certificates of title, to be
unfounded and unjustified.
First, under the expropriation judgment
the payment of just compensation is not subject
to any condition. Second, it is a recognized rule
that although the right to enter upon and
appropriate the land to public use is completed
prior to payment, title to the property
expropriated shall pass from the owner to the
expropriator only upon full payment of the just
compensation.
With respect to the amount of the just
compensation still due and demandable from
respondent NHA, the lower courts erred in not
awarding interest computed from the time the
property is actually taken to the time when
compensation is actually paid or deposited in
court. It is not disputed that respondent NHA took
actual possession of the expropriated properties
in 1977. Perforce, while petitioners are not
entitled to the return of the expropriated
property, they are entitled to be paid the balance
of P1,218,574.35 with legal interest thereon at
12% per annum computed from the taking of the
property in 1977 until the due amount shall have
been fully paid.
REPUBLIC vs. LIM
462 SCRA 265 (2005)

ACADCOM 2010; Contributors: Gene Geocaniga, Jarissa Guiani, Darlene Magabilen


TAU MU Page 109 of 179

Facts: On April 5, 1948 an entry of judgment was


made in the expropriation case filed by the
Republic against the Denzons. The judgment
allowed the Republic to expropriate their property
after payment of just compensation in the
amount of P4,062.10. After the lapse of more
than 50 years, the Republic failed to pay the
amount, though it took possession of the
property. May the landowner be allowed to
recover?

TAU MU
TAU MU
TAU MU
TAU MU
TAU MU
TAU MU
TAU MU

Held: In Panes v. Visayas, 264 SCRA 708 (1996),


we ruled that the determination of the amount of
provisional deposit for the issuance of the writ of
possession is a judicial function. Consequently, a
summary hearing must first be done by the judge
to determine provisionally the value of the
property, which full amount should be deposited
with the national or provincial treasurer.
However, under the 1987 Rules on Civil
Procedure, it is provided that upon filing of the
complain and after due notice to the defendant
the plaintif shall have the right to take or enter
upon the possession of the real property involved
if he deposits with the authorized government
depositary an amount equivalent to the assessed
value of the property for purposes of taxation to
be held by such bank subject to the orders of the
court. There is therefore no need of a hearing,
as the issuance of the writ of possession by the

TAU MU

Facts: Robern Development Corporation owns


parcels of land in Davao City intended for a lowcost housing project.
The National Power
Corporation filed a complaint to expropriate
them.
Upon deposit of P6,121.20 with the
Philippine National Bank representing the
assessed value of the property, the Regional Trial
Court issued a writ of possession in NPCs favor
as authorized by PD No. 42. Can the judge issue
the writ without first conducting a hearing on the
amount of just compensation?

TAU MU

ROBERN vs. QUITAIN


315 SCRA 150 (1999)

TAU MU
TAU MU

-WRIT OF POSSESSION-

The Fraternal
Ateneo de Davao

TAU MU
TAU MU

Held: Yes. While we are aware of the doctrine


that non-payment of just compensation does not
entitle the landowner to recover possession, the
facts of the present case do not justify its
application. It bears stressing that the Republic
was ordered to pay just compensation twice, the
first was in the expropriation proceedings and the
second in, Valdehueza. Fifty-seven years have
passed since then. We cannot but construe the
Republics failure to pay just compensation as a
deliberate refusal on its part. We thus rule that
the special circumstances prevailing in this case
entitle respondent to recover possession of the
expropriated lot from the Republic. Unless this
form of swift an efective relief is granted to him,
the grave injustice committed against his
predecessors-in-interest, though no fault or
negligence on their part, will be perpetuated.

KITY

TAU MU TAU MU TAU MU


TAU MU TAU MU TAU MU

CONSTITUTIONAL LAW II
Order of Saint Thomas More
Atty. Philip John Pojas/Atty. Rovyne G. Jumao-as, RN
University College of Law

court becomes ministerial once the provisional


compensation is deposited.

CITY of ILOILO vs. LEGASPI


444 SCRA 329 (2004)
Facts: In 2001, the Sangguniang Panlungsod of
the City of Iloilo enacted Regulation Ordinance
granting authority to its City Mayor to institute
expropriation proceedings on Lot No. 935,
registered in the name of Manuela Yusay. The
regulation ordinance was approved by then City
Mayor Malabor. Mayor Malabor wrote the heirs of
Yusay, through Mrs. Sylvia, Administratrix of the
estate of Manuela Yusay, making a formal ofer to
purchase their property known as Cadastral Lot
No. 935 with an area of 85,320 square meters for
P250 per square meter for the purpose of
converting the same as an on-site relocation for
the poor and landless residents of the city in line
with the citys housing development program.
Mayor Malabor informed the administrators,
Sylvia and Enrique, that their counter-proposal to
the Citys proposal to purchase Lot No. 935 was
not acceptable to the City Government,
particularly to the City Council, which insisted
that an expropriation case be filed. With their
apparent refusal to sell the property, the City
terminated further proceedings on the matter.
Petitioner City of Iloilo, represented by
Mayor Treas, filed an Amended Complaint or
Eminent Domain against private respondents
Heirs of Yusay. Public respondent Judge Legaspi
found the motion to be in order and meritorious,
and the grounds of the opposition to be
untenable; thus, he set the case for Preliminary
Hearing on the Special and Affirmative Defenses.
Petitioner filed a Motion for Issuance of Writ of
Possession alleging that since it has deposited
with the Court the amount of Php2,809,696.50
representing 15% of the fair market value of the
property sought to be expropriated based on its
current tax declaration, it may immediately take
possession of the property in accordance with
Section 19, Republic Act No. 7160.
Issue: WON petitioner may immediately take
possession
of
the
property
subject
of
expropriation by virtue of its power of eminent
domain.
Ruling: Petitioner has the irrefutable right to
exercise its power of eminent domain. It being a
local government unit, the basis for its exercise is
granted under Section 19 of Rep. Act No. 7160.
The requisites for authorizing immediate entry
are as follows: (1) the filing of a complaint for
expropriation sufficient in form and substance;
and (2) the deposit of the amount equivalent to
15% of the fair market value of the property to be
expropriated based on its current tax declaration.
Upon compliance with these requirements, the
issuance of a writ of possession becomes
ministerial.

ACADCOM 2010; Contributors: Gene Geocaniga, Jarissa Guiani, Darlene Magabilen


TAU MU Page 110 of 179

TAU MU
TAU MU
TAU MU
TAU MU
TAU MU
TAU MU
TAU MU
TAU MU
TAU MU

Facts: Petitioner, Capitol Steel, is a domestic


corporation which owns 65 parcels of land located
in the Municipality of Tagoloan, Province of
Misamis Oriental. PHIVIDEC, is a governmentowned and controlled corporation organized and
existing under PD 538, as amended, which is
vested with governmental and proprietary
functions including the power of eminent domain
for the purpose of acquiring rights of way or any
property for the establishment or expansion of
the Phividec Industrial Areas. The properties of
Capitol Steel were identified as the most ideal
site for the Mindanao International Container
Terminal Project (MICTP), a PHIVIDEC project
which involves the phased production of an 800meter berth and the acquisition of port
equipment to handle the volume of seaborne
break-bulk and container traffic in Mindanao.
Hence, PHIVIDEC filed an expropriation case. The
trial court issued a writ of possession in favor of
PHIVIDEC.
Due, however, to the unauthorized
engagement by PHIVIDEC of the legal services of
a private lawyer, the expropriation case was
dismissed. Capitol Steel requested the Technical
Committee on Real Property Valuation (TCRPV) of
the BIR for a revaluation of its properties. TCPRV
issued a resolution fixing the reasonable and
realistic zonal valuation of the properties at
Php700 per square meter. PHIVIDEC informed
Capitol Steel that it would file anew an
expropriation case and that it had deposited
Php116,563,500 in the name of Capitol
Steel.PHIVIDEC further informed Capitol Steel that

TAU MU
TAU MU

CAPITOL STEEL vs. PHIVIDEC


510 SCRA 590 (2006)

The Fraternal
Ateneo de Davao

TAU MU
TAU MU

In the case at bar, petitioner avers that


the Amended Complaint it filed complies with
both requisites, thus entitling it to a writ of
possession as a matter of right and the issuance
thereof becoming ministerial on the part of the
lower court even without any hearing. On the
other hand, private respondents allege that the
Amended Complaint is not sufficient in form and
substance since it failed to allege compliance
with the mandatory requirements for the exercise
of the power of eminent domain for purposes of
socialized housing. The Court finds the Amended
Complaint sufficient in form and substance, and
the amount of Php2,809,696.50 deposited with
the Regional Trial Court of Iloilo is equivalent to
15% of the fair market value of the property
sought to be expropriated per current tax
declaration..
For a writ of possession to issue, only two
requirements are required: the sufficiency in form
and substance of the complaint and the required
provisional deposit. In fact, no hearing is required
for the issuance of a writ of possession. The
sufficiency in form and substance of the
complaint for expropriation can be determined by
the mere examination of the allegations of the
complaint. In this case, the sufficiency of the
Amended Complaint was further confirmed by
public respondent when he set the case for pretrial and hearing.

KITY

TAU MU TAU MU TAU MU


TAU MU TAU MU TAU MU

CONSTITUTIONAL LAW II
Order of Saint Thomas More
Atty. Philip John Pojas/Atty. Rovyne G. Jumao-as, RN
University College of Law

the total amount deposited represents the zonal


value of the properties, and may be withdrawn at
any time. PHIVIDEC filed an Urgent Motion for the
Issuance of a Writ of Possession, to which it
attached a Certificate of Availability of Funds, and
Certifications from the Landbank and the DBP
that it deposited the total amount of
Php116,563,500 required under RA 8974.
Issue: WON the appellate court erred in ordering
the RTC to issue a writ of possession in favor of
respondent for the expropriation of the property.
Ruling: After a writ of possession was issued in
favor of respondent in the first expropriation
case, respondent commenced the construction of
infrastructure buildings and container port
terminals. Possession of the properties has since
remained with respondent, with the MICTP now
complete and fully operational. When the second
expropriation case was re-filed, R.A. 8974, which
provides for substantive requirements before a
writ of possession is issued, was already in force
and in efect. Under R.A. 8974, the requirements
for authorizing immediate entry in expropriation
proceedings involving real property are: (1) the
filing of a complaint for expropriation sufficient in
form and substance; (2) due notice to the
defendant; (3) payment of an amount equivalent
to 100% of the value of the property based on the
current relevant zonal valuation of the BIR
including payment of the value of the
improvements and/or structures if any, or if no
such valuation is available and in cases of utmost
urgency, the payment of the profered value of
the property to be seized; and (4) presentation to
the court of a certificate of availability of funds
from the proper officials.
Upon compliance with the requirements, a
petitioner in an expropriation case, in this case
the respondent, is entitled to a writ of possession
as a matter of right and it becomes the
ministerial duty of the trial court to forthwith
issue the writ of possession. No hearing is
required and the court neither exercises its
discretion or judgment in determining the amount
of the provisional value of the properties to be
expropriated as the legislature has fixed the
amount under Section 4 of R.A. 8974. The
payment of the provisional value as a prerequisite
to the issuance of a writ of possession difers
from the payment of just compensation for the
expropriated property. While the provisional
value is based on the current relevant zonal
valuation, just compensation is based on the
prevailing fair market value of the property. There
is no need for the determination with reasonable
certainty of the final amount of just compensation
before the writ of possession may be issued.
-EXPROPRIATION BY LGUSHEIRS OF SUGUITAN vs. CITY
328 SCRA 137 (2000)
Facts: In 1994, the Sangguniang Panglungsod ng
Mandaluyong City issued a resolution authorizing
the mayor to institute expropriation proceedings

ACADCOM 2010; Contributors: Gene Geocaniga, Jarissa Guiani, Darlene Magabilen


TAU MU Page 111 of 179

over the lot of Saguitan. Saguitan filed a motion


to dismiss arguing that the Local Government
Code requires an ordinance, not a mere
resolution. The trial court, however, denied the
motion reasoning that an ordinance would only
be required to appropriate funds in payment of
the expropriated property once the trial court has
determined its value. Is the court correct?

TAU MU
TAU MU
TAU MU
TAU MU
TAU MU
TAU MU

Facts: Respondent filed a complaint for


expropriation against petitioners. Respondent
needed petitioners' 16,256 sq. m. idle property at
the junction of the North Expressway, MalhacanIba-Camalig main road artery and the MacArthur
Highway. It planned to use it to establish a
common public terminal for all types of public

TAU MU

FRANCIA Jr. vs. MUN. OF MEYCAUAYAN,


BULACAN
549 SCRA 424 (2008)

TAU MU

Held: The applicable law as to the point of


reckoning for the determination of just
compensation is the Local Government Code.
The 1997 Rules on Civil Procedure cannot prevail
over RA No. 7160, which is a substantive law.
However, since the parties voluntarily agreed to
be bound by the report of the commission and
approved by the trial court, said valuation of the
property has the force of law between the parties
and should be complied with in good faith.

TAU MU

Facts: On Sept. 17, 1993, the City of Cebu filed


expropriation proceedings against respondents
who owned parcels of land which the former
wanted to use for the construction of a public
road. On Sept. 21, 1994, the court issued a writ
of possession in favor of the City. Sec. 19 of the
Local Government Code of 1991 provides that
just compensation shall be determined as of the
time of actual taking. However, Sec. 4, Rule 67 of
the 1997 Rules of Procedure provides that just
compensation shall be determined at the time of
the filing of the complaint for expropriation.
Which shall prevail?

TAU MU
TAU MU

CITY OF CEBU vs. DEDAMO


387 SCRA 754 May 7, 2002

The Fraternal
Ateneo de Davao

TAU MU
TAU MU

Held: No. Under the Local Government Code,


the exercise of eminent domain must comply with
the following requisites: 1) an ordinance enacted
by the council authorizing the chief executive; 2)
the power is exercised for public use, purpose or
welfare, or for the benefit of the poor and the
landless; 3) payment of just compensation; and
4) a definite ofer was previously made which was
not accepted by the property owner.
An
examination of the law shows that an ordinance
is necessary to authorize the filing of the
complaint with the proper court since, beginning
at that point, the power of eminent domain stats
to be exercised. The determination of the award
of just compensation is but the last stage which
cannot be arrived at without an initial finding by
the court that plaintif has a right to take
property.

KITY

TAU MU TAU MU TAU MU


TAU MU TAU MU TAU MU

CONSTITUTIONAL LAW II
Order of Saint Thomas More
Atty. Philip John Pojas/Atty. Rovyne G. Jumao-as, RN
University College of Law

utility vehicles with a weighing scale for heavy


trucks.
Petitioners denied that the property
sought to be expropriated was raw land. It was in
fact developed and there were plans for further
development. For this reason, respondents ofer
price of Php2,333,500 was too low. After trial, the
RTC ruled that the expropriation was for a public
purpose. The construction of a common terminal
for all public utility conveyances (serving as a
two-way loading and unloading point for
commuters and goods) would improve the flow of
vehicular traffic during rush hours. Moreover, the
property was the best site for the proposed
terminal because of its accessibility.
Issue: WON prior determination of the existence
of a public purpose was not necessary for the
issuance of a writ of possession in expropriation
cases.
Ruling: Section 19 of Republic Act 7160 provides
that: Section 19. Eminent Domain. A local
government unit may, through its chief executive
and acting pursuant to an ordinance, exercise the
power of eminent domain for public use, or
purpose, or welfare for the benefit of the poor
and the landless, upon payment of just
compensation, pursuant to the provisions of the
Constitution and pertinent laws; Provided,
however, That the power of eminent domain may
not be exercised unless a valid and definite ofer
has been previously made to the owner, and that
such ofer was not accepted; Provided, further,
That the local government unit may immediately
take possession of the property upon the filing of
the expropriation proceedings and upon making a
deposit with the proper court of at least 15% of
the fair market value of the property based on
the current tax declaration of the property to be
expropriated; Provided, finally, That, the amount
to be paid for the expropriated property shall be
determined by the proper court, based on the fair
market value at the time of the taking of the
property.
Before a local government unit may enter
into the possession of the property sought to be
expropriated, it must (1) file a complaint for
expropriation sufficient in form and substance in
the proper court and (2) deposit with the said
court at least 15% of the property's fair market
value based on its current tax declaration. The
law does not make the determination of a public
purpose a condition precedent to the issuance of
a writ of possession.
i.
NECESSITY
Questions of necessity or wisdom are essentially
political when decided by the national legislature
and are usually not subject to judicial review
[Cruz, Constitutional Law 2007, p.65]. But when
exercised by a delegate, the determination of
whether there is genuine necessity for the
exercise is a justiciable question [Nachura,
Outline Reviewer in Political Law 2006, p. 49].
The issue of the necessity of the expropriation is
a matter properly addressed to the RTC in the

ACADCOM 2010; Contributors: Gene Geocaniga, Jarissa Guiani, Darlene Magabilen


TAU MU Page 112 of 179

course of the expropriation proceedings. If the


property owner objects to the necessity of the
takeover, he should say so in his Answer to the
Complaint. The RTC has the power to inquire into
the legality of the exercise of the right of eminent
domain and to determine whether there is a
genuine necessity for it. [Nachura, Outline
Reviewer in Political Law 2006, p. 49].

TAU MU
TAU MU
TAU MU
TAU MU
TAU MU
TAU MU

1. The amount of just compensation


2. The choice of the property (Note: that
private property should be the last resort)
3. To determine the necessity of the taking
4. To determine the "public use" character
of the taking. However, if the expropriation is
pursuant to a specific law passed by Congress,

TAU MU

Matters under Judicial review of the


exercise of the power of eminent domain
(FYI: Res Judicate does not apply)

TAU MU

PROCEDURE:
1. Expropriating authority files a complaint for
expropriation against the landowner;
2. Expropriating authority must deposit in an
amount equivalent to the assessed value of the
property for the purposes of taxation in the bank
account of the land owner. (This is what you call
the provisional compensation, the basis is the
assessed value for purposes of taxation.)
3. After due notice to the land owner that the
amount has been deposited, the government can
now take possession because the court will issue
a writ of possession after the provisional
compensation has been deposited.

TAU MU

Property already devoted to public use is still


subject to expropriation, provided this is done
directly by the national legislature or under a
specific grant of authority to the delegate. A mere
general authority may not suffice. In such a case,
the courts will have authority to inquire into the
necessity of the expropriation and, finding none,
refuse to allow it [Cruz, Constitutional Law 2007,
p.68].

TAU MU
TAU MU

Even services may be subject to eminent domain


[Nachura, Outline Reviewer in Political Law 2006,
p. 50].

The Fraternal
Ateneo de Davao

TAU MU
TAU MU

j.
PRIVATE PROPERTY
Anything that can come under the dominion of
man is subject to expropriation. This will include
real and personal, tangible and intangible
properties [Cruz, Constitutional Law 2007, p.67].
All private property capable of ownership may be
expropriated, except money and choses in action
[Nachura, Outline Reviewer in Political Law 2006,
p. 50]. Expropriation of money would be a futile
act because of the requirement for the payment
of just compensation, usually also in money. A
chose in action is a personal right not reduced
into possession but recoverable by a suit at law, a
right to receive, demand or recover a debt,
demand or damages on a cause of action ex
contractu or for a tort or omission of duty. It is
essentially conjectural both as to its validity and
its value [Cruz, Constitutional Law 2007, p.68].

KITY

TAU MU TAU MU TAU MU


TAU MU TAU MU TAU MU

CONSTITUTIONAL LAW II
Order of Saint Thomas More
Atty. Philip John Pojas/Atty. Rovyne G. Jumao-as, RN
University College of Law

the courts cannot question the public use


character of the taking.
5. Authority
of
the
expropriating
authorities;
6. Whether the formalities have been
complied with.
Note: If the power is being exercised by an
agency possessing power delegated by Congress,
meaning the LGU's, President, Public utilities, all
issues are subject to judicial review. Including the
necessity and the purpose of public taking.

Issues which are NOT subject to


judicial review:
1. When the power is exercised by
Congress and it involves the issue of
necessity. It is the Congress will decide
if the taking is necessary.
2. The public use character of the
purpose of the taking property. If the
Congres will decide that this is the
purpose and this is for public use, the
court cannot look into that.

CASE: JUDICIAL REVIEW


DE KNECHT vs. BAUTISTA
100 SCRA 660 (1980)
Facts: The plan to extend EDSA to Roxas
Boulevard to be ultimately linked to the Cavite
Coastal Road Project, originally called for the
expropriation of properties along Cuneta Avenue
in Pasay City. Later on, however, the Ministry of
Public Highways decided to make the proposed
extension pass through Fernando Rein and Del
Pan Streets. Because of the protests of residents
of the latter, the Commission on Human
Settlements recommended the reversion to the
original plan, but the Ministry argued the new
route withh save the government P2 million. The
government filed expropriation proceedings
against the owners of Fernando Rein and Del Pan
streets, among whom was petitioner.
Held: The choice of Fernando Rein and Del Pan
streets is arbitrary and should not receive judicial
aprpoval. The Human Settlements Commission
concluded that the cost factor is so minimal that
it can be disregarded in making a choice between
the two lines. The factor of functionality strongly
militates against the choice of Fernando Rein and
Del Pan streets, while the factor of social and
economic impact bears grievously on the
residents of Cuneta Avenue. While the issue
would seem to boil down to a choice between
people, on one hand, and progress and
development, on the other, it is to be
remembered that progress and development are
carried out for the benefit of the people.
Point of reference for valuating a piece of
property:
General rule: The value must be that as of the
time of the filing of the complaint for
expropriation.

ACADCOM 2010; Contributors: Gene Geocaniga, Jarissa Guiani, Darlene Magabilen


TAU MU Page 113 of 179

Exception: When the filing of the case comes


later than the time of taking and meanwhile the
value of the property has increased because of
the use to which the expropriator has put it, the
value is that of the time of the earlier taking.
BUT if the value increased independently of what
the expropriator did, then the value is that of the
latter filing of the case.

TAU MU
TAU MU
TAU MU
TAU MU
TAU MU
TAU MU
TAU MU

Consequential
Damages/Consequential
Benefits
Consequential damages consist of injuries directly
caused on the residue of the private property
taken by the reason of expropriation. Where, for
example, the expropriator takes only a part of a
parcel of land, leaving the remainder with an odd
shape or area as to be virtually unusable, the
owner can claim consequential damages. On the
other hand, if the remainder is as a result of the
expropriation placed in a better location, such as
fronting a street where it used to be an interior
lot, the owner will enjoy consequential benefits
which should be deducted from the consequential
damages.
Consequential
benefits,
like
consequential damages, must be direct and
particular and not merely shared with the rest of
the properties in the area, as where there is a
general appreciation of lad values because of the
public use to which the condemned properties are
devoted [Cruz, Constitutional Law 2007, p.79].

TAU MU

Title to the property does not pass until after


payment, except in agrarian reform [Nachura,
Outline Reviewer in Political Law 2006, p. 57].

TAU MU

When a parcel of land is taken by eminent


domain, the owner of the fee is not necessarily
the only person who is entitled to compensation.
Every person having interest at law or in equity in
the land taken is entitled to share in the award. If
a person claiming an interest in the land sought
to be condemned is not made a party, he is given
the right to intervene and lay claim to the
compensation [Bernas, The 1987 Philippine
Constitution: A Comprehensive Reviewer, p. 108].

TAU MU
TAU MU

Owner (Concept)
According to Knecht vs. CA, the term owner as
applied in eminent domain cases refers to all
those who have lawful interest in the property to
be condemned, including a mortgagee, a lessee
and a vendee in possession under an executory
contract [Cruz, Constitutional Law 2007, p.78].

The Fraternal
Ateneo de Davao

TAU MU
TAU MU

Market Value (Definition)


Market value has been described in a variety of
ways. It is the price fixed by the buyer and seller
in the open market in the usual and ordinary
course of legal trade and competition; the price
and value of the article established or shown by
sale, public or private, in the ordinary way of
business; the fair value of property as between
one who desires to purchase and one who desires
to sell; the current price; the general or ordinary
price for which property may be sold in that
locality
[Bernas,
The
1987
Philippine
Constitution: A Comprehensive Reviewer, p. 104].

KITY

TAU MU TAU MU TAU MU


TAU MU TAU MU TAU MU

CONSTITUTIONAL LAW II
Order of Saint Thomas More
Atty. Philip John Pojas/Atty. Rovyne G. Jumao-as, RN
University College of Law

In such a case, the owner is not restricted to


payment of the market value of the portion
actually taken. In addition to the market value of
the portion taken, he is also entitled to payment
of consequential damages, if any to the
remaining part of the property. At the same time,
from the total compensation must be deducted
the value of consequential benefits, if any,
provided consequential benefits shall not exceed
consequential
damages
[Nachura,
Outline
Reviewer in Political Law 2006, p. 51]. If the
consequential benefits exceed the consequential
damages, these items should be disregarded
altogether as the basic value of the property
should be paid in every case [Cruz, Constitutional
Law 2007, p.79].
Reasonable Period
Just compensation includes not only the correct
determination of the amount to be paid to owner
of the land but also the payment of the land
within a reasonable period of time from its taking
[Bernas, The 1987 Philippine Constitution: A
Comprehensive Reviewer, p. 105].
Without prompt payment, compensation cannot
be considered just, for the property owner is
made to sufer the consequence of being
immediately deprived of his land while being
made to wait for a decade or more before
actually receiving the amount necessary to cope
with his loss.
To allow the taking of the
landowners properties and in the meantime
leave them empty-handed by withholding
payment of just compensation while the
government speculates on whether or not it will
pursue expropriation, or worse, for government to
subsequently decide to abandon the property and
return it to the landowner when it has already
been rendered useless by force majeure, it is
undoubtedly an oppressive exercise of eminent
domain that must never be sanctioned [Nachura,
Outline Reviewer in Political Law 2006, p. 54].
Property Assessment
The property taken should be assessed as of the
time of the taking, which usually coincides with
the
commencement
of
the
expropriation
proceedings. Where entry precedes the filing of
the complaint for expropriation, the assessment
should be made as of the time of the entry [Cruz,
Constitutional Law 2007, p.83].
The general rule is that the value must be that as
of the time of the filing of the complaint for
expropriation (Section 4, Rule 67, Rules of Court).
Moreover, the filing of the case generally
coincides with the taking. When, however, the
filing of the case comes later than the time of
taking and meanwhile the value of the property
has increased because of the use to which the
expropriator has put it, the value is that of the
time of the earlier taking. Otherwise, the owner
would gain undeserved profit. But if the value
increased independently of what the expropriator
did, then the value is that of the later filing of the
case [Bernas, The 1987 Philippine Constitution: A
Comprehensive Reviewer, p. 109].

ACADCOM 2010; Contributors: Gene Geocaniga, Jarissa Guiani, Darlene Magabilen


TAU MU Page 114 of 179

When eminent domain is exercised by a local


government unit, the amount to be paid for the
expropriated property shall be determined by the
proper court, based on the fair market value at
the time of the taking of the property (Sec. 19,
RA7160) precisely because the Rules of Court
cannot prevail over RA7160, a substantive law
[Nachura, Outline Reviewer in Political Law 2006,
p. 56].

TAU MU
TAU MU
TAU MU
TAU MU
TAU MU
TAU MU

Right of Owner in case of Non-payment of


Just Compensation
As a rule, non-payment of just compensation in
an expropriation proceeding does not entitle the
private landowner to recover possession of the
expropriated lots, but only to demand payment of
the fair market value of the property.

TAU MU

In some expropriation cases, the Court imposed


an interest of 12% per annum on the just
compensation due the landowner. It must be
stressed, however, that in these cases, the
imposition of interest was in the nature of
damages for delay in payment which, in efect,
makes the on the part of the government one of
forbearance. It follows that the interest in the
form of damages cannot be applied where there
was prompt and valid payment of just
compensation. Conversely, where there was
delay in tendering a valid payment of just
compensation, imposition of interest is in order
[Nachura, Outline Reviewer in Political Law 2006,
p. 56].

TAU MU

When there is delay in the payment of just


compensation, the owner is entitled to payment
of interest, if claimed; otherwise, interest is
deemed waived. In NAPOCOR vs. Angas, the
Supreme Court held that the interest due the
property owner is at the rate of 6% per annum,
prescribed in Article 2209 of the Civil Code, and
not 12% per annum under Central Bank Circular
No. 416, because the latter applies to loans or
forbearance of money, goods or credits, or
judgments involving such loans or forbearance of
money goods or credits. The kind of interest
involved here is by way of damages, hence Art.
2209 of the Civil Code applies.

TAU MU
TAU MU

Form of Compensation
As explicitly provided by Sec. 16(e), RA 6657, the
deposit of compensation must be in cash or in
Land Bank bonds, not in any other form, and
certainly not in a trust account.
Entitlement of Owner to Interest

The Fraternal
Ateneo de Davao

TAU MU
TAU MU

Determination of Just Compensation


The principal criterion in determining just
compensation is the character of the land at the
time of the taking. The tax declaration is only one
of the factors to be used in determining the
market value of the property for purposes of
arriving at the amount to be paid by way of just
compensation
[Nachura, Outline Reviewer in
Political Law 2006, p. 55]

KITY

TAU MU TAU MU TAU MU


TAU MU TAU MU TAU MU

CONSTITUTIONAL LAW II
Order of Saint Thomas More
Atty. Philip John Pojas/Atty. Rovyne G. Jumao-as, RN
University College of Law

However, in Republic vs. Vicente Lim, the


Supreme Court said that the facts of the case do
not justify the application of the rule. It was held
that where the government failed to pay just
compensation within five (5) years from the
finality of the judgment in the expropriation
proceedings, the owners concerned shall have
the right to recover possession of their property
[Nachura, Outline Reviewer in Political Law 2006,
p.57].
Writ of Possession
The issuance of the writ of possession becomes
ministerial upon:
1. Filing of a complaint for expropriation
sufficient in form and substance;
2. Upon deposit made by the government of
the amount equivalent to fifteen percent (15%) of
the fair market value of the property sought to be
expropriated
per
current
tax
declaration
[Nachura, Outline Reviewer in Political Law 2006,
p. 58].
Plaintifs right to dismiss the complaint in
eminent domain
In expropriation cases, there is no such thing as
the plaintifs matter-of-right to dismiss the
complaint, precisely because the landowner may
have already sufered damages at the start of the
taking. The plaintifs right to dismiss the
complaint has always been subject to Court
approval and to certain conditions [Nachura,
Outline Reviewer in Political Law 2006, p. 58].
Right to repurchase or re-acquire the
property
The property owners right to repurchase the
property depends upon the character of the title
acquired by the expropriator, e.g. if land is
expropriated for a particular purpose with the
condition that when that purpose is ended or
abandoned, the property shall revert to the
former owner, then the former owner can
reacquire the property
[Nachura, Outline
Reviewer in Political Law 2006, p. 58].
Temporary takeover of private business or
utility
Temporary takeover by the government extends
only to the operation of the business and not to
the ownership thereof. As such, the government
is not required to compensate the private entityowner of the said business as there is not transfer
of ownership, whether permanent or temporary.
The private entity-owner afected cannot,
likewise, claim just compensation for the use of
said business and its properties, as the temporary
takeover by the government is in the exercise of
the police power and not the power of eminent
domain [Nachura, Outline Reviewer in Political
Law 2006, p. 59].

TAU MU

RA7279 (Urban Development and Housing


Act of 1992)
Under RA7279, lands for socialized housing are to
be acquired in the following order:
1. Government lands
2. Alienable lands of the public domain

ACADCOM 2010; Contributors: Gene Geocaniga, Jarissa Guiani, Darlene Magabilen


TAU MU Page 115 of 179

3. Unregistered, abandoned, or idle lands


4. Lands within the declared Areas for
Priority Development, Zonal Improvement
Program sites, Slum Improvement and
Resettlement sites which have not yet
been acquired
5. BLISS site which have not yet been
acquired
6. Privately owned lands

TAU MU
TAU MU
TAU MU
TAU MU
TAU MU
TAU MU
TAU MU

While the principle of res judicata does not


denigrate the right of the State to exercise
eminent domain, it does not apply to specific
issues decided in a previous case. For example, a
final judgment dismissing an expropriation suit on
the ground that there was not prior ofer
precluded another suit raising the same issue; it
cannot, however, bar the State or its agent from
thereafter complying with this requirement, as
prescribed by law, and subsequently exercising
its power of eminent domain over the same

TAU MU

Res Judicata
The principle of res judicata does not bar the
right of the State or its agent to expropriate
private property. The very nature of the eminent
domain, as an inherent power of the State,
dictates that the right to exercise the power be
absolute and unfettered by a prior judgment or
res judicata. The scope of eminent domain is
plenary and, like police power, can reach every
form of property which the State might need for
public use.

TAU MU

Instance where exercise of the power of


eminent domain is not subject to judicial
review
When land is expropriated for subdivision and
resale for social justice purposes directly by the
legislature and not through an inferior agency of
the state, the necessity and public purpose of the
taking are not subject to judicial review. Article
XIII, Section 4, constitutes a textual commitment
of discretion on the subject to the legislature.
However, if the power is exercised by an agency
possessing power delegated by the legislature or
when the issue is the adequacy of the
compensation, judicial review is still proper.

TAU MU
TAU MU

The exercise of the power of eminent domain is


subject to judicial review. The following aspects of
the exercise of the power have been subject to
judicial scrutiny:
1. The adequacy of the compensation
2. The necessity of the taking
3. The public use character of the purpose
of the taking

The Fraternal
Ateneo de Davao

TAU MU
TAU MU

The mode of expropriation is subject to two


conditions, namely:
1. It shall be resorted to only when the other
modes of acquisition have been exhausted
2. Parcels owned by small property owners
are exempt from such acquisition
[Nachura, Outline Reviewer in Political Law 2006,
p. 60]
Judicial Review [Bernas, The 1987 Philippine
Constitution: A Comprehensive Reviewer, p.112]

KITY

TAU MU TAU MU TAU MU


TAU MU TAU MU TAU MU

CONSTITUTIONAL LAW II
Order of Saint Thomas More
Atty. Philip John Pojas/Atty. Rovyne G. Jumao-as, RN
University College of Law

property. (Municipality of Paranaque vs. VM


Realty Corporation)
Expropriation
as
substitute
for
the
enforcement of a valid contract
Expropriation lies only when it is made necessary
by the opposition of the owner to the sale or by
the lack of any agreement as to price. Where
there is a valid and subsisting contract, between
the owners of the property and the expropriating
authority, there is no reason for the expropriation.
The State may not enter into a contract which in
efect binds it not to exercise the power of
eminent domain. Like police power and the power
of taxation, the power of eminent domain is
inalienable. There can be no right to restrain by
contract the power of eminent domain, it must
also of necessity to follow that any contract by
which it was sought to accomplish that result
would be inefficacious for want of power.
Exercise of Eminent Domain by Local
Government Unit
By express legislative authority granted by
Congress
in
Section
19,
RA7160,
local
government units may expropriate private
property for public use, or purpose, or welfare, for
the benefit of the poor and the landless. Thus, in
Moday vs. CA, the Supreme Court held that the
Sangguniang Panlalawigan of Agusan del Sur was
without authority to disapprove Bunawan
Municipal Reso 43-89 because clearly, the
Municipality of Bunawan has the authority to
exercise the power of eminent domain and its
Sangguniang Bayan the capacity to promulgate
the assailed resolution
[Nachura, Outline
Reviewer in Political Law 2006, p. 52].
Ordinance (Requisite for Valid Expropriation)
In Municipality of Paranaque vs. VM Realty
Corporation, the Supreme Court declared that
there was lack of compliance with Sec. 19,
RA7160, where the Municipal Mayor filed a
complaint for eminent domain over two parcels of
land on the strength of a resolution passed by the
Sangguniang Bayan, because what is required by
law is an ordinance.
Essential requisites for LGU to validly exercise the
power of eminent domain (Section 19, RA7160):
e. An ordinance is enacted by the local
legislative council authorizing the local
chief executive, in behalf of the LGU, to
exercise the power of eminent domain or
pursue expropriation proceedings over a
particular private property;
f. The power of eminent domain is exercised
for public use, purpose or welfare, or for
the benefit of the poor and the landless;
g. There is payment of just compensation as
required under Sec. 9, Article III of the
Constitution, and other pertinent laws;
h. A valid and definite ofer has been
previously made to the owner of the
property sought to be expropriated, but
said ofer was not accepted.

ACADCOM 2010; Contributors: Gene Geocaniga, Jarissa Guiani, Darlene Magabilen


TAU MU Page 116 of 179

Limitations on power of eminent domain of LGU:


(Estate of JBL Reyes vs. City of Manila)
Since local governments possess only delegated
power of eminent domain, it is subject to
limitations specified by law on the delegated
power. Thus there are mandatory limits with
respect to:
3. The order of priority in acquiring land for
socialized housing; and
4. The resort to expropriation proceedings as
a means to acquiring it.

TAU MU
TAU MU
TAU MU
TAU MU
TAU MU
TAU MU
TAU MU

LIMITATIONS TO THE NON-IMPAIRMENT


CLAUSE
1. If the law is a valid exercise of police power it
will prevail over the contract.
Into each contract are read the
provisions of existing law, and always a
reservation of the police power as long as the
agreement deals w/ a matter afecting the public
welfare.
The legislature cannot bargain away the
police power through the medium of contract.
Neither may the private parties fetter the
legislative authority by contracting on matters
that are essentially within the power of the
lawmaking body to regulate.

TAU MU

Q: Is the protection provided in this provision


absolute?
A: NO. There are instances when contracts valid
@ the time of their conclusion may become
invalid, or some of their provisions may be
rendered inoperative or illegal, by virtue of
supervening legislation.

TAU MU

NON-IMPAIRMENT CLAUSE
The PURPOSE of this clause is to safeguard
the integrity of valid contractual agreements
against the warranted interference by the
state.
Contracts should be respected by the
legislature and not tampered w/ by
subsequent laws that will
1. change the intention of the parties or
2. will modify the parties rights and
obligations.
The will of the obligor and the obligee must
be observed.
This right is a limitation to Congress power to
enact laws.

TAU MU
TAU MU

Section 10. No law impairing the obligation


of contracts shall be passed.

The Fraternal
Ateneo de Davao

TAU MU
TAU MU

Private lands rank last in the order of priority for


purposes of socialized housing. In the same vein,
expropriation proceedings may not be resorted to
only after the other modes of acquisition are
exhausted. Compliance with these conditions are
mandatory because these are the only
safeguards of often time helpless owners of
private property against what may be tyrannical
violation of due process when their property is
forcibly taken from them allegedly for public use.

KITY

TAU MU TAU MU TAU MU


TAU MU TAU MU TAU MU

CONSTITUTIONAL LAW II
Order of Saint Thomas More
Atty. Philip John Pojas/Atty. Rovyne G. Jumao-as, RN
University College of Law

2. The
freedom
of
contract
w/c
is
constitutionally protected is freedom to enter into
LAWFUL contracts.
Contracts that contravene public policy
are not lawful.
3. The inherent powers of taxation and
imminent domain may also limit the nonimpairment clause.
A lawful tax on a new subject or an
increased tax on an old one does not interfere w/
a contract or impair its obligation w/i the meaning
of the constitution. Even though such taxation
may afect particular contracts, as it may
increase the debt of one person and lessen the
security of another, or may impose additional
burdens upon one class and release the burdens
of another, still the tax must be paid unless
prohibited by the constitution, nor can it be said
that it impairs the obligation of any contract in its
true legal sense.
Where a law grants a tax exemption in
exchange for a valuable consideration, such
exemption is considered a contract and cannot be
repealed because of this section [CASSANOVA vs.
HORD].
All other tax exemptions are not
contractual and so may be revokes @ will by the
legislature.
4. Freedom of Speech. Expression and of the
Press (Sec. 4)
5. Right to Assemble and Petition (Sec. 4).
6. Freedom of Religion (Sec. 5).
CONTRACT
any lawful agreement on property or property
rights, whether real or personal, tangible or
intangible
the agreement may be executed or executory
the PARTIES may be private persons only,
natural or artificial, or private persons and the
govt. or its agencies.
It does not cover licenses
NOT COVERED BY SEC. 10
1. licenses because they only involve grants
of privileges that are essentially revocable
2. marriage contract because marriage is
regarded as more than a an agreement
between the spouses as it is a social
institution subject @ all times to regulation
by the legislature and to change of the
original conditions
3. public office
EXCEPTION: where the salary has already
been earned, it will be deemed vested property
right that cannot be withdrawn or reduced by
retroactive legislation
LAW
includes statutes enacted by the national
legislature, executive orders & administrative
regulations promulgated under a valid power, and
municipal ordinances passes by local legislative
bodies

ACADCOM 2010; Contributors: Gene Geocaniga, Jarissa Guiani, Darlene Magabilen


TAU MU Page 117 of 179

it does NOT include judicial decisions or


adjudication made by administrative bodies in
their QJ functions
TO IMPAIR it (law) must retroact so as to afect
existing contracts concluded before its enactment
NO IMPAIRMENT if it (law) is made to operate
prospectively only --- to cover contracts entered
into after its enactment

TAU MU
TAU MU
TAU MU
TAU MU
TAU MU

Coverage: Any lawful contract agreement; any


contract which involves property, i.e. intellectual
property, personal, movable, etc.; franchises or

TAU MU

2. It can be invoked against a law, an executive


act, an ordinance, but not against executive
officials performing judicial actions

TAU MU

Note:
1. A mere change in PROCEDURAL REMEDIES
which does not change the substance of the
contract, and which still leaves an efficacious
remedy for enforcement does NOT impair the
obligation of contracts.

TAU MU

BUT in the case of REMEDIES, there will be


impairment only when all of them are withdrawn,
w/ the result that either of the parties will be
unable to enforce his rights under the original
agreement.
There is NO IMPAIRMENT as long as
substantial and efficacious remedy remains. And
this rule holds true even if the remedy retained is
the most difficult to employ and it is the easier
ones that are withdrawn [MANILA TRADING CO.
vs. REYES (62 Phil. 461)].

TAU MU

Q: When is there impairment?


A:
1. When the law changes the terms of the
contract;
2. When the law imposes new obligations
or conditions;
3. When the law dispenses w/ those agreed
upon; or
4. When the law withdraws the remedies
for the enforcement of the rights of the
parties.

TAU MU
TAU MU

IMPAIRMENT
is anything that diminishes the efficacy of the
contract
the degree of diminution is immaterial as long
as the rights of either party are changed to his
prejudice there is impairment of the obligation of
the contract.
To impair, the law must retroact so as to afect
existing
contracts
concluded
before
its
enactment. There will be no impairment if the law
is made to operate prospectively only, to cover
contracts entered into after its enactment.

The Fraternal
Ateneo de Davao

TAU MU
TAU MU

OBLIGATION
is the vinculum juris --- the tie that binds the
parties to each other
it refers to the law/duty that binds the parties
to
perform
their
undertaking/agreement
according to its terms and intent

KITY

TAU MU TAU MU TAU MU


TAU MU TAU MU TAU MU

CONSTITUTIONAL LAW II
Order of Saint Thomas More
Atty. Philip John Pojas/Atty. Rovyne G. Jumao-as, RN
University College of Law

charters granted to a private person; timber


licenses.
What can be impaired: Licenses issued by the
Government, licenses for exploiting mining,
marbles; marriage contracts; public office
The following are superior to the non
impairment clause:
1. Police Power; (reclassification of property)
2. Eminent domain; (lessor for lease but
expropriated)
3. Taxation power
4. Freedom of religion
*The right to non-impairment of contracts cannot
be invoked against judicial acts.

CASES: NON-IMPAIRMENT OF CONTRACTS


UNITED BF HOMEOWNERS vs. CITY MAYOR
515 SCRA 1 (2007)
Facts: BF Homes Paraaque Subdivision is the
largest subdivision in the country. In 1997, the
Municipal Council of Paraaque enacted a
Municipal Ordinance entitled, "An Ordinance
Prescribing the Comprehensive Land Use Plan &
Zoning of the Municipality of Paraaque Pursuant
to the Local Government Code of 1991 and Other
Pertinent Laws." Sections 11.5 and 11.6 of the
Ordinance seeks to reclassify El Grande and
Aguirre Avenues in BF Homes Paraaque from
residential to commercial areas.
Petitioners filed a petition for prohibition
with an application for TRO and preliminary
injunction. They questioned the constitutionality
of certain sections of the municipal ordinance and
alleged that the reclassification of certain
portions of BF Homes Paraaque from residential
to commercial zone is unconstitutional because it
amounts to impairment of the contracts between
the developer of BF Homes Paraaque and the lot
buyers. Petitioners cited the annotation on the lot
buyers titles which provides that "the property
shall be used for residential purposes only and for
no other purpose."
On the other hand, public respondents
alleged that the passage of Municipal Ordinance
is a valid exercise of police power by the
Municipal Council of Paraaque and that such
ordinance
can
nullify
or
supersede
the
contractual obligations entered into by the
petitioners and the developer. Meanwhile, EL
ACTO claimed that its members are lot owners,
residents,
and
operators
of
commercial
establishments along El Grande and Aguirre
Avenues in BF Homes Paraaque, who will be
afected if Municipal Ordinance is declared
unconstitutional. EL ACTO asserted that Municipal
Ordinance is a valid exercise of police power and
that petitioners are guilty of estoppel since
petitioners endorsed the opening of many of
these commercial establishments in BF Homes
Paraaque. EL ACTO further alleged that the
instant petition should have been initially filed

ACADCOM 2010; Contributors: Gene Geocaniga, Jarissa Guiani, Darlene Magabilen


TAU MU Page 118 of 179

with the Regional Trial Court in accordance with


the principle of hierarchy of courts.
Issue: WON the ordinance impaired the
contractual obligation annotated in homeowners
titles.

TAU MU
TAU MU
TAU MU
TAU MU
TAU MU
TAU MU
TAU MU
TAU MU
TAU MU

Facts: In June 1987, Harrison Motors Corporation


sold 2 Isuzu Elf trucks to private respondent
Rachel Navarro. Petitioner, a known importer,
assembler and manufacturer, assembled the 2
trucks using imported component parts. Prior to
the sale, Claros represented to Navarro that all
the BIR taxes and customs duties for the parts
used on the 2 trucks had been paid for. In
September 1987, BIR and the LTO entered into a
MOA which provided that prior to registration in
the LTO of any assembled or re-assembled motor
vehicle which used imported parts, a Certificate
of Payment should first be obtained from the BIR
to prove payment of all taxes required under
existing laws. The Bureau of Customs issued
Customs
Memorandum
Order
No.
44-87
promulgating rules, regulations and procedure for
the voluntary payment of duties and taxes on

TAU MU
TAU MU

HARRISON vs. NAVARRO


331 SCRA 202 (2000)

The Fraternal
Ateneo de Davao

TAU MU
TAU MU

Ruling: The Court has upheld in several cases


the superiority of police power over the nonimpairment clause. The constitutional guaranty of
non-impairment of contracts is limited by the
exercise of the police power of the State, in the
interest of public health, safety, morals and
general welfare. Contractual restrictions on the
use of property could not prevail over the
reasonable exercise of police power through
zoning regulations. While non-impairment of
contracts is constitutionally guaranteed, the rule
is not absolute, since it has to be reconciled with
the legitimate exercise of police power, i.e., "the
power to prescribe regulations to promote the
health, morals, peace, education, good order or
safety and general welfare of the people."
Invariably described as "the most essential,
insistent, and illimitable of powers" and "in a
sense, the greatest and most powerful attribute
of government," the exercise of the power may
be judicially inquired into and corrected only if it
is capricious, whimsical, unjust or unreasonable,
there having been a denial of due process or a
violation of any other applicable constitutional
guarantee. The State guarantees sanctity of
contract and is said to be the "law between the
contracting parties," but while it is so, it cannot
contravene "law, morals, good customs, public
order, or public policy." Above all, it cannot be
raised as a deterrent to police power, designed
precisely to promote health, safety, peace, and
enhance the common good, at the expense of
contractual rights, whenever necessary.
Hence, the ordinance, as held by the SC, is
a legitimate exercise of police power and the
reclassification of El Grande and Aguirre Avenues
in BF Homes Paraaque is not arbitrary or
unreasonable.

KITY

TAU MU TAU MU TAU MU


TAU MU TAU MU TAU MU

CONSTITUTIONAL LAW II
Order of Saint Thomas More
Atty. Philip John Pojas/Atty. Rovyne G. Jumao-as, RN
University College of Law

imported motor vehicles assembled by nonassemblers.Pursuant to the MOA between the BIR
and the LTO, the BIR issued Revenue
Memorandum Order No. 44-87 which provided
the procedure governing the processing and
issuance of the Certificate of Payment of internal
revenue taxes for purposes of registering motor
vehicles.
In 1988, the BIR, BOC and LTO entered
into a tripartite MOA which provided that prior to
the registration in the LTO of any locally
assembled motor vehicle using imported
component parts, a Certificate of Payment should
first be obtained from the BIR and the BOC to
prove that all existing taxes and customs duties
have been paid. On that same year, government
agents seized and detained the 2 Elf trucks of
respondent after discovering that there were still
unpaid BIR taxes and customs duties thereon.
The BIR and the BOC ordered Navarro to pay the
proper assessments or her trucks would be
impounded. She went to Claros to ask for the
receipts evidencing payment of BIR taxes and
customs duties; however, Claros refused to
comply. She then demanded from Claros that he
pay the assessed taxes and warned him that he
would have to reimburse her should she be forced
to pay for the assessments herself. Her demands
were again ignored.But wanting to secure the
immediate release of the trucks to comply with
her business commitments, Navarro paid the
assessed BIR taxes and customs duties.
Consequently, she returned to petitioner's office
to ask for reimbursement, but petitioner again
refused.
Issue: WON the payment of the taxes specified
in the administrative regulations violated the
constitutional mandate of non-impairment clause.
Ruling: The SC ruled that the Memorandum
Orders and the MOA did not impose any
additional BIR taxes or customs duties. What Sec.
10, Art. III, of the Constitution prohibits is the
passage of a law which enlarges, abridges or in
any manner changes the intention of the
contracting parties. The Memorandum Orders and
the 2 Memoranda of Agreement do not impose
any additional taxes which would unduly impair
the contract of sale between petitioner and
private respondent. Instead, these administrative
regulations were passed to enforce payment of
existing BIR taxes and customs duties at the time
of importation. Administrative rulings and
regulations are generally prospective in nature.
An inspection of the 2 Memoranda of Agreement
however demonstrates that their intent is to
enforce
payment
of
taxes
on
assemblers/manufacturers
who
import
component parts without paying the correct
assessments. Thus, although Navarro is the one
required by the administrative regulations to
secure the Certificate of Payment for the purpose
of registration, petitioner as the importer and the
assembler/manufacturer of the two 2 Elf trucks is
still the one liable for payment of revenue taxes
and customs duties. Petitioner's obligation to pay
does not arise from the administrative regulations

ACADCOM 2010; Contributors: Gene Geocaniga, Jarissa Guiani, Darlene Magabilen


TAU MU Page 119 of 179

TAU MU
TAU MU
TAU MU
TAU MU
TAU MU
TAU MU
TAU MU
TAU MU
TAU MU

Facts: The instant case involves a rich tract of


mineral land known as the "Diwalwal Gold Rush
Area." On March 10, 1988, Marcopper Mining
Corporation was granted Exploration Permit No.
133 which included the hotly-contested Diwalwal
area. Marcopper's acquisition of mining rights
over Diwalwal under its EP No. 133 was
subsequently challenged where Marcopper's
claim was sustained over that of another mining
firm, Apex Mining Corporation. In 1991, Republic
Act 7076 or the People's Small-Scale Mining Act
was enacted. The law established a People's
Small-Scale Mining Program to be implemented
by the Secretary of the DENR and created the
Provincial Mining Regulatory Board under the
DENR Secretary's direct supervision and control.
The statute also authorized the PMRB to declare
and set aside small-scale mining areas subject to
review by the DENR Secretary 5 and award mining
contracts to small-scale miners under certain
conditions. In 1991, DENR Sec. Factoran issued
Department Administrative Order No. 66,
declaring 729 hectares of the Diwalwal area as
non-forest land open to small-scale mining.
Subsequently, a petition for the cancellation of EP
No. 133 and the admission of a Mineral
Production Sharing Arrangement (MPSA) proposal
over Diwalwal was filed before the DENR Regional
Executive Director. While the RED Mines case was
pending, Marcopper assigned its EP No. 133 to
petitioner Southeast Mindanao Gold Mining
Corporation (SEM), which in turn applied for an
integrated MPSA over the land covered by the
permit. The Mines and Geosciences Bureau
accepted and registered the integrated MPSA
application of petitioner. In the meantime,
Republic Act 7942, the Philippine Mining Act, was
enacted. Provincial Mining Regulatory Board of
Davao passed Resolution No. 26 authorizing the
issuance of ore transport permits to small-scale
miners operating in the Diwalwal mines. Thus,
petitioner filed a complaint for damages against
the DENR Secretary and PMRB-Davao. SEM
alleged that the illegal issuance of the OTPs

TAU MU
TAU MU

SMGMC vs. BALITE PORTAL


380 SCRA 145 (2002)

The Fraternal
Ateneo de Davao

TAU MU
TAU MU

but from the tax laws existing at the time of


importation. Hence, even if private respondent
already owned the 2 trucks when the
Memorandum
Orders
and
Memoranda
of
Agreement took efect, the fact remains that
petitioner was still the one duty-bound to pay for
the BIR taxes and customs duties.
As between petitioner, who is the
importer-assembler/manufacturer, and Navarro,
who is merely the buyer, it is petitioner which has
the obligation to pay taxes to the BIR and the
BOC. Petitioner would be unjustly enriched if
Navarro should be denied reimbursement. It
would inequitably amass profits from selling
assembled trucks even if it did not pay the taxes
due on its imported spare parts. Imposing the tax
burden on private respondent would only
encourage the proliferation of smugglers who
scheme to evade taxes by passing on their tax
obligations to their unsuspecting buyers.

KITY

TAU MU TAU MU TAU MU


TAU MU TAU MU TAU MU

CONSTITUTIONAL LAW II
Order of Saint Thomas More
Atty. Philip John Pojas/Atty. Rovyne G. Jumao-as, RN
University College of Law

allowed the extraction and hauling of P60,000.00


worth of gold ore per truckload from SEM's mining
claim.
Meanwhile, the DENR Secretary issued
Memorandum Order No. 97-03. With this,
petitioner filed a special civil action for certiorari,
prohibition and mandamus against PMRB-Davao,
the DENR Secretary and Balite Communal Portal
Mining Cooperative (BCPMC), which represented
all the OTP grantees. It prayed for the nullification
of the Memorandum Order No. 97-03 on the
ground that the "direct state utilization" espoused
therein would efectively impair its vested rights
under EP No. 133; that the DENR Secretary
unduly usurped and interfered with the
jurisdiction of the RPA which had dismissed all
adverse claims against SEM in the Consolidated
Mines cases; and that the memorandum order
arbitrarily imposed the unwarranted condition
that certain studies be conducted before mining
and environmental laws are enforced by the
DENR.
Issue:
WON
the
non-impairment
clause
mandated in the Constitution was violated.
Ruling: Under no circumstances may petitioner's
rights under EP No. 133 be regarded as total and
absolute. EP No. 133 merely evidences a privilege
granted by the State, which may be amended,
modified or rescinded when the national interest
so requires. This is necessarily so since the
exploration, development and utilization of the
country's natural mineral resources are matters
impressed with great public interest. Like timber
permits, mining exploration permits do not vest in
the grantee any permanent or irrevocable right
within the purview of the non-impairment of
contract and due process clauses of the
Constitution, since the State, under its allencompassing police power, may alter, modify or
amend the same, in accordance with the
demands of the general welfare.
Thus, the State may pursue the
constitutional
policy
of
full
control
and
supervision of the exploration, development and
utilization of the country's natural mineral
resources, by either directly undertaking the
same or by entering into agreements with
qualified entities. The DENR Secretary acted
within his authority when he ordered a study of
the first option, which may be undertaken
consistently in accordance with the constitutional
policy enunciated above. Obviously, the State
may not be precluded from considering a direct
takeover of the mines, if it is the only plausible
remedy in sight to the gnawing complexities
generated by the gold rush. The State need be
guided only by the demands of public interest in
settling for this option, as well as its material and
logistic feasibility.
PICOP RESOURCES vs. BASE METALS
510 SCRA 400 (2006)
Facts: In 1987, the Central Mindanao Mining and
Development Corporation entered into a Mines
Operating Agreement with Banahaw Mining and

ACADCOM 2010; Contributors: Gene Geocaniga, Jarissa Guiani, Darlene Magabilen


TAU MU Page 120 of 179

TAU MU
TAU MU
TAU MU
TAU MU
TAU MU
TAU MU
TAU MU
TAU MU
TAU MU

Ruling: An examination of the Presidential


Warranty at once reveals that it simply reassures
PICOP of the government's commitment to
uphold the terms and conditions of its timber
license and guarantees PICOP's peaceful and
adequate possession and enjoyment of the areas
which are the basic sources of raw materials for
its wood processing complex. The warranty
covers only the right to cut, collect, and remove
timber in its concession area, and does not
extend to the utilization of other resources, such
as mineral resources, occurring within the
concession. The Presidential Warranty cannot be
considered a contract distinct from PTLA No. 47
and IFMA No. 35. The SC agreed with the OSG's
position that it is merely a collateral undertaking

TAU MU
TAU MU

Issue: WON the approval of the application and


the issuance of the MPSA of Base Metals will
violate the constitutional mandate against
impairment of obligation in a contract.

The Fraternal
Ateneo de Davao

TAU MU
TAU MU

Development Corporation whereby the latter


agreed to act as Mine Operator for the
exploration,
development,
and
eventual
commercial operation of CMMCI's 18 mining
claims. Pursuant to the terms of the Agreement,
Banahaw Mining filed applications for Mining
Lease Contracts over the mining claims with the
Bureau of Mines. Banahaw Mining was issued a
Mines Temporary Permit authorizing it to extract
and dispose of precious minerals found within its
mining claims. Upon its expiration, the temporary
permit was subsequently renewed thrice by the
Bureau of Mines. Since a portion of Banahaw
Mining's mining claims was located in petitioner
PICOP's logging concession, Banahaw Mining and
petitioner PICOP entered into a Memorandum of
Agreement, whereby, in mutual recognition of
each other's right to the area concerned,
petitioner PICOP allowed Banahaw Mining an
access/right of way to its mining claims.
In 1991, Banahaw Mining converted its
mining claims to applications for Mineral
Production Sharing Agreements. While the MPSA
were pending, Banahaw Mining decided to
sell/assign its rights and interests over 37 mining
claims in favor of private respondent Base Metals
Mineral Resources Corporation. The transfer
included mining claims held by Banahaw Mining
in its own right as claim owner, as well as those
covered by its mining operating agreement with
CMMCI. Upon being informed of the development,
CMMCI, as claim owner, immediately approved
the assignment made by Banahaw Mining in favor
of private respondent Base Metals, thereby
recognizing private respondent Base Metals as
the new operator of its claims. Pivate respondent
Base Metals amended Banahaw Mining's pending
MPSA applications with the Bureau of Mines to
substitute itself as applicant and to submit
additional documents in support of the
application. Area clearances were submitted, as
required. Thereafter, Base Metals amended MPSA
applications were published. Hence, petitioner
PICOP filed with the Mines Geo-Sciences Bureau
an Adverse Claim and/or Opposition to private
respondent Base Metals' application.

KITY

TAU MU TAU MU TAU MU


TAU MU TAU MU TAU MU

CONSTITUTIONAL LAW II
Order of Saint Thomas More
Atty. Philip John Pojas/Atty. Rovyne G. Jumao-as, RN
University College of Law

which cannot amplify PICOP's rights under its


timber license. A timber license is not a contract
within the purview of the non-impairment clause
is edifying. A timber license is not a contract
within the purview of the due process clause; it is
only a license or a privilege, which can be validly
withdrawn whenever dictated by public interest
or public welfare. A license is merely a permit or
privilege to do what otherwise would be unlawful,
and is not a contract between the authority,
federal, state, or municipal, granting it and the
person to whom it is granted; neither is it a
property or a property right, nor does it create a
vested right; nor is it taxation. Thus, he granting
of license does not create irrevocable rights,
neither is it property or property rights.
Timber licenses, permits and license
agreements are the principal instruments by
which the State regulates the utilization and
disposition of forest resources to the end that
public welfare is promoted. And it can hardly be
gainsaid that they merely evidence a privilege
granted by the State to qualified entities, and do
not vest in the latter a permanent or irrevocable
right to the particular concession area and the
forest products therein. They may be validly
amended, modified, replaced or rescinded by the
Chief Executive when national interests so
require. Thus, they are not deemed contracts
within the purview of the due process of law
clause. Since timber licenses are not contracts,
the non-impairment clause, cannot be invoked.
REPUBLIC vs. RMDC
426 SCRA 517 (2004)
Facts: The four petitioners, after having been
granted permission to prospect for marble
deposits in the mountains of Biak-na-Bato, San
Miguel, Bulacan, succeeded in discovering marble
deposits of high quality and in commercial
quantities in Mount Mabio which forms part of the
Biak-na-Bato mountain range. Having succeeded
in discovering said marble deposits, and as a
result of their tedious eforts and substantial
expenses, the petitioners applied with the Bureau
of Mines, now Mines and Geosciences Bureau, for
the issuance of the corresponding license to
exploit said marble deposits. Thereafter, License
No. 33 was issued by the Bureau of Mines in favor
of
the
herein
petitioners.
Shortly
after
Respondent Maceda was appointed Minister of
the DENR, petitioners License No. 33 was
cancelled by him, by virtue of Proclamation No.
84, through his letter to ROSEMOOR MINING AND
DEVELOPMENT CORPORATION for the reasons
stated therein.
Issue: WON Proclamation No. 84 which confirmed
the cancellation of the license of the petitioners
violated the non-impairment clause.
Ruling: Proclamation No. 84 cannot be
stigmatized as a violation of the non-impairment
clause. As pointed out earlier, respondents
license is not a contract to which the protection
accorded by the non-impairment clause may
extend. Even if the license were, it is settled that

ACADCOM 2010; Contributors: Gene Geocaniga, Jarissa Guiani, Darlene Magabilen


TAU MU Page 121 of 179

provisions of existing laws and a reservation of


police power are deemed read into it, because it
concerns a subject impressed with public welfare.
As it is, the non-impairment clause must yield to
the police power of the state. Thus, a mining
license that contravenes a mandatory provision
of the law under which it is granted is void. Being
a mere privilege, a license does not vest absolute
rights in the holder. Thus, without ofending the
due process and the non-impairment clauses of
the Constitution, it can be revoked by the State in
the public interest.
BELTRAN vs. SECRETARY
476 SCRA 168 (2005)

TAU MU
TAU MU
TAU MU
TAU MU
TAU MU
TAU MU
TAU MU
TAU MU
TAU MU
TAU MU

Sale of Human Blood, and the Establishment and


Operation of Blood Banks and Blood Processing
Laboratories. The law, which was enacted in
1956, allowed the establishment and operation
by licensed physicians of blood banks and blood
processing laboratories.
Petitioners assail the constitutionality of the
questioned legal provisions, namely, Section 7 of
RA 7719, which provides: Section 7.
Phaseout of Commercial Blood Banks - All
commercial blood banks shall be phased-out over
a period of two (2) years after the efectivity of
this Act, extendable to a maximum period of two
(2) years by the Secretary. and Section 23 of
Administrative Order No. 9, which provides that:
Section 23. Process of Phasing Out. -- The
Department shall efect the phasing-out of all
commercial blood banks over a period of two (2)
years, extendible for a maximum period of two
(2) years after the efectivity of R.A. 7719. The
decision to extend shall be based on the result of
a careful study and review of the blood supply
and demand and public safety.

The Fraternal
Ateneo de Davao

TAU MU
TAU MU

Facts: RA 7719 or the National Blood Services


Act of 1994 seeks to provide an adequate supply
of safe blood by promoting voluntary blood
donation and by regulating blood banks in the
country. It was approved by then President Ramos
on May 15, 1994 and it law took efect on August
23, 1994. Blood banking and blood transfusion
services in the country have been arranged in 4
categories: blood centers run by the PNRC,
government-run blood services, private hospital
blood banks, and commercial blood services.
Years prior to the passage of the RA 7719,
petitioners
have
already
been
operating
commercial blood banks under RA 1517, entitled
An Act Regulating the Collection, Processing and
PAUPER
INDIGENT

need
not
be persons who have no
persons so poor that property or sources of
they
must
be income
sufficient
for
supported at public their support aside from
expense
their own labor though

those protected self-supporting


when
include
lowly-paid able to work and in
employees, domes- employment
tic
servants
and
laborers

KITY

TAU MU TAU MU TAU MU


TAU MU TAU MU TAU MU

CONSTITUTIONAL LAW II
Order of Saint Thomas More
Atty. Philip John Pojas/Atty. Rovyne G. Jumao-as, RN
University College of Law

Issue: WON Section 7 of R.A. 7719 and its


implementing rules and regulations violated the
non-impairment clause.
Ruling: The National Blood Services Act is a valid
exercise of the States police power. Therefore,
the Legislature, under the circumstances,
adopted a course of action that is both necessary
and reasonable for the common good. Police
power is the State authority to enact legislation
that may interfere with personal liberty or
property in order to promote the general welfare.
The State, in order to promote the general
welfare, may interfere with personal liberty, with
property, and with business and occupations.
Thus, persons may be subjected to certain kinds
of restraints and burdens in order to secure the
general welfare of the State and to this
fundamental aim of government, the rights of the
individual may be subordinated. hat the nonimpairment clause of the Constitution must yield
to the loftier purposes targeted by the
government. The right granted by this provision
must submit to the demands and necessities of
the States power of regulation. While the Court
understands the grave implications of Section 7
of the law in question, the concern of the
Government in this case, however, is not
necessarily to maintain profits of business firms.
In the ordinary sequence of events, it is profits
that sufer as a result of government regulation.
Furthermore, the freedom to contract is
not absolute; all contracts and all rights are
subject to the police power of the State and not
only may regulations which afect them be
established by the State, but all such regulations
must be subject to change from time to time, as
the general well-being of the community may
require, or as the circumstances may change, or
as experience may demonstrate the necessity. As
found by the SC, individual rights to contract and
to property have to give way to police power
exercised for public welfare.
BPI vs. SEC
541 SCRA 294 (2007)

TAU MU

Facts: BPI, through its predecessor-in- interest,


Far East
Bank
and
Trust
Company,
extended credit accommodations to the ASB
Group secured by a real estate mortgage over 2
properties. The ASB Group filed a petition for
rehabilitation and suspension of payments before
the SEC. Thereafter, the interim receiver
submitted its Proposed Rehabilitation Plan for the
ASB Group. The Rehabilitation Plan provides,
among others, a dacion en pago by the ASB
Group to BPI of one of the properties mortgaged
to the latter at the ASB Group against the total
amount of the ASB Groups exposure to the
bank. In turn, ASB Group would require the
release of the other property mortgaged to BPI, to
be thereafter placed in the asset pool. The dacion
would constitute full payment of the entire
obligation due to BPI because the balance was
then to be considered waived, as per the
Rehabilitation Plan.BPI opposed the Rehabilitation

ACADCOM 2010; Contributors: Gene Geocaniga, Jarissa Guiani, Darlene Magabilen


TAU MU Page 122 of 179

Plan and moved for the dismissal of the ASB


Groups petition for rehabilitation. However, the
SEC hearing panel issued an order approving ASB
Groups
proposed
rehabilitation
plan
and
appointed Mr. Fortunato Cruz as rehabilitation
receiver. BPI filed a petition for review before the
SEC en banc, imputing grave abuse of discretion
on the part of the hearing panel. It argued that
the Order constituted an arbitrary violation of
BPIs freedom and right to contract since the
Rehabilitation Plan compelled BPI to enter into a
dacion en pago agreement with the ASB Group.
The SEC en banc denied the petition.

TAU MU
TAU MU
TAU MU
TAU MU
TAU MU
TAU MU

Ruling: SECs approval of the Rehabilitation Plan


did not impair BPIs right to contract. As correctly
contended by private respondents, the nonimpairment clause is a limit on the exercise of
legislative power and not of judicial or quasijudicial power. The SEC, through the hearing
panel that heard the petition for approval of the
Rehabilitation Plan, was acting as a quasi-judicial
body and thus, its order approving the plan
cannot constitute an impairment of the right and
the freedom to contract. Besides, the mere fact
that the Rehabilitation Plan proposes a dacion en
pago approach does not render it defective on
the ground of impairment of the right to
contract. Dacion en pago is a special mode of
payment where the debtor ofers another thing to
the creditor who accepts it as equivalent of
payment
of
an
outstanding
debt.
The
undertaking really partakes in a sense of the
nature of sale, that is, the creditor is really buying
the thing or property of the debtor, the payment
for which is to be charged against the debtors
debt. As such, the essential elements of a
contract of sale, namely; consent, object certain,
and cause or consideration must be present.
Being a form of contract, the dacion en pago
agreement cannot be perfected without the
consent of the parties involved.
Section 11. Free access to the courts and
quasi-judicial bodies and adequate legal
assistance shall not be denied to any
person by reason of poverty.

TAU MU

The courts of justice should be as available to


the pauper as to the affluent in the protection
of their respective rights --- Rule 3 Sec. 22
Rules of Court.

TAU MU

MARTINEZ vs. PEOPLE


332 SCRA 694 (2000)

TAU MU
TAU MU

Facts: Martinez was accused of homicide. While


his petition for certiorari was pending before the
Court of Appeals, he filed a motion with said court
to litigate as pauper litigant. The court denied it
because under Sec. 16, Rule 41, of the Revised
Rules of Court, appellate courts are not allowed to
entertain such petitions

TAU MU

CASE: FREE ACCESS TO COURTS

The Fraternal
Ateneo de Davao

TAU MU
TAU MU

Issue: WON there was a violation of the nonimpairment clause.

KITY

TAU MU TAU MU TAU MU


TAU MU TAU MU TAU MU

CONSTITUTIONAL LAW II
Order of Saint Thomas More
Atty. Philip John Pojas/Atty. Rovyne G. Jumao-as, RN
University College of Law

Held: The 1997 Rules of Civil Procedure has


removed the prohibition on appellate courts from
entertaining petitions to litigate as paper. While
accused filed his petition in 1994, the new policy
shall be made applicable to pending actions. This
interpretation is more in keeping with Art. III. Sec.
11 of the Constitution which decrees that free
access to the courts and quasi-judicial bodies and
adequate legal assistance shall not be denied to
any person by reason of poverty.
ACAR vs. ROSAL
Facts: This is a civil case which was filed by
Sacada workers against the Sugar Central.
Complainants
prayed
that
they
be
authorized to sue as pauper litigants (to be
exempted to pay certain fees). The court
denied their motion because they were
workers, and not paupers.
Ruling: The court held that there was a
denial of access to courts by reason of
poverty. An indigent was defined as persons
who have no property or source of income
sufficient for their own labor, though selfsupporting when able to work and in
employment.
Section
12.
(1)
Any
person
under
investigation for the commission of an
offense shall have the right to be informed
of his right to remain silent and to have
competent
and
independent
counsel
preferably of his own choice. If the person
cannot afford the services of counsel, he
must be provided with one. These rights
cannot be waived except in writing and in
the presence of counsel.
(2) No torture, force, violence, threat,
intimidation, or any other means which
vitiate the free will shall be used against
him. Secret detention places, solitary,
incommunicado, or other similar forms of
detention are prohibited.
(3) Any confession or admission obtained in
violation of this or Section 17 hereof shall
be inadmissible in evidence against him.
(4) The law shall provide for penal and civil
sanctions for violations of this section as
well as compensation to the rehabilitation
of victims of torture or similar practices,
and their families.
SECTION 12 (1):
CUSTODIAL INVESTIGATION
Time when the investigation is no longer a
general inquiry into an unsolved crime BUT has
begun to focus on a particular suspect who has
been taken into police custody and subjected by

ACADCOM 2010; Contributors: Gene Geocaniga, Jarissa Guiani, Darlene Magabilen


TAU MU Page 123 of 179

TAU MU
TAU MU

confessions made prior to its efectivity. The


same is true with the Morales decision which is a
judge-made rule. Prior to April 26, 1983, the
guidelines requiring that waiver of the right to
counsel by an accused-can be properly made only
with the assistance of counsel, had yet to be
promulgated by the Court.
The principle of
prospectivity of statutes shall apply to judicial
decisions, which although in themselves are not
laws, are nevertheless evidences of what the law
means.
RIGHTS OF PERSON UNDER CUSTODIAL
INVESTIGATION:
(1) Right to remain silent

Only an accused has the absolute right to


remain silent.

A person who is not an accused may


assume the stance of silence only when asked an
incriminating question.

A person under custodial investigation has


the right to refuse to answer any question.
Moreover, his silence may NOT be used against
him [PEOPLE vs. FRAGO,232 SCRA 653].
(2) Right
to
have
competent
and
independent counsel, preferably of his own
choice

The counsel made available to the person


under custodial investigation must be competent
and independent and preferably to the detainees
own choice.
Q: What is the privilege given to that person
under investigation?
A: If a person cannot aford a counsel, he must be
provided with one. So, that is a right and a
privilege. It is diferent from the right to counsel
because the right to counsel, everybody has the
right to counsel, but the person who cannot
aford counsel has the privilege and is given one.
(3) Right to be provided with the
services of counsel if he cannot afford the
services of one.
(4) Right to be informed of these rights.

TAU MU
TAU MU
TAU MU
TAU MU
TAU MU

Ruling:
Yes.
Sec. 12, Art III of the 1987
Constitution cannot be applied to extra-judicial

TAU MU

Facts: Accused are charged with Estafa Through


Falsification of Public Documents. On February
12, 1982, an agent of the National Bureau of
Investigation took their statements after they
orally waived their right to counsel without the
assistance of a lawyer.
Under the 1973
Constitution, there was no requirement that the
waiver must be in writing and made with the
assistance of counsel. However, on April 26,
1983, the Supreme Court rendered the decision in
Morales v. Enrile requiring that the waiver be
made with the assistance of counsel to be valid.
Considering that trial took place under the 1987
Constitution which bars such evidence, are the
statements admissible?

TAU MU

Santos v. Sandiganbayan
347 SCRA 386 (2000)

TAU MU
TAU MU

Essence of protection: What is being eschewed


is the evil of extorting a confession from the
mouth of the person being interrogated. As
defined, extortion is an act or practice of taking
or obtaining anything from a person by illegal use
of fear, whether by force, threats or any undue
exercise of power. In the context of obtaining an
admission, extorting means compelling or
coercing a confession or information by any
means serving to overcome his power of
resistance, or making the confession or admission
involuntary. [PEOPLE vs. TIN LANG UY, 475
SCRA 248]

The Fraternal
Ateneo de Davao

TAU MU
TAU MU

the police to a process of interrogations that


lends itself to eliciting incriminating statements
Any questioning initiated by the law enforcers
after a person has been taken into custody or
otherwise deprived of his freedom

It exists only in custodial interrogations, or


in-custody interrogation of accused persons. By
custodial interrogation is meant questioning
initiated by law enforcement officers after a
person has been taken into custody or otherwise
deprived of his freedom of action in any
significant way.

It refers to a custodial investigation where a


suspect has already been taken into police
custody and the investigating officers begin to
ask
questions
to
elicit
information
and
confessions or admissions from the suspect.

Involves any questioning initiated by law


enforcement authorities after a person is taken
into custody or otherwise deprived of his freedom
of action in any significant manner. And, the rule
begins to operate at once as soon as the
investigation ceases to be a general inquiry into
an unsolved crime and direction is then aimed
upon a particular suspect who has been taken
into custody and to whom the police would then
direct interrogatory question which tend to elicit
incriminating statements. It refers to the critical
pre-trial stage when the investigation ceases to
be a general inquiry into an unsolved crime but
has begun to focus on a particular person as a
suspect.

KITY

TAU MU TAU MU TAU MU


TAU MU TAU MU TAU MU

CONSTITUTIONAL LAW II
Order of Saint Thomas More
Atty. Philip John Pojas/Atty. Rovyne G. Jumao-as, RN
University College of Law

CRIMINAL PROCESS [MIRANDA Case]:


1.
Investigation prior to the filing of the
charges;
2.
Preliminary Examination and Investigation
after charges are filed; and
3.
Trial Period.
Q: When do the rights under Sec. 12 (1) end?
A: The rights under this section end when the
investigation prior to the filing of the charges also
end.
GENERAL RULE: The rights can be waived. BUT
the waiver must be done in writing and in the
presence of counsel. The provisions say that
these rights can be waived, if it is in writing and
in the presence of counsel.
Q: What cannot be waived from the abovementioned rights?

ACADCOM 2010; Contributors: Gene Geocaniga, Jarissa Guiani, Darlene Magabilen


TAU MU Page 124 of 179

A: The right to be informed cannot be waived, it


is an absolute right.
PURPOSES OF THE RIGHTS IN SEC. 12:
(1) To prohibit incommunicado investigation in
a police dominated atmosphere resulting in selfincriminating statements without the Miranda
Warning.
(2) To prevent coerced confessions given in a
police dominated atmosphere.
NOTE: In Galman case, there was a law
compelling the accused to testify under the pain
of contempt while in Ayson casem there is none.

TAU MU
TAU MU
TAU MU
TAU MU
TAU MU
TAU MU
TAU MU
TAU MU
TAU MU

WHEN THE RIGHT ATTACHES OR WHEN CAN


A PERSON INVOKE THESE RIGHTS:
1. Such person must be under investigation;

The right attaches at the start of the


investigation when the investigating officers start
to ask and question immediate information. Now,
the question has to be specific, meaning you are
being singled out, targeted as a perpetrator. If
only asked a general question, that is not
considered under investigation because it says
here that you must be interrogated or questioned
about matters relating to the crime, so it must be
a specific kind of investigation.

When a suspect is being invited to


the police station and asked questions about the
crime, even if he is not arrested. But if all the
requisites are present, then the right attaches.
[PEOPLE vs. TAN]

When the questioning is not a


general inquiry for any possible lead to the
perpetration of the crime under investigation.
[PEOPLE vs. BRAVO, 318 SCRA 812]

The accused was arrested and


confession was made while walking along the
highway with the policemen. The admission is
inadmissible. [PEOPLE vs. BARIQUIT, 341 SCRA
600]

TAU MU
TAU MU

REQUISITES WHEN RIGHT ATTACHES:


1. He must be under the custody of law
enforcers or is in jail or deprived of freedom of
action in a significant way.
2. He must be under investigation subjected to
questioning or interrogation by a law enforcer.
3. Such questioning must be on relation to a
crime.

The Fraternal
Ateneo de Davao

TAU MU
TAU MU

GENERAL
RULE:
In
administrative
investigations, the rights under Sec. 12 do not
attach.
EXCEPTION: Galman case because there was a
law or EO compelling the accused to testify
WHEN RIGHTS ARE AVAILABLE:
(1) AFTER a person has been taken into
custody or
(2) When a person is otherwise deprived of
his freedom of action in any significant way.
(3) When the investigation is being conducted
by the government (police, DOJ, NBI) with respect
to a criminal ofense.
(4) Signing of arrest reports and booking
sheets.

KITY

TAU MU TAU MU TAU MU


TAU MU TAU MU TAU MU

CONSTITUTIONAL LAW II
Order of Saint Thomas More
Atty. Philip John Pojas/Atty. Rovyne G. Jumao-as, RN
University College of Law

2. The person being investigated must be:


a. In the custody of a law enforcer, or
b. Must be in jail, or
c. Deprived of freedom of action in
significant way.

He was arrested, he was on board a


police vehicle already, he was not in jail. But he
was already deprived of his freedom of action in a
significant way because he was already in the
vehicle of the police and it was the police who
questioned him. [PEOPLE vs. BOLANOS]
3. The investigation must be made by a law
enforcer.
[Note: ALL elements must concur.]
CASES: RIGHT APPLIES
PEOPLE vs. MALNGAN
503 SCRA 204 (2006)
Facts: Malngan was accused of burning her
employers house. She was apprehended by the
Brgy. Chairman and the tanods and brought her
to the Barangay Hall. Upon inspection, a
disposable lighter was found inside Malngans
bag. Thereafter, she confessed to Bernardo in the
presence of multitudes of angry residents that
she set her employers house on fire because she
had not been paid her salary for about a year and
that she wanted to go home to her province but
her employer told her to just ride a broomstick in
going home.
Malngan was then turned over to arson
investigators headed by SFO4 Danilo Talusan,
who brought her to the San Lazaro Fire Station in
Sta. Cruz, Manila where she was further
investigated
and
then
detained.
When
interviewed by a reporter of ABS-CBN Network,
Malngan while under detention was heard by
SFO4 Talusan as having admitted the crime and
even narrated the manner how she accomplished
it. He was able to hear the same confession, this
time at his home, while watching the television
program True Crime hosted by Gus Abelgas also
of ABS-CBN Network. Hence, she was charged
with the crime of arson with multiple homicide.
Issue: WON the uncounselled extrajudicial
confession of Malngan to Bernardo and to the
media is inadmissible in evidence as being
violative of Article 3, Sec. 12(1) of the
Constitution.
Ruling: The provision under Sec. 12 Article 3 of
the Constitution applies to the stage of custodial
investigation when the investigation is no
longer a general inquiry into an unsolved crime
but starts to focus on a particular person as a
suspect. Said constitutional guarantee has also
been extended to situations in which an
individual has not been formally arrested but has
merely been invited for questioning. To be
admissible in evidence against an accused, the
extrajudicial confessions made must satisfy the

ACADCOM 2010; Contributors: Gene Geocaniga, Jarissa Guiani, Darlene Magabilen


TAU MU Page 125 of 179

TAU MU
TAU MU
TAU MU
TAU MU
TAU MU
TAU MU

Facts: Mojello was found guilty beyond


reasonable doubt for the crime of rape with
homicide. He admitted the perpetrated crime and

TAU MU

PEOPLE vs. MOJELLO


425 SCRA 11 (2004)

TAU MU

Ruling: No.
The settled rule is that an
uncouselled extra-judicial confession without a
valid waiver of the right to counselthat is, in
writing and in the presence of counselis
inadmissible in evidence. The belated presence
of a lawyer the following say even if prior to the
actual signing of the uncounselled confession
does not cure the defect for the investigator was
already able to extract incriminatory statements
from the suspects. We have previously held that
admissions
obtained
during
custodial
interrogations without the benefit of counsel
although later reduced to writing and in the
presence of counsel are still flawed under the
Constitution.

TAU MU

Facts: Since accused had no counsel and there


was no lawyer in Kaputian, Island Garden City of
Samal, the police investigator took their
confessions but did not require them to sign. The
following day, he escorted the two accused to
Davao City, along with their affidavits, before the
Public Attorneys Office.
The lawyer, after
advising them of their constitutional rights and
explaining the contents of their statements, and
after they affirmed the veracity and voluntariness
of their execution, asked them to affix their
signatures. Are the confessions admissible?

TAU MU
TAU MU

PEOPLE vs. QUIDATO JR.


297 SCRA 1 (1998)

The Fraternal
Ateneo de Davao

TAU MU
TAU MU

following requirements: (1) it must be voluntary;


(2) it must be made with the assistance of
competent and independent counsel; (3) it must
be express; and (4)
it must be in writing.
The
inadmissibility
of
Malngans
confession to Bernardo and the lighter does not
automatically lead to her acquittal. It should well
be recalled that the constitutional safeguards
during custodial investigations do not apply to
those not elicited through questioning by the
police or their agents but given in an ordinary
manner whereby the accused verbally admits to
having committed the ofense as what happened
in the case at bar when Malngan admitted to one
of the witnesses to having started the fire in the
Separas house. The testimony of the said
witness recounting said admission is admissible
in evidence against her and is not covered by the
constitutional guarantee. Article III of the
Constitution, or the Bill of Rights, solely governs
the relationship between the individual on one
hand and the State (and its agents) on the other;
it does not concern itself with the relation
between a private individual and another private
individual as both Malngan and prosecution
witness undoubtedly are. There was also no
evidence on record to show that said witness was
acting under police authority.

KITY

TAU MU TAU MU TAU MU


TAU MU TAU MU TAU MU

CONSTITUTIONAL LAW II
Order of Saint Thomas More
Atty. Philip John Pojas/Atty. Rovyne G. Jumao-as, RN
University College of Law

assisted
by
counsel
during
custodial
investigation.
Issue:
WON
the
extrajudicial
confession
executed is admissible in evidence.
Ruling: The extra-judicial confession of the
appellant is valid and admissible in evidence. The
phrase preferably of his own choice does not
convey the message that the choice of a lawyer
by a person under investigation is exclusive to
preclude
other
equally
competent
and
independent attorneys from handling the case,
otherwise, the temporariness of custodial
investigation will be solely in the hands of the
accused. The suspect has the final choice as he
may reject the counsel. A lawyer is deemed
engaged when the accused does not raise any
objection during the investigation. The first
custodial investigation violated the Miranda
doctrine since no counsel was present and
employees waiver of right of counsel. The Dec.
23, 1996 custodial investigation was upheld for
having complied with Art. 3, Section 12. Even
though improper interrogation methods were
used at the outset, there is still a possibility of
obtaining a legally valid confession later on by
properly interrogating the subject under diferent
conditions and circumstances than those which
prevailed originally.
People vs. Arondain
366 SCRA 98 (2001)
Facts: After the robbing and killing of the taxi
driver, accused was arrested by the police not far
from the crime scene. The gun used in the killing
was confiscated from him. When asked by the
arresting officers why he shot the victim, accused
answered that the latter resisted his demand for
money. He added that after the shooting the
driver, he panicked and immediately scampered
away, leaving the money bill scattered on the
floor of the taxi cab. Is his statement admissible
in evidence?
Ruling:
No.
The statement made by the
accused admitting the crime cannot be admitted
even as part of res gestae. It must be stressed
that said statement was obtained in violation of
his constitutional rights. Said confession was
giver after he was arrested and without the
assistance of counsel. He was not even informed
of his right to remain silent or to counsel. From
the time he was arrested and deprived of his
freedom, all the questions propounded on him by
the police for the purpose of eliciting, admissions,
confessions or any information came within the
ambit of custodial investigation.
Failing to
observe
the
constitutional
mandate
the
confession cannot be admitted as evidence
against him.
PEOPLE vs. TAN
286 SCRA 207 (1998)
Facts: In connection with the crime highway
robbery with murder, the police invited accused,
then a suspect, to the police station. In the
course of the conversation inside the station, and
without being warned of his right to counsel, but

ACADCOM 2010; Contributors: Gene Geocaniga, Jarissa Guiani, Darlene Magabilen


TAU MU Page 126 of 179

without being subjected to any pressure, accused


allegedly admitted authorship of the crime. Is his
oral confession admissible?

PEOPLE VS. BRAVO


318 SCRA 812 (1999)

TAU MU
TAU MU
TAU MU
TAU MU
TAU MU
TAU MU

Ruling: No. The interrogation by the police falls


under the term custodial investigation. It is of
no moment that the questioning was done along

TAU MU

Facts: Accused were suspects in a crime of


robbery with homicide. Upon their arrest, the
police immediately commenced investigation by
asking them questions regarding the commission
of the crime even while they were still walking
along the highway, on their way to the police
station. Without any lawyer to assist them, the 2
confessed to the crime. Are the confessions
admissible?

TAU MU

PEOPLE VS. BARINQUIT


341 SCRA 600 (2000)

TAU MU

Ruling: No. Accused was under arrest for the


rape and killing and any statements made by him
pertaining to possible complicity in the crime
without prior warning of his rights is inadmissible
in evidence. The policemans apparent attempt
to circumvent the rule by insisting that the
admission was made during an informal talk
prior to custodial investigation proper is not
tenable. Appellant was not invited to the police
station as part of a general inquiry for any
possible lead to the perpetrators of the crime
under investigation. Courts are not allowed to
distinguish between preliminary questioning and
custodial investigation proper when applying the
exclusionary rule.

TAU MU
TAU MU

Facts: On Jan. 15, 1994, the decomposing body


of a child was found. After interviewing a witness
who saw accused with the child on the night of
Jan. 12, the police located accused at his work
place. He was informed that he was a suspect
and asked to come to the precinct for
questioning. Accused was not informed of his
constitutional rights, and while he was being
informally interviewed by a policeman before
being turned over to the assigned investigator,
accused admitted that he was with the girl and
he carried her on his shoulder but he was so
drunk that night that he did not remember what
he did to her. Is the statement admissible?

The Fraternal
Ateneo de Davao

TAU MU
TAU MU

Ruling: No. Under the Constitution and RA No.


7438, a confession to be admissible must satisfy
the following requirements: (1) it must be
voluntary; (2) it must be made with the
assistance of competent and independent
counsel; (3) it must be express; and (4) it must be
in writing. Further, under RA No. 7438 it is
provided that custodial investigation shall
include the practice of issuing an invitation to a
person who is investigated in connection with an
ofense he is suspected to have committed,
without prejudice to the liability of the inviting
officer for any violation of law.

KITY

TAU MU TAU MU TAU MU


TAU MU TAU MU TAU MU

CONSTITUTIONAL LAW II
Order of Saint Thomas More
Atty. Philip John Pojas/Atty. Rovyne G. Jumao-as, RN
University College of Law

the highway while accused were being led by the


police to the station. To put it diferently, the
place of interrogation is not at all a reliable
barometer to determine the existence or absence
of custodial investigation. It is significant that at
the time the police already presumed accused as
the perpetrators of the crime and singled them
out as the authors thereof. The police should
have
properly
apprised
them
of
their
constitutional
rights,
without
which
such
uncounselled admissions are inadmissible in
evidence.
GUTANG vs. PEOPLE
335 SCRA 479 (2000)
Facts: After conducting a search by virtue of a
warrant in the residence of accused and the
confiscation
of
marijuana,
shabu
and
paraphernalia for the use of shabu, the police
prepared a receipt.
Accused, without the
assistance of counsel, signed the receipt. The
Philippine National Police Crime Laboratory
subsequently conducted an examination of the
specimens and it rendered a report that they
were positive to test for the presence of
prohibited and/or regulated drugs.
Are the
Receipts of Property Seized and Chemistry and
Physical Science Reports admissible in evidence?
Ruling:
The Receipt of Property Seized
describing the property taken from accused is
inadmissible in evidence. It was signed by him
without the assistance of a lawyer.
It is
tantamount to an uncounselled extra-judicial
confession. However, the Physical Science and
Chemistry Reports are admissible inasmuch as
the examined materials were legally seized on
the strength of a valid warrant. Thus, the tests
conducted thereon were legally done. In other
words, even without the Receipt of Property
Seized, the guilt of the accused for the crimes
charged was proven beyond reasonable doubt.
WHEN RIGHTS ARE NOT AVAILABLE:
(1) During a police line-up. Exception: Once
there is a move among the investigators to elicit
admissions or confessions from the suspect, or
once you are targeted as a suspect.
(2) During administrative investigations.
(3) Confessions made by an accused at the
time he voluntarily surrendered to the police or
outside the context of a formal investigation.
(4) Statements made to a private person.
WHEN THE RIGHT DOES NOT ATTACH:
(1) General inquiries
(2) General rule for police line-up
(3) when person investigating is not a police
officer or not a law enforcer
(4) Administrative investigations because sec.
12 only covers criminal investigations
(5) Fact finding investigations even if akin to a
criminal investigation, still look at the person
investigating: not a police officer or law enforcer,
but a fact finding body (so it is still not allowed)

ACADCOM 2010; Contributors: Gene Geocaniga, Jarissa Guiani, Darlene Magabilen


TAU MU Page 127 of 179

(6) A barangay captain is not considered a


police officer or law enforcer.
[Note: The evidence is admissible perhaps
because one or all of the requisites are missing.]
REQUISITES OF A VALID WAIVER:
(1)Waiver should be made in WRITING.
(2)Waiver should be made in the PRESENCE OF
COUNSEL.

TAU MU
TAU MU
TAU MU
TAU MU
TAU MU
TAU MU
TAU MU
TAU MU

[Miranda Warning by CJ Warren]


He must be warned prior to any questioning that
he has the right to remain silent, that anything he

TAU MU

It is now incumbent upon the prosecution to


prove during trial that prior to questioning the
confessant was warned of his constitutional
rights.
The rights under this section are available
only to a person facing custodial investigation.
These rights do not apply to a spontaneous
statement, NOT elicited through questioning by
the authorities, BUT given in an ordinary manner
whereby the accused orally admits having
committed the crime [PEOPLE vs. BALOLOY
(4/12/2002)].
The rights under this section can be lost by
neglect as when the defense fails to object to the
admissibility of the evidence immediately as
required by Rule 132 Sec. 36 --- deemed WAIVER.

TAU MU
TAU MU

MIRANDSA RIGHTS/WARNING:
1.
The person must be informed in the
outset in clear and unequivocal terms that he has
the right to remain silent;
2.
Thereafter he must be told that anything
he says can and will be used against him in court;
3.
He must be clearly informed that he has
the right to consult with a lawyer and to have the
lawyer with him during the interrogation;
4.
He should be warned that not only has he
the right to consult with a lawyer but also that if
he is indigent, a lawyer will be appointed to
represent him;
5.
Even if the person consents to answer
questions without assistance of counsel, the
moment he asks for a lawyer at any point in the
investigation, the interrogation must cease until
an attorney is present;
6.
If the foregoing protections and warnings
are NOT demonstrated to have been observed by
the prosecution, no evidence obtained as a result
of the interrogation can be used against the
accused.

The Fraternal
Ateneo de Davao

TAU MU
TAU MU

[PEOPLE vs. GOMEZ]: The guy was in HK, he was


in jail in HK. The NBI or some government agency
sent some people to investigate, but these are
law enforcers. Now, there was an admission. Can
this be used against him? The SC said that no, he
can invoke his rights under sec. 12 even if the
investigation is being done in a foreign country
for as long as he is under investigation. For as
long as he is in custody of a law enforcer, (he was
in jail in HK), and for as long as the person
investigating is a law enforcer, he can invoke his
right because at that time that he was
investigated in HK, there was no lawyer present.

KITY

TAU MU TAU MU TAU MU


TAU MU TAU MU TAU MU

CONSTITUTIONAL LAW II
Order of Saint Thomas More
Atty. Philip John Pojas/Atty. Rovyne G. Jumao-as, RN
University College of Law

says can be used against him in a court of law,


that he has the right to the presence of an
attorney, and that if he cannot aford an attorney
one will be appointed for him prior to any
questioning if he so desires. Opportunity to
exercise those rights must be aforded to him
throughout the interrogation. After such warnings
have been given, such opportunity aforded him,
the individual may knowingly and intelligently
waive these rights and agree to answer or make a
statement. But unless and until such warnings
and waivers are demonstrated by the prosecution
at the trial, no evidence obtained as a result of
interrogation can be used against him.
The
objective
is
to
prohibit
incommunicado interrogation of individuals in a
police-dominated atmosphere, resulting in selfincriminating statement without full warnings of
constitutional rights.

The employee may, of course, refuse to


submit any statement at the investigation; that is
his privilege. But if he should opt to do so, in his
defense to the accusation against him, it would
be absurd to reject his statements, whether at
the administrative investigation, or at a
subsequent criminal action brought against him,
because he had not been accorded, prior to his
making and presenting them, his Miranda rights
(to silence and to counsel and to be informed
thereof, etc) which, to repeat, are relevant in
custodial investigations. [PEOPLE vs. AYSON]
Note: An interrogation of the accused by the
police, if any there had been would already have
been ended at the time of the filing of the
criminal case in court (or the public prosecutors
office). Hence, with respect to a defendant in a
criminal case already pending in court (or the
public prosecutors office), there is no occasion to
speak of his right while under custodial
interrogation laid down by the second and
subsequent sentences of Section 20, Article IV of
the 1973 Constitution [now Section 12, Article III
of the 1987 Constitution], for the obvious reason
that he is no longer under custodial
interrogation.
RIGHTS OF THE ACCUSED WHETHER IN
COURT
OR
UNDERGOING
PRELIMINARY
INVESTIGATION
BEFORE
PUBLIC
PROSECUTOR:
1) The right to refuse to be made witnesses;
2) The right not to have any prejudice whatsoever
imputed to them by such refusal;
3) The right to testify on their own behalf, subject
to cross-examination by the prosecution; and
4) While testifying, the right to refuse to answer a
specific question that tends to incriminate them
for some crime other than that for which they are
being prosecuted.
[LADIANA vs. PEOPLE, 393 SCRA 419]
CASES: RIGHT DOES NOT APPLY
PEOPLE vs. BALOLOY
381 SCRA 31 (2002)

ACADCOM 2010; Contributors: Gene Geocaniga, Jarissa Guiani, Darlene Magabilen


TAU MU Page 128 of 179

CONSTITUTIONAL LAW II
Order of Saint Thomas More
Atty. Philip John Pojas/Atty. Rovyne G. Jumao-as, RN
University College of Law

KITY

Ateneo de Davao

TAU MU
TAU MU
TAU MU
TAU MU
TAU MU
TAU MU
TAU MU
TAU MU
TAU MU
TAU MU
TAU MU

Ruling: In the case at bar, there is merit in


Baloloys claim that his constitutional rights
during custodial investigation were violated by
Judge Dicon when the latter propounded to him
incriminating questions without informing him of
his constitutional rights. It is settled that at the

TAU MU
TAU MU

of

TAU MU TAU MU TAU MU


TAU MU TAU MU TAU MU

Facts: A dead body of an 11-year-old girl


Genelyn Camacho was found. The one who
caused its discovery was accused-appellant
Baloloy himself, who claimed that he had caught
sight of it while he was catching frogs in a nearby
creek. At Genelyns wake, Brgy. Captain Ceniza
asked Baloloy why his rope was found where
Genelyns body was discovered. He answered, I
have to claim this as my rope because I can
commit sin to God if I will not claim this as mine
because this is mine. Baloloy told Ceniza that his
intention was only to frighten the girl, not to
molest and kill her. When Genelyn ran away, he
chased her. As to how he raped her, Baloloy told
Ceniza that he first inserted his fingers into
GENELYNs vagina and then raped her.
Thereafter, he threw her body into the ravine.
After such confession, Ceniza examined
his body and found a wound on his right shoulder,
as well as abrasions and scratches on other parts
of his body. Upon further inquiry, Baloloy told her
that the wound on his shoulder was caused by
the bite of Genelyn. Ceniza then turned over
Baloloy to a policeman for his own protection, as
the crowd became unruly when she announced to
them that Baloloy was the culprit. He was
forthwith brought to the police headquarters.
Ceniza informed him the police officer that
Baloloy was the suspect in the killing of Genelyn,
and she turned over to him a black rope which
belonged to Baloloy. He wanted to interrogate
Baloloy, but Ceniza cautioned him not to proceed
with his inquiry because the people around were
getting unruly and might hurt Baloloy. He
immediately brought Baloloy to the police station,
and on that same day, he took the affidavits of
the witnesses.
Judge. Dicon said that when he arrived in
his office, several people, including Ceniza, were
already in his courtroom. He learned that they
came to swear to their affidavits before him.
After reading the affidavit of Ceniza, he asked
Ceniza whether her statements were true.
Ceniza answered in the affirmative and pointed to
Baloloy as the culprit. Judge Dicon turned to him
and asked him whether the charge against him
was true. He replied: I was demonized. While
Judge Dicon realized that he should not have
asked him as to the truthfulness of the
allegations against him, he felt justified in doing
so because the latter was not under custodial
investigation. Judge Dicon thus proceeded to ask
Baloloy whether he had a daughter as old as the
victim and whether he was aware of what he had
done to the girl Again, Baloloy responded that he
was demonized, and he spontaneously narrated
that after he struck Genelyns head with a stone
he dropped her body into the precipice.
Issue: WON the extrajudicial confession
Baloloy to Ceniza is admissible in evidence.

The Fraternal

moment the accused voluntarily surrenders to, or


is arrested by, the police officers, the custodial
investigation is deemed to have started. So, he
could not thenceforth be asked about his
complicity in the ofense without the assistance
of counsel. Judge Dicons claim that no complaint
has yet been filed and that neither was he
conducting a preliminary investigation deserves
scant consideration. The fact remains that at
that time JUANITO was already under the custody
of the police authorities, who had already taken
the statement of the witnesses who were then
before Judge Dicon for the administration of their
oaths on their statements.
Moreover, contrary to what the police
officer claims that Baloloy was not arrested but
was rather brought to the police headquarters for
his protection, the records reveal that he was in
fact arrested. If indeed Baloloys safety was the
primordial concern of the police authorities, the
need to detain and deprive him of his freedom of
action would not have been necessary. Arrest is
the taking of a person into custody in order that
he may be bound to answer for the commission
of an ofense, and it is made by an actual
restraint of the person to be arrested, or by his
submission to the person making the arrest. At
any rate, while it is true that his extrajudicial
confession before Judge Dicon was made without
the advice and assistance of counsel and hence
inadmissible in evidence, it could however be
treated as a verbal admission of the accused,
which could be established through the
testimonies of the persons who heard it or who
conducted the investigation of the accused.
PEOPLE vs. ENDINO
352 SCRA 307 (2001)
Facts: For a murder that took place in Palawan,
policemen arrested accused in Antipolo. On their
way to the airport, they stopped at ABS-CBN
television station where reporters interviewed
accused. Video footages were taken that showed
accused admitting his guilt. The interview was
shown over ABS-CBN evening news program TV
Patrol?
Is the video footage admissible in
evidence?
Ruling: Yes. The confession does not form part
of custodial investigation as it was not given to
police officers but to media men.
However,
because of inherent danger in the use of the
television as a medium for admitting ones guilt,
and the recurrence of the phenomenon in several
cases, it is prudent that trial courts are reminded
that extreme caution must be taken in further
admitting similar confessions. For all probability,
the police, with the connivance of unscrupulous
media practitioners, may attempt to legitimize
coerced confessions and place them beyond the
exclusionary rule by having an accused admit an
ofense on television.
Courts should never
presume that all media confessions described as
voluntary have been freely given. This type of
confession always remains suspect and therefore
should be thoroughly scrutinized.

ACADCOM 2010; Contributors: Gene Geocaniga, Jarissa Guiani, Darlene Magabilen


TAU MU Page 129 of 179

PEOPLE vs. TABOGA


376 SCRA 505 (2002)
Facts: Accused was charged with Robbery with
Homicide, as well as Arson, for robbing and killing
an old woman and burning her house. After he
was arrested, he was interviewed by a radio
reporter inside the investigation room of the
police station were policemen were present. The
radio announcer informed him at the outset that
he was a reporter who will be interviewing him to
get side of the incident. During the interview, he
admitted authorship of the crimes.
Is his
confession admissible?

TAU MU
TAU MU
TAU MU
TAU MU
TAU MU
TAU MU

Ruling: The SC ruled that although the testimony


of SFO4 Talusan is a hearsay because he was not
present when Gus Abelgas interviewed Malngan,
it may nevertheless be admitted in evidence as

TAU MU

Issue: WON the testimony of SF04 Talusan is


inadmissible for being a hearsay.

TAU MU

Malngan was then turned over to arson


investigators headed by SFO4 Danilo Talusan,
who brought her to the San Lazaro Fire Station in
Sta. Cruz, Manila where she was further
investigated
and
then
detained.
When
interviewed by a reporter of ABS-CBN Network,
Malngan while under detention was heard by
SFO4 Talusan as having admitted the crime and
even narrated the manner how she accomplished
it. He was able to hear the same confession, this
time at his home, while watching the television
program True Crime hosted by Gus Abelgas also
of ABS-CBN Network. Hence, she was charged
with the crime of arson with multiple homicide.

TAU MU

Facts: Malngan was accused of burning her


employers house. She was apprehended by the
Brgy. Chairman and the tanods and brought her
to the Barangay Hall. Upon inspection, a
disposable lighter was found inside Malngans
bag. Thereafter, she confessed to Bernardo in the
presence of multitudes of angry residents that
she set her employers house on fire because she
had not been paid her salary for about a year and
that she wanted to go home to her province but
her employer told her to just ride a broomstick in
going home.

TAU MU
TAU MU

PEOPLE vs. MALNGAN


503 SCRA 204 (2006)

The Fraternal
Ateneo de Davao

TAU MU
TAU MU

Ruling:
Yes.
The confession made by the
accused did not form part of custodial
investigation. It was not given to police officers
but to a media man in an apparent attempt to
elicit sympathy. There is nothing in the record to
show that the radio announcer colluded with the
police authorities to elicit inculpatory evidence
against accused. Nether is there anything on
record which even remotely suggests that the
radio announcer was instructed by the police to
extract information from him on the details of the
crimes.
Thus, assistance of counsel is not
required.

KITY

TAU MU TAU MU TAU MU


TAU MU TAU MU TAU MU

CONSTITUTIONAL LAW II
Order of Saint Thomas More
Atty. Philip John Pojas/Atty. Rovyne G. Jumao-as, RN
University College of Law

an independently relevant statement to establish


not the truth but the tenor of the statement or
the fact that the statement was made. Under the
doctrine of independently relevant statements,
regardless of their truth or falsity, the fact that
such statements have been made is relevant. The
hearsay rule does not apply, and the statements
are admissible as evidence. Evidence as to the
making of such statement is not secondary but
primary, for the statement itself may constitute a
fact in issue or be circumstantially relevant as to
the existence of such a fact.
BPI vs. CASA MONTESSORI, YABUT
430 SCRA 261 (2004)
Facts: Casa opened an account with BPI. BPI
discovered that 9 checks were encashed by
Sonny Santos, a fictitious name used by Yabut,
external auditor of Casa. Casa filed a complaint
for collection with damages against BPI.
Issue: WON the voluntary admission of Yabut
violated his constitutional rights on custodial
investigation.
Ruling: He was not under custodial investigation.
His affidavit was executed in private and before
private individuals. To fall within the ambit of Sec.
12, there must be an arrest or a deprivation of
freedom. It does not apply to spontaneous
statements made in a voluntary manner whereby
an individual orally admits to authorship of a
crime. What the constitution proscribes is the
compulsory
or
coercive
disclosure
of
incriminating facts. The Bill of Rights does not
concern itself with the relation between a private
individual and another individual. It governs the
relationship between the individual and the State.
ASTUDILLO and ORELLANA vs. PEOPLE
509 SCRA 509 (2006)
Facts: Petitioners were hired by Western
Marketing Corporation, a chain of appliance
stores, as salespersons. It was found out later on
that the daily cash collection report did not reflect
any
remittance
of
payments
from
the
transactions covered by certain invoices. It was
also discovered that Flormarie was the one who
filled it up and received the payment reflected
therein and that the goods covered thereby were
missing. Concluding that the transactions under
the said invoices were made but no payment was
remitted to Western, the accountant reported the
matter to the branch manager. Benitez and
Orellana pleaded with them not to report the
matter to the management. Flormarie made a
similar plea as she admitted to stealing the
missing booklet of invoices, she explaining that
her father was sick and had to undergo medical
operation, and ofering to pay for the goods
covered thereby. In a subsequent meeting with
Lily, Orellana admitted having brought home
some
appliances
while
Benitez
gave
a
handwritten statement asking for apology and
explaining why there was a short-over. In a still
subsequent meeting with the representative of

ACADCOM 2010; Contributors: Gene Geocaniga, Jarissa Guiani, Darlene Magabilen


TAU MU Page 130 of 179

Western, Orellana made a written statement in


the formers presence stating that she will not
repeat what she did when she brought items
outside the establishment. Also in a meeting with
the representative, Astudillo, who was earlier
implicated by Flormaries husband in his
telephone conversation with the branch manager,
wrote in her written statement her apology and
her regrets for what she did. Flormarie
subsequently appeared before a notary public to
execute a similar statement. She later executed a
statement before SPO1 Gregorio.

The Fraternal
Ateneo de Davao

TAU MU
TAU MU
TAU MU
TAU MU

In an inventory of stocks conducted at the


branch office of Western, several other appliances
were found missing as were unauthorized
deductions from the cash collections. The total
missing merchandise was valued at P797,984.00
as reflected in the inventory report. And
discrepancies between the actual sales per cash
sales invoice and the cash remittance to the
company in the sum of P34,376.00 were also
discovered, prompting Western to initiate the
criminal complaints for Qualified Theft.

KITY

TAU MU TAU MU TAU MU


TAU MU TAU MU TAU MU

CONSTITUTIONAL LAW II
Order of Saint Thomas More
Atty. Philip John Pojas/Atty. Rovyne G. Jumao-as, RN
University College of Law

TAU MU

Ruling: In the case at bar, petitioners were not


under custodial investigation to call for the
presence of counsel of their own choice, hence,
their written incriminatory statements are
admissible in evidence. The extra-judicial
confession before the police of Flormarie in which
she incriminated petitioners bears a diferent
complexion, however, as it was made under
custodial investigation. When she gave the
statement, the investigation was no longer a
general inquiry into an unsolved crime but had
begun to focus on a particular suspect. The
records show that the accountant had priorly
reported the thievery to the same police
authorities and identified Flormarie and Benitez
as initial suspects.
It is always incumbent upon the
prosecution to prove at the trial that prior to incustody
questioning,
the
confessant
was
informed of his constitutional rights. The
presumption of regularity of official acts does not
prevail over the constitutional presumption of
innocence. Hence, in the absence of proof that
the arresting officers complied with these
constitutional
safeguards,
extrajudicial
statements, whether inculpatory or exculpatory,
made
during
custodial
investigation
are
inadmissible and cannot be considered in the
adjudication of a case.
In other words,
confessions and admissions in violation of Section
12 (1), Article III of the Constitution are
inadmissible in evidence against the declarant
and more so against third persons. This is so even
if such statements are gospel truth and
voluntarily given.
There is nothing on record, however,
buttressing petitioners claim other than their
self-serving assertion. The presumption that no

TAU MU
TAU MU

Issue: WON the employees extra-judicial


admissions taken before an employer in the
course of an administrative inquiry are admissible
in a criminal case filed against them.

person of normal mind would deliberately and


knowingly confess to a crime unless prompted by
truth and conscience such that it is presumed to
be voluntary until the contrary is proved thus
stands. Only Astudillo was acquitted.
SEBASTIAN V. GARCHITORENA
343 SCRA 463 (2000)
Facts: Petitioners were postal employees
charged with Malversation of Public Funds.
During a fact-finding investigation conducted
relative to missing postage stamps they executed
sworn statements without the assistance of
counsel and without being warned of their rights.
Their written statements were ofered by the
prosecution in evidence during the trial in the
criminal case. Is there a violation of the right to
counsel?
Ruling: No. The rights under Sec. 12 apply only
to custodial investigation or questioning initiated
by law enforcement officers after a person has
been taken into custody or otherwise deprived of
his freedom of action in any significant way. The
fact finding relative to the missing stamps
conducted by the Chief Postal Service Office is
not a custodial investigation but a mere
administrative
investigation.
While
an
administrative investigation may be akin to a
criminal proceeding, irrespective of the nature of
the charges and of the respondents capacity to
represent himself, no duty rests on the
investigator to provide the person being
investigated with counsel.
REMOLINA VS. CSC
362 SCRA 304 (2001)

TAU MU
TAU MU

Facts: For having secured for his wife a fake


Report of Rating with a passing mark in the
teachers board examination, petitioner was
dismissed
from
government
service
for
dishonesty. Among the evidence against him was
his written statement made during the
investigation by the Civil Service Commission
admitting his guilt. No counsel assisted him nor
was he advised of rights under the Constitution
when he signed the statement. Is it admissible?

TAU MU
TAU MU
TAU MU
TAU MU

Ruling: Yes. While investigations conducted by


an administrative body may at times be akin to a
criminal proceedings, the fact remains that under
existing laws, a party in an administrative inquiry
may not be assisted by counsel, irrespective of
the nature of the charges or the respondents
capacity to represent himself. Under the Civil
Service Act, a respondent has the option of
engaging the services of counsel or not. The
right to counsel is not always imperative in an
administrative
investigation
because
such
inquiries are conducted merely to determine
whether there are facts that merit disciplinary
measure against erring pubic officials. In this
case, respondents was not accused of any crime
in the investigation conducted by the CSC field
office.
It was conducted for the purpose of

ACADCOM 2010; Contributors: Gene Geocaniga, Jarissa Guiani, Darlene Magabilen


TAU MU Page 131 of 179

CONSTITUTIONAL LAW II
Order of Saint Thomas More
Atty. Philip John Pojas/Atty. Rovyne G. Jumao-as, RN
University College of Law

he

should

The Fraternal
Ateneo de Davao

be

PEOPLE vs. TING LAN UY


475 SCRA 248 (2005)

TAU MU
TAU MU
TAU MU
TAU MU
TAU MU
TAU MU
TAU MU
TAU MU
TAU MU

Ruling:
The rights enumerated by the
constitutional provision invoked by accusedappellant are not available before government
investigators enter the picture. Thus, as held in
one case, the admissions made during the course
of an administrative investigation by Philippine
Airlines do not come within the purview of
Section 12. The protective mantle of the
constitutional provision also does not extend to
admissions or confessions made to a private
individual, or to a verbal admission made to a
radio announcer who was not part of the
investigation, or even to a mayor approached as
a personal confidante and not in his official
capacity.
Neither does the constitutional provision
on custodial investigation extends to a
spontaneous statement, not elicited through
questioning by the authorities, but given in an
ordinary manner whereby the accused orally
admits having committed the crime, nor to a
person undergoing an audit examination because
an audit examiner is not a law enforcement
officer.
Thus, the flaw in appellants argument in
this regard becomes immediately apparent vis--

TAU MU
TAU MU

Issue: WON a sworn statement which was taken


without the benefit of counsel violated Section
12.

TAU MU
TAU MU

Facts: The prosecution theorizes that accused


diverted the funds covered by the two PNB
Managers checks by falsifying a commercial
document called an Application for Cashiers
Check by inserting an account number of a
private individual after the name of the payee,
UCPB. It claims that NPC did not authorize the
insertion considering that the Payment Instruction
issued by NPC instructing PNB to prepare a
Managers check to be charged to NPCs savings
account did not contain any account number.
Through the insertion, the accused allegedly
succeeded in diverting the funds from the UCPB
in favor of several persons.
In his defense, appellant asserts that there
was no evidence that he committed any of the
acts alleged in the information, particularly the
intercalation on the ACC; that he deposited the
checks subsequently issued or that he received
the proceeds thereof; or that he conspired with
any of his co-accused. He claims that his
conviction was based on the alleged sworn
statement and the transcript of stenographic
notes of a supposed interview with appellant by
the NPC personnel and the report of the NBI.
Appellant maintains that he signed the sworn
statement while confined at the Philippine Heart
Center and upon assurance that it would not be
used against him. He was not assisted by
counsel nor was he apprised of his constitutional
rights when he executed the affidavit.

TAU MU TAU MU TAU MU


TAU MU TAU MU TAU MU

ascertaining
whether
administratively charged.

KITY

vis the foregoing legal yardsticks, considering


that his statement was taken during the
administrative investigation of NPCs audit team
and before he was taken into custody. As such,
the inquest was still a general inquiry into an
unsolved ofense at the time and there was, as
yet, no specific suspect. The fact that an NBI
investigation was being contemporaneously
conducted at the time the sworn statement was
taken will not extricate appellant from his
predicament. The essence of the constitutional
safeguard is protection from coercion. The
interview where the sworn statement is based
was conducted by NPC personnel for the NPCs
administrative investigation. Any investigation
conducted by the NBI is a proceeding separate,
distinct and independent from the NPC inquiry
and should not be confused or lumped together
with the latter.
PEOPLE vs. SALONGA
359 SCRA 310 (2001)
Facts: Accused was an Acting Assistant Cashier
of Metrobank. During a spot audit conducted by
the Department of Internal Afairs of the bank, he
admitted having issued a cashiers check without
any legitimate transaction and that he benefited
P8,500.00 from the amount of the check. His
admissions were reduced into writing and ofered
in evidence by the prosecution. When he made
the admissions, however, there was no lawyer
assisting him. Is it admissible?
Ruling: Yes. The constitutional right to counsel
may be invoked only by a person under custodial
investigation. Custodial investigation is the stage
where the police investigation is no longer a
general inquiry into an unsolved crime but has
begun to focus on a particular suspect taken into
custody by the police who carry out an
interrogation to elicit incriminating statements.
In this case, the bank auditor who questioned
accused was not a police officer but a private
person. He was not under custodial investigation
so that the legal formalities required by the
fundamental law do not apply.
PEOPLE vs. BALOLOY
supra.
The constitutional provision on custodial
investigation does not apply to a spontaneous
statement, not elicited through questioning by
the authorities but given in an ordinary manner
whereby the suspect orally admits having
committed the crime. Neither can it apply to
admissions or confessions made by a suspect in
the commission of a crime before he is placed
under investigation. What the Constitution bars
is the compulsory disclosure of incriminating facts
or confessions. The rights under Section 12 of
the Constitution are guaranteed to preclude the
slightest use of coercion by the state as would
lead the accused to admit something false, not to
prevent him from freely and voluntarily telling the
truth.

ACADCOM 2010; Contributors: Gene Geocaniga, Jarissa Guiani, Darlene Magabilen


TAU MU Page 132 of 179

PEOPLE vs. PAVILLARE


329 SCRA 684 (2000)
Facts: Accused was suspect in a kidnapping of an
Indian national.
After his arrest for another
ofense, he was identified in a police line-up by
his victim at the police station as one of the
kidnappers. Considering that accused was not
assisted by counsel, is he identification
admissible?

TAU MU
TAU MU
TAU MU
TAU MU
TAU MU
TAU MU
TAU MU

Ruling: The legal formalities required by the


fundamental law of the land apply only to extrajudicial confessions or admissions obtained
during custodial investigations. Indeed, the rights
enumerated in the constitutional provision exist
only in custodial interrogations, or in-custody
interrogation of accused persons.

TAU MU

Issue (1): WON absence of the counsel when


petitioner executed the affidavit violated his right
under Section 12.

TAU MU

Facts: Caridad declared that she is the wife of


Barangay Captain Francisco, the victim in the
case at bar. Caridad testified that Francisco was
shot and killed by accused Ladiana, who happens
to be also a distant relative of the decedent Prior
to the conduct of the examination-in-chief on
Cortez, a retired assistant prosecutor, the defense
counsel made an admission as to the authorship,
authenticity, and voluntariness of the execution
of the counter-affidavit of Ladiana, which was
subscribed and sworn to before Cortez. In said
counter-affidavit, accused Ladiana allegedly
admitted to making the fatal shots on Francisco.
However, Ladiana allegedly did so in self-defense
as Francisco was then purportedly attacking
Ladiana and had, in fact, already inflicted a stab
wound on the arm of accused Ladiana. However,
Cortez emphasized that he was not the one who
conducted the preliminary investigation of the
complaint which led to the filing of the subject
case. Additionally, Cortez testified that he would
not be able to anymore recognize the face of the
affiant in the said counter-affidavit, but
maintained that there was a person who
appeared and identified himself as Ladiana before
he affixed his signature on the counter-affidavit.

TAU MU
TAU MU

LADIANA vs. PEOPLE


393 SCRA 419 (2002)

The Fraternal
Ateneo de Davao

TAU MU
TAU MU

Ruling: Yes. The right to counsel under Sec. 12


(1) of the Bill of Rights does not extend to a
person in a police line-up because that stage of
criminal law enforcement process is not yet part
of custodial investigation. Custodial investigation
commences when a person is taken into custody
and is singled out as a suspect in the commission
of the crime under investigation and the police
officers begin to ask questions on the suspects
participation therein which tend to elicit an
admission. A police line-up has been held to the
outside the mantle of protection of the right to
counsel because it involves a general inquiry into
an unsolved crime.

KITY

TAU MU TAU MU TAU MU


TAU MU TAU MU TAU MU

CONSTITUTIONAL LAW II
Order of Saint Thomas More
Atty. Philip John Pojas/Atty. Rovyne G. Jumao-as, RN
University College of Law

Contrary to what Ladiana claims that the


questioned statements were made during the
preliminary investigation, not during the custodial
investigation, hence, the right to competent and
independent
counsel
also
applies
during
preliminary investigations is untenable. A
preliminary investigation is an inquiry or a
proceeding to determine whether there is
sufficient ground to engender a well-founded
belief that a crime has been committed, and that
the respondent is probably guilty thereof and
should be held for trial.
A
person
undergoing
preliminary
investigation before the public prosecutor cannot
be considered as being under custodial
investigation.
In
fact,
the
Court
has
unequivocally declared that a defendant on trial
or under preliminary investigation is not under
custodial interrogation.
There is no question that even in the
absence of counsel; the admissions made by
petitioner in his Counter-Affidavit are not violative
of his constitutional rights. It is clear from the
undisputed facts that it was not exacted by the
police while he was under custody or
interrogation. Hence, the constitutional rights of
a person under custodial investigation as
embodied in Article III, Section 12 of the 1987
Constitution, are not at issue in this case.
Issue (2): WON the counter-affidavit filed by
petitioner
during
preliminary
investigation
constitutes as an extra-judicial admission.
Ruling: Petitioners counter-affidavit is not an
extrajudicial admission but only an admission. In
a confession, there is an acknowledgment of
guilt; in an admission, there is merely a
statement of fact not directly involving an
acknowledgment of guilt or of the criminal intent
to commit the offense with which one is charged.
RIGHT TO COUNSEL
The right to
independent counsel,
choice.

have competent
preferably of his

and
own

REQUISITES
FOR
COUNSEL
DURING
CUSTODIAL INVESTIGATION:
a.
The
counsel must be preferably of the own choice of
the person being investigated.
b. The counsel must be competent and
independent; and according to the SC
c. The counsel must be vigilant and
efective.
WHEN IS THE PERSON WHO IS BEING
INVESTIGATED NOT DEPRIVED OF HIS RIGHT
TO COUNSEL:
(1) When he personally has his own lawyer;
(2) When the person expressly agrees to
the lawyer chosen or given to him by the
investigator;
(3) When the person gives his implied
consent to the lawyer chosen by the police
investigator [PEOPLE Vs. PORIO and TIZON]

ACADCOM 2010; Contributors: Gene Geocaniga, Jarissa Guiani, Darlene Magabilen


TAU MU Page 133 of 179

Because if you do not object, you impliedly


consented to the lawyer given to you.
EXCEPTION: if the person had no opportunity to
object to the lawyer assigned to him because the
environment is intimidating. It cannot be
considered as implied consent.

TAU MU
TAU MU
TAU MU
TAU MU
TAU MU
TAU MU
TAU MU

(b) Any public officer or employee, or anyone


acting under his order or his place, who arrests,
detains or investigates any person for the
commission of an ofense shall inform the latter,
in a language known to and understood by him,

TAU MU

Section 2. Rights of Persons Arrested, Detained


or Under Custodial Investigation; Duties of Public
Officers. - (a) Any person arrested detained or
under custodial investigation shall at all times be
assisted by counsel.

TAU MU

REPUBLIC ACT No. 7438 -AN ACT DEFINING


CERTAIN RIGHTS OF PERSON ARRESTED,
DETAINED
OR
UNDER
CUSTODIAL
INVESTIGATION AS WELL AS THE DUTIES OF
THE
ARRESTING,
DETAINING
AND
INVESTIGATING OFFICERS, AND PROVIDING
PENALTIES FOR VIOLATIONS THEREOF

TAU MU
TAU MU

[PEOPLE vs. TOMAQUIN]: There was a violation of


the right to counsel when the accused confessed
to the barangay captain and there was no lawyer.
Although the barangay captain was a lawyer, he
was not considered independent.
WAIVER OF THE RIGHT TO COUNSEL DURING
INVESTIGATION
History of the waiver.
1.
1935 Constitution- no such thing.
2.
Under
the
1973
Constitution
protected the right to remain silent and the right
to counsel. However, the right to remain silent
can be waived just by talking. But the right to
counsel can be waived, there were no formalities.
The only requirement was that it was done
voluntarily and intelligently.
3.
1983:
Enrile
case:
By
jurisprudence, the SC held that the right to
counsel can be waived but only with the
assistance of counsel. There's no form, as
long as the counsel is present. You need to
have a lawyer to waive your right to a
lawyer during investigation
4.
1987: The right of counsel can be
waived but only with the assistance of counsel
and the waiver must be in writing.

The Fraternal
Ateneo de Davao

TAU MU
TAU MU

WHEN IS A PERSON BEING INVESIGATED


BEING
DEPRIVED
OF
HIS
RIGHT
TO
COUNSEL:
(1) If he was not asked if he has a counsel of
choice,
(2) And whether he could hire such counsel,
and if he could not,
(3) Whether he would agree to the one that
would be provided for him. [PEOPLE vs. AGUSTIN]
(4) When the lawyer is not vigilant and
efective;
(5) When the lawyer is not independent
[PEOPLE vs. CULALA]

KITY

TAU MU TAU MU TAU MU


TAU MU TAU MU TAU MU

CONSTITUTIONAL LAW II
Order of Saint Thomas More
Atty. Philip John Pojas/Atty. Rovyne G. Jumao-as, RN
University College of Law

of his rights to remain silent and to have


competent and independent counsel, preferably
of his own choice, who shall at all times be
allowed to confer privately with the person
arrested,
detained
or
under
custodial
investigation. If such person cannot aford the
services of his own counsel, he must be provided
with a competent and independent counsel by
the investigating officer.lawphi1
(c) The custodial investigation report shall be
reduced to writing by the investigating officer,
provided that before such report is signed, or
thumbmarked if the person arrested or detained
does not know how to read and write, it shall be
read and adequately explained to him by his
counsel or by the assisting counsel provided by
the investigating officer in the language or dialect
known to such arrested or detained person,
otherwise, such investigation report shall be null
and void and of no efect whatsoever.
(d) Any extrajudicial confession made by a
person
arrested,
detained
or
under
custodial investigation shall be in writing
and signed by such person in the presence
of his counsel or in the latter's absence,
upon a valid waiver, and in the presence of
any of the parents, elder brothers and
sisters, his spouse, the municipal mayor,
the
municipal
judge,
district
school
supervisor, or priest or minister of the
gospel as chosen by him; otherwise, such
extrajudicial
confession
shall
be
inadmissible as evidence in any proceeding.
(e) Any waiver by a person arrested or
detained under the provisions of Article 125
of the Revised Penal Code, or under
custodial investigation, shall be in writing
and signed by such person in the presence
of his counsel; otherwise the waiver shall
be null and void and of no effect.
(f) Any person arrested or detained or under
custodial investigation shall be allowed visits by
or conferences with any member of his
immediate family, or any medical doctor or priest
or religious minister chosen by him or by any
member of his immediate family or by his
counsel, or by any national non-governmental
organization duly accredited by the Commission
on Human Rights of by any international nongovernmental organization duly accredited by the
Office of the President. The person's "immediate
family" shall include his or her spouse, fianc,
parent or child, brother or sister, grandparent or
grandchild, uncle or aunt, nephew or niece, and
guardian or ward.
As used in this Act, "custodial investigation" shall
include the practice of issuing an "invitation" to a
person who is investigated in connection with an
ofense he is suspected to have committed,
without prejudice to the liability of the "inviting"
officer for any violation of law.

ACADCOM 2010; Contributors: Gene Geocaniga, Jarissa Guiani, Darlene Magabilen


TAU MU Page 134 of 179

In the absence of any lawyer, no custodial


investigation shall be conducted and the
suspected person can only be detained by the
investigating officer in accordance with the
provisions of Article 125 of the Revised Penal
Code.

TAU MU
TAU MU
TAU MU
TAU MU
TAU MU
TAU MU
TAU MU

Facts: The appellants were charged with 4


counts of rape with homicide. During trial, the
police officers invariably testified that they
informed the appellants of their rights and of the
consequences of their acts before their
statements were taken down. Atty. Guinalon
purportedly acted as counsel for all the suspects
at the time their statements were taken
down. Known to the four as a leader in the
community, Atty. Guinalon was requested to
assist in the execution of their extra-judicial
statements. The lawyer asked the suspects why
they requested him in particular. They replied
that they knew him and that he, in turn, knew all
of them. Atty. Guinalon conferred with the
suspects, who expressly signified their intention
to put into writing what happened that fateful
night. He explained to them that by making a
confession, they would be admitting to the
commission of a grave crime, which carried with
it a severe penalty. After Atty. Guinalon apprised
them of their constitutional rights, the four
proceeded
to
execute
their
respective
statements. Atty. Guinalon was in front of the
suspects when they gave their statements and
was present during the entire investigation.
Assistant Prosecutor Cardinal subscribed
the extra-judicial statements of the four
accused. He explained to the suspect the
consequences of his action, making sure that the
latter understood the contents of his statement.
The Assistant City Prosecutor told Tizon, Jr. that
the same could be used against him and that he
could be severely punished for his crime. Cardinal
also subscribed the extra-judicial confessions of
the other appellants. Prior to the signing of these
statements, he asked the suspects if the police

Issue (1): WON the rights of the accused under


Section 12 was violated by reason of the
extrajudicial declarations extracted from them.

TAU MU

PEOPLE vs. TIZON JR.


385 SCRA 364

threatened them or forced them to sign the


statements. They answered that they were not.
To avert any compulsion, the prosecutor even
asked the police officers to leave his cubicle
before asking the suspects any question. He also
examined their hands and bodies for any injuries
and asked them whether they were promised any
reward. He found no signs of injury on the
suspects, who categorically declared that they
were not threatened and that no reward was
promised them.
Assistant City Prosecutor
Cardinal then instructed them to examine every
page of the documents and to sign the
statements in the presence of their counsel

TAU MU

CASES: COUNSEL OF CHOICE

Ateneo de Davao

TAU MU
TAU MU

RA 7438 has extended the guaranty to


situations in which an individual has not been
formally arrested but has been merely invited
for questioning.
RA 7438 which expanded the provision in the
1987 Constitution by providing that: "If the
accused waives his right to counsel, the waiver
must be waived (1) with the assistance of
counsel, and (2) the waiver must be in writing
and signed in the presence of parents, brothers
or sisters, spouse, municipal mayor, municipal
judge, district or school supervisor, etc.

The Fraternal

TAU MU
TAU MU

Section 3. Assisting Counsel. - Assisting counsel


is any lawyer, except those directly afected by
the case, those charged with conducting
preliminary investigation or those charged with
the prosecution of crimes.

KITY

TAU MU TAU MU TAU MU


TAU MU TAU MU TAU MU

CONSTITUTIONAL LAW II
Order of Saint Thomas More
Atty. Philip John Pojas/Atty. Rovyne G. Jumao-as, RN
University College of Law

Ruling: The right to be informed of one's


constitutional rights during custodial investigation
refers to an efective communication between the
investigating officer and the suspected individual,
with the purpose of making the latter understand
these rights. Understanding would mean that
information transmitted was efectively received
and comprehended. Hence, the Constitution
does not merely require the investigating officers
to "inform" the person under investigation;
rather, it requires that the latter be informed.
Appellants' extra-judicial statements were
subscribed to before an assistant prosecutor who
testified that he asked the appellants if they were
forced by the police to sign their statements.
They declared that everything was of their own
free will. The prosecutor examined appellants
bodies to determine if they were harmed. The
prosecutor even told the police to get out of the
office to preclude any intimidation. Moved by
remorse, appellants said they were giving their
statements freely and voluntarily.
Issue (2): WON the appellants were not
accorded a competent and independent counsel
of their own choice as provided for under Section
12.
Ruling: Appellants deny that they sought the
services of Atty. Guinalon as their counsel during
the taking of their extra-judicial statements. If
Atty. Guinalon was not really appellants chosen
counsel, they could have requested for another
lawyer or voiced their objection. Appellants
never did but instead voluntarily executed their
extra-judicial statements. While the initial choice
of the lawyer in cases where a person under
custodial investigation cannot aford the services
of a lawyer is naturally lodged in the police
investigators, the accused really has the final
choice as he may reject the counsel chosen for
him and ask for another one. A lawyer provided
by the investigators is deemed engaged by the
accused where he never raised any objection
against the lawyers appointment during the
course of the investigation and the accused
thereafter subscribes to the veracity of his
statement before the swearing officer.

ACADCOM 2010; Contributors: Gene Geocaniga, Jarissa Guiani, Darlene Magabilen


TAU MU Page 135 of 179

TAU MU
TAU MU
TAU MU
TAU MU

Ruling: No. A Municipal Attorney cannot be an


independent counsel as required by the
Constitution.
As a legal officer of the
municipality, he provides legal assistance and
support to the mayor of the municipality in
carrying out the delivery of basic services to the
people, including the maintenance of peace and
order. It is therefore seriously doubted whether
he can efectively undertake the defense of the
accused without running into conflict of interest.
He is no better that a fiscal or prosecutor who
cannot represent the accused during custodial
investigations.

TAU MU

Facts: Accused was a suspect in a robbery with


homicide case. After his arrest, he executed an
extra-judicial
confession
admitting
the
commission of the crime with the assistance of
the Municipal Attorney of Valenzuela, Metro
Manila. The lawyer testified that he apprised
accused of his constitutional rights.
Is the
confession admissible?

TAU MU

PEOPLE VS. CULALA


316 SCRA 582 (1999)

TAU MU

Ruling: The accused has the final choice of the


lawyer as he may reject the counsel chosen for
him and ask for another one. However, a lawyer
provided by the investigator is deemed engaged
by the accused where, as in this case, he never
raised any objection against the officers
appointment during the course of investigation
and the accused thereafter subscribed to the
veracity of his statement before the swearing
officer.

TAU MU

Facts: Accused was convicted of murder. Among


the evidence presented during the trial was his
extra-judicial confession wherein he admitted the
crime. The document was signed in the presence
of Atty. Teofilo Tumulak and subscribed before the
municipal mayor. He now alleges that the lawyer
was not his choice.

TAU MU
TAU MU

PEOPLE VS. GONZALES


311 SCRA 551 (1999)

The Fraternal
Ateneo de Davao

TAU MU
TAU MU

Atty. Guinalon is a competent and independent


counsel, having been a public prosecutor, a
member of the Practicing Lawyer Association of
Negros Occidental, and of the Integrated Bar of
the Philippines and a civil leader in the
community, and the way he steadfastly and
painstakingly assisted the four accused during
the entire proceedings
in the custodial
investigation to safeguard their constitutional
rights and his attendance before Asst. City
Prosecutor Manuel Cardinal when the four
accused subscribed to their corresponding extrajudicial statements. He even signed these
statements to attest to his presence and the
regularity of the proceedings. Thus, the
appellants were accorded a competent and
independent counsel in the person of Atty.
Guinalon.

KITY

TAU MU TAU MU TAU MU


TAU MU TAU MU TAU MU

CONSTITUTIONAL LAW II
Order of Saint Thomas More
Atty. Philip John Pojas/Atty. Rovyne G. Jumao-as, RN
University College of Law

PEOPLE vs. VELARDE


384 SCRA 646 (2002)
Facts: Accused declared that he has been
detained for more than one year already because
he was told that he was the one who committed a
crime against his cousin. According to him, he
was arrested while selling balot by four Barangay
Officials. When they asked him where he brought
the victim, he told them he dont know. He did
not insist answering them because I dont know
what they were asking about the child. He just
went with them because if he will not go with
them di nila lulubayan and pamilya ko. He was
brought to the Barangay Hall. He was kicked and
mauled by the father and brothers of Brenda, the
victim. After hurting him inside the Barangay Hall
he was made to sign by one of the Barangay
Officials. He signed without reading what he
signed because he cannot read very well.
During trial, the accused was candid
enough to admit that the signature is his
signature; that Atty. Domingo is known to him
because he was then the Mayor of Malolos; that
he hired or engaged the services of Atty.
Domingo; that he was also candid enough to
testify that wala akong alam diyan. His
educational attainment was up to Grade four (4)
only. He claims that he does not know the police
investigator
who
typed
the
Sinumpaang
Salaysay.
Issue (1): WON the extrajudicial confession
taken during the investigation with the presence
of Atty. Domingo is inadmissible in evidence.
Ruling: Based on the facts of the case, upon the
advice of the Mayor, Atty. Domingo, Velardes
written extrajudicial confession was taken.
During the investigation, appellant was assisted
by the mayor as counsel. In this case, Atty.
Domingo
cannot
be
considered
as
an
independent counsel. He was the mayor of
Malolos at the time. As such, he exercised
operational supervision and control over the PNP
unit in that municipality. His powers included the
utilization of the elements thereof for the
maintenance of peace and order, the prevention
of crimes, the arrest of criminal ofenders and the
bringing of ofenders to justice.
Hence, the
extrajudicial confession taken from Velarde
cannot be used as evidence.
Issue (2): WON appellant was assisted by a
competent and independent counsel during the
custodial investigation.

TAU MU

Ruling: The right to counsel is a fundamental


right and contemplates not just the mere
presence of a lawyer beside the accused. The
competent and independent lawyer so engaged
should be present at all stages of the interview,
counseling or advising caution reasonably at
every turn of the investigation, and stopping the
interrogation once in a while either to give advice
to the accused that he may either continue,
choose to remain silent or terminate the
interview. The desired role of counsel in the

ACADCOM 2010; Contributors: Gene Geocaniga, Jarissa Guiani, Darlene Magabilen


TAU MU Page 136 of 179

TAU MU
TAU MU
TAU MU
TAU MU
TAU MU
TAU MU
TAU MU
TAU MU

Ruling: No. In providing that during the taking


of the extra-judicial confession the parents of the
accused, older brother and sisters, his spouse,
the municipal mayor, municipal judge, district
school supervisor, priest or minister of the gospel
may be present, RA No. 7438 does not suppose
that they appear as substitute for counsel. It
applies only when counsel of the accused is
absent and a valid waiver had been executed.
Hence, in this case, in the absence of such valid
waiver, the priest, mayor or relatives of the
accused could not stand in lieu of counsel. The
apparent consent of the accused in continuing
with the investigation was of no moment as a

TAU MU

Facts: Accused were suspects in a rape with


homicide case. They voluntarily went to the
police station to confess. Since there was no
lawyer to assist them, the police requested the
presence of the parish priest and the municipal
mayor as well as the relatives of the accused to
obviate the possibility of coercion.
Are their
confessions admissible in evidence?

TAU MU
TAU MU

PEOPLE vs. ORDONO


334 SCRA 673 (2000)

The Fraternal
Ateneo de Davao

TAU MU
TAU MU

process of custodial investigation is rendered


meaningless if the lawyer merely gives
perfunctory advice as opposed to a meaningful
advocacy of the rights of the person undergoing
questioning. If the advice given is so cursory as
to be useless, voluntariness is impaired.
As mayor of Malolos, his duties were
inconsistent with those of his responsibilities to
appellant, who was already incarcerated and
tagged as the main suspect in the rape-slay
case. Serving as counsel of appellant placed him
in direct conflict with his duty of operational
supervision and control over the police. What
the Constitution requires in Article III Section 12
(1) is the presence of competent and
independent counsel, one who will efectively
undertake his clients defense without any
intervening conflict of interest. Evidently Atty.
Domingo, being the mayor of the place where the
investigation was taken, could not act as counsel,
independent or otherwise, of appellant.
During the investigation, Atty. Domingo
failed to act as the independent and competent
counsel envisioned by the Constitution. He failed
to give any meaningful advice to protect the
rights of appellant. The former did not even
bother to inform the latter of the consequences of
an extrajudicial confession. It is significant to
point out that, during the cross-examination and
perhaps in total confusion, the investigator even
went so far as to state that Atty. Domingo had not
acted as appellants lawyer. If this were so, then
appellant had absolutely no counsel when his
extra-judicial confession was taken.
Thus, it is clear that, in palpable violation
of the Constitution, appellant was not assisted by
a competent and independent counsel during the
custodial investigation and the taking of his
extra-judicial confession. Hence, the Court is
duty-bound to disregard it.

KITY

TAU MU TAU MU TAU MU


TAU MU TAU MU TAU MU

CONSTITUTIONAL LAW II
Order of Saint Thomas More
Atty. Philip John Pojas/Atty. Rovyne G. Jumao-as, RN
University College of Law

waiver to be efective must be made in writing


and in the presence of counsel.
PEOPLE vs. TOMAQUIN
435 SCRA 23 (2004)
Facts: Appellant was convicted for the crime of
murder.
Issue: WON the extra-judicial confession
executed by appellant is admissible in evidence
against him.
Ruling: It is not legally sociable to consider a
brgy. captain an independent counsel of
appellant. What the Constitution requires is the
presence of an independent and competent
counsel, one who will efectively undertake his
clients defense without conflict of interest. The
appellant chose Atty. Parawan who did not stop
him from complaining about the latters failure to
safeguard his rights. Atty. Parawan failed to meet
the exacting standards of an independent and
competent counsel as required by Constitution.
The extra-judicial confession executed by
appellant, even if gospel truth, is deemed an
uncounselled
confession
and
therefore,
inadmissible in evidence.
PEOPLE vs. LABTAN
320 SCRA 140 (1999)
Facts: Accused was charged with highway
robbery and robbery with homicide. After trial, he
was convicted mainly based on his extra-judicial
confession. While he agreed to be assisted by a
lawyer given to him by the investigator, said
lawyer has been engaged by the police 3 times
and would sometimes be paid by the police for
his services. He was also the one who notarized
the confession. Is the confession admissible?
Ruling: No. The Constitution requires that the
counsel assisting the suspect must be competent
and independent.
The independence of the
lawyer in this case is suspect since he was
regularly engaged by the police as counsel for
suspects who cannot aford the services of a
lawyer and even received money from the police
as payment of his services. His act of notarizing
the statement also seriously compromised his
independence. By doing so, he vouched for the
regularity of the circumstances surrounding the
taking of the sworn statement by the police. He
cannot serve as counsel of the accused and the
police at the same time. There was a serious
conflict of interest on his part.
PEOPLE vs. OBERO
332 SCRA 190 (2000)
Facts: Accused was a suspect in a robbery with
homicide case.
He signified his intention to
confess but had no lawyer to assist him.
Coincidentally, the Station Commander of the
WPD Headquarters, UN Avenue, Manila, who was
a lawyer, happened to be at Station 7 of the WPD,

ACADCOM 2010; Contributors: Gene Geocaniga, Jarissa Guiani, Darlene Magabilen


TAU MU Page 137 of 179

so that the investigator requested him to assist


accused. After the lawyer apprised him of his
constitutional rights, accused confessed to the
commission of the crime.
Is the confession
admissible?

TAU MU
TAU MU
TAU MU
TAU MU
TAU MU
TAU MU

RIGHT TO BE INFORMED
This presupposes a transmission of
meaningful information, not just a ceremonial
recitation of an abstract principle.
Presupposes a transmission of meaningful
information, not just ceremonial recitation of an
abstract principle. The police must explain the
meaning of the provision in a language the
accused fairly understands. This will depend on
factors like, the educational background and the
intelligence of the accused.

TAU MU

Ruling: No. Even as the person under custodial


investigation enjoys the right to counsel from its
inception, so does he enjoy such right until its
termination indeed, in every phase of the
investigation. An efective and vigilant counsel
necessarily and logically requires that the lawyer
be present and able to advise and assist his client
from the time the confessant answers the first
question asked by the investigating officer until
the signing of the extra-judicial confession. The
lawyer in this case can hardly be the counsel that
the framers contemplated as competent.
Neither can he be described as vigilant and
efective which jurisprudence require.

TAU MU

Facts: Accused was convicted of robbery with


homicide.
At the start of the custodial
investigation he was assisted by a lawyer who
warned him of his rights and asked him if he is
willing to answer the questions of the police
investigator. During the investigation, the lawyer
would come and go within the police station.
Midway along, after all the material points have
been asked, the lawyer left for an important
engagement with the consent of accused. The
lawyer also instructed the police to bring the
accused and the document to him after the
written confession had been prepared for further
examination. Is the confession admissible?

TAU MU
TAU MU

PEOPLE vs. MORIAL


363 SCRA 96 (20001)

The Fraternal
Ateneo de Davao

TAU MU
TAU MU

Held: No. Accused was assisted by a lawyer,


who, though presumably competent, cannot be
considered
as
independent
counsel
as
contemplated by law for the reason that he was
station commander of the WPD. The lawyer was
part of the police force who could not be
expected to have efectively and scrupulously
assist accused in the investigation, his claim to
the contrary notwithstanding. To allow him to
assist in confessions would render illusory the
protection given to the suspect during custodial
investigation.

KITY

TAU MU TAU MU TAU MU


TAU MU TAU MU TAU MU

CONSTITUTIONAL LAW II
Order of Saint Thomas More
Atty. Philip John Pojas/Atty. Rovyne G. Jumao-as, RN
University College of Law

Q: Is the legal officer of a municipality qualified to


assist the accused?
A: He is not an independent counsel because as
legal officer, it is his function to assist the mayor
including the reinforcement of peace and order.
Obviously, there is conflict of interest.
CASES: TO BE INFORMED
PEOPLE vs. CANELA
Facts: Prior to the investigation, the police let the
accused read the rights. When the case reached
the SC, the accused raised the issue on whether
he was sufficiently informed in accordance with
the constitutional requirement.
Ruling: The court held that asking the accused
just to read his rights will not be sufficient under
the idea of meaningful transmission of
information. The accused should be made sure
to have understood his rights.
PEOPLE vs. NICANDRO
Facts: Police investigator was presented by the
fiscal. The fiscal asked about the investigation.
The policeman answered the first thing I did was
to inform the accused of his rights. And the next I
questioned her about the marijuana.
Issue: What is the meaning of meaningful
transmission of information?
Ruling: The court held that there was no
compliance with the requirement of the
Constitution. If this statement only appeared
during the direct examination, it is obvious that
the police did not inform her as to what these
specific rights are: This is a general statement.
The policeman, must explain to her in practical
terms which she can understand.
PEOPLE vs. KADIWA
Facts: The accused, upon being informed of his
rights was only made to answer one word ,
OPO.
Ruling: The court ruled there was no sufficient
compliance of the right to be informed. The
function was kilometric while the answer was
monosyllabic, which does not show that the
accused properly understood his rights. (NOTE:
There is no formula but merely depends on the
education, literacy, etc. of the accused)
PEOPLE vs. BANDULA
Facts: Two people were arrested and they were
immediately investigated without counsel. Next
day, some of the accused executed extra judicial
confessions assisted by counsel. Two weeks later,
the other one also made a confession.

TAU MU

Issue: Whether the confession of


accused be considered as admissible.

ACADCOM 2010; Contributors: Gene Geocaniga, Jarissa Guiani, Darlene Magabilen


TAU MU Page 138 of 179

the

two

Ruling: The court rules the confession as not


admissible. Confession made without counsel
although later reduced to writing with the
assistance of counsel is not admissible. Late
arrival of counsel will not cure the defect.
ADMISSIBILITY
OF
CONFESSIONS
1987 Constitution

JUDICIAL

RA
7438
(April
1992)
1. Must be in writing
under the situation
wherein the right to
counsel attaches.
2. Must be with
counsel

TAU MU
TAU MU
TAU MU
TAU MU

A mayor cannot be considered the independent


lawyer referred to by the Constitution. [PEOPLE
vs. TALIMAN, 342 SCRA 534]

TAU MU

A municipal attorney could not be an independent


counsel as required by the Constitution. We
reasoned that as legal officer of the municipality,
he provides legal assistance and support to the
mayor and the municipality in carrying out the
delivery of basic services to the people, including
the maintenance of peace and order. It is
therefore seriously doubted whether he can
efectively undertake the defense of the accused
without running into conflict of interests.
[PEOPLE vs. BANDULA, ]

TAU MU

The extrajudicial confession of the accusedappellant was inadmissible as he was assisted


by the incumbent municipal attorney. [PEOPLE
vs. CULALA]

TAU MU
TAU MU

3. With counsel in
writing -- admissible
with counsel, not in
writing
-not
admissible.
(When
is
it
admissible? When the
right does not attach)

The Fraternal
Ateneo de Davao

TAU MU
TAU MU

1. Need not be in
writing, but must be
done in the presence
of counsel
2. if extra judicial
confession made w/o
counsel - inadmissible,
because it has to be in
the
presence
of
counsel
3. If made in the
presence of counsel,
but not in writingvalid,
admissible
(because need not be
in writing. If the police
will testify that the
confession was made
in the presence of
counsel,
then
that
testimony
can
be
admitted in evidence)

EXTRA

KITY

TAU MU TAU MU TAU MU


TAU MU TAU MU TAU MU

CONSTITUTIONAL LAW II
Order of Saint Thomas More
Atty. Philip John Pojas/Atty. Rovyne G. Jumao-as, RN
University College of Law

WAIVER of the rights under Sec. 12 (1):


The law says that these rights cannot be
waived except in writing and in the presence of
counsel.

TAU MU

Q: What rights may be waived?


A: Only the right to remain silent and the right to
counsel may be waived. The right to be informed
of the said rights cannot be waived.

TAU MU

Q: How can the waiver be made?

A: The waiver must be:


1. In writing and
2. Done in the presence of counsel [Sec.
12 (1)]
3. The waiver must be in language w/c
clearly manifests the desire to waive the right
[PEOPLE vs. GALIT, 135 SCRA 465].
In localities where there are no lawyers, the
state must bring the individual to a place where
there is one or bring counsel to the place where
the person is held.

WAIVER
It is defined as the abandonment or
relinquishment of a right.
What can the accused waive?
- right to remain silent
- right to counsel
NOTE: The right to be informed cannot be waived
APPLICATION OF THE RIGHTS IN SECTION
12:
NOTE: Enactment of 1935 Constitution January
17, 1973
Moncupa ruling was promulgated April
26, 1983
Enactment of the 1987 Constitution
February 2, 1987
Enactment of RA 7438
Prior to January 17, 1973
- There was no right to counsel and
the right to remain silent under the 1935
Constitution. (Miranda vs State of Arizona was
only decided on 1956)
MAGTOTO vs. MANGERA
Facts: The accused made an extra judicial
confession prior to Jan. 17, 1973. He had not
been informed and had not availed of his right to
the assistance of counsel. He was tried after the
efectivity of the 1935 Constitution.
RULING: The court held the 1935 Constitution
had no retroactive efect. And the confession he
made was held admissible against him.

TAU MU

After January 17, 1973


- The right to remain silent and the
right to counsel is now protected and that those
rights may be waived, and there is no formality. It
can be waived in any manner, but must be
voluntary and intelligently made even without a
lawyer to assist the accused. (No specific form of
waiver).
From April 26, 1983
MONCUPA VS ENRILE
The right to counsel may only be waived
with the assistance of counsel. And it must also

ACADCOM 2010; Contributors: Gene Geocaniga, Jarissa Guiani, Darlene Magabilen


TAU MU Page 139 of 179

CONSTITUTIONAL LAW II
Order of Saint Thomas More
Atty. Philip John Pojas/Atty. Rovyne G. Jumao-as, RN
University College of Law

KITY

Ateneo de Davao

PEOPLE vs. DACUYCUY


Facts: Confession was obtained prior to April 26,
1983 and accused waived his right without the
assistance of a counsel. The issue is whether
there is a retroactive efect in this rule.
Ruling: The court held that this rule should be
applied even to confessions obtained prior to
April 26, 1983. This was given a retroactive
efect.

TAU MU
TAU MU
TAU MU
TAU MU
TAU MU

Under RA 7438, Sec. 2(1)


- There is the requirement of a form
in order for the waiver to be valid.
- After the lawyer signs the waiver,
he can already leave and need not be present
during the time when the confession is made.
- However, under RA 7438, after a
valid waiver, signed by the lawyer, it is now
required that the confession is signed by the
accused in the presence of other additional
people although they need not sign anymore. And
these are the either of the following:
- parents
- brothers
- sisters
- spouse
- mayor
- priest

TAU MU

NOTE: Under the 1987 Constitution, there is no


requirement that the confession should be in
writing. The requirement is only that the waiver
should be in writing.

TAU MU

TAU MU

According to Fr. Bernas, if asked in the BAR, just


cite the two sets of cases.

TAU MU

Prior to the Moncupa rule, it is unfair to expect


the police to follow the said rule since the
decision has not been promulgated. So why give
efect to it?
IMPORTANT: There are two sets of decisions in
this matter that do not reach a consensus one
agrees that there is retroactive efect and the
other agrees that it has none.

TAU MU
TAU MU

There was retroactive efect to the rule in


Moncupa. However, in People vs Magalona and
People vs Ponce, the Court stated that the rule
that waiver must be done with the assistance of
counsel has no retroactive efect. The rule on
Moncupa should only be used as a guideline after
the decision on that case.

TAU MU
TAU MU

PEOPLE vs. PECARDAL

TAU MU TAU MU TAU MU


TAU MU TAU MU TAU MU

be made in writing. Otherwise, the confession is


not admissible in court.

From February 2, 1987


- The waiver must be done:
- In the presence of
counsel
- It must be in writing

The Fraternal

- etc.
- In addition to the requirements of
the waiver to be with assistance of counsel and in
writing, the confession itself must be signed in
the presence of the other people (exclusively
enumerated in the law, but their signature is not
required.
- If there is no waiver, there is no
requirement that the other people should be
present, only his counsel.
OTHER CHANGES BROUGHT ABOUT BY
RA7438
Under RA 7438, it requires that the
Custodial Investigation report same with the
confession must be in writing, otherwise, such
report shall be void. Oral confessions are now
also void.
The list of visitors are now expanded
Section 2(f)
A penalty is provided for failure to inform
and to provide competent counsel to the
accused. (8 to 10 years of imprisonment).
It also defined as to who can assist the
accused as counsel Section 3
It also interpreted the meaning of whether
the police or investigator can change the lawyer
for the accused Section 8
Custodial investigation now includes
invitations.
SUMMARY OF RIGHTS GRANTED BY SECTION
12(1):
1. The right to remain silent: meaning he
has the right to refuse to answer any question
and his silence cannot be used against him.
2. The right to competent and independent
counsel preferably of his own choice: to preclude
the slightest coercion that would be used against
the accused to confess something
3. The right to be informed of these rights: it
is not enough that the police officer will just
repeat to the person under investigation the
constitutional provision, he must also explain the
efects in practical terms. It contemplates an
efective communication that results in the
understanding of the rights contemplated.
4. (Right added by 7438): Visitorial rights:
The right to be visited by family, friends, lawyers,
doctors, etc. The person being investigated can
be visited.
SECTION 12 (2)
This provision contains the right against
coerced confessions.
SECTION 12 (3)
The exclusionary rule. The law says Any
confession or admission obtained in violation of
this or Sec. 17 hereof shall be inadmissible in
evidence against him.
Confessions made in violation of Sec. 12 are
inadmissible in evidence. How do you
understand "made in violation of Sec. 12"?
a. If the confession is done without the
lawyer; (uncounseled confession)

ACADCOM 2010; Contributors: Gene Geocaniga, Jarissa Guiani, Darlene Magabilen


TAU MU Page 140 of 179

b. When the waiver is not valid (when


without presence of lawyer, and not in writing)
c. Confession is gathered through force,
intimidation, etc. (sec. 12 paragraph 2)
d. No meaningful transmission of right.
e. Oral confession (in violation of RA
1348), because confessions have to be in writing.
OTHER
CONFESSIONS
WHICH
ARE
EXCLUDED: (even with counsel or if such
was true)

TAU MU
TAU MU

According to the Court, there is no


presumption that the police have given the
Miranda warning to the accused. During the trial,

TAU MU

PEOPLE vs. TOLENTINO

TAU MU

Miranda

TAU MU

1.
No presumptions that the
Warning has been given. (General Rule)

TAU MU

PRESUMPTIONS
Presumptions are Assumptions of Fact which
are made by law in order to dispense with
evidence.

TAU MU

EXCEPTIONS:
(A) To prove that there is a conspiracy.
(B) To be used as evidence against police
abuse.
(C) To
impeach
the
accused
persons
credibility.

TAU MU

SCOPE OF INADMISSIBILITY:
(1) They are inadmissible against the
confessant.
(2) They are inadmissible for the
purpose of any proceeding: (criminal, civil and
administrative proceedings). However, if a
confession is made in an administrative
proceeding, then that is admissible.
(3) When the confession was made
and where the confession was made are diferent.
You have to look into both.
(4) The confession is inadmissible
against third persons.

TAU MU

1. Principle of Humanity the law abhors all


forms of violence inflicted upon another.
2. Also, these confessions are generally
unreliable.

TAU MU
TAU MU

REASONS WHY CONFESSIONS OBTAINED


THROUGH
DURESS
OR
TORTURE
ARE
INADMISSIBLE:

The Fraternal
Ateneo de Davao

TAU MU
TAU MU

1. Where the accused is promised that he would


be released if he signs. (People vs Eglipa)
2. When the accused is promised a better
treatment if he signs. (People vs Albano, 145 S
155)
3. All oral confessions are inadmissible. It should
always be in writing to be admissible as evidence.
4. Confessions obtained after a valid waiver, but
not signed in the presence of person enumerated
in RA 7438
5. All other means which vitiate the persons
will.

KITY

TAU MU TAU MU TAU MU


TAU MU TAU MU TAU MU

CONSTITUTIONAL LAW II
Order of Saint Thomas More
Atty. Philip John Pojas/Atty. Rovyne G. Jumao-as, RN
University College of Law

the prosecution must prove that the police have


warned the accused of his rights under Section 12
in the absence of proof. It is presumed that there
was no warning given and the evidence is
inadmissible. There is no presumption of
regularity of official acts under the Miranda Rule.
2.

No presumption on the validity of a waiver.

PEOPLE vs. JARA


Whenever there is a waiver of constitutional
rights, then presumption is always against the
waiver. The presumption is that there was no
waiver given. If the prosecution claims there was
a waiver. They must prove with strong and
convincing evidence to the satisfaction of the
Court. Also, there is no presumption of the
regularity of official acts in this case.
What must the prosecution prove?
The presumption is that there was
no valid waiver except if the prosecution can
prove:
That the accused willingly and
voluntarily made his confession
That
the
accused
was
not
interested in having a lawyer
That the waiver complied with all
the requirements to make a valid one
There was a valid Miranda Warning
Mere presentation of the text of the waiver is not
sufficient to prove the above- mentioned.
3.
There
now
the
presumption
Voluntariness of the Confession.

of

PEOPLE vs. ENANORIA


An opposite presumption. Atty. Jocon was present
to assist the accused in giving his statement.
There is now a presumption on voluntariness.
There is now a presumption of the regularity of
official acts. This means that there is a
presumption that the police regularly perform and
comply with proper procedures, and that they do
not have to torture the accused to get the
confession. It is presumed that the confession
was voluntarily made. The police do not have to
prove that they did not torture the accused in
order to get the confession. The law does not
presume that the police are torturers. The burden
is now on the accused to prove that he was
tortured and the confession was involuntarily
made.
NOTE: There was strong evidence in this case
that the accused gave his testimony uncoerced
and freely.
PEOPLE VS BALLISTEROS
An obiter dictum. One accused signed without the
assistance of a lawyer. Later, during the trial, he
was made a State witness against his co-accused.
He repeated his confession in open court. His
confession was considered admissible as
evidence.

ACADCOM 2010; Contributors: Gene Geocaniga, Jarissa Guiani, Darlene Magabilen


TAU MU Page 141 of 179

The rule stands that any confession obtained with


violation of the Constitution shall be held
inadmissible as evidence in Court. This is based
on the 1987 Constitution. His confession shall be
inadmissible in Court only if this is used against
him, but this does not include confessions to be
used as evidence against third persons. (A
dangerous ruling is made. This should not be
considered as a new string.)

CASES: EXCLUSIONARY RULE

TAU MU
TAU MU

Ruling: The alleged extra-judicial confession of


Janson cannot be admitted in evidence. The
manner which it was obtained violated accused
constitutional right to counsel. Even if the
confession contains a grain of truth, it was made
without the assistance of counsel, it becomes
inadmissible in evidence, regardless of the
absence of coercion or even if it had been
voluntarily given. Assistance of counsel is

TAU MU

Issue: WON the alleged extra-judicial confession


of Janson is inadmissible because it was obtained
in violation of the constitutional rights of the
accused.

TAU MU

Facts: The RTC declared the appellants guilty for


the crime of robbery with rape. Jansons
statement was signed by him without force. No
lawyer was present during the custodial
investigation. The affidavit was signed by Atty.
Zerudo after the investigation was conducted.

TAU MU

PEOPLE vs. JANSON


400 SCRA 584 (2003)

TAU MU

Q: Are they admissible against the persons who


obtained the confession/admission?
A: YES. They may be used as evidence against
the person violating the constitutional prohibition.

TAU MU

It is the person alone who made the


confession/admission who can ask for its
exclusion [PEOPLE vs. BALISTEROS, 237 SCRA
499].

TAU MU

ADMISSION
is
the
act,
declaration
or
omission of a
party as to a
relevant
fact
[Rule 130 Sec.
26]

TAU MU

CONFESSION

the declaration of an
accused acknowledging
his guilt of the ofense
charged, or of any
ofense
necessarily
included therein [Rule
130 Sec. 33]

TAU MU
TAU MU

Q: Against whom does the confession/admission


be inadmissible as evidence?
A: Against the person making the confession or
admission.

The Fraternal
Ateneo de Davao

TAU MU
TAU MU

PRESUMPTIONS:
1. There is no presumption that Miranda warning
has been given;
2. No presumption that the waiver is valid;
3. Presumption that the confession is voluntarily
made;

KITY

TAU MU TAU MU TAU MU


TAU MU TAU MU TAU MU

CONSTITUTIONAL LAW II
Order of Saint Thomas More
Atty. Philip John Pojas/Atty. Rovyne G. Jumao-as, RN
University College of Law

required even when he waives the right to


counsel. A confession, to be admissible, must
satisfy the following requirement: (1) it must be
voluntary, (2) it must be made with the
assistance of an independent and competent
counsel, (3) it must be express and (4) it must be
in writing. The purpose is to curb the uncivilized
practice of extracting confession by coercion no
matter how slight, as would lead the accused to
admit something false. Any waiver of the right of
counsel without the assistance of counsel has no
evidentiary rule. In this case, it cannot be said
that the waiver of the right to counsel was made
knowingly and intelligently. Admissions obtained
during custodial investigation without the benefit
of counsel, although later reduced in writing and
signed in the presence of counsel, are still flawed
under the Constitution. If the lawyers role is
reduced to being that of a mere witness to the
signing of a priority prepared document albeit
indicating thereon compliance with the accused
constitutional rights, the constitutional standard
is not met.
MARCELO vs. SANDIGANBAYAN
302 SCRA 102 (1999)
Facts: Accused was an emergency laborer of
Makati Central Post Office.
He was caught
stealing 622 letters some of them consisting US
dollar bills. After his arrest, he was asked to affix
his signatures on the envelopes of the letters in
the presence of National Bureau of Investigation
(NBI) agents but without the assistance of
counsel. Are the letters admissible in evidence?
Ruling: Accused signed the envelopes following
his arrest. Hence, he was at that time under
custodial investigation. Under the Constitution,
at this stage he had the right to counsel. Since
the signatures are actually evidence of admission
obtained
from
accused
under
custodial
investigation, they should be excluded. However,
the letters are themselves not inadmissible. The
letters were validly seized from accused as an
incident to a valid arrest.
A ruling that his
admission that the letters in question were those
seized from him does not extend to the letter
themselves.
SECTION 12 (4)
The way the law is worded, it seems that it will
need implementing legislation.

SEC. 13 BAIL All persons, except those


charged with offenses punishable by
Reclusion Perpetua when evidence of guilt
is strong, shall, before conviction, be
bailable by sufficient sureties, or be
released on recognizance as my be provided
by law. The right to bail shall not be
impaired even when the privilege of the
writ of habeas corpus is suspended.
Excessive bail shall not be required.

ACADCOM 2010; Contributors: Gene Geocaniga, Jarissa Guiani, Darlene Magabilen


TAU MU Page 142 of 179

BAIL

is a mode short of confinement w/c would, w/


reasonable certainty, insure the attendance of
the accused [DE LA CAMARA vs. ENAGE, 41 SCRA
1]

It is the security given for the provisional


release of a person accused of a crime while the
case is still pending or trial is still going on.

TAU MU
TAU MU
TAU MU
TAU MU
TAU MU
TAU MU

Sec. 5. Bail, when discretionary. Upon conviction by


the Regional Trial Court of an ofense not punishable
by death, reclusion perpetua, or life imprisonment,
admission to bail is discretionary. The application for
bail may be filed and acted upon by the trial court
despite the filing of a notice of appeal, provided it
has not transmitted the original record to the
appellate court. However, if the decision of the trial
court conviction the accused changed the nature of
the ofense from non-bailable to bailable, the
application for bail can only be filed with and

TAU MU

Sec. 4. Bail, a matter of right; exception. All


persons in custody shall be admitted to bail as a
matter of right, with sufficient sureties, or released
on recognizance as prescribed by law or this Rule (a)
before or after conviction by the Metropolitan Trial
Court, Municipal Trial Court, Municipal Trial Court in
Cities, or Municipal Circuit Trial Court, and (b) before
conviction by the Regional Trial court of an ofense
not punishable by death, reclusion perpetua, or life
imprisonment.

TAU MU

Sec. 3. No release or transfer except on court order


or bail. No person under detention by legal process
shall be released or transferred except upon order of
the court or when he is admitted to bail.

TAU MU
TAU MU

Sec. 2. Conditions of the bail; requirements. All


kinds of bail are subject to the following conditions:
(a) The undertaking shall be efective upon
approval, and unless cancelled, shall remain in
force at all stages of the case until
promulgation of the judgment of the Regional
Trial Court, irrespective of whether the case
was originally filed in or appealed to it;
(b) The accused shall appear before the proper
court whenever required by the court of these
Rules;
(c) The failure of the accused to appear at the
trial without justification and despite due
notice shall be deemed a waiver of his right to
be present thereat. In such case, the trial may
proceed in absentia; and
(d) The bondsman shall surrender the accused to
the court for execution of the final judgment.
The original papers shall state the full name and
address of the accused, the amount of the
undertaking and the conditions required by this
section. Photographs (passport size) taken within the
last six (6) months showing the face, left and right
profiles of the accused must be attached to the bail.

The Fraternal
Ateneo de Davao

TAU MU
TAU MU

RULE 114 BAIL


Section 1. Bail defined. Bail is the security given
for the release of a person in custody of the law,
furnished by him or a bondsman, to guarantee his
appearance before any court as required under the
conditions hereinafter specified. Bail may be given in
the form of corporate surety, property bond, cash
deposit or recognizance.

KITY

TAU MU TAU MU TAU MU


TAU MU TAU MU TAU MU

CONSTITUTIONAL LAW II
Order of Saint Thomas More
Atty. Philip John Pojas/Atty. Rovyne G. Jumao-as, RN
University College of Law

If the penalty imposed by the trial court is


imprisonment exceeding six (6) years, the accused
shall be denied bail, or his bail shall be cancelled
upon a showing by the prosecution, with notice to
the accuse, of the following or other similar
circumstances:
(a) That he is a recidivist, quasi-recidivist, or
habitual delinquent, or has committed the
crime aggravated by the circumstance of
reiteration;
(b) That he has previously escaped from legal
confinement, evaded sentence, or violated the
conditions of his bail without valid justification;
(c) That he committed the ofense while under
probation, parole, or conditional pardon;
(d) That the circumstances of his case indicate the
probability of flight if released on bail; or
(e) That there is undue risk that he may commit
another crime during the pendency of the
appeal.
The appellate court may, motu proprio or on motion
of any party, review the resolution of the Regional
Trial Court after notice to the adverse party in either
case.
Sec. 6. Capital offense defined. A capital ofense is
an ofense which, under the law existing at the time
of its commission and of the application for
admission to bail, may be punished with death.
Sec. 7. Capital offense or an offense punishable by
reclusion perpetua or life imprisonment, not bailable.
No person charged with a capital ofense, or an
ofense punishable by reclusion perpetua or life
imprisonment, shall be admitted to bail when
evidence of guilt is strong, regardless of the state of
the criminal prosecution.
Sec. 8. Burden of proof in bail application. At the
hearing of an application for bail filed by a person
who is in custody for the commission of an ofense
punishable by death, reclusion perpetua, or life
imprisonment, the prosecution has the burden of
showing that evidence of guilt is strong. The
evidence presented during the bail hearing shall be
considered automatically reproduced at the trial but,
upon motion of either party, the court may recall any
witness for additional examination unless the latter is
dead, outside the Philippines, or otherwise unable to
testify.
Sec. 9. Amount of bail; guidelines. The judge who
issued the warrant or granted the application shall fix
a reasonable amount of bail considering primarily,
but not limited to, the following factors:
(a) Financial liability of the accused to give bail;
(b) Nature and circumstance of the ofense;
(c) Penalty for the ofense charged;
(d) Character and reputation of the accused;
(e) Age and health of the accused;
(f) Weight of the evidence against the accused;
(g) Probability of the accused appearing at the
trial;
(h) Forfeiture of other bail;
(i) The fact that the accused was a fugitive from
justice when arrested; and
(j) Pendency of other cases where the accused is
on bail.
Excessive bail shall not be required.

TAU MU

Sec. 10. Corporate surety. Any domestic or foreign


corporation, licensed as a surety in accordance with
law and
currently
authorized to act as such, may
ACADCOM 2010; Contributors: Gene Geocaniga, Jarissa Guiani,
Darlene
Magabilen
provide
bail
by
a
bond subscribed jointly by the
TAU MU Page 143 of 179
accused and an officer of the corporation duly
authorized by its board of directors.

tax declaration in the office of the provincial, city and


municipal assessor concerned.
Within the same period, the accused shall submit to
the court his compliance and his failure to do so shall
be sufficient cause for the cancellation of the property
bond and his re-arrest and detention.

TAU MU
TAU MU
TAU MU
TAU MU
TAU MU
TAU MU

When a person has been in custody for a period equal


to or more than the possible maximum imprisonment
prescribed for the ofense charged, he shall be
released immediately, without prejudice to the
continuation of the trial or the proceedings on appeal.
If the maximum penalty to which the accused may be
sentenced is destierro, he shall be released after
thirty (30) days of preventive imprisonment.

TAU MU

Sec. 16. Bail, when not required; reduced bail or


recognizance. No bail shall be required when the
law or these Rules so provide.

TAU MU

Sec. 15. Recognizance. Whenever allowed by law or


these Rules, the court may release a person in
custody on his own recognizance or that of a
responsible person.

TAU MU

Sec. 14. Deposit of cash as bail. The accused or any


person acting in his behalf may deposit in cash with
the nearest collector of internal revenue or provincial,
city, or municipal treasurer the amount of bail fixed
by the court, or recommended by the prosecutor who
investigated or filed the case. Upon submission of a
proper certificate of deposit and a written
undertaking
showing
compliance
with
the
requirements of section 2 of this Rule, the accused
shall be discharged from custody. The money
deposited shall be considered as bail and applied to
the payment of fine and costs while the excess, if
any, shall be returned to the accused or to whoever
made the deposit.

TAU MU
TAU MU

Sec. 13. Justification of sureties. Every surety shall


justify by affidavit taken before the judge that he
possesses the qualification prescribed in the
preceding section. He shall describe the property
given as security, stating the nature of his title, its
encumbrances, the number and amount of other bails
entered into by him and still undischarged, and his
other liabilities. The court may examine the sureties
upon oath concerning their sufficiency in such
manner as it may deem proper. No bail shall be
approved unless the surety is qualified.

The Fraternal
Ateneo de Davao

TAU MU
TAU MU

Sec. 12. Qualifications of sureties in property bond.


The qualifications of sureties in a property bond shall
be as follows:
(a) Each must be a resident owner of real estate
within the Philippines;
(b) Where there is only one surety, his real estate
must be worth at least the amount of
undertaking;
(c) If there are two or more sureties, each may
justify in an amount less than that expressed in
the undertaking but the aggregate of the
justified sums must be equivalent to the whole
amount of the bail demanded.
In all cases, every surety must be worth the amount
specified in his own undertaking over and above all
just debts, obligations and properties exempt from
execution.

KITY

TAU MU TAU MU TAU MU


TAU MU TAU MU TAU MU

CONSTITUTIONAL LAW II
Order of Saint Thomas More
Atty. Philip John Pojas/Atty. Rovyne G. Jumao-as, RN
University College of Law

trial judge in the province, city or municipality. If


the accused is arrested in a province, city, or
municipality other than where the case is pending,
bail may also be filed with any regional trial court of
said place, of if no judge thereof is available, with
any metropolitan trial judge, municipal trial judge,
or municipal circuit trial judge therein.
(b) Where the grant of bail is a matter of discretion,
or the accused seeks to be released on
recognizance, the application may only be filed in
the court where the case is pending, whether on
preliminary investigation, trial, or appeal.
Any person in custody who is not yet charged in
court may apply for bail with any court in the
province, city, or municipality where he is held.
Sec. 18. Notice of application to prosecutor. In the
application for bail under section 8 of this Rule, the
court must give reasonable notice of the hearing to
the prosecutor or require him to submit his
recommendation.
Sec. 19. Release on bail. The accused must be
discharged upon approval of the bail by the judge
with whom it was filed in accordance with section
17 of this Rule.
When bail is filed with a court other than where the
case is pending, the judge who accepted the bail
shall forward it, together with the order of release
and other supporting papers, to the court where the
case is pending, which may, for good reason,
require a diferent one to be filed.
Sec. 20. Increase or reduction of bail. After the
accused is admitted to bail, the court may, upon
good cause, either increase or reduce its amount.
When increased, the accused may be committed to
custody if he does not give bail in the increased
amount within a reasonable period. An accused
held to answer a criminal charge, who is released
without bail upon filing of the complaint or
information, may, at any subsequent stage of the
proceedings and whenever a strong showing of
guilt appears to the court, be required to give bail
in the amount fixed, or in lieu thereof, committed to
custody.
Sec. 21. Forfeiture of bail. When the presence of
the accused is required by the court or these Rules,
his bondsmen shall be notified to produce him
before the court on a given date and time. If the
accused fails to appear in person as required, his
bail shall be declared forfeited and the bondsmen
given thirty (30) days within which to produce their
principal and to show why no judgment should be
rendered against them for the amount of their bail.
Within the said period, the bondsmen must:
(a) produce the body of their principal or give
the reason for his non-production; and
(b) explain why the accused did not appear
before the court when first required to do so.
Failing in these two requisites, a judgment shall be
rendered against the bondsmen, jointly and
severally, for the amount of the bail. The court shall
not reduce or otherwise mitigate the liability of the
bondsmen, unless the accused has been
surrendered or is acquitted.

Sec. 22. Cancellation of bail. Upon application of


A person in custody for a period equal to or more
the bondsmen, with due notice to the prosecutor,
ACADCOM
2010; Contributors:
Geocaniga,
Darlene
Magabilen
than
the minimum
of the principalGene
penalty
prescribed Jarissa Guiani,
the bail
may be
cancelled upon surrender of the
TAU
MU
Page
144
of
179
for the ofense charged, without application of the
accused or proof of his death.
Indeterminate Sentence Law or any modifying
circumstance, shall be released on a reduced bail or
The bail shall be deemed automatically cancelled
on his own recognizance, at the discretion of the
upon acquittal of the accused, dismissal of the

KITY

undertaking, cause him to be arrested by a police


officer or any other person of suitable age and
discretion.

TAU MU
TAU MU

An accused released on bail may be re-arrested


without the necessity of a warrant if he attempts to
depart from the Philippines without permission of the
court where the case is pending.

TAU MU
TAU MU

Sec. 24. No bail after final judgment; exception. No


bail shall be allowed after a judgment of conviction
has become final. If before such finality, the accused
applies for probation, he may be allowed temporary
liberty under his bail. When no bail was filed or the
accused is incapable of filing one, the court may
allow his release on recognizance to the custody of a
responsible member of the community. In no case
shall bail be allowed after the accused has
commenced to serve sentence.

1. that the accused shall appear before the


proper court whenever required by the court or
by the Rules
2. absence without justification is a waiver of
his right to be present in the hearing
WHO ARE ENTITLED TO BAIL:
1) All persons ACTUALLY DETAINED
2) shall, BEFORE CONVICTION
3) Be entitled to bail.
WHO ARE NOT ENTITLED TO BAIL:
1) Persons charged with ofenses PUNISHABLE
by RECLUSION PERPETUA or DEATH, when
evidence of guilt is strong
2) Persons CONVICTED by the trial court. Bail is
only discretionary pending appeal.
3) Persons who are members of the AFP facing a
court martial.
OTHER RIGHTS IN RELATION TO BAIL:
1) The right to bail shall NOT be impaired even
when the privilege of the writ of habeas
corpus is suspended.
2) Excessive bail shall not be required.
As soon as one is placed under the
custody of law, the accused that was confined in
the hospital was allowed to post bail. The reason
is he expressly submitted to physical and legal
control of the court over his person by filing of
application for bail, furnishing the court of his
whereabouts and unequivocally recognizing the
jurisdiction of the court. [PADERANGA vs. CATheory of Constructive Custody]
WHEN IS BAIL A MATTER OF RIGHT:
1. Before conviction in the MTC
2. All appealed cases from the MTC to the RTC
and to the CA or the SC, but only when original
court is MTC
3. Original cases decided by the RTC when the
imposable penalty is less than RP
4. Original cases decided by the RTC when the
penalty is RP or above but the evidence of guilt is
not strong.

TAU MU

IMPLICIT LIMITATIONS ON THE RIGHT TO


BAIL:
1. The person claiming the right must be in
actual detention or custody of the law.
2. The constitutional right is available only in
criminal
cases, not,
e.g. in deportation
proceedings.

TAU MU

Note:
1. Right to bail is not available in the military.
2. Apart from bail, a person may attain
provisional liberty through recognizance.

TAU MU

TAU MU

DUTIES IMPOSED ON THE JUDGE WHEN AN


APPLICATION FOR BAIL IS FILED:

TAU MU

CONDITIONS OF BAIL

TAU MU

Sec. 26. Bail not a bar to objections on illegal arrest,


lack of or irregular preliminary investigation. An
application for or admission to bail shall not bar the
accused from challenging the validity of his arrest or
the legality of the warrant issued therefore, or from

TAU MU

A monthly report of such visitation shall be submitted


by the executive judges to the Court Administrator
which shall state the total number of detainees, the
names of those held for more than thirty (30) days,
the duration of detention, the crime charged, the
status of the case, the cause for detention, and other
pertinent information.

TAU MU

In cities and municipalities to be specified by the


Supreme Court, the municipal trial judges or
municipal circuit trial judges shall conduct monthly
personal inspections of the municipal jails in their
respective municipalities and submit a report to the
executive judge of the Regional Trial Court having
jurisdiction therein.

TAU MU

Sec. 25. Court supervision of detainees. The court


shall exercise supervision over all persons in custody
for the purpose of eliminating unnecessary detention.
The executive judges of the Regional Trial Courts shall
conduct monthly personal inspections of provincial,
city, and municipal jails and the prisoners within their
respective jurisdictions. They shall ascertain the
number of detainees, inquire on their proper
accommodation and health and examine the
condition of the jail facilities. They shall order the
segregation of sexes and of minors from adults,
ensure the observance of the right of detainees to
confer privately with counsel, and strive to eliminate
conditions inimical to the detainees.

PURPOSE OF BAIL
To recognize the presumption of innocence
accorded to every accused upon whom should
not be inflicted incarceration at the outset since
after trial he would be entitled to acquittal, unless
his guilt be established beyond reasonable doubt.

The Fraternal
Ateneo de Davao

TAU MU TAU MU TAU MU


TAU MU TAU MU TAU MU

CONSTITUTIONAL LAW II
Order of Saint Thomas More
Atty. Philip John Pojas/Atty. Rovyne G. Jumao-as, RN
University College of Law

1. notify the prosecutor of the hearing of the


application for bail and require him to submit his
recommendation
2. conduct a hearing (mandatory!) of the
application regardless of whether the prosecution

ACADCOM 2010; Contributors: Gene Geocaniga, Jarissa Guiani, Darlene Magabilen


TAU MU Page 145 of 179

presents evidence or not that the guilt of the


accused is strong;
3. decide whether or not the evidence of
accuseds guilt is strong based on the summary
of prosecutions evidence;
4. if accuseds guilt is NOT strong, discharge
him upon approval of the bailbond.

TAU MU
TAU MU
TAU MU
TAU MU
TAU MU
TAU MU

BAIL SHOULD BE
DENIED in Rule 114

TAU MU

before conviction
by the RTC on an

TAU MU

Bail is a MATTER
Bail is a MATTER OF
OF RIGHT
DISCRETION

all
persons persons charged w/
charged
w/
an ofenses punishable by
ofenses
NOT RP when evidence of
punishable
by
RP guilt is strong [Sec. 13]
[Sec. 13]

upon conviction by

before or after the RTC of an ofense


conviction
by
the NOT
punishable
by
MTC, MTCC, MeTC or death,
RP,
or
life
MCTC [Rule 114 Sec. imprisonment [Rule 114
4]
Sec. 5]

TAU MU

BAIL AS A MATTER OF DISCRETION:


1. Cases decided by the RTC, appealed to the
CA/SC and the circumstances of AO 12-94 is not
present.

TAU MU
TAU MU

WHEN SHOULD BAIL BE DENIED:


1. After final judgment by any court
2. Before
conviction
for
an
ofense
punishable by death or reclusion perpetua where
the evidence of guilt is strong.
3. After conviction for a crime punishable by
reclusion perpetua or death while the case is on
appeal [PEOPLE vs. VALERIANO]
4. After conviction for an ofense with the
penalty exceeding 6 years but not more than 20
years, if:
a. accused is a recidivist, quasirecidivist, habitual delinquent or has
committed a crime aggravated by
reiteracion
b. accused is found to have
previously
escaped
from
legal
confinement
c. accused committed the ofense
while on probation, parole or conditional
pardon
d. circumstances of accused or his
case indicate the probability of flight
e. there is undue risk that during the
pendency of the appeal, accused may
commit another crime. (SC Administrative
Circular No. 12-94)
This includes punishment of life imprisonment.

The Fraternal
Ateneo de Davao

TAU MU
TAU MU

The court order granting/refusing bail should


contain a summary of the prosecutions evidence
followed by the conclusion on whether or not the
evidence of guilt is strong.
The court cannot merely rely on the affidavits
and recitals of its contents, if timely objected to,
because they are only hearsay --- insufficient to
establish quantum of proof needed w/c is strong
evidence.

KITY

TAU MU TAU MU TAU MU


TAU MU TAU MU TAU MU

CONSTITUTIONAL LAW II
Order of Saint Thomas More
Atty. Philip John Pojas/Atty. Rovyne G. Jumao-as, RN
University College of Law

ofense
NOT
punishable by death,
RP,
or
life
imprisonment
[Rule
114 Sec. 4]

Sec. 5 par. 3

WHERE SHOULD BAIL BE FILED (SEC 17)


General rule:
in the court where the case
pending
Exceptions:
0
(1) Where the judge of the court where the
case is pending is absent or unavailable, bail may
be filed to any RTC or MTC of the province, city or
municipality where the case pending.
1
(2) Where the accused is arrested in a
province, city, or municipality other than where
the case is pending, bail may be filed in any RTC
or MTC of that place
2
(3) Any person in custody who is not yet
charged, in the RTC or MTC of the province, city or
municipality where he is held.
Mitigating circumstances are not yet
considered to determine the penalty because in
the end, it will only result in a full blown trial.
Although mitigating circumstances in general are
not considered yet. The mitigating circumstance
of MINORITY may be considered in granting bail.
AC # 12-94 is constitutional because it only
limits the right to bail after conviction while the
case is on appeal. The right to bail is only
guaranteed by the Constitution before conviction,
not after conviction while the case is on appeal.
The Constitution does not distinguish whether
before final decision or before appeal.
WHEN BAIL NOT REQUIRED
1. When a person has been in custody for a
period equal to or more than the maximum
imposable penalty for the ofense charged.
2. After 30 days of preventive imprisonment if
the maximum penalty for the ofense is destierro.
BAIL IN MILITARY COURTS
Bail has traditionally not been recognized
and is not available in the military as an
exception to the general rule embodied in the Bill
of Rights. The unique structure of the military
should be enough reason to exempt military men
from the constitutional coverage of the right to
bail. Military are allowed the fiduciary use of
firearms which could very well result in the
overthrow of duly constituted authorities.
POLICY
ARGUMENTS
(NOT
LEGAL
ARGUMENTS)
WHICH
COULD
BE
CONSIDERED IMPORTANT:
1. Military officers are diferent from ordinary
criminals because they are entrusted with trust
and confidence of the people.
2. It would also be dangerous if there are
armed coup plotters walking in the streets
because they are allowed to go out on bail.
NOTE: These are only policy arguments and
decisions should not be based solely on these.

ACADCOM 2010; Contributors: Gene Geocaniga, Jarissa Guiani, Darlene Magabilen


TAU MU Page 146 of 179

COMMENDADOR vs. DE VILLA


The accused was charged with violation of
the Articles of War before a Court Martial.
ISSUE: Whether or not a person charged before a
court martial has a right to bail.
RULING: The court decided that traditionally, the
right to bail has not been recognized in the
military courts.

PEOPLE vs. DONATO


June 5, 1991
BAIL
RIGHT

MTC (6
YRS AND
BELOW)
Matter of
right
all
the
time

NOT
ALLOW
ED/
DENIED

N/A
because it
is always a
matter of
right.

DISCRE
TION

N/a

RTC (6
YRS 1 DAY
-)
if
the accused
appeals, all
appealed
cases from
MTC, for as
long as
hindi pa
final
judgment,
bail is a
matter of
right;
if
penalty
imposed is
less than
RP
pena
lty is RP or
higher, BUT
the
evidence of
guilt is
strong
if
the
penalty
is
RP
or
higher and
the
evidence of
guilt
is
strong
N/A

TAU MU
TAU MU
TAU MU
TAU MU

Efect of conviction: imprisoned.

TAU MU

4. Recognizance- the accused is entrusted to


the custody of a prominent person who is likely to
guarantee his appearance in court. Only for minor
ofenses.

TAU MU

Efect of conviction or acquittal: the title is


returned to the accused. This is limited to real
property only.

TAU MU

3. Property bond- if the accused has no cash


and no surety, title of real property is deposited in
court. It does not need to be in the name of the
accused, he can borrow. Upon tax assessment, if
the value of the property is double the value of
the recommended bail, the court accepts it and
the accused is free to go. The accused may also
use the title of another person provided he is
authorized to do so.

WAIVER OF RIGHT TO BAIL

TAU MU

2. Surety Bond - a bonding company


guarantees the appearance in court. Bonding
company guarantee the appearance of the
accused in Court, provided he pays a certain
percentage of the recommended bail to the
bonding company (15-20%). It should be renewed
annually.
The accused pays a premium every year and he
cannot recover that what he has paid. (same as
in an insurance premium)

Efect of acquittal: he goes free.

TAU MU

Efect of conviction or acquittal: the bond is


returned.
Exception: if the crime involves civil liability, the
cash may go to the aggrieved party

Ateneo de Davao

TAU MU
TAU MU

FORMS OF BAIL
1. Cash Bond- when the fiscal recommends a
bail, and the judge approves. All the accused has
to do is to deposit the cash bond to the clerk of
court, and then he is free.

The Fraternal

TAU MU
TAU MU

FACTORS CONSIDERED IN SETTING THE


AMOUNT OF BAIL:
1) Ability to post bail
2) Nature of the ofense
3) Penalty imposed by law
4) Character and reputation of the accused
5) Health of the accused
6) Strength of the evidence
7) Probability of appearing at the trial
8) Forfeiture of previous bail bonds
9) Whether accused was a fugitive from justice
when arrested
10) If accused is under bond in other cases

KITY

TAU MU TAU MU TAU MU


TAU MU TAU MU TAU MU

CONSTITUTIONAL LAW II
Order of Saint Thomas More
Atty. Philip John Pojas/Atty. Rovyne G. Jumao-as, RN
University College of Law

ACADCOM 2010; Contributors: Gene Geocaniga, Jarissa Guiani, Darlene Magabilen


TAU MU Page 147 of 179

CA/SC
Wh
en the
case was
originally
decided by
the MTC,
appealed
to the RTC
and
appealed
to the
higher
courts.

Only
applicable
here.
Because it
only
involves
cases
decided by
the
RTC
and
the
penalty is
Prision
Mayor -- RT
[If any of
the
instances
in AO 1294
exists
then
bail
should be
denied
If none of
the
instances
exist, then
bail is a
matter of
discretion.]

Salas was charged with rebellion with a


penalty of prision mayor. (Bail here is a matter of
right). The fiscal opposed the fixing of the bail
bond saying that Salas is dangerous, and at the
same time, citing many circumstances.
RULING: The court held that if the bail is a
matter of right, there is no more need of a
hearing to determine whether he can enjoy the
right or not, even if he is a very notorious
criminal. The right is absolute. Even the fear of
possibility that the accused will evade sentence is
not a valid ground for denial of bail. A hearing
may only be needed to fix the amount of bail and
not to determine whether the accused can enjoy
the right or not.

TAU MU
TAU MU
TAU MU
TAU MU
TAU MU
TAU MU

Q: If a person is out on bail and he commits


a crime, then he applies for bail again, can
bail be granted?
A: Depends. If the crime is punished by 6 years
and below, kahit pa 100 times nya gawin yung
crime, pwede syang magbail kasi bail is a matter
of right sa MTC diba? Pero matamaan sya sa
amount of bail.
The right to bail, a corollary to the right to be
presumed innocent, is like the privilege of habeas

TAU MU

WHEN IS THE ACCUSED EXEMPT FROM BAIL


(RA6036):
1. If ofense is punishable is 6 months or less (he
is released on recognizance)
2. When the crime is covered by summary rule
because there is no arrest, only notices. No
arrest, no bail.
3. Rule 112 Sec. 9 (B)- if judge is satisfied that
there is no necessity for placing the accused on
custody, then he may issue summons instead of a
warrant of arrest.

TAU MU

Note: Accused failed to raise the issue of not


hearing his application the soonest possible. He
only raised it during appeal. [PEOPLE vs. PANES]

TAU MU

Rights covered in the second type of rights:


1.
Right to due process
2.
Right against torture
3.
right to free access to courts

TAU MU
TAU MU

Is this considered a valid waiver of his right to


bail?
The court distinguished two rights:
1. Rights which are purely personal to
the accused. This right can be waived.
2. Rights where the accused as well
as the government are interested in. This right
cannot be waived because of public policy.
According to the Constitution, the right to
bail is purely personal, thus, it can be waived.
Hence, the waiver of Salas is valid and binding.

The Fraternal
Ateneo de Davao

TAU MU
TAU MU

There is also a second issue. The Solicitor


General and the lawyers for Salas entered into an
agreement whereby the companion of Salas
would be released while Salas remains in jail.
Immediately after, he filed the petition for bail.
The government contended that he cannot do so
because has waived his right to bail when he
entered into the agreement.

KITY

TAU MU TAU MU TAU MU


TAU MU TAU MU TAU MU

CONSTITUTIONAL LAW II
Order of Saint Thomas More
Atty. Philip John Pojas/Atty. Rovyne G. Jumao-as, RN
University College of Law

corpus, another means of obtaining liberty albeit


only provisional.
Q: May an alien invoke the Constitutional right to
bail during the pendency of deportation
proceedings?
A: Yes. According to the Lao Gi case, the court
held that due to the harsh consequences of
deportation proceedings to the life and liberty of
a person, the rules on criminal procedure
including the right to bail, should be applied to
deportation proceedings.
NOTE: Section 13 also covers proceedings
not criminal in nature.
STRONG EVIDENCE
PROOF EVIDENT or
EVIDENT PROOF

clear,
strong
evidence w/c leads a
well-guarded
dispassionate
judgment
to
the
conclusion that:
a. the ofense has
been committed as
charged,
b. accused is the
guilty agent, and
c. he
will
probably be punished
capitally if the law is
administered

PRESUMPTION
GREAT

exists when the


circumstances testified
to are such that the
inference
of
guilt
naturally to be drawn
therefrom is strong,
clear and convincing to
an unbiased judgment
and
excludes
all
reasonable probability
of any other conclusion

The person claiming the right under this


provision must be under actual detention or
custody of the law [MENDOZA vs. CFI, 51 SCRA
369].
One is under the custody of the law either
when he has been arrested or has surrendered
himself to the jurisdiction of the court.
The right may NOT be impaired even when
the privilege for habeas corpus is suspended.

GENERAL RULE: The constitutional right to bail


is available only in criminal proceedings.
Q: How do you determine whether the
evidence of guilt is strong?
A: There has to be a summary hearing to
determine if the evidence guilt is strong. The
prosecution must be given the opportunity to
present evidence, and only then can the judge
grant or deny bail if the evidence of guilt is strong
or not. And if the accused does not apply for bail,
the judge has no right to outright fix the amount
of bail.
LIFE
IMPRISONMENT

a
penalty
in
special laws

does
not
carry
accessory penalties

is
indefinite
in
duration

ACADCOM 2010; Contributors: Gene Geocaniga, Jarissa Guiani, Darlene Magabilen


TAU MU Page 148 of 179

RECLUSION
PERPETUA

is imposed by the
RPC

carries
w/
it
accessory penalties

is for a duration of
30 years, after w/c the
prisoner is eligible for

RECOGNIZANCE
is an obligation of record entered into before a
court guaranteeing the appearance of the
accused for trial
it is in the nature of a contract between the
surety and the state [PEOPLE vs. ABNER (87 Phil.
569)]
the details on how this can be obtained or
when it is applicable is left to legislation
RIGHT AGAINST EXCESSIVE BAIL
The right to bail can be rendered useless by a
bail bond set at an exorbitant amount.

CASES: RIGHT TO BAIL


PEOPLE vs. MANES
303 SCRA 231 (1999)

TAU MU
TAU MU
TAU MU

Facts: Fitzgerald, an Australian Citizen, actuated


by lust and by the use of a laced drug, induced
a 13 year old minor to engage in prostitution and

TAU MU

PEOPLE vs. FITZGERALD


505 SCRA 573 (2006)

TAU MU

Held: No. Trial proceeded without the accused


calling the attention of the trial court to his
unresolved petition. It was only appeal that he
raised the issue. Thus, for failure to raise the
issue at the earliest opportune time, accused is
deemed to have waived the right to bail. In
addition, the issue has been rendered academic
by the conviction of accused. When an accused
is charged with an ofense punishable b reclusion
perpetua of life imprisonment or death, and the
evidence of guilt is strong, bail must be denied.

TAU MU

Facts: Accused as charged with murder.


He
applied for bail but the judge set the case for trial
without acting on his applications.
He was
convicted and on appeal he contended that the
judge committed a serious error of law when he
tried the case without resolving the application
for bail. Is the contention of accused correct?

TAU MU

NOTE: However, it is very hard to determine


what is excessive and what is not. It depends on
many factors.

Issue: WON the CA erred when it granted


Fitzgeralds application for bail.

TAU MU

RULING: The court held that the amount of bail


is excessive and it violates the Constitution. It
would have been more honest if the judge denied
bail rather than to grant bail on an amount
beyond the persons reach. It is like not granting
bail at all.

had carnal knowledge of her. He was then


charged for violating certain provisions of RA
7160. He was found guilty by the trial court.
Hence, he applied for bail which was denied. He
filed for a motion for new trial before the CA on
the ground that new and material evidence not
previously available had surfaced. The CA
granted the motion. And subsequently, CA
granted Fitzgeralds application for bail. Hence,
petitioner opposed and argued that CA erred in
granting Fitzgeralds Motion for Bail despite the
fact that the latter was charged with a crime
punishable by reclusion perpetua and the
evidence of his guilt is strong.

TAU MU

The mayor of Magsaysay was implicated in


the killing of 14 people and injuring 12 others in
Agusan. The judge ordered his bail at
P1,195,200.00.

Ateneo de Davao

TAU MU
TAU MU

DELA CAMARA vs. ENAGE

The Fraternal

TAU MU
TAU MU

pardon
bail is a matter of discretion --- should be
denied if evidence of guilt is strong!

KITY

TAU MU TAU MU TAU MU


TAU MU TAU MU TAU MU

CONSTITUTIONAL LAW II
Order of Saint Thomas More
Atty. Philip John Pojas/Atty. Rovyne G. Jumao-as, RN
University College of Law

Ruling: The right to bail emenates from of the


right to be presumed innocent. It is accorded to a
person in the custody of the law who may, by
reason of the presumption of innocence he
enjoys, be allowed provisional liberty upon filing
of a security to guarantee his appearance before
any court, as required under specified conditions.
In this case, bail was not a matter of right
for Fitzgerald but a mere privilege subject to the
discretion of the CA to be exercised in accordance
with the stringent requirements of Sec. 5, Rule
114. However, it is bad enough that the CA
granted bail on grounds other than those stated
in the Motion filed by Fitzgerald; it is worse that it
granted bail on the mere claim of his illness. Bail
is not a sick pass for an ailing or aged detainee or
prisoner needing medical care outside the prison
facility. A mere claim of illness is not a ground for
bail. It may be that the trend now is for courts to
permit bail for prisoners who are seriously sick.
There may also be an existing proposition for the
selective decarceration of older prisoners based
on findings that recidivism rates decrease as age
increases. But, in this particular case, the CA
made no specific finding that respondent sufers
from an ailment of such gravity that his continued
confinement during trial will permanently impair
his health or put his life in danger. It merely
declared respondent not in the best of health
even when the only evidence on record as to the
latters state of health is an unverified medical
certificate. Hence, the CA erred when it granted
Fitzgeralds application for bail.
PEOPLE vs. SANDIGANBAYAN and ESTRADA
529 SCRA 764 (2007)
Facts: Jinggoy Estrada was charged, together
with ERAP and several others, for the crime of
plunder, defined and penalized under RA No.
7080. When he was arrested, he filed a motion
alleging that: (1) no probable cause exists to put
him on trial and hold him liable for plunder, it
appearing that he was only allegedly involved in
illegal gambling and not in a series or
combination of overt or criminal acts as required
in RA 7080; and (2) he is entitled to bail as a
matter of right. By reason of the denial of his
motion, he interposed a petition for certiorari
before the SC claiming that the respondent

ACADCOM 2010; Contributors: Gene Geocaniga, Jarissa Guiani, Darlene Magabilen


TAU MU Page 149 of 179

Issue: WON the grant of bail was proper


considering the fact that Jinggoy is equally guilty
and liable as ERAP by his indispensable
cooperation and/or direct participation in the
commission of the crime of plunder.

TAU MU
TAU MU
TAU MU
TAU MU
TAU MU
TAU MU
TAU MU

Ruling: No. Extradition proceedings are not


criminal in nature. By using the term before
conviction, it is apparent that the right to bail in
Sec. 13, Art III of the Constitution does not apply
to extradition proceedings because extradition
courts do not render judgments of conviction or
acquittal. Moreover, the right to bail flows from

Facts: The Republic of the Philippines and the


then British Crown Colony of Hong Kong signed
an Agreement for the Surrender of Accused and
Convicted Persons. Two years after the
agreement was signed, Hong Kong reverted back
to the Peoples Republic of China and became the
Hong Kong Special Administrative Region. Muoz
was charged before the Hong Kong Court with 3
counts of the ofense of accepting an advantage
as agent. He also faces 7 counts of the ofense
of conspiracy to defraud, penalized by the
common law of Hong Kong. Thereafter, warrants
of arrest were issued against him. If convicted,
he faces a jail term of 7 to 14 years for each
charge.
The DOJ received from the Hong Kong DOJ
a request for the provisional arrest of Muoz. The
DOJ then forwarded the request to the NBI which,
in turn, filed an application for the provisional
arrest of private respondent. Hence, an order of
arrest was issued against Muoz. Petitioner filed a
petition for the extradition of Muoz. For his part,
Muoz filed, in the same case, a petition for bail
which was opposed by petitioner. The application
for bail was denied holding that there is no
Philippine law granting bail in extradition cases
and that private respondent is a high flight risk.
However, when the case was raffled to PR Judge
Olalia since the previous judge inhibited himself,
he granted Muozs application for bail.

TAU MU

Facts: The Government of the United States of


America, through the Department of Justice, filed
with the Regional Trial Court a petition for the
extradition of Mark Jimenez who was facing
various criminal charges in Florida. Petitioners
prayed for his immediate arrest, and the judge
set it for hearing. Thereafter, the judge issued an
order for his arrest and fixed bail for his
temporary liberty at P1 million. In extradition
proceedings, is the prospective extraditee
entitled to post bail while the extradition
proceedings are pending?

the presumption of innocence in favor of accused


who is entitled to acquittal unless his guilt is
proved beyond reasonable doubt.
It follows,
therefore, that the provision on bail will not apply
to extradition he is sought to be extradited are
bailable in the United States is no argument.
Extradition proceedings are separate and distinct
from the trial for the ofenses for which he is
charged. He should apply for bail before the US
court, not before the extradition court.

TAU MU

GOVERNMENT OF USA vs. PURGANAN


September 24, 2002

Ateneo de Davao

TAU MU
TAU MU

Ruling: The SC ruled that the imputation of


grave abuse of discretion to Sandiganbayn was
untenable. To begin with, Section 13 of Article III
(Bill of Rights) of the Constitution mandates:
Section 13. All persons, except those charged
with ofenses punishable by reclusion perpetua
when evidence of guilt is strong, shall, before
conviction, be bailable by sufficient sureties, or
be released on recognizance as may be provided
by law. Even if the capital ofense charged is
bailable owing to the weakness of the evidence of
guilt, the right to bail may justifiably still be
denied if the probability of escape is great. In this
case, Jinggoy does not, as determined by
Sandiganbayan, seem to be a flight risk.
The likelihood of escape on the part
Jinggoy is now almost nil, given his election on
May 2004, as Senator of the Republic of the
Philippines. The Court takes stock of the fact that
those who usually jump bail are shadowy
characters mindless of their reputation in the
eyes of the people for as long as they can flee
from the retribution of justice. On the other
hand, those with a reputation and a respectable
name to protect and preserve are very unlikely to
jump bail. The Court, to be sure, cannot accept
any suggestion that someone who has a popular
mandate to serve as Senator is harboring any
plan to give up his Senate seat in exchange for
becoming a fugitive from justice.

The Fraternal

TAU MU
TAU MU

Sandiganbayan committed grave abuse of


discretion in (a) sustaining the charge against
him for alleged ofenses and with alleged
conspirators with whom he is not even
connected, and (b) in not fixing bail for him. But
later on, Sandiganbayan granted his application
for bail. Hence, this petition.

KITY

TAU MU TAU MU TAU MU


TAU MU TAU MU TAU MU

CONSTITUTIONAL LAW II
Order of Saint Thomas More
Atty. Philip John Pojas/Atty. Rovyne G. Jumao-as, RN
University College of Law

GOVERNMENT OF HONG KONG vs. OLALIA


AND MUNOZ
521 SCRA 470 (2007)

Issue: WON a prospective extraditee may be


granted bail.
Ruling: Philippine jurisprudence has not limited
the exercise of the right to bail to criminal
proceedings only. This Court has admitted to bail
persons who are not involved in criminal
proceedings. In fact, bail has been allowed in this
jurisdiction to persons in detention during the
pendency of administrative proceedings, taking
into cognizance the obligation of the Philippines
under international conventions to uphold human
rights.
The right of a prospective extraditee to
apply for bail in this jurisdiction must be viewed
in the light of the various treaty obligations of the
Philippines concerning respect for the promotion
and protection of human rights. Under these
treaties, the presumption lies in favor of human
liberty. Thus, the Philippines should see to it that
the right to liberty of every individual is not
impaired.

ACADCOM 2010; Contributors: Gene Geocaniga, Jarissa Guiani, Darlene Magabilen


TAU MU Page 150 of 179

TAU MU
TAU MU
TAU MU
TAU MU
TAU MU
TAU MU
TAU MU

Ruling: In extradition cases, prior notice and


hearing should be aforded the extraditee even
when a possible extradition is still being
evaluated. The Court qualified and declared that
prospective extraditees are entitled to notice and
hearing only when the case is filed in court and
not during the process of evaluation.
The SC ruled that the benefits of
continued temporary liberty on bail should not be
revoked and their grant of bail should not be
cancelled, without the co-petitioner, Imelda
Rodriguez in this case, being given notice and
without her being heard why her temporary
liberty should not be discontinued. Imeldas copetitioner was already facing his charges against
him, hence, the issue posted is already moot and
academic. Thus, it emphasized that bail may be
granted to a possible extraditee only upon a clear

TAU MU

Issue (1): WON a prospective extraditee is


entitled to notice and hearing before the
cancellation of his or her bail.

TAU MU

Facts: This case stemmed from a petition for


extradition which was filed by the Government of
the United States of America through the DOJ
against the petitioners. After their arrest,
petitioners applied for bail which the trial court
granted. The bail was set for one million pesos
for each. Petitioners then posted cash bonds.
The US government moved for reconsideration of
the grant of bail, but the motion was denied by
the trial court.
The SC directed the trial court to resolve
the matter of bail which shall be subject to
whatever ruling that it may have in the similar
case of Mark Jimenez. In compliance with the SCs
directive, the trial court, without prior notice and
hearing, cancelled the cash bond of the
petitioners and ordered the issuance of a warrant
of arrest. Thus, petitioners filed a motion for
reconsideration of the cancellation of their bail.

TAU MU
TAU MU

RODRIGUEZ vs. PRESIDING JUDGE and US


GOVERNMENT
483 SCRA 290 (2006)

The Fraternal
Ateneo de Davao

TAU MU
TAU MU

Muoz was arrested on September 23,


1999, and remained incarcerated until December
20, 2001, when the trial court ordered his
admission to bail. In other words, he had been
detained for over two (2) years without having
been convicted of any crime. By any standard,
such an extended period of detention is a serious
deprivation of his fundamental right to liberty. In
fact, it was this prolonged deprivation of liberty
which prompted the extradition court to grant
him bail. While our extradition law does not
provide for the grant of bail to an extraditee,
however, there is no provision prohibiting him or
her from filing a motion for bail, a right to due
process under the Constitution.
Thus, in this case, there is no showing that
Muoz presented evidence to show that he is not
a flight risk. Consequently, this case should be
remanded to the trial court to determine whether
private respondent may be granted bail on the
basis of clear and convincing evidence.

KITY

TAU MU TAU MU TAU MU


TAU MU TAU MU TAU MU

CONSTITUTIONAL LAW II
Order of Saint Thomas More
Atty. Philip John Pojas/Atty. Rovyne G. Jumao-as, RN
University College of Law

and convincing showing (1) that he will not be a


flight risk or a danger to the community, and (2)
that there exist special, humanitarian and
compelling circumstances. Therefore, the SC held
that Imeldas bail should be restored.
Issue (2): WON an order to issue warrant of
arrest against petitioners and to cancel the bail of
extraditees was a grave abuse of discretion of the
trial court.
Ruling: The cancellation of co-petitioners bail,
without prior notice and hearing, could be
considered a violation of co-petitioners right to
due process tantamount to grave abuse of
discretion. Considering that Imelda has not been
shown to be a flight risk nor a danger to the
community, she is entitled to notice and hearing
before her bail could be cancelled. Thus, absent
prior notice and hearing, the bails cancellation
was in violation of Imeldas right to due process.
YAP, JR. V. CA
358 SCRA 564 (2001)
Facts:
Accused
was
convicted
of
misappropriating P5.5M and was sentenced to a
jail term of 8 years to 20 years. While his appeal
was pending before the Court of Appeals, he
applied for bail which was granted. However, the
Court of Appeals fixed it at P5.5M, with a
restriction that he could not change residence
without the approval of the court and a directive
to the Commissioner of Immigration and
Deportation to issue a hold departure order
against him. Was his right against excessive bail
violated?
Ruling:
Yes.
The amount of P5.5M is
unreasonable, excessive and constitutes an
efective denial of petitioners right to bail. The
purpose for bail is to guarantee the presence of
accused whenever required by the court. The
amount should be high enough to assure his
presence, but no higher than is reasonably
calculated to fulfill this purpose. To fix bail at an
amount equivalent to the civil liability of which
petitioner is charged is to permit the impression
that the amount paid as bail is an exaction of the
civil liability that accused is charged of, thus the
court cannot allow because bail is not intended as
a punishment, nor a satisfaction of civil liability
which should necessarily await the judgment of
the appellate court.
OBOSA vs. CA
266 SCRA 281 (1997)
Facts: Accused was charged with two counts of
murder, a capital ofense, but after trial he was
conceived only of two counts of homicide, a noncapital ofense. On appeal, is he entitled to bail
as a matter of right?
Ruling: Petitioner applied for bail after the
efectivity of Administrative Circular No. 12-94.
Under the Circular, if the court imposed a penalty
of imprisonment exceeding six years but not

ACADCOM 2010; Contributors: Gene Geocaniga, Jarissa Guiani, Darlene Magabilen


TAU MU Page 151 of 179

more that twenty ears, then bail is a matter of


discretion, except when any of the circumstances
mentioned in paragraph 3, Sec. 5 of the Circular
is present, in which case bail has to be denied.
Since petitioner was convicted of two counts of
homicide, his circumstances found in the Circular
justify the denial of bail, except that a retroactive
application of the Circular is barred as it would be
unfavorable to the petitioner.

TAU MU
TAU MU
TAU MU
TAU MU
TAU MU
TAU MU
TAU MU
TAU MU

Q: Why does the accused has more rights?


A: (1) Because it is better to free to guilty people than
to convict 1 innocent person; (2) In a criminal trial, it is
one person against the people of the state with all its
machinations; and (3) It should not be proof by
preponderance of evidence since it would result in the
conviction of more innocent people.

TAU MU

RIGHTS OF A PERSON CHARGED WITH A


CRIMINAL OFFENSE:
1. Right to due process of law waivable by making a
plea of guilty
2. Right to be presumed innocent waivable by
making a plea of guilty
3. Right to be heard by himself and counsel waivable
4. Right to be informed of the nature and cause of the
accusation against him not waivable
5. Right to have a speedy trial waivable
6. Right to an impartial trial not waivable
7. Right to public trial waivable
8. Right to meet the witness face to face waivable
9. Right to have compulsory process to secure the
attendance of witnesses and the production of
evidence in his behalf waivable by making an
advance waiver.

TAU MU
TAU MU

Section 14. (1) No person shall be held to answer


for a criminal offense without due process of law.
(2) In all criminal prosecutions, the accused shall
be presumed innocent until the contrary is
proved, and shall enjoy the right to be heard by
himself and counsel, to be informed of the
nature and cause of the accusation against him,
to have a speedy, impartial, and public trial, to
meet the witnesses face to face, and to have
compulsory process to secure the attendance of
witnesses and the production of evidence in his
behalf. However, after arraignment, trial may
proceed notwithstanding the absence of the
accused: Provided, that he has been duly
notified and his failure to appear is unjustifiable.

The Fraternal
Ateneo de Davao

TAU MU
TAU MU

Be that as it may, the rules on bail prior to


the efectivity of Administrative Circular No. 12-94
does not favor him either. In accordance with
previous decisions, when he appealed he opened
the whole case for review so that the possibility
of conviction upon the original charge of murder
is ever present. Likewise, since the prosecution
has previously demonstrated that evidence of
guilt is strong, such determination subsists on
appeal, despite conviction for a lesser ofense,
since such determination is for the purpose of
resolving whether to grant or deny bail and does
not have any bearing on whether petitioner will
ultimately be acquitted or convicted of the
charge.

KITY

TAU MU TAU MU TAU MU


TAU MU TAU MU TAU MU

CONSTITUTIONAL LAW II
Order of Saint Thomas More
Atty. Philip John Pojas/Atty. Rovyne G. Jumao-as, RN
University College of Law

NOTE: The rights of the accused apply only to


PROCEDURAL
JUDICIAL
PROCESS
IN
CRIMINAL
PROCEEDINGS as compared to Section 1 of Article III of
the Constitution that applies to all types of
proceedings.
1. DUE PROCESS
a. The right to due process is the biggest right of all.
According to Justice Cruz, the right to due process
mentioned here is only a procedural due process, the
procedure laid down by law in trying an accused who is
being charged of a crime.
b. This means that the accused can only be convicted
by a tribunal which is required to comply with the
stringent requirements of the rules of criminal
procedure.

Criminal due process requires that the accused be


tried by an impartial and competent court in
accordance with the procedure prescribed by law and
with proper observance of all the rights accorded him
under the Constitution and the applicable statutes.
Accordingly, to illustrate, denial from him of the right to
preliminary investigation, as required by law, will
constitute a denial of due process.
It should be noted that the right to a preliminary
investigation is not among the rights granted to the
accused in the Bill of Rights. It is purely statutory. Even
so, denial of this right, in the absence of a valid waiver,
will violate due process.
Obviously, the basic ingredient of criminal due
process is a trial conducted in accordance with the
rudiments of fair play. Hence, the accused has a right
to complain if the judge has a personal or pecuniary
interest in the outcome of the case, as where he is
allowed to share in the fines he may impose or where
he is covered by the disqualification enumerated in the
rules of court.
Due process is also denied where a person is
impleaded for violation of a law, administrative
regulation or municipal ordinance not previously
published as he would not know what acts he must do
or avoid to prevent prosecution. Where appeal is
permitted by the Constitution or by statute, denial
thereof will also militate against due process.
ELEMENTS OF CRIMINAL DUE PROCESS [BANCO
ESPANOLA vs. PALANCA, 37 PHIL 921]:
1) There must be an impartial court or tribunal clothed
with judicial power to hear and determine the matter
before it.
2) Jurisdiction must be lawfully acquired over the
person of the accused and over the property which is
the subject matter of the proceeding.
3) Accused was given an opportunity to be heard
4) Judgment must be rendered upon lawful hearing.
NOTE: All elements must be present. The absence of
one means there is a violation of criminal due process.
PROCEDURAL
GUARANTY
IN
CRIMINAL
PROCEEDINGS [NUNEZ vs. SANDIGANBAYAN 11
SCRA 542]:
1. The accused must be informed why is he being
charged and what are the charges against him;
2. He should have the full opportunity to rebut it;
3 The conviction should rest on evidence that is not
tainted with falsity
4. The sentence imposed against him should be in
accordance with a valid law;
5. The court that rendered judgment is one of
competent jurisdiction.

ACADCOM 2010; Contributors: Gene Geocaniga, Jarissa Guiani, Darlene Magabilen


TAU MU Page 152 of 179

TAU MU
TAU MU
TAU MU
TAU MU

Issue: WON the appellant was deprived of his right to


due process.

TAU MU

Facts: Macarang was sentenced to the penalty of


death for qualified rape. The trial court considered
appellant to have waived his right to present his
evidence without any showing that the latter was fully
aware of the consequence of such waiver.

TAU MU

Ruling: The right of the accused to due process was


violated.
No trial was conducted based on the
procedure in the Rules of Court and accused was not
given his full day in court. It cannot be argued that
accused waived his right to confront and cross-examine
the witness because the existence of the waiver must
be shown to have been done knowingly and with
sufficient awareness of the consequence. The case
should be remanded for further proceedings.
PEOPLE vs. MACARANG
424 SCRA 18 (2004)

TAU MU

Facts: After accused was arraigned, the prosecution


presented the rape victim who identified her affidavit
of desistance and reaffirmed that she had no further
interest in prosecuting accused. The judge then asked
clarificatory questions to determine the truth and
volutariness of both her affidavit-complaint and
affidavit of desistance. Counsel for the accused did not
anymore cross-examine the witness. The prosecutor
then moved to dismiss the case as she could no longer
prove the guilt of the accused. About two months
later, the court convicted accused of rape and
sentenced him to reclusion perpetua.

defense evidence, he would have deemed his right to


present it, did not satisfy appellants constitutional
right to due process. The trial court granted Motion of
Appellants counsel to withdraw his appearance.
Appellant, therefore, had no more counsel. It is obvious
that the appellant was deprived of his right to due
process.
PAGASIAN vs. AZURA
Facts: In a criminal case for theft entitled People vs
Dumo, Barangay Captain Pagasian was a witness. It
appeared that because of a report, and accompanied
by police, he seized the carabao from the house of the
accused which allegedly is the subject matter of the
theft. After trial, the Judge acquitted Dumo but
convicted Pagasian for clear violation of the
fundamental law of the land and against human rights.
He was sentenced to two days of jail term and a fine of
P200.00.
Ruling: The court ruled that the judge violated due
process. The barangay captain was not informed of the
charges against him and in fact, he had no idea that he
was on trial. He did not even present evidences in his
own behalf.
OLAGUER vs. MILITARY COMMISSION

TAU MU

ALONTE vs. SAVELLANO JR.


287 SCRA 245 (1998)

TAU MU
TAU MU

CASES ON DUE PROCESS:

The Fraternal
Ateneo de Davao

TAU MU
TAU MU

REQUIREMENTS FOR DUE PROCEES CLAUSE IF


THE ACCUSED PLEADS GUILTY TO A CAPITAL
OFFENSE [PEOPLE vs. STA. TERESA] (although no
more capital punishment):
1. The trial court must conduct a searching
inquiry into the voluntariness of the plea and the full
comprehension of what he confessed thereof.
2. The court must order the prosecution to
present evidence to prove the guilt of the accused
beyond reasonable doubt and the precise degree of his
liability
3. The accused must be asked if he would like to
present evidence on his behalf, and allow him to do so,
if he desires.

KITY

TAU MU TAU MU TAU MU


TAU MU TAU MU TAU MU

CONSTITUTIONAL LAW II
Order of Saint Thomas More
Atty. Philip John Pojas/Atty. Rovyne G. Jumao-as, RN
University College of Law
[OLAGUER vs. MILITARY COMMISSION] The court
held that Due Process in Section 14 means trial by
judicial process not by executive or military process.
Military tribunal is not part of judiciary, but of executive
to discipline the army. Therefore, there is a violation of
due process.
Even during martial law, all cases should be tried
under civilian courts. AS LONG AS THE CIVILIAN
COURTS ARE OPEN and FUNCTIONING, decisions by
military tribunals on civilians are null and void.
Pending cases in Tribunals should be transferred to
civilian courts when national emergency already
ceases to exist. Military Tribunals have jurisdiction only
as long as national emergency exists. Double jeopardy
will not attach.

TAU MU

Ruling: In criminal cases where the impossible penalty


may be death, the presiding judge is called upon to see
to it that the accused is made aware of the
consequences of not heeding the warrant given by the
trial court. A simple forewarning to the appellant that
the next time that he would not be ready with his
ACADCOM 2010; Contributors: Gene Geocaniga, Jarissa
TAU MU Page 153 of 179

Facts: Olaguer was a civilian tried and sentenced to


die by the military tribunal during Martial Law. After
Marcos was deposed, Olaguer went to SC challenging
the validity of his conviction saying that his conviction
was a violation of his right to due process.
Ruling: The court held that due process in Sec. 14
means trial by judicial process not by executive of
military process. Military tribunal is not part of judiciary
but of the executive branch for discipline of army.
Therefore there is a violation of due process. Even
during martial law, all cases should be tried under
civilian courts. As long as civilian courts are open and
functioning, the decisions of the military tribunal on
civilians are null and void.
Pending cases involving civilians in military tribunals
should be transferred to civilian courts when national
emergency already ceased to exist. Military tribunals
have jurisdiction only as long as national emergency
exists. Double jeopardy will not lie here.
2. PRESUMPTION OF INNOCENCE
In all criminal prosecution, the accused shall be
presumed innocent until the contrary is proved.
DEMURRER OF EVIDENCE
It is motion to dismiss based on insufficiency of
evidence.
The government has not overcome the presumption of
innocence since it is the duty of the prosecution to
present evidence against the accused.
The evidences presented will not be based on the
strength or weakness of the accused but of the
PROSECUTION.
The Constitution does not prohibit the legislature

from providing that proof of certain facts leads to a


prima facie presumption of guilt, provided that the
facts proved have a reasonable connection to the
ultimate fact presumed.
Presumption of guilt should not be conclusive.
Q: When is there a REVERSE TRIAL?
A: (1) When the accused puts up an affirmative
defense (like self-defense) and; (2) When a law is made
by the Congress that there is presumption of guilt
Guiani, Darlene Magabilen

Q: Why does it not violate the Constitution?


A: (1) There is a logical connection between the fact
presumed and the fact proved; and (2) The
presumption is rebuttable.

TAU MU
TAU MU
TAU MU
TAU MU
TAU MU

Facts: Petitioners challenge the provision of PD No.


704, the Fisheries Decree, which provides that the
discovery of explosives or obnoxious substance in any
fishing boat shall constitute a presumption that the
owner or operator were fishing with the use of

TAU MU

HIZON vs. CA
supra

Ruling: It is conceded that the legislature has the


power to provide that proof of certain facts can
constitute a prima facie evidence of guilt provided
there is a rational connection between the fact proved
and the fact presumed. To avoid any constitutional
infirmity, the inference of one form proof of the other
must not be arbitrary and unreasonable. P.D. 704
creates a presumption of guilt based on facts proved
and hence is not constitutionally impermissible.
However, the statutory presumption can only be prima
facie. It cannot, under the guise of regulating the
presentation of evidence, operate to preclude accused
from presenting his defense to rebut the fact presumed

THE RIGH TO PRESENT EVIDENCE


BEHALF INCLUDES:
a) The right to testify in ones favor;
b) The right to call witnesses.
THE RIGHT TO BE
FOLLOWING RIGHTS:

TAU MU

CASE ON PRESUMPTION OF INNOCENCE:

3. RIGHT TO BE HEARD BY HIMSELF AND


COUNSEL
Simply stated, this is the right to present evidence on
one's behalf, which includes the right to testify in one's
favor and the right to call witnesses. It includes the
right to defend yourself, the right to confrontation, and
the right to call witnesses.

TAU MU

[DIZON-PAMINTUAN vs. PEOPLE, 234 SCRA 63]:


There is no constitutional infirmity to the reversed
presumptions.
[HIZON vs. CA]: Laws providing presumptions in
criminal cases that a crime being or has been
committed is not a violation of the presumption of
innocence right provided that they are based on facts
and these facts must be part of the crime when
committed.
To avoid any constitutional infirmity, the inference
of one from proof of the other must not be arbitrary
and unreasonable.
The legislature has the power to provide that proof
of certain facts can constitute prima facie evidence of
guilt of the accused and shift the burden of proof to the
accused provided that there is a rational connection
between the facts proved and the ultimate fact
presumed. (Natural inference)
The statutory presumption is merely prima facie
and cannot preclude the accused from presenting his
defense to rebut it and at no instance can the accused
be denied the right to rebut the presumption.

explosives or poisonous substance, as a violation of the


constitutional presumption of innocence.

TAU MU

Q: Are these presumptions constitutional?


A: Yes. Clearly, the fact presumed is but a natural
inference from the fact proved so that it cannot be said
that there is no rational connection between the two.
Furter, the statute establishes only a prima facie
presumption thus giving the accused an opportunity to
rebut it.

Ateneo de Davao

TAU MU
TAU MU

GENERAL RULE: The accused shall be presumed


innocence until the contrary is proved.
EXCEPTIONS:
1. Article 217 of RPC, the failure of the public
officer to produce money in his charge is prima facie
evidence of malversation. So the burden of proof is
shifted to the defense to prove otherwise (that he is
not guilty).
2. Violation of the Anti-fencing Law
3. Possession of poisonous substances if
you are on board a fishing vessel. (Reason: There
is a presumption of involvement in illegal fishing)

The Fraternal

TAU MU
TAU MU

Q: Do laws create by Congress to create a


certain presumption of guilt violate the
presumption of innocence?
A: NO, because clearly, the fact presumed is but a
natural inference from the fact proved so that it cannot
be said that there is no rational connection between
the 2. Furthermore, the statute establishes only a
prima facie presumption, thus giving the accused an
opportunity to rebut it.

KITY

TAU MU TAU MU TAU MU


TAU MU TAU MU TAU MU

CONSTITUTIONAL LAW II
Order of Saint Thomas More
Atty. Philip John Pojas/Atty. Rovyne G. Jumao-as, RN
University College of Law
Note: In case of a reverse trial, the burden of evidence
shifts to the accused to present the existence of factual
evidence on whatever exculpatory defense he
presents.

HEARD

IN

YOUR

INCLUDES

THE

Right to be present at the trial

A. The right to be present covers the


period from ARRAIGNMENT to PROMULGATION of
sentence.
B. After arraignment, trial may proceed
notwithstanding absence of accused, provided 2
requisites are met. Note, that trial in absentia is
allowed only if the accused has been validly arraigned.
(i) Accused has been duly notified; and
(ii) His failure to appear is unjustifiable.
C. The accused may waive the right to be
present at the trial by not showing up. However, the
court can still compel the attendance of the accused if
necessary for identification purposes. EXCEPTION: If
the accused, after arraignment, has stipulated that he
is indeed the person charged with the ofense and
named in the information, and that any time a witness
refers to a name by which he is known, the witness is
to be understood as referring to him.
D. While the accused is entitled to be
present during promulgation of judgement, the
absence of his counsel during such promulgation does
not afect its validity.

Right to counsel

A. Right to counsel means the right to EFFECTIVE


REPRESENTATION.
B. If the accused appears at arraignment without
counsel, the judge must:
(i) Inform the accused that he has a right
to a counsel before arraignment
(ii) Ask the accused if he desires the aid of
counsel
(iii) If the accused desires counsel, but
cannot aford one, a counsel de oficio must be
appointed
(iv) If the accused desires to obtain his own
counsel, the court must give him a reasonable
time to get one.
BASIC ELEMENTS OF THE RIGHT TO COUNSEL:

ACADCOM 2010; Contributors: Gene Geocaniga, Jarissa Guiani, Darlene Magabilen


TAU MU Page 154 of 179

TAU MU
TAU MU
TAU MU
TAU MU
TAU MU
TAU MU
TAU MU

DOCTRINAL CASES:
[PEOPLE vs. JOSE 37 SCRA 450]. BUT under Sec. 12
(1) the right to counsel also exists before arraignment.
[DELGADO vs. CA]: Deprivation of right to counsel
because the accused was represented by a fake
lawyer;
Waiver of this right to counsel need not be done in
the presence of counsel because under our procedure,
the accused can defend himself;
[PEOPLE vs. NADERA]: If the evidence presented by
the prosecution is strong, it is the duty of the defense
to try to rebut it in anyway he can. Here, the defense
lawyer believed the victim. That cannot be done, the
defense lawyer must try his best to rebut the evidence.
He must not be swayed by the testimony of the victim
because it is his duty to defend his client, and if he
does not do so then there is a deprivation of the right
to counsel.
[CHIONGBIAN CASE]: The Court said that the
counsel of choice thing applies only to cases when the
accused is under custodial investigation. There are
other rights that cannot be violated, like the right to
speedy trial. So the accused cannot delay the case just
because he wants to get his own lawyer
[PEOPLE vs. TULIN]: Accused was first represented
by a non lawyer, who did a good job. When he found
out, he got a real lawyer, who adopted the nonlawyer's strategy. Accused was convicted. Hence, there
was no deprivation because he should have said to his

TAU MU

when the accused has no counsel of choice and desires


to employ the services of one is MANDATORY ONLY AT
THE TIME OF ARRAIGMENT. The counsel need not be
one who is the choice of the accused.
The right to counsel may be waived.
The rights enumerated in Sec. 14 (2) are rights in all
criminal prosecutions which cover the period from
arraignment to rendition of judgment. The right to
counsel exists only during that period.

TAU MU

The duty of the court to appoint counsel de officio

TAU MU
TAU MU

Note: The duty of the court is not ended with such


appointment of a counsel de oficio, as it should also
see to it that the counsel does his duty by the
defendant.

The Fraternal
Ateneo de Davao

TAU MU
TAU MU

NOTE: The right to counsel during trial can be waived.


The accused may represent himself in any litigation.
PRE ARRAIGNMENT DUTIES OF THE JUDGE [Rule
116 Sec.1]:
a. to inform the accused that he has the
right to have his own counsel before being arraigned;
b. after informing the accused he has to ask
the latter whether he desires the aid of counsel;
c. if the accused so desires to procure the
services of counsel, the court must grant him
reasonable time to do so;
d. if ha so desires to have counsel but is
unable to employ one, the court must assign counsel
de officio.

KITY

TAU MU TAU MU TAU MU


TAU MU TAU MU TAU MU

CONSTITUTIONAL LAW II
Order of Saint Thomas More
Atty. Philip John Pojas/Atty. Rovyne G. Jumao-as, RN
University College of Law
a. The court is duty bound to inform the
accused that he has the right to an attorney before he
is arraigned;
b. The court must ask him if he desires the
service of counsel;
c. If he does, and is unable to get one, the
court must assign a counsel de officio;
d. Or, if the accused wishes to procure private
counsel, the court must give him time to obtain one
[PEOPLE vs. HOLGADO (85 Phil. 753)];
e. Where the duly authorized members of the
bar are not available, the court may appoint a person
resident of the province and of good repute for probity
and ability [Rule 116 Sec. 1];

counsel that he does not want to adopt the strategy of


the non-lawyer. There was waiver on the part of the
accused.
Right to an impartial judge
Right of confrontation and crossexamination
Right to compulsory process to
secure the attendance of witnesses

CASES ON
COUNSEL:

RIGHT

TO

BE

HEARD

AND

TO

PEOPLE vs. SANTOCILDES


321 SCRA 310 (1999)
Facts: Accused was charged and convicted of the
crime of rape and sentenced to reclusion perpetua.
During trial, he was represented by a certain Gualberto
C. Ompong, who turned out to be a non-lawyer. On
appeal, he argues that his right to counsel was violated
so that he should be acquitted of the crime charged.
Ruling: The right of accused to counsel was violated,
no matter that the person who represented him had
the ability of a seasoned lawyer and handled the case
in a professional and skillful manner.
This is so
because an accused person is entitled to be
represented by a member of the bar in a criminal case
filed against him. Unless he is represented by a
lawyer, there is a great danger that any defense
presented will be inadequate considering the legal
skills needed in court proceedings. The judgment is set
aside, and the case remanded for new trial.
PEOPLE vs. TULIN
364 SCRA 10 (2001)
Facts: Accused was tried for piracy.
They were
represented by Tomas Posadas who presented and
examined 7 witnesses before the court discovered that
he was a non-lawyer. Their new lawyer, however,
manifested that accused were adopting the evidence
adduced when they were represented by a non-lawyer.
Convicted by the trial court, they now claim that their
right to counsel was violated.
Ruling: The law entitles the accused to be present
and to defend himself in a person and by counsel at
every stage of the proceedings. However, it is also
provided that rights may be waived, unless the waiver
is contrary to law, public order, public policy, etc. The
Rules also states that upon motion, the accused may
be allowed to defend himself in person when it
sufficiently appears to the court that he can properly
protect his rights without the assistance of counsel. By
analogy, it is amply shown that the rights of accused
were sufficiently and properly protected by Tomas
Posadas. He knew the technical rules of procedure.
Hence, there was a valid waiver of the right to
sufficient representation during the trial, considering
that it was unequivocally, knowingly, and intelligently
made with the full assistance of their new bona fide
lawyer.
PEOPLE vs. BERMAS
306 SCRA 135 (1999)
Facts: Accused, an indigent, was charged with rape,
convicted and sentenced to death. His first lawyer
withdrew after the direct examination of the victim and
cross-examination was done by another de oficio
lawyer who only prepare for 10 minutes. The third de
oficio lawyer also wanted to withdrew but was
prevailed upon by the court and he presented the
accused as witness, but later also ceased to appear for
unknown reasons.

ACADCOM 2010; Contributors: Gene Geocaniga, Jarissa Guiani, Darlene Magabilen


TAU MU Page 155 of 179

TAU MU
TAU MU
TAU MU
TAU MU
TAU MU

Ruling: There is no denial of the right to counsel de


officio was appointed during the absence of the
accuseds counsel de parte, pursuant to the courts
desire to finish the case as easy as practicable under
the continuous trial system. The appointment of
counsel de officio under such circumstances is not
proscribed by the Constitution. The preference in the
choice of counsel pertains more aptly and specifically
to a person under investigation. Neither is there a
violation of appellants right to counsel just because
the trial court did not grant their request for
suspension of the hearing pending their search for new
counsel. The right of the accused to select his own
counsel must be exercised on a reasonable time and in
a reasonable manner. The court may deny accuseds
application is not made for purpose of delay. That the
trial court limited the length of time to cross examine a
witness cannot be labelled as a violation of a
constitutional right. Due process of law is not denied by
the reclusion of irrelevant, immaterial or incompetent
evidence, or testimony of an incompetent witness.

TAU MU

Issue: Whether there is a violation of appellants right


to due process.

TAU MU

Facts: Appellants were found guilty beyond reasonable


doubt of the crimes of kidnapping and serious illegal
detention.

TAU MU

Ruling: Accused has the right to be heard by himself


and counsel.
He has also the right to present
evidence. Accordingly, denial of due process can be
successfully invoked where no valid waiver of rights
has been made. In this case, we find that under the
circumstances, the accused was denied due process
when the successive non-appearance of his counsel
was construed as a waiver of his right to present
evidence. Since he imposable penalty may be death,
the trial court should have been more circumspect in
outrightly denying accused his opportunity to present
his side, particularly since he himself was present
during the four hearings. Clearly, such presence is a
strong indication that accused was interested in
defending himself.
PEOPLE vs. LARRANAGA
421 SCRA 530 (2004)

TAU MU
TAU MU

Facts: Accused was convicted of raping his own


daughter and sentenced to death. In appears that
after the prosecution rested, the case was set for
reception of defense evidence. However, in all four
settings counsel for the accused failed to appear
despite notice. This was treated by the trial court as a
waiver by the accused of his right to present evidence,
and it considered the case submitted for decision.

The Fraternal
Ateneo de Davao

TAU MU
TAU MU

PEOPLE vs. DIAZ


311 SCRA 585 (1999)

KITY

TAU MU TAU MU TAU MU


TAU MU TAU MU TAU MU

CONSTITUTIONAL LAW II
Order of Saint Thomas More
Atty. Philip John Pojas/Atty. Rovyne G. Jumao-as, RN
University College of Law
Ruling: Accused was not properly and efectively
accorded the right to counsel. The right to counsel
proceeds from the fundamental principle of due
process which basically means that a person must be
heard before being condemned. The right to counsel
must be more than just the presence of a lawyer in the
court room or the mere propounding of standard
questions and objections. The right to counsel means
that the accused is amply accorded legal assistance
extended by a counsel who commits himself to the
cause of the defense and acts accordingly. The right
assumes an active involvement by a lawyer in the
proceedings, his bearing constantly in mind of the
basic rights of the accused, his being well-versed on
the case and his knowing the fundamental procedures,
essential laws and existing jurisprudence.

PEOPLE vs. CHIONGBIAN


301 SCRA 614 (1999)
Facts: Due to several postponements sought by a
lawyer of accused, the judge appointed a counsel de
oficio for him during trial. Accused objected to the
services of counsel de oficio since he can aford to hire
a counsel de parte of his own choice. Was there a
violation of the right of the accused to counsel of his
own choice?
Ruling: No. An examination of related provisions in
the Constitution concerning the right to counsel will
show that the preference in the choice of counsel
pertains more aptly and specifically to a person under
custodial investigation rather than one who is an
accused in a criminal prosecution. But even if we were
to extend the application of the concept of preference
in the choice of counsel to an accused in a criminal
prosecution, such preferential discretion cannot
partake of a discretion so absolute and arbitrary as
would make the choice of counsel refer exclusively to
the predilection of accused. Otherwise, the pace of
criminal prosecutions would be entirely dictated by the
accused.
PEOPLE vs. RIVERA
362 SCRA 153 (2001)
Facts: Accused was charged with and convicted of
incestuous rape. During order trial date his counsel
failed to appear so that the judge assigned him a
counsel de officio.
The lawyer moved for a
postponement of the cross-examination of the
physician-witness, but the judge denied the motion
because the witness was a very busy person. Thus,
the lawyer decided not to cross-examine. The accused
now argues that the assignment by a counsel de officio
in the absence of his own lawyer violated the right to
counsel?
Ruling: While the Constitution recognized the right of
the accused to be represented b counsel of his own
choice, his option to secure the services of a private
counsel is not absolute. For considering the States
and the ofended partys right to speedy and adequate
justice, the court may restrict the option of the accused
to retain a private counsel if he insists on a counsel he
cannot aford, or if the chosen counsel is not a member
of the bar, or the latter declines to represent accused.
Courts are not required to wait indefinitely the pleasure
and convenience of the accused as they are also
mandated
to
promote
speedy
and
orderly
administration of justice.
PEOPLE vs. LIWANAG
363 SCRA 62 (2001)
Facts: Accused was convicted of highway robbery with
multiple rape. During trial he was assisted b counsel
de oficio, a PAO lawyer. In the middle of the trial, he
retained the services of counsel de parte. After he was
convicted by the trial court, another lawyer filed the
notice of appeal but failed to file the appellants brief.
Before the Supreme Court he was represented by
another counsel de officio. He now claims that his right
to counsel was violated because his counsel made
insufficient cross-examination of the prosecution
witnesses, and failed to impeach the testimony of
complainant through the use of contradictory evidence.

TAU MU

Ruling: The right to be assisted by counsel does not


presuppose the right to an intelligent counsel. The
requirement is not for counsel to be intelligent, but to
be efective. While fairness is the object of Art. III, Sec.
14 (2) of the Constitution, the assistance aforded by
the counsel to an accused need only be in accordance
ACADCOM 2010; Contributors: Gene Geocaniga, Jarissa Guiani, Darlene Magabilen
TAU MU Page 156 of 179

TAU MU
TAU MU
TAU MU

PURPOSES OF THE RIGHT:


1) To furnish the accused with a description of the
charge against him as will enable him to make his
defenses
2) To avail himself of his conviction or acquittal against
a further prosecution for the same cause
3) To inform the court of the facts alleged.

TAU MU

4. RIGHT TO BE INFORMED OF THE NATURE


AND CAUSE OF ACCUSATION AGAINST HIM
In all criminal prosecutions, the accused shall have the
right to be informed of the nature and cause of the
accusation against him.

TAU MU

Ruling: Counsels decision not to cross-examine the


witness is a glaring example of his manifest lack of
enthusiasm for his clients clause. It may be that
defense counsel personally found the testimony to be
believable. Nevertheless, he had the bounded duty to
scrutinize private complainants testimony to ensure
that the accuseds constitutional rights to confront and
examine the witnesses against him was not rendered
for naught.

TAU MU

Facts: After the rape victim testified, the following


dialogue occurred:
COURT: Any cross?
ATTY. BROTONEL:
If Your Honor please,
we are not conducting any cross-examination, because
this representation, from eh demeanor of the witness, I
am convinced that she is telling the truth.

[PEOPLE vs. GRADA]: It was not the accused name


listed in the information, but that of his brother.
Deprivation? No, because he knew of the nature and
the cause of the accusation against him. It was just a
wrong name; it does not vitiate the case. Besides, he
did not raise it upon arraignment, so he is deemed to
have waived the right.
Higher crime charged but convicted of lower
ofense, there is no violation of this right; lower ofense
charged but convicted of higher ofense, there is a
violation of the right to be informed because lower
crime does not involve the higher crime;
VOID FOR VAGUENESS RULE: The accused is
denied of his right to be informed of the charge against
him and to due process as well, when the statute itself
is stated in such indefinite language that it is not
possible for men of ordinary intelligence to determine
what acts or omissions are punished. In such a case,
the law is deemed void.
The accused must be informed of the facts that are
imputed to him (cardinal requisite). Thus, the
information referred to must describe the act with
sufficient particularity.
The right to assail the sufficiency of the
information or the admission of evidence may be
waived by the accused if he fails to object to its
sufficiency during trial and the deficiency is cured by
competent evidence presented therein [PEOPLE vs.
PALARCA May 29, 2002)].
The information need not allege the precise
time of commission of the ofense, unless the time of
an essential element of the crime charged.

TAU MU

PEOPLE vs. NADERA


324 SCRA 490 (2000)

2. You can be convicted in possession if acquitted


in using prohibited drugs.
3. The word minor is not sufficient, age must be
mentioned and if uncle did the act of rape, it must be
stated that it is within the 4th civil degree.

TAU MU

Ruling: Accused acted as his own counsel. To allege


now that his right to be assisted by counsel was
violated is to bend the truth too far. The constitutional
right of the accused to counsel is not violated where he
was represented by a prominent and competent
member of the Bar, namely himself, even if there were
others available. He is now estopped from claiming
that the trial court violated his right to be represented
by counsel of his own choice.

Ateneo de Davao

TAU MU
TAU MU

Facts: Accused was charged with murder. Being a


practicing lawyer, he insisted on representing himself.
Despite proddings by the court and an ofer of the
possibility of assistance from the Public Attorneys
Office, he handled his own defense and was convicted.
On appeal, he claims that his right to counsel was
violated.

The Fraternal

TAU MU
TAU MU

PEOPLE vs. SESBRENO


314 SCRA 87 (1999)

KITY

TAU MU TAU MU TAU MU


TAU MU TAU MU TAU MU

CONSTITUTIONAL LAW II
Order of Saint Thomas More
Atty. Philip John Pojas/Atty. Rovyne G. Jumao-as, RN
University College of Law
with the provisions of Rules of Court and the Code of
Professional Responsibility. In the Philippine setting, a
counsel assisting an accused is presumed to be
providing all the necessary legal defense which are
reasonable under the circumstances in accordance
with said norms. The proper measure of attorney
performance remains simply reasonableness under
prevailing norms. Coupled with the presumption that
counsels performance was reasonable under the
circumstances, as long as the trial was fair in that the
accused was accorded due process by means of an
efective assistance of counsel, then the constitutional
requirement that an accused shall have the right to be
heard by himself and counsel is satisfied.

If the information fails to allege the material


elements of the ofense, the accused cannot be
convicted thereof even if the prosecution is able to
present evidence during the trial with respect to such
elements.
The real nature of the crime charged is determined
from the recital of facts in the information. It is not
determined based on the caption or preamble thereof
or from the specification of the provision of law
allegedly violated.

RULE 110 PROSECUTION OF OFFENSES


Sec. 6. Sufficiency of complaint or information. A
complaint or information is sufficient if it states the name
of the accused; the designation of the ofense given by the
statute; the acts or omissions complained of as
constituting the ofense; the name of the ofended party;
the approximate date of the commission of the ofense;
and the place where the ofense was committed.
xxx
xxx
xxx
Sec. 8. Designation of the offense. The complaint or
information shall state the designation of the ofense given
by the statute, aver the acts or omissions constituting the
ofense, and specify its qualifying and aggravating
circumstances. If there is no designation of the ofense,
reference shall be made to the section or subsection of the
statute punishing it.

What is essential is that the accused be


informed of the facts alleged against him, not the
characterization of the crime which is a conclusion of
law.
5. RIGHT TO SPEEDY, IMPARTIAL AND PUBLIC
TRIAL

TAU MU

ADEQUACY OF INFORMATION
1. If the information is more serious to less
IMPARTIAL TRIAL
serious it can be done.
ACADCOM 2010; Contributors: Gene Geocaniga, Jarissa Guiani, Darlene Magabilen
TAU MU Page 157 of 179

TAU MU
TAU MU
TAU MU
TAU MU
TAU MU

The right to a speedy disposition of a case, like the


right to speedy trial, is deemed violated only when the
proceeding is attended by vexatious, capricious, and
oppressive delays; or when unjustified postponements
of the trial are asked for and secured, or when without
cause or justifiable motive a long period of time is

TAU MU

REMEDY OF THE ACCUSED IF HIS RIGHT TO


SPEEDY TRIAL HAS BEEN VIOLATED:
1. He can move for the dismissal of the case.
2. If he is detained, he can file a petition for the
issuance of writ of habeas corpus.

TAU MU

EFFECT OF DISMISSAL BASED ON THE GROUND


OF VIOLATION OF THE ACCUSEDS RIGHT TO
SPEEDY TRIAL:
If the dismissal is valid, it amounts to an acquittal and
can be used as basis to claim double jeopardy. This
would be the efect even if the dismissal was made
with the consent of the accused

TAU MU

FACTORS USED IN DETERMINING WHETHER THE


RIGHT TO A SPEEDY TRIAL HAS BEEN
VIOLATED:
1) Time expired from the filing of the information
2) Length of delay involved or extent of delay (actual
lapse of time)
3) Reasons for the delay
4) Assertion or non-assertion of the right by the
accused or the invocation of the accused of this right.
Take Note: This right can be waived and when
there is no objection against any postponement, then
there is implied waiver).
5) Prejudice caused to the defendant.

TAU MU

Right to speedy trial

TAU MU
TAU MU

SPEEDY TRIAL
It is one free from vexatious, capricious and
oppressive delays and is in inconveniences before
sentence is pronounced upon him. This would also be
consonant with Section 16 of the Bill of Rights
providing that all persons shall have the right to a
speedy disposition of their cases before all judicial,
quasi-judicial, or administrative bodies.
Although the trial is an indispensable and,
indeed, the most important part of the proceedings
against the accused, it has been held that the right to
be present thereat is a personal right and therefore
may be validly waived.

The Fraternal
Ateneo de Davao

TAU MU
TAU MU

PUBLICITY OF THE TRIAL


It is necessary to prevent abuses that may be
committed by the court to the prejudice of the
defendant. The people have the right to attend the
proceedings not only because of their interest therein
but also so they can see whether or not the
constitutional safeguards for the benefit of the accused
are being observed. The accused is also entitled to the
company of his relatives and friends to give him the
moral support he needs during his ordeal.
This rule is not absolute, however, for it is
competent for the court to bar the public in certain
cases, like rape trials, where the purpose of the
spectators in attending the proceedings might be only
to pander to their morbid curiosity, more since their
presence is likely to inhibit testimony and embarrass
some of the parties.

KITY

TAU MU TAU MU TAU MU


TAU MU TAU MU TAU MU

CONSTITUTIONAL LAW II
Order of Saint Thomas More
Atty. Philip John Pojas/Atty. Rovyne G. Jumao-as, RN
University College of Law
It has been previously remarked that this requirement
will call for no less than the cold neutrality of an
IMPARTIAL judge, to insure that justice is done to the
defendant. In another case, it was held as part of the
rule that the judge must not only be impartial but must
also appear to be impartial.

allowed to elapse without the party having his case


tried. Equally applicable is the balancing test used to
determine whether a defendant has been denied his
right to a speedy trial, or a speedy disposition of a case
for that matter, in which the conduct of both the
prosecution and the defendant are weighed, and such
factors as length of the delay, reason for the delay, the
defendants assertion or non-assertion of his right, and
prejudice to the defendant resulting from the delay, are
considered. [SOLAR TEAM ENTERTAINMENT vs. HOW,
338 SCRA 511]
Right to impartial trial
Definition of impartial trial
The accused is entitled to the full cold neutrality of an
impartial judge. It is an element of due process. The
right to an impartial judge must not only be in reality,
but also in appearance. The judge must not only be
impartial but must look impartial; otherwise, the judge
must inhibit himself from trying the case.
[TABUENA vs. SANDIGANBAYAN]: Judges can only
clarify matters, and not impeach the accused. The
judge cannot ask questions to find out whether the
accused is guilty or not. The judge should not act as
the prosecutor of the accused.
The right to have an impartial Judge. Impartiality
must not only be in reality, but also even in
appearance.
The judge should not be bias but objective; ruling
based on information.
The right to an impartial trial is not a bar to a
judges intervention in cross-examination.
A litigant is entitled to the cold neutrality of an
impartial judge.
The judge should file a motion for inhibition if he is
impartial.
TWO PROCEEDINGS AGAINST A BIASED JUDGE:
a) Disqualification;
b) Inhibition.
PUBLIC TRIAL
The attendance at the trial is open to all irrespective of
their relationship to the accused. However, if the
evidence to be adduced is ofensive to decency or
public morals, the public may be excluded.
The right of the accused to a public trial is not
violated if the hearings are conducted on Saturdays,
either with the consent of the accused or if failed to
object thereto.
PURPOSE OF PUBLIC TRIAL
1. To prevent possible abuses of judicial power
that could be committed against him.
2. To prevent the courts from being instruments
of persecution.
NOTE: The right belongs to the accused and therefore,
it can be WAIVED.
But waiver is not to be inferred from mere failure to
urge the trial of the case. Waiver or abandonment may
be presumed only when the postponement of the trial
has been sought and obtained by the accused himself
or by his attorney [KALAW vs. APOSTOL (64 Phil. 859)].

The presumption is always against the waiver of


constitutionally protected rights.

The REMEDY of the accused for VIOLATION of his


right to SPEEDY TRIAL is dismissal of the case and, if he
is under detention, release by habeas corpus.
Moreover, dismissal for violation of the right to speedy
trial is equivalent to acquittal and is therefore a bar to
a subsequent prosecution for the same ofense.

ACADCOM 2010; Contributors: Gene Geocaniga, Jarissa Guiani, Darlene Magabilen


TAU MU Page 158 of 179

TAU MU
TAU MU
TAU MU

PEOPLE vs. SANCHEZ


302 SCRA 21 (1999)

TAU MU

CASES ON RIGHT TO SPEEDY, IMPARTIAL AND


PUBLIC TRIAL

TAU MU

only because of their interest therein but also so they


can see whether or not the constitutional safeguards
for the benefit of the accused are being observed.
This rule is not absolute, however, for it is
competent for the court to bar the public in certain
cases, like rape trials, where the purpose of the
spectators in attending the proceedings might be only
to pander to their morbid curiosity, more since their
presence is likely to inhibit testimony and embarrass
some of the parties.

TAU MU

People have a right to attend the proceedings not

TAU MU

TRIAL WITH LIVE MEDIA COVERAGE (ESTRADA


CASE):
Balancing of rights present in the case:
a) Accused Fair Trial
b) Press Freedom of the Press
c) Public Right to information
d) Court Proper Administration of Justice
There would be no live media coverage.
The Estrada case covers immunity by suit, thus it is
not yet a trial.
EXCEPTION: 1 Camera for the purpose of history
but not simultaneous with the trial.

TAU MU

[CRUZ vs. SILVA]: Where the Fiscal chose the


courtroom, installed microphones and invited the
press, allowing them to ask questions, in a preliminary
investigation involving a sensational killing. The court
held the Fiscal in contempt. This is a case of trial by
publicity.

TAU MU

TRIAL BY PUBLICITY
This is a violation of the right to an impartial trial as too
much publicity pressures the judge. However, it is not
that easy to use this ground as a violation of this right.
The test to be passed is the test of actual prejudice.
To warrant a finding of prejudicial trial by publicity,
there must be allegation and proof that the judges
have been actually influenced, and not simply that
they might be influenced. So, there is a diference
between the judges MAY BE influenced, that is not
enough, there must be proof that the judges were
actually influenced.

TAU MU
TAU MU

A PUBLIC TRIAL WHICH IS NOT HELD IN COURT IS


VALID IF:
1) Public was not excluded.
2) Accused was not prejudiced.
3) Accused did not object during trial

The Fraternal
Ateneo de Davao

TAU MU
TAU MU

Q: What is the reason why trial has to be public?


A: So that the public may see that the accused is fairly
dealt with, and to give the judge a line to attend to his
responsibility.

KITY

TAU MU TAU MU TAU MU


TAU MU TAU MU TAU MU

CONSTITUTIONAL LAW II
Order of Saint Thomas More
Atty. Philip John Pojas/Atty. Rovyne G. Jumao-as, RN
University College of Law

The general public may be excluded from trial


when the evidence to be presented in the proceeding is
ofensive to public decency and morals [Rule 119
Sec. 13]. BUT even in such instances the accused
should be allowed to have his friends, relatives and
counsel present [IN RE: OLIVER 333 US 257].

The purpose of the RIGHT TO A PUBLIC TRIAL is to


safeguard against any attempt to employ our courts as
instruments of persecution. IT is an efective restraint
against possible abuse of judicial power because it the
trial will be subject to contemporaneous review in the
form of public opinion.

Facts: Mayor Sanchez was convicted of seven courts


of rape with homicide. Considering the position of
accused, the trial was accompanied by widespread
media coverage. On appeal, Sanchez claims that his
right to a fair trial was violated due to the intense
publicity.
Ruling: The right of an accused to a fair trial is not
incompatible with a free press. Pervasive publicity is
not per se prejudicial to the right of an accused to fair
trial. It does not by itself prove that the publicity so
permeated the mind of the trial judge and impaired his
impartiality.
At best appellant can only conjure
possibility of prejudice on the part of the trial judge due
to the barrage of publicity. But the test is not the
possibility of prejudice but actual prejudice. To warrant
a finding of prejudicial publicity, there must be
allegation and proof that judges have been unduly
influenced, not simply that they might be. Appellant
must discharge this burden. In this case, there is no
proof that the judge acquired a fixed opinion, or actual
bias as a consequence of extensive media coverage.
ESTRADA vs. DESIERTO
356 SCRA 109 (2001)
Facts: Petitioner seeks a reconsideration of the
decision of the Supreme Court declaring that having
resigned from the presidency, he may be prosecuted
for Plunder. Among others, he contends that his right
to an impartial trial has been afected by the prejudicial
pre-trial publicity of the proceedings before the
Ombudsman.
He also points to the alleged hate
campaign launched by some newspapers so that the
prosecution and the judiciary can no longer assure him
of a sporting chance. He urges the Court to apply the
rule on res ipsa loquitor. Has petitioners right to fair
trial been violated?
Ruling: No. The mere fact that the proceedings was
given a day to day coverage does not prove that the
publicity so permeated the mind of the tribunal and
impaired his impartiality.
To warrant a finding of
prejudicial publicity, there must be allegation and proof
that the judges have been unduly influenced, not
simply that they might be. In this case, petitioner has
failed to adduce any proof of actual prejudice
developed by the members of the Panel of
Investigators of the Ombudsman. We cannot replace
this test of actual prejudice with the rule of res ipsa
loquitur. The latter rule assumes that an injury has
been sufered and then shifts the burden to the panel
of investigators to prove that the impartiality of its
members has not been afected by said publicity. Such
a rule will overturn our case law that pervasive
publicity is not per se prejudicial to the right of an
accused to fair trial. For this reason, we continue to
hold that it is not enough for petitioner to conjure
possibility of prejudice but must prove actual prejudice
on the part of his investigators for the Court to sustain
his plea.
RE: REQUEST RADIO
360 SCRA 248 (2001)
Facts: The Kapisanan ng mga BroadKaster ng
Pilipinas, an association representing duly franchised
and authorized television and radio networks,
requested the Supreme Court to allow live media
coverage of the trial of former President Estrada. The
request was anchored on the need to assure the public
of full transparency in the proceedings. In efect, the
request seeks reconsideration of the 1991 resolution of
the Court which barred live media coverage of all court
proceedings.

ACADCOM 2010; Contributors: Gene Geocaniga, Jarissa Guiani, Darlene Magabilen


TAU MU Page 159 of 179

TAU MU
TAU MU
TAU MU
TAU MU

Facts: The direct examination of the witness presented


by the accused for his defense consisted of 16
questions, while the prosecutor propounded 6 crossexamination questions. The court asked 27 questions
and added 10 more later. When accused himself
testified, the prosecutor asked 14 cross-examination
questions, while the court asked 67 questions which
were in the nature of cross-examination. Was there a
violation of the right to due process?
Ruling: The accused basic constitutional right to due
process was violated and this appears to be a more
compelling reason for their acquittal. While a trial
judge has a right to question witnesses, his
examination should be limited to asking clarificatory
questions. The impression cannot be avoided that the
Sandiganbayan had taken the cudgels for the
prosecution.
The cold neutrality of an impartial
judge requirement of due process was denied the
accused

TAU MU

TABUENA vs. SANDIGANBAYAN


268 SCRA 332 (1997)

TAU MU

Ruling: The motion is denied. However, because of


the significance of the trial and the importance of
preserving the records, there should be an audio visual
recording of the proceedings. The recordings will not
be for live or real time broadcast but for documentary
purposes. Only later will they be available for public
showing, after the Sandiganbayan shall have
promulgated its decision. The master film shall be
deposited in the National Museum and the Records
Management ad Archives Office for historical
preservation and exhibition pursuant to law.
The
audio-visual recording shall be made under the
supervision and control of the Sandiganbayan.

TAU MU
TAU MU

Facts: The Secretary of Justice seeks a reconsideration


of the resolution denying permission to televise and
broadcast live the trial of President Estrada before the
Sandiganbayan. Among others, he argues that if there
is a clash between the rights of the people to public
information and the freedom of the press, on the one
hand, and the right of the accused to fair trial, it should
be resolved in favor of the right of the people, because
the people, as repository of democracy are entitled to
information; and that live media coverage is a
safeguard against attempts by any party to use the
courts as instruments for the pursuit of selfish interest

The Fraternal
Ateneo de Davao

TAU MU
TAU MU

RE: REQUEST FOR LIVE


365 SCRA 62 (2001)

KITY

TAU MU TAU MU TAU MU


TAU MU TAU MU TAU MU

CONSTITUTIONAL LAW II
Order of Saint Thomas More
Atty. Philip John Pojas/Atty. Rovyne G. Jumao-as, RN
University College of Law
Ruling:
The issue involves the weighing out of
constitutional guarantees of freedom of the press and
the right to public information, of the one hand, and
the fundamental rights of the accused, on the other
hand, along with the constitutional power of a court to
control its proceedings in ensuring a fair and impartial
trial. When these rights race against each other, the
right of the accused must be preferred. With the
possibility of losing his life or liberty, it must be
ensured that accused receives a verdict decreed by an
imprejudiced judge. Television coverage, however, can
impair the testimony in criminal trials, can the afect
the performance of the judge, and can destroy the case
of the accused in the eyes of the public. Accordingly,
to protect the parties right to due process, to prevent
the distraction of the participants in the proceedings
and in the last analysis, to avoid miscarriage of justice,
the request is denied.

Facts: Accused was charged with violation of the


Dangerous Drugs Act. Arraigned on August 8, 1995,
trial never commenced despite the lapse of 1 year and
3 months due to 11 postponements, 9 of which were
secured by the prosecution.
The grounds for
postponements ranged from the absence of witnesses
for the prosecution, absence of the prosecutor and reraffling of the case to another branch. Must the case
be dismissed on the ground of speedy trial?
Ruling:
No.
The reasons for the prosecutions
postponements were reasonable and were not
intended merely to delay the proceedings of the case.
It would be unjust to pounce on the absence of the
witnesses as a basis for dismissing the case when
there was a valid excuse for their absence, that is,
there was no proof that they were duly served with
subpoena. The other reasons for postponements were
due to circumstances beyond the control of the
prosecution. The right of the accused to speedy trial
should not be utilized to deprive the State of a
reasonable opportunity of fairly prosecuting criminals.
DE ZUZUARREGUI vs. ROSETE
382 SCRA 1 (2002)
Facts: A complaint was filed against Judge Rosete for
gross ignorance of the law for dismissing with
prejudiced criminal case for falsification of a private
document filed by a complainant.
Issue: Whether there was an abuse of discretion on
the part of the respondent.
Ruling: The requirement for postponement was not for
any flimsy excuse but complainant was in the US for an
operation. The right to a speedy trial is a relative one.
Courts should do more than a mathematical
computation of the number of postponements. What
ofends the right to speedy trial are unjustified
postponements which prolong trial for an unreasonable
length of time.
LUMANLAW vs. PERALTA
482 SCRA 396 (2006)
Facts: Lumanlaw was apprehended for illegal
possession of a dangerous drug. A year had passed
since the filing of the Information, yet Lumanlaw
remained uninformed of the charges against him, while
continuing to be in detention and despair all
throughout that period of limbo. Owing to this
insuferable state of afairs, petitioners counsel
manifested his intention to file a motion to dismiss on
account of the violation of his clients right to a speedy
trial.
Issue: WON there was a violation of Lumanlaws
constitutional right to speedy trial.

TAU MU

Ruling: There were fourteen postponements in this


case. Given the length and the unreasonableness of
the majority of the delays, a violation of the right of
petitioner to speedy trial becomes manifest. Almost
two years elapsed from the filing of the Information
against him until the filing of this Petition; incredibly,
he has not been arraigned. An arraignment takes, at
most, ten minutes of the courts business and does not
normally entail legal gymnastics. It consists simply of
reading to the accused the charges leveled against
them, ensuring their understanding of those charges,
and obtaining their plea to the charges. A prudent and
TAI LIM vs. CA
resolute judge can conduct an arraignment as soon as
317 SCRA 521 (1999)
the accused are presented before the court.
In fact, by fixing a period of only thirty days from the
filing of the information to the conduct of an
ACADCOM 2010; Contributors: Gene Geocaniga, Jarissa Guiani, Darlene Magabilen
TAU MU Page 160 of 179

TAU MU

TAU MU

SUMBANG vs. GENERAL COURT MARTIAL


337 SCRA 227 (2000)

TAU MU
TAU MU
TAU MU
TAU MU
TAU MU

FAILURE OF THE ACCUSED TO CROSS-EXAMINE A


WITNESS:
If the failure of the accused to cross-examine a witness
is due to his own fault or was not due to the fault of the
prosecution, the testimony of the witness should be
excluded.

TAU MU

Testimony not subjected to cross-examination must


be excluded from consideration. HOWEVER if crossexamine actually commenced, but for lack of material
time, it was not completed and the witness died before
it could be resumed, so much of the testimony already
covered by cross-examination is admissible [PEOPLE
vs. SEBERIS 99 SCRA 92].
An exception to the requisite of confrontation is a
dying declaration w/c are declarations made in
extremity, when the party is at the point of death, and
when every hope of this world is gone; when every
motive to falsehood is silenced and the mind is induced
by the most powerful considerations to speak the truth.
A situation so solemn and so awful as to be considered
by the law as creating an obligation equal to that which
is imposed by a positive oath in a court of justice [US
vs. GIL (1909)].

TAU MU

PURPOSES:
a.
to aford the accused an opportunity to test the
testimony of the witness by cross-examination and
b.
to allow the judge to observe the deportment of
the witness
[GO vs. COLLECTOR OF CUSTOMS (1934)].

TAU MU

6. RIGHT TO MEET WITNESS FACE TO FACE


(CONFRONTATION)

TAU MU
TAU MU

Ruling: The determination of whether an accused has


been denied the right to speedy trial must have to
depend on the surrounding circumstances of each
case. Although it is unfortunate that it took about 8
years form 1991 before the trial resumed in 1999, the
delay does not amount to violation of the right
considering that it could not be attributable to the
prosecution. The delay was due to the changes in the
composition of the court martial. Notably, from the
time petitioners motion to dismiss was file in 1991, he
did not take action to assert his right. Thus, the
supervening delay seems to have been without his
objection hence impliedly with his acquiescence. The
right to speedy trial may be waived.

The Fraternal
Ateneo de Davao

TAU MU
TAU MU

Facts: Petitioner was a member of the Philippine


Constabulary accused before a court martial of killing a
civilian on May 29, 1988. In view of the efectivity of
RA No. 6975 otherwise known as the Philippine
National Police Law, the composition of the court
martial was changed so that the case remained
pending for years.
On Sept. 29, 1999, petitioner
moved for the dismissal of the case on the ground of
violation of the right to speedy trial.

KITY

TAU MU TAU MU TAU MU


TAU MU TAU MU TAU MU

CONSTITUTIONAL LAW II
Order of Saint Thomas More
Atty. Philip John Pojas/Atty. Rovyne G. Jumao-as, RN
University College of Law
arraignment, RA 8493 recognizes that this fundamental
right should and can be done with minimal delay. The
protracted delay became all the more oppressive and
vexatious when viewed from the perspective that the
liberty of the accused was being curtailed for the entire
duration. Hence, in the light of the numerous and
unreasonable delays in the arraignment of petitioner,
the sought for dismissal of the Information filed against
him is in order.

WHEN IS THE RIGHT TO CROSS-EXAMINE


DEMANDABLE:
It is demandable only during trials. Thus, it cannot be
availed of during preliminary investigations.
PRINCIPAL EXCEPTIONS TO THE RIGHT OF
CONFRONTATION:
1. The admissibility of dying declarations
2. Trial in absentia under Section 14(2)
3. With respect to child testimony
Q: What are not admissible as evidence in court?
A: (1) Extra judicial statements of an accused
implicating another when not repeated in open court;
and (2) Affidavits of Witnesses not produced in Court to
be cross-examined.
7. RIGHT TO HAVE COMPULSORY PROCESS
TO SECURE THE ATTENDANCE OF WITNESSES
AND HE PRODUCTION OF EVIDENCE IN HIS
BEHALF
This right to compulsory process must be invoked
during the trial. Failure to do so constitutes a waiver
that cannot be rectified or undone on appeal.

A person accused can obtain a subpoena from the


court in order to compel the attendance of witnesses in
his behalf.
A refusal of subpoena will result to detention until
such time that compliance is made.
EXCEPTION: If the person resides more than 100
kilometers from the place of trial, he is not bound by a
subpoena. This rule applies only to Civil cases, not to
Criminal cases.
.
This right includes the right to testify in ones favor
[PEOPLE vs. SANTIAGO (46 Phil. 734)] and the right to
be given time to call witnesses [US vs. LAO CHUECO
(37 Phil. 53)].
This right may NOT be invoked on appeal if the
accused made no efort during trial to avail of himself
of it [US vs. GARCIA (1908)].
To establish the right to continuance by reason of the
absence of witnesses, the accused must show that:
a. the witness is really material;
b. he is guilty of no neglect of previously obtaining
attendance of said witness;
c. the witness will be available at the time desired;
and
d. no similar evidence could be obtained.
[PEOPLE vs. SANDAL (54 Phil. 883)]
TRIAL IN ABSENCIA
The right of the accused to be present during trial
after arraignment may be waived because this
provision allows trial to continue even w/o the presence
of the accused.
The accuseds present during trial may be totally
waived except when, after arraignment, his presence is
needed for purposes of identification. The reason is to
forestall the accused from using the defense that he
was never identified as the person charge in the
information and therefor entitled to acquittal.
For an accused to be excused from attending trial,
he must unqualifiedly admit that every time a witness
mentions a name by w/c he is known, the witness is to
be understood as referring to him [VILLAVICENCIO vs.
LUKBAN (39 Phil. 778)].
The purpose of this manner of trial is to prevent
unnecessary delays of the trial.

TAU MU

REQUISITES of TRIAL IN ABSENCIA:


1. Accused has been arraigned;
ACADCOM 2010; Contributors: Gene Geocaniga, Jarissa Guiani, Darlene Magabilen
TAU MU Page 161 of 179

GENERAL RULE: The right to attend trial is waivable.


EXCEPTION:
1. Arraignment
2. During the identification stage (witness
testifies to the identity of the accused)
3. Promulgation of judgment
Q: When can an accused be compelled by the
court to attend trial?
A: (1) Arraignment; (2) During the identification stage
(witness testifies to identify the accused); and (3)
Promulgation of judgment.

KITY

TAU MU
TAU MU

Note: The accused may not invoke such right on


appeal if he made no efort during the trial to avail
himself of it.
CASE ON TRIAL IN ABSENTIA:

TAU MU
TAU MU
TAU MU

Ruling: No. Once jurisdiction is acquired, it is never


lost.

TAU MU

Issue 1: Whether the court loses jurisdiction over a


person who escapes.

TAU MU

Facts: There were several accused and after he was


arraigned accused escaped. The trial proceeded in his
absence and the judge convicted his co-accused.
However, insofar as the one who escaped, the judge
withheld his decision that he can enjoy his
constitutional right to confrontation if he is arrested.
(Some think that this is wrong. Accused should be
judged basing on the evidences presented during the
trial in his absence.)

TAU MU

JIMENEZ vs. NAZARENO

TAU MU

Ruling: No. Under Sec. 14 (2) of the Bill of Rights, the


following are the requisites of a valid trial in absentia:
1) accused had already been arraigned; 2) he has been
duly notified of the trial, and 3) his failure to appear is
unjustifiable. In this case accused had not been duly
notified of the trial because notice of hearing was sent
to his former address despite the fact he notified the
court of his change of address.

TAU MU

Facts: Accused was charged with estafa and was out


on bail. While trial was going on, accused changed his
address notifying the court through his counsel as well
as the bonding company. When accused failed to
appear during a hearing because notice was sent to his
old address, the judge issued a warrant for his arrest,
appointed a counsel de officio for him, ordered a trial in
absentia, and convicted him on the theory that he
waived his right to present evidence. Was there a valid
trial in absentia?

TAU MU
TAU MU

TO ESTABLISH THE RIGHT TO CONTINUANCE BY


REASON OF THE ABSENCE OF WITNESS, THE
ACCUSED MUST SHOW:
1) That the witness is really material;
2) That he is guilty of not neglect previously obtaining
attendance of said witness;
3) That the witness will be available at the time
desired.
4) That no similar evidence could be obtained from
other witnesses.

PARADA vs. VENERACION


269 SCRA 371 (1997)

The Fraternal
Ateneo de Davao

TAU MU TAU MU TAU MU


TAU MU TAU MU TAU MU

CONSTITUTIONAL LAW II
Order of Saint Thomas More
Atty. Philip John Pojas/Atty. Rovyne G. Jumao-as, RN
University College of Law
2. Notice of the trial was duly served to him and
properly returned;
3. His failure to appear is unjustified or he waives his
right to be present.

Issue 2: Whether the accused who escaped retains his


right to confrontation to present evidence to cross
examination and to be heard.
Ruling: These rights are no longer retained once the
accused escapes. The provision on trial in absentia will
be useless. Escape tantamounts tp a waiver to your
right to present evidence, confrontation, and etc.
Sec. 15: The privilege of the writ of habeas
corpus shall not be suspended except in cases of
invasion or rebellion when public safety requires
it.
WRIT OF HABEAS CORPUS
It is a prerogative writ of liberty employed to test
the validity of a persons detention
It is directed to the person detaining another,
commanding him to produce the body of the prisoner
@ a designated time and place, w/ the day and cause
of his caption and detention, to do, to submit to, and
receive whatever the court or judge awarding the writ
shall consider in his behalf

The OBJECT of this writ is the liberty of those who


may be in prison w/o sufficient cause

An order issued by a court directed to a person


(illegally) detaining another, commanding him to
produce the body of the prisoner at a designated time
and place, and to explain the cause of his detention.
(Cross reference: Special Proceedings)

The Habeas Corpus is not filed against a person. It


is a special proceeding and not a civil action. If it were
civil, then there is the plaintif and defendant. Here,
there is only the petitioner. You petition the court to
issue a writ directing someone to return the person
illegally detained.

If a person is restrained of his liberty, he or


someone acting on his behalf may file a petition for
habeas corpus to secure his release. This action shall
take precedence in the calendar of the court and must
be acted upon immediately.

Mere delay in the resolution of the issue will by


itself constitute an invalid derogation of a persons
right to unlawful restraint.
Rule 102, Rules of Court: Sec. 1. To what habeas
corpus extends. - Except as otherwise expressly
provided by law, the writ of habeas corpus shall extend
to all cases of illegal confinement or detention by
which any person is deprived of his liberty, or by which
the rightful custody of any person is withheld from the
person entitled thereto.
The following grounds are recognized by
the SC for the issuance of the writ of habeas
corpus:
4. deprivation
of
any
fundamental
or
constitutional right;
5. lack of jurisdiction of the court to impose the
sentence;
6. imposition of excessive penalties;
Q: When is this remedy available?
A: The writ may be resorted to where a persons liberty
is subjected to physical restraint.
Q: Is this right/remedy limited to cases of
physical restrain?
A: NO. Even moral restraint is a ground for the
issuance of this writ [CUANCA vs. SALAZAR].

TAU MU

INSTANCES WHEN THE WRIT CAN BE AVAILED OF:


1. If a person has finished serving his
sentence, his incarceration has become unlawful.

ACADCOM 2010; Contributors: Gene Geocaniga, Jarissa Guiani, Darlene Magabilen


TAU MU Page 162 of 179

TAU MU
TAU MU
TAU MU
TAU MU
TAU MU
TAU MU

INSTANCES WHEN THE JUDGE WILL NOT ISSUE


THE WRIT:
1. If the person is in the custody of an officer and
under process issued by a court or judge or by virtue of
a judgment or order of a court of record, and that the
court or judge had jurisdiction to issue the process,
render the judgment, or make the order. Meaning legal
detention. No way will the court issue the writ;
2. When a person has been judicially charged
with or convicted or a crime in the Philippines. Again,
legal detention. You cannot use the writ if the ofense is
bailable and you cannot post bail. You cannot use the
writ to get out of jail;
3. If the jurisdiction appears after the writ is
allowed, the person shall not be discharged by reason
of any informality or defect in the process, judgment or
order;
4. A person sufering imprisonment under lawful
judgment;
5. In the case of Panillo vs. Salonga (June 24,
1994), if the issue of detention raised in the petition for
a writ of habeas corpus is necessarily related with
another pending case;
6. If the petitioner has the remedy of appeal;
7. When the writ is suspended and the person is
being detained for the crimes covered by the
suspension.

TAU MU

Note: Non-production of the person detained is


sufficient ground to hold the ofender in contempt of
court. The ofender must exert all eforts to produce
the body, mere writing of letters is not enough.

TAU MU
TAU MU

THREE WAYS OF COMPLYING WITH THE WRIT OF


HABEAS CORPUS:
1. Produce the body;
2. If the person is sick and cannot be safely
brought to court, affidavit;
3. Proving, by means of affidavit, that the person
detained has waived his right to be present (the person
who is being detained will produce the affidavit)

The Fraternal
Ateneo de Davao

TAU MU
TAU MU

PURPOSE: Best remedy for personal freedom because


by the Writ of HC, the object is the speedy release by
judicial decree of persons who are illegally detained or
deprived of their liberty. This is a weapon against
arbitrary use of State power.

KITY

TAU MU TAU MU TAU MU


TAU MU TAU MU TAU MU

CONSTITUTIONAL LAW II
Order of Saint Thomas More
Atty. Philip John Pojas/Atty. Rovyne G. Jumao-as, RN
University College of Law
The authorities no longer have any right to keep him in
prison. If he is not released, then his relatives can file
for Habeas Corpus.
2. When patients are detained in hospitals
for failure to pay the bill. If the corpse is not
released, will the writ apply? No, because the Bill of
Rights can only be availed of by living citizens of the
Philippines, or aliens.
3. Custody of Children
4. When a person is denied of one or more
of his constitutional rights.
5. When
restraint
is
involuntary
and
unnecessary
6. When the deprivation of freedom which
was originally valid becomes arbitrary
later on
7. When the court has no jurisdiction to
impose the sentence
8. Imposition of excessive penalties

Q: Can the judge inquire as to the reason of the


detention of the detainee?
A: No, because the detainee is already covered. And
the judge can no longer order the release of the
detainee.
What happens? Is the writ really suspended? It is only
the privilege of the writ that is suspended. So, even if
there is rebellion, invasion, anybody can still petition
for habeas corpus. And just because there is rebellion,
the judge cannot say " I will not issue a writ.". So if
there is an application, and it is shown that there is
illegal detention, the judge will issue the writ. Unless
the decision is legal.
Now, when the judge issues the writ, what happens. It
is addressed to the person detaining another. As a
detaining officer, you have to prove, you have to show
the judge that he (the person detained) is covered by
the suspension, and once he is covered by the
detention, the judge cannot do anything anymore. But
he (the judge) has to issue the writ, even if the
privilege is suspended.
Art. 7, Sec. 18: The suspension of the privilege of
the writ shall apply only to persons judicially charged
with rebellion or ofenses inherent in, or directly
connected with invasion.
LIMITATIONS ON THE POWER OF THE PRESIDENT
TO SUSPEND THE WRIT:
1. Suspension is limited to 60 days only;
2. Congress may extend the period, upon
initiative of the President, if the invasion or rebellion
still exists after the 60 day period, with the condition
that public safety requires it.
3. Congress by a majority vote can revoke the
suspension, and this suspension cannot be set aside by
the president.
4. Any citizen can file a petition in the SC
challenging the decision and the court can look into the
legality of the decision. (The issue now becomes a
political question, and the Court must promulgate its
decision within 30 days)
5. If the person is detained, these authorities can
only detain him for 3 days. (if he is arrested without a
warrant) He has to be charged with an ofense.
6. The suspension of the privilege of the writ shall
apply only to persons judicially charges with rebellion
or ofenses inherent in, or directly connected with
invasion.
A prisoner may avail of this remedy to secure his
release if he was convicted by a court w/o jurisdiction
or his sentence had become invalid [ALCANTARA vs.
DIR. OF PRISONS 75 Phil. 749].
The right is also available to a person sentenced to
a longer penalty than that subsequently meted out to
another person convicted of the same ofense
[GUMABON vs. DIR. OF PRISONS 37 SCRA 420].
Habeas Corpus may also be resorted to in case of a
wrongful denial of bail [ZAFRA vs. CITY WARDEN].
HOWEVER where a decision is only tainted w/
errors of law, a petition for habeas corpus will not lie. If
the error does not go to the jurisdiction of the court,
the proper remedy would not be the prerogative writ
but an ordinary appeal [CELESTE vs. PEOPLE 31 SCRA
391].
BUT if the error alleged denies the right to a
speedy trial, the same is considered jurisdictional and
so may be corrected on habeas corpus [CONDE vs.
RIVERA 45 Phil. 650].

TAU MU

Note: Even if the President suspends the privilege of


the Writ, anybody can still file a petition of Habeas
Corpus. The judge can still issue the Writ. But once the
writ reaches the detaining officer and it is proven that
the detainee is covered by the suspension that is the
time that the writ shall not be efective. The detaining
officer merely has to prove that the person detained is
covered.
ACADCOM 2010; Contributors: Gene Geocaniga, Jarissa Guiani, Darlene Magabilen
TAU MU Page 163 of 179

TAU MU

TAU MU
TAU MU
TAU MU

Facts: Petitioner was convicted of 3 counts of


falsification of public documents, which convictions
were affirmed by the Court of Appeals and the
Supreme Court. Claiming that the proceedings were
attended by violations of her constitutional rights,
consisting of the failure of the judge to appreciate
certain evidence in favor, as well as the presentation
by the prosecution of a witness who might have been
perjured, petitioner prays for the issuance of a writ of
habeas corpus. Is the remedy proper?

TAU MU

IN RE: GARCIA
339 SCRA 292 (2000)

TAU MU

Ruling: Petitioners are correct that the extra-ordinary


writ of habeas corpus is the appropriate remedy to
inquire into questions of violations of constitutional
rights and that this Court has jurisdiction to entertain
the review.
However, in this case there was no
violation of the constitutional rights of the accused
since they did not make any confession or admission
during the identification in a police line-up. Thus, there
was no need for them to be assisted by counsel.

TAU MU

Facts: Accused was sentenced to death by the


Regional Trial Court for rape with homicide which
decision had been affirmed by the Supreme Court.
They later filed a petition for habeas corpus on the
ground that the trial court was ousted of jurisdiction to
try their case since the pre-trial identification was
made without the assistance of counsel. The quoted
Justice Teehankee in Olagner who stated that: Once a
deprivation of a constitutional right is shown to exist,
the court that rendered the judgment is deemed
ousted of its jurisdiction and habeas corpus is the
appropriate remedy to assail the legality of detention.

TAU MU
TAU MU

ANDAL vs. PEOPLE


307 SCRA 650 (1999)

The Fraternal
Ateneo de Davao

TAU MU
TAU MU

CASES: HABEAS CORPUS

KITY

TAU MU TAU MU TAU MU


TAU MU TAU MU TAU MU

CONSTITUTIONAL LAW II
Order of Saint Thomas More
Atty. Philip John Pojas/Atty. Rovyne G. Jumao-as, RN
University College of Law
The writ itself is not suspended --- thus it may be
issued --- but only the privilege --- meaning it cannot
be given efect.
NOTE: See also ARTICLE VII Sec. 18.
The SC has the power to annul the suspension of
the privilege if the same is not based on the grounds
enumerated in the constitution:
1.invasion or rebellion
2.when the public safety so requires.
While the President may suspend the privilege of
the writ or impose martial law for 60 days, Congress
may, on his initiative, extend such measures with the
concurrence of only a majority of its members voting
jointly --- same vote needed to revoke presidential
acts. The extension may be indefinite, depending on
the discretion of the legislature; which may be dictated
by the President.
The SC has the power to revoke extension if it is
without factual basis.

TAU MU

Ruling: The high prerogative writ of habeas corpus


was devised as a speedy and efectual remedy in cases
of unlawful restraint. Its object is to inquire into the
legality of ones detention, and if found illegal, to order
the release of he detainee. However, it will not issue
where the person in whose behalf the writ is sought is
in custody under process issued by a court with
jurisdiction to issue the process or order. In this case,
petitioner is not entitled to the relief via a petition for
habeas corpus since she has been convicted by final
judgment of the crime of falsification of public
document.
While it has been held that once a
deprivation of a constitutional right is shown to exist,
the court that rendered judgment is deemed ousted of
ACADCOM 2010; Contributors: Gene Geocaniga, Jarissa
TAU MU Page 164 of 179

jurisdiction and habeas corpus is the proper remedy,


petitioner has failed to persuade the Court that the
proceedings before the trial court were attended by
violations of her constitutional rights.
FERIA vs. CA
325 SCRA 525 (2002)
Facts: Petitioner has been in jail since May 1981 and
convicted of Robbery with Homicide. In 1993, he
sought to be transferred to Muntinlupa but the transfer
could not be efected because the documents relating
to his conviction were not submitted. Further inquiry
revealed that the entries records appear to have been
destroyed in a fire. Does the destruction of the records
render the conviction void as to warrant the release of
the convict by virtue of a writ of habeas corpus?
Ruling:
No.
The absence of a copy of a valid
judgment does not mean that there is no legal basis to
detain petitioner. Based on the hearings conducted,
there is sufficient evidence to establish the fact of his
conviction. Petitioner himself admitted in one of his
motions that he was actually convicted. The mere loss
or destruction of the records of the case does not
invalidate the judgment of the commitment, or
authorize the prisoners release. His remedy is not a
petition for habeas corpus but a proceeding for the
reconstitution of judicial records.
CALVAN vs. CA, SALES
341 SCRA 806 (2000)
Facts: MFR of a decision of the SC. Sales shot former
Mayor Benemerito. After the incident, he placed
himself under the custody of the municipal police. A
criminal complaint for murder was charged against
him. Judge Calvan concluded the preliminary
investigation without allowing him to submit counteraffidavit and present his witnesses. Sales filed a
petition for habeas corpus. CA dismissed the petition
initially but granted it thereafter.
Issue: Whether the petition for writ of habeas corpus
should be granted.
Ruling: A judge has the constructive custody over
Sales for having issued the order and warrant for his
arrest. In habeas corpus, the concern is not merely
ordering or holding the petitioner in custody, but
whether such error is sufficient to render void the
judgment. The illegal order and warrant of arrest
issued by Judge subsists and ofered no speedy,
adequate remedy or appeal in the ordinary court of
law. The writ of habeas corpus can be invoked by the
attendance of a special circumstance that requires
immediate action. Respondent cannot resort to the
remedy of a motion to quash, the case no longer being
with the judge.
IN RE: AQUINO vs. ESPERON
August 31, 2007

TAU MU
TAU MU

Facts: The facts leading to the arrest of Major Aquino,


show that, along with several military men allegedly
met to plot a breach of the Camp Defense Plan of
Camp General Emilio Aguinaldo and to take over Camp
Aquinaldo, as well as the Headquarters of the
Philippine Army. In the wake of the groups alleged
withdrawal of support from the Armed Forces of the
Philippines chain of command and the current
administration of PGMA, Major Aquino was ordered
arrested and confined at the Intelligence Service
Group, upon the order Lt. Gen. Esperon.
Hence, petitioner filed a petition for habeas
corpus with the CA praying that the AFP Chief of Staf
and the Commanding General of the Philippine Army,
Guiani, Darlene Magabilen

TAU MU
TAU MU
TAU MU
TAU MU

WRIT OF HABEAS DATA

Section 6 of the Rule on the Writ of Habeas Data


requires the following material allegations of ultimate
facts in a petition for the issuance of a writ of habeas
data:
(a) The personal circumstances of the petitioner
and the respondent;
(b) The manner the right to privacy is violated or
threatened and how it afects the right to life, liberty or
security of the aggrieved party;

TAU MU

RULING: The court held a release that renders a


petition for habeas corpus moot and academic is one
that is unconditional. In this case, the restrictions
efectively denied him of his constitutional rights.
Restrictions are declared null and void.

TAU MU

Facts: Moncupa was arrested for being a member of


the NDF. His lawyer filed a petition for habeas corpus.
While the petition was pending, the military released
him. After his release, the military attached many
conditions (an example is he cannot travel without
permission). After the military argued that the petition
was moot and academic since he was already released.

TAU MU

MONCUPA vs. ENRILE

TAPUZ vs. DEL ROSARIO


554 SCRA 768

TAU MU

Ruling: The court held that any restraint which will


preclude freedom of action is sufficient reason to issue
the writ. The forcible taking of the women by the city
officials who handed those to other parties and
deposited them in a distant region deprived them of
locomotion as efectively as if they had been
imprisoned. Without money or belongings, they were
prevented from exercising their right of going where
they want. So restraint continues until they are brought
back to Manila, unless they do not want to return.
NOTE: The petition for habeas corpus can be filed by
anybody in behalf of the detainee.

(c) The actions and recourses taken by the


petitioner to secure the data or information;
(d) The location of the files, registers or databases,
the government office, and the person in charge, in
possession or in control of the data or information, if
known;
(e) The reliefs prayed for, which may include the
updating, rectification, suppression or destruction of
the database or information or files kept by the
respondent.
In case of threats, the relief may include a prayer
for an order enjoining the act complained of; and
(f) Such other relevant reliefs as are just and
equitable."

TAU MU

Facts: Mayor Lukban of Manila herded 120 prostitutes


into a beer and brought from Manila to Davao. The
relatives of some went and filed a petition for habeas
corpus. Mayors defense was that habeas corpus was
not the proper remedy because there was no restraint
of liberty.

Ateneo de Davao

TAU MU
TAU MU

Ruling: A writ of habeas corpus extends to all cases of


illegal confinement or detention by which any person is
deprived of his liberty, or by which the rightful custody
of any person is withheld from the person entitled to
it. As a general rule, the writ of habeas corpus will not
issue where the person alleged to be restrained of his
liberty is in the custody of an officer under a process
issued by the court which has jurisdiction to do so. Its
essential object and purpose is to inquire into all
manner of involuntary restraint and to relieve a person
from it if such restraint is illegal.
In the case at bar, Major Aquino stands
charged in court martial proceedings for alleged
violations of Article 67 (Attempting to Begin or Create
Mutiny) and Article 96 (Conduct Unbecoming an Officer
and Gentleman) of the Articles of War. The legality of
Major Aquinos restraint having been settled, the
privilege of the writ is unavailing.
VILLAVICENCIO vs. LUKBAN

The Fraternal

TAU MU
TAU MU

Issue: WON the denial of petitioners petition for writ


of habeas corpus in the person of Major Aquino was
valid.

KITY

TAU MU TAU MU TAU MU


TAU MU TAU MU TAU MU

CONSTITUTIONAL LAW II
Order of Saint Thomas More
Atty. Philip John Pojas/Atty. Rovyne G. Jumao-as, RN
University College of Law
or whoever are acting in their place and stead, be
directed to immediately produce the body of Major
Aquino and explain forthwith why he should not be set
at liberty without delay.

CASE: WRIT OF HABEAS DATA

Facts: The private respondents spouses Sanson filed a


complaint for forcible entry and damages against the.
The private respondents alleged in their complaint
that: (1) they are the registered owners the disputed
land; (2) they were the disputed land's prior possessors
when the petitioners - armed with bolos and carrying
suspected firearms and together with unidentified
persons numbering 120 - entered the disputed land by
force
and
intimidation,
without
the
private
respondents' permission and against the objections of
the private respondents' security men, and built
thereon a nipa and bamboo structure.
The petitioners denied the material allegations
of the complaint. They essentially claimed that: (1)
they are the actual and prior possessors of the
disputed land; (2) on the contrary, the private
respondents are the intruders; and (3) the private
respondents' certificate of title to the disputed property
is spurious. They asked for the dismissal of the
complaint and interposed a counterclaim for damages.
Subsequently, private respondents filed a
motion for demolition. Meanwhile, the petitioners
opposed the motion for demolition. The respondent
Judge nevertheless issued via a Special Order a writ of
demolition after the Sherif's written notice to the
petitioners to voluntarily demolish their house/s to
allow the private respondents to efectively take actual
possession of the land. When the sheriif used the
notice to vacate and for demolition, petitioners filed a
petition which contains and prays for three remedies,
namely: a petition for certiorari; the issuance of a writ
of habeas data under the Rule on the Writ of Habeas
Data; and finally, the issuance of the writ of amparo
under the Rule on the Writ of Amparo.
The petitioners alleged in their motion that a
petition for a WRIT OF HABEAS DATA is prayed for so
that the PNP may release the report on the burning of
the homes of the petitioners and the acts of violence
employed against them by the private respondents,
furnishing the Court and the petitioners with copy of
the same; and that petitioners apply for a WRIT OF
HABEAS DATA commanding the PNP to produce the
police report pertaining to the burning of the houses of
the petitioners in the land in dispute and likewise the
investigation report if an investigation was conducted
by the PNP."
Issue: WON the issuance of the writ of habeas date is
valid.
Ruling: The SC ruled that the petitions for certiorari
and issuance of a writ of habeas data fatally defective,
both in substance and in form. The petition for the
issuance of the writ of amparo, on the other hand, is
fatally defective with respect to content and substance.

ACADCOM 2010; Contributors: Gene Geocaniga, Jarissa Guiani, Darlene Magabilen


TAU MU Page 165 of 179

that it breaks the expectation of impunity in the


commission of these ofenses; it is curative in that it
facilitates the subsequent punishment of perpetrators
as it will inevitably yield leads to subsequent
investigation and action. In the long run, the goal of
both the preventive and curative roles is to deter the
further commission of extralegal killings and enforced
disappearances.
It is similar to habeas corpus, only broader. This is
practiced in Mexico. And petition can be filed for
deprivation of any constitutional right.

Facts: Petitioners are settlers in a certain parcel of


land situated in Pasig City. Their dwellings/houses have
either been demolished as of the time of filing of the
petition, or is about to be demolished pursuant to a
court judgment. Hence, they filed a petition for writ of
Amparo before the SC alleging that they were deprived
of their liberty, freedom and/or rights to shelter
enshrined and embodied in our Constitution, as the
result of these nefarious activities of both the Private
and Public Respondents.

TAU MU
TAU MU
TAU MU
TAU MU
TAU MU
TAU MU
TAU MU
TAU MU

The1987 Constitution does not explicitly provide


for the writ of Amparo. But the second paragraph of
Article VIII, Section 1 of the 1987 Constitution, the
Grave Abuse Clause, provides for the judicial power "to
determine whether or not there has been a grave
abuse of discretion amounting to lack or excess of
jurisdiction on the part of any branch or instrumentality
of the Government." The Clause accords a similar
general protection to human rights extended by the
Amparo contra leyes, Amparo casacion, and Amparo
administrativo. Amparo libertad is comparable to the
remedy of habeas corpus found in several provisions of
the 1987 Constitution.
Writ of Amparo ofers a better remedy to extralegal
killings and enforced disappearances and threats
thereof. The remedy provides rapid judicial relief as it
partakes of a summary proceeding that requires only
substantial evidence to make the appropriate reliefs
available to the petitioner; it is not an action to
determine criminal guilt requiring proof beyond
reasonable doubt, or liability for damages requiring
preponderance
of
evidence,
or
administrative
responsibility requiring substantial evidence that will
require full and exhaustive proceedings.
The writ of Amparo serves both preventive and
curative roles in addressing the problem of extralegal
killings and enforced disappearances. It is preventive in

Ateneo de Davao

TAU MU
TAU MU

CHARACTERISTICS
OF
ENFORCED
DISAPPEARANCES:
1. An arrest, detention or abduction of a person
by a government official or organized groups or private
individuals acting with the direct or indirect
acquiescence of the government;
2. The refusal of the State to disclose the fate or
whereabouts of the person concerned or a refusal to
acknowledge the deprivation of liberty which places
such persons outside the protection of law.

The Fraternal

TAU MU
TAU MU

WRIT OF AMPARO
It was promulgated on October 24, 2007 "in light of
the prevalence of extralegal killing and enforced
disappearances." It was an exercise for the Court's
expanded power to promulgate rules to protect the
people's constitutional rights, which made its maiden
appearance in the 1987 Constitution in response to the
Filipino experience of the martial law regime.
The Amparo Rule was intended to address the
intractable problem of "extralegal killings" and
"enforced disappearances," its coverage, in its present
form, is confined to these two instances or to threats
thereof. "Extralegal killings" are "killings committed
without due process of law, i.e., without legal
safeguards or judicial proceedings."

KITY

TAU MU TAU MU TAU MU


TAU MU TAU MU TAU MU

CONSTITUTIONAL LAW II
Order of Saint Thomas More
Atty. Philip John Pojas/Atty. Rovyne G. Jumao-as, RN
University College of Law
The petitioners allegations obviously lack what
the Rule on Writ of Habeas Data requires as a
minimum, thus rendering the petition fatally deficient.
Specifically, there were no concrete allegations of
unjustified or unlawful violation of the right to privacy
related to the right to life, liberty or security. The
petition likewise has not alleged, much less
demonstrated, any need for information under the
control of police authorities other than those it has
already set forth as integral annexes. The necessity or
justification for the issuance of the writ, based on the
insufficiency of previous eforts made to secure
information, has not also been shown. In sum, the
prayer for the issuance of a writ of habeas data is
nothing more than the "fishing expedition" that the SC
- in the course of drafting the Rule on habeas data had in mind in defining what the purpose of a writ of
habeas data is not. In these lights, the outright denial
of the petition for the issuance of the writ of habeas
data is fully in order.

CASES: WRIT OF AMPARO


CANLAS vs. NAPICO HOMEOWNERS
554 SCRA 208

Issue: WON the issuance of a writ of Amparo is proper.


Ruling: The Rule on the Writ of Amparo provides that,
The petition for a writ of amparo is a remedy available
to any person whose right to life, liberty and security is
violated or threatened with violation by an unlawful act
or omission of a public official or employee, or of a
private individual or entity. The writ shall cover
extralegal killings and enforced disappearances or
threats thereof. The threatened demolition of a
dwelling by virtue of a final judgment of the court,
which in this case was affirmed with finality by this
Court is not included among the enumeration of rights
as stated in the above-mentioned provision for which
the remedy of a writ of amparo is made available.
Their claim to their dwelling, assuming they still have
any despite the final and executory judgment adverse
to them, does not constitute right to life, liberty and
security. There is, therefore, no legal basis for the
issuance of the writ of amparo.
Under Section 6 of the same rules, the court
shall issue the writ upon the filing of the petition, only
if on its face, the court ought to issue said writ. Thus,
under said provision,. Upon the filing of the petition,
the court, justice or judge shall immediately order the
issuance of the writ if on its face it ought to issue. The
clerk of court shall issue the writ under the seal of the
court; or in case of urgent necessity, the justice or the
judge may issue the writ under his or her own hand,
and may deputize any officer or person to serve it. The
writ shall also set the date and time for summary
hearing of the petition which shall not be later than
seven (7) days from the date of its issuance.
Considering that there is no legal basis for its
issuance, as in this case, the writ will not be issued and
the petition will be dismissed outright. This new
remedy of writ of amparo is intended for the protection
of the highest possible rights of any person, which is
his or her right to life, liberty and security.
SECRETARY OF NATIONAL DEFENSE vs. MANALO
OCTOBER 7, 2008

TAU MU

Facts: The case at bar involves the rights to life,


liberty and security in the first petition for a writ of
Amparo filed before the SC. This case was originally a
Petition for Prohibition, Injunction, and Temporary
ACADCOM 2010; Contributors: Gene Geocaniga, Jarissa Guiani, Darlene Magabilen
TAU MU Page 166 of 179

The Fraternal
Ateneo de Davao

TAU MU
TAU MU
TAU MU
TAU MU

from victims of extralegal killings and enforced


disappearances. The writ of Amparo is a tool that gives
voice to preys of silent guns and prisoners behind
secret walls. Hence, the petition was dismissed.

Section 16. All persons shall have the right to a


speedy disposition of their cases before all
judicial, quasi-judicial, or administrative bodies.
This provision guarantees the right generally to a

speedy disposition of cases. It covers the periods


before, during and after trial.
It gives a broader protection that Sec. 14 (2) --- right
to speedy trial.
This provision applies to civil, criminal, and
administrative cases.
SECTION 14 vs. SECTION 16
While the rights of an accused only apply to the trial
phase of criminal cases, the right to a speedy
disposition of cases covers ALL phases of JUDICIAL,
QUASI-JUDICIAL or ADMINISTRATIVE proceedings. In
short, the right to a speedy trial granted in Sec. 14 is
applicable only to criminal cases.

TAU MU

FACTORS TO BE CONSIDERED:
1. length of delay
2. reason for the delay
3. assertion of the right or failure to assert it
4. prejudice caused by the delay

TAU MU

Factors to be considered are the same as the


factors of speedy trial. As mentioned, the right of
speedy disposition of cases is broader than the rights
of the accused to speedy trial. If the delay will focus on
the trial itself, then it falls under speedy trial.

TAU MU

Issue: WON respondents have a right to the privilege


of the writ of Amparo.

KITY

TAU MU TAU MU TAU MU


TAU MU TAU MU TAU MU

CONSTITUTIONAL LAW II
Order of Saint Thomas More
Atty. Philip John Pojas/Atty. Rovyne G. Jumao-as, RN
University College of Law
Restraining Order (TRO) filed before the SC
respondents (therein petitioners) to stop herein
petitioners (therein respondents) and/or their officers
and agents from depriving them of their right to liberty
and other basic rights. In SCs resolution, it ordered (1)
ordered the Secretary of the Department of National
Defense and the Chief of Staf of the AFP, their agents,
representatives, or persons acting in their stead,
including but not limited to the Citizens Armed Forces
Geographical Unit (CAFGU) to submit their Comment;
and (2) enjoined them from causing the arrest of
therein petitioners, or otherwise restricting, curtailing,
abridging, or depriving them of their right to life,
liberty, and other basic rights as guaranteed under
Article III, Section 1 of the 1987 Constitution.
While the Petition was pending, the Rule on the
Writ of Amparo took efect. Petitioners filed a
Manifestation and Omnibus Motion to Treat Existing
Petition as Amparo Petition, to Admit Supporting
Affidavits, and to Grant Interim and Final Amparo
Reliefs. They prayed that: (1) the petition be
considered a Petition for the Writ of Amparo under Sec.
26 of the Amparo Rule; (2) the Court issue the writ
commanding therein respondents to make a verified
return within the period provided by law and containing
the specific matter required by law; (3) they be granted
the interim reliefs allowed by the Amparo Rule and all
other reliefs prayed for in the petition but not covered
by the Amparo Rule; (4) the Court, after hearing,
render judgment as required in Sec. 18 of the Amparo
Rule; and (5) all other just and equitable reliefs. The
Court resolved to treat the Petition as a petition under
the Amparo Rule and accordingly, the privilege of writ
of Amparo was granted.

TAU MU

Ruling: The production order under the Amparo Rule


should not be confused with a search warrant for law
enforcement under Article III, Section 2 of the 1987
Constitution. This Constitutional provision is a
protection of the people from the unreasonable
intrusion of the government, not a protection of the
government from the demand of the people such as
respondents.
Instead, the Amparo production order may be
likened to the production of documents or things under
Section 1, Rule 27 of the Rules of Civil Procedure which
provides in relevant part. Petitioners assert that the
disclosure of the present places of assignment of
M/Sgt. Hilario aka Rollie Castillo and Donald Caigas, as
well as the submission of a list of medical personnel, is
irrelevant, improper, immaterial, and unnecessary in
the resolution of the petition for a writ of Amparo. They
add that it will unnecessarily compromise and
jeopardize the exercise of official functions and duties
of military officers and even unwittingly and
unnecessarily expose them to threat of personal injury
or even death.
On the contrary, the disclosure of the present
places of assignment of M/Sgt. Hilario aka Rollie
Castillo and Donald Caigas, whom respondents both
directly implicated as perpetrators behind their
abduction and detention, is relevant in ensuring the
safety of respondents by avoiding their areas of
territorial jurisdiction. Such disclosure would also help
ensure that these military officers can be served with
notices and court processes in relation to any
investigation and action for violation of the
respondents' rights. The list of medical personnel is
also relevant in securing information to create the
medical history of respondents and make appropriate
medical interventions, when applicable and necessary.
In blatant violation of our hard-won guarantees
to life, liberty and security, these rights are snufed out
ACADCOM 2010; Contributors: Gene Geocaniga, Jarissa
TAU MU Page 167 of 179

The remedy for the violation of this right is


MANDAMUS --- same remedy as to violation of the right
to a speedy trial [ROQUE vs. OMBUDSMAN
(5/12/1999)].
CASES: RIGHT VIOLATED
CERVANTES vs. SANDIGANBAYAN
307 SCRA 149 (1999)

TAU MU
TAU MU

Facts: On March 6, 1986, a complaint for falsification


was filed against accused before the Tanodbayan. On
Oct. 2, 1986, accused submitted his counter-affidavit.
On May 18, 1992, or more than 6 years later, the Office
of the Special Prosecutor filed an information before
the Sandiganbayan against him for violation of R.A. No.
3019. Accused filed a motion to quash on the ground
of violation of his right to speedy disposition of cases.

TAU MU
TAU MU

Ruling: The information should be quashed. Accused


was deprived of his right to speedy disposition of the
case. It took 6 years from the filing of the initiatory
complaint before the information for the ofense was
filed in court. It is the duty of the Special Prosecutor to
speedily resolve the complaint, as mandated by the
Constitution, regardless of whether accused did not
object to the delay or that the delay was with his
acquiescence provided that it was not due to causes
directly attributable to him.
LOPEZ vs. OMBUDSMAN
364 SCRA 568 (2001)

TAU MU

Facts: On Dec. 20, 1993, members of the COA Audit


Team submitted a Joint-Affidavit Complaint against
respondent before the Office of the Ombudsman. On
April 22, 1994, respondent submitted his counterGuiani, Darlene Magabilen

TAU MU
TAU MU
TAU MU
TAU MU
TAU MU

Facts: Licaros was Vice Chairman of Home Savings


Bank who was charged as an accessory to the crime of
robber before the Sandiganbayan in an information
dated July 6, 1982. he asked for a separate trial and on
August 26, 1986, completed the presentation of his
evidence and submitted the case for decision. The
Sandiganbayan deferred decision until such time that
the other accused have completed presentation of
their own evidence, which took place on June 20, 1990.
On March 23, 2000, or after about 10 years, he filed a
motion to dismiss the case on the ground that the
Sandiganbayan failed to decide the case thus violating
his right to speedy disposition of cases. Should the
case be dismissed?

TAU MU

LICAROS vs. SANDIGANBAYAN


370 SCRA 394 (2001)

HOFER vs. HRET


428 SCRA 383

TAU MU

Ruling: Yes. From the time of the arraignment of


accused in 1992, there was an unexplained interval of
inactivity of close to 5 years in the Sandiganbayan.
Granting that the delay was caused by the separate
motions for reinvestigation filed by the other accused,
there was no explanation why the reinvestigation was
unduly stretched. Apparently, the Ombudsman did not
complete the reinvestigation. In addition, it cannot be
said that petitioner failed to assert his right because he
wrote the Ombudsman about the prejudice caused him
by the cases. For all these past 11 years, petitioner
has remained under a cloud and has been deprived of
his retirement after serving the government for over 40
years because of the inaction of the Ombudsman. The
delay here measures up to unreasonableness which
entitles him to a dismissal of the cases.

the time it is deemed submitted for decision. Even if


we were to consider the period provided under Sec. 15
(1) VIII, of the Constitution, which is 12 months from
submission of the case for decision, the Sandiganbayan
still failed to perform its duty to decide within the
period fixed by law. Indubitably, there has been a
transgression of the right to a speedy disposition of his
case due to inaction on the part of the Sandiganbayan.
Considering the 10 year-delay in the resolution of the
case; the fact that petitioner has sufered vexation and
oppression due to the delay ; that he did not sleep on
his rights but has in fact asserted it; that he did not
contribute in any manner to the delay; that the
Sandiganbayan did not give any valid reason to justify
the delay; and that he is charged as a mere accessory,
dismissal is justified.

TAU MU

Facts: On May 1991, accused was charged with 2


counts of falsification of a public document. Because
of some pre-trial motions, accused was arraigned only
on July 28, 1992. Since his co-accused sought a
reinvestigation of the cases trial did not proceed.
Accused retired in 1994 but failed to get his benefits
due to the pendency of the criminal cases. On March
20, 1997, accused wrote to the Ombudsman a request
for the payment of his retirement benefits. On October
12, 1998, accused filed a motion to dismiss invoking
his right to speedy trial.
Should the cases be
dismissed?

Ateneo de Davao

TAU MU
TAU MU

ABARDO vs. SANDIGANBAYAN


355 SCRA 64 (2001)

The Fraternal

TAU MU
TAU MU

Ruling: Yes.
The preliminary investigation was
resolved close to 4 years after the last reply affidavit
was submitted. It took the Deputy Ombudsman 8
months to approve the resolution of the investigator
and close to another year for the Ombudsman to
approve the recommendation.
The issues are not
sufficiently complex to justify the length of time for
their resolution.
There is no statement that
voluminous documentary and testimonial evidence are
involved. The failure of the Ombudsman to resolve the
complaints for almost 4 years is violative of the right of
respondent.
In such event, he is entitled to the
dismissal of the cases filed against him.

KITY

TAU MU TAU MU TAU MU


TAU MU TAU MU TAU MU

CONSTITUTIONAL LAW II
Order of Saint Thomas More
Atty. Philip John Pojas/Atty. Rovyne G. Jumao-as, RN
University College of Law
affidavit. More than four years later, on July 17, 1988,
the Office of the Ombudsman-Mindanao issued a
resolution recommending that respondent be charged
with 30 counts of violation of the Anti-Graft and
Corrupt Practices Act. Was there an unjustifiable delay
on the part of the Ombudsman in resolving the
complaint against respondent which violated his right
to a speedy disposition of cases?

Facts: Petitioner Hofer and respondent Cabilao, were


congressional candidates in the lone congressional
district of Zamboanga Sibugay during the May 2001
national and local elections. Respondent was
proclaimed
the
duly
elected
congressional
representative. Claiming that massive vote buying,
tampering of election returns and other irregularities
were committed, petitioner filed with the HRET an
election protest. HRET dismissed petitioners protest
and ruled that petitioners failure to take necessary
steps to prosecute this case justify its dismissal.
Issue: WON there was a violation of respondent
Cabilaos right to speedy disposition of case.
Ruling: The election protest filed by petitioner is a
serious charge which, if true, could unseat protestee as
Representative of her district. Hence, the observance
of the HRET Rules in conjunction with the Rules of
Court, must be taken seriously. Section 59 of the 1998
HRET Rules is explicit. Unfortunately, petitioner did not
comply with it. In fact, despite the lapse of six (6)
months, she failed to present her evidence. Such
inaction shows her utter lack of interest to prosecute
her case.
As held in one case, by their very nature and
given the public interest involved in the determination
of the results of an election, the controversies arising
from the canvass must be resolved speedily, otherwise
the will of the electorate would be frustrated. And the
delay brought about by the tactics resorted to by
petitioner is precisely the very evil sought to be
prevented by election statutes and controlling case law
on the matter."
TATAD vs. SANDIGANBAYAN

TAU MU

Facts: Initially, the complaints were filed in 1974


against Tatad while he was still connected with
Malacaang. In 1979, he resigned so the complaints
were filed with the Tanodbayan. In 1982, the
submission of evidence was completed. In 1985, the
Tanodbayan recommended the filing of charges against
him. Tatad claimed that there was a violation of his
right to speedy disposition. Subject matter was the
time between 1982 to 1985 since it was this time when
the evidence were presented and the Tanodbayan
should have recommended the filing of the charges at
that time. The argument of the Solicitor General is that:
In past decisions, absence of preliminary investigation
is not a fatal defect and the may be cured, thus, the
more reason that delay should not also be a fatal
defect (NOTE: In past cases, fiscal can just file charges
without PI).

Ruling: Yes. Under Sec. 6 of PD No. 1606, the


Sandiganbayan has only 90 days to decide a case from
ACADCOM 2010; Contributors: Gene Geocaniga, Jarissa Guiani, Darlene Magabilen
TAU MU Page 168 of 179

CASES: RIGHT NOT VIOLATED


BINAY vs. SANDIGANBAYAN
316 SCRA 65 (1999)

TAU MU
TAU MU
TAU MU
TAU MU

DIMAYACYAC vs. CA
430 SCRA 121 (2004)

TAU MU

Ruling: No. The concept of speedy disposition of


cases, like speedy trial is flexible. It is consistent
with reasonable delay. The preliminary investigation in
the subject cases against accused took more than 1
year and 4 months to finish. But such a happenstance
alone, or any like delay, for that matter, should not be
cause for an unfettered abdication by the court of its
duty to try cases. The delay adverted to in the cases
under consideration does not measure up to
unreasonableness. It cannot be said that petitioners
found themselves in a situation oppressive to their
rights simply by reason of the delay without more.

TAU MU

Facts: Accused were employed with the National Food


Authority. On Dec. 1991, a complaint for falsification of
public document was filed against him before the
Ombudsman. On Jan. 15, 1993, the last rejoinder was
filed and the complaint was submitted for resolution.
On May 30, 1994, or after 1 year and 4 months, the
office of the Ombudsman issued a resolution ordering
that they be charged with various crimes. Was their
right to speedy disposition of cases violated?

TAU MU

DANSAL vs. FERNANDEZ


327 SCRA 145 (2000)

TAU MU
TAU MU

Ruling: No. The concept of speedy disposition is a


relative term. Factors to determine whether the right
has been denied include the length of the delay, the
reasons for such delay, the assertion or failure to
assert such right by the accused and the prejudice
caused by the delay. A mere mathematical reckoning
of the time involved, therefore, would not be sufficient.
In this case, there was no undue delay. The cases
involved complex issues and voluminous documents.
The prosecution had to review the findings of the
Commission on Audit to determine probable cause.
Thus, the length of time consumed before the
resolution was issued is justified.

The Fraternal
Ateneo de Davao

TAU MU
TAU MU

Facts: On Sept. 7, 1994, the Office of the Ombudsman


filed 3 information against Mayor Binay for violations of
Art. 220 of the Revised Penal Code and 2 for violations
of Sec. 3 (e) of the Anti-Graft Law before the
Sandiganbayan.
Considering that the affidavitcomplaint was filed before the Office of the
Tanodbayan on July 27, 1988 for preliminary
investigation, was there a violation of the right of
accused to speedy disposition of cases?

KITY

TAU MU TAU MU TAU MU


TAU MU TAU MU TAU MU

CONSTITUTIONAL LAW II
Order of Saint Thomas More
Atty. Philip John Pojas/Atty. Rovyne G. Jumao-as, RN
University College of Law
Ruling: The court held that long delayed in the
termination of the preliminary investigation is violative
of the right to speedy trial. The court also held that the
absence of PI can be corrected. But undue delay in the
conduct of the preliminary investigation cannot be
corrected because until now, man still has not invented
a device to set back time.

dismissed for the inordinate delay in the conduct of


preliminary investigation for the purpose of filing the
proper information, which is a violation of the
accuseds constitutional right to due process of law and
to speedy disposition of cases.
Issue: WON petitioners constitutional right to a
speedy disposition of his case has been violated.
Ruling: The SC ruled in the negative. The right to a
speedy disposition of cases, like the right to a speedy
trial, is deemed violated only when the proceedings is
attended by vexatious, capricious, and oppressive
delays; or when unjustified postponements of the trial
are asked for and secured, or when without cause or
unjustifiable motive, a long period of time is allowed to
elapse without the party having his case tried. In the
determination of whether or not that right has been
violated, the factors that may be considered and
balanced are: the length of the delay the reasons for
such delay, the assertion or failure to assert such right
by the accused, and the prejudice caused by the delay.
A mere mathematical reckoning of the time involved,
therefore, would not be sufficient. In the application of
the constitutional guarantee of the right to speedy
disposition of cases, particular regard must also be
taken of the facts and circumstances peculiar to each
case.
The records clearly show is that petitioner
never asserted his right to a speedy disposition of his
case. The only ground he raised in assailing the
subsequent filing of the two informations is that he will
be subjected to double jeopardy. It was only the OSG
that brought to light the issue on petitioners right to a
speedy disposition of his case, and only when the case
was brought to the appellate court on certiorari. Even
in this petition before the SC, petitioner did not raise
the issue of his right to a speedy disposition of his
case. Again, it was only the OSG that presented such
issue in the Brief for the State which was only then
adopted by petitioner through a Manifestation. The SC
is not convinced that the filing of the informations
against petitioner after two years was an unreasonable
delay. Petitioner himself did not really believe that
there was any violation of his right to a speedy
disposition of the case against him. As held in one
case, petitioners failure to assert their right to a
speedy disposition of their cases was deemed to have
waived such. Hence, the SC dismissed/denied this
case.
BERNAT vs. SANDIGANBAYAN
428 SCRA 787 (2004)
Facts: Special civil action in the SC. Certiorari. On
August 14, 1991, petitioner along with several coaccused, were charged by the Sandiganbayan with
violation of the Section 3(e) of RA 3019, otherwise
known as the Anti-Graft and Corruption Practices Act.
The case submitted for decision more than 8 years had
lapsed.
Issue: Whether there is a violation of petitioners
constitutional right to a speedy disposition of his case.
Ruling: The right is deemed violated only when the
proceedings are attended by vexations, capricious and
oppressive delays. It is not a mere mathematical
reckoning of time. Factors that should be considered
are (1) length of delay; (2) reasons for the delay: (3)
assertion or failure to assert such right by the accused:
and (4) prejudice caused by the delay. Petitioner failed
reasonably to assert his constitutional right to speedy
trial.

TAU MU

Facts: Information for falsification of public documents


was filed against petitioner along with some others.
Before his arraignment, petitioner moved to quash the
information on two (2) grounds. First, that the officer
who filed the information had no legal authority to do
so, and second, that more than one ofense was
charged in the information. Pending resolution of the
motion to quash, petitioner was arraigned.
Subsequently., the OSG advances the view that
CASTILLO vs. SANDIGANBAYAN
the criminal case against herein petitioner may be
328 SCRA 69 (2000)
ACADCOM 2010; Contributors: Gene Geocaniga, Jarissa Guiani, Darlene Magabilen
TAU MU Page 169 of 179

TAU MU

Facts: The office of the Tanodbayan recommended the


filing of an Information against accused for violation of
RA No. 3019 on Oct. 10, 1987. however, it was only on
Nov. 5, 1990, or after 3 years when an Information was
filed before the Sandiganbayan by the Ombudsman.
Was the right of accused to speedy disposition of cases
violate?

TAU MU
TAU MU
TAU MU
TAU MU
TAU MU

TY-DAZO vs. SANDIGANBAYAN


374 SCRA 200 (2001)

TAU MU

Facts: As a result of an audit by the Commission on


Audit on the Davao del Norte School of Fisheries,
petitioners were ordered to submit their counteraffidavits on August 14, 1992, which they did on Dec.
3, 1992. On October 10, 1994, the investigator came
up with a resolution which was approved by the Deputy
Ombudsman on March 13, 1997 and by the
Ombudsman on April 28, 1998.
Based on the
resolution, an information for violation of Republic Act.
No. 3019 was filed against petitioners before the
Sandiganbayan. Was there a violation of their right to
speedy disposition of cases?
Ruling: No. The right to speedy disposition is deemed
violated only when the proceedings is attended by
vexations, capricious and oppressive delays.
The
concept is relative and flexible.
In determining
whether the right has been violated, the factors that
may be considered and balanced are as follows: (1)
the length of the delay; (2) the reasons for the delay;
(3) the assertion or failure to assert such right by the
accused; and (4) the prejudice caused by the delay. In
this case, while it took the investigator some 4 years to
prepare a resolution, he had to go over the lengthy
COA report and counter-affidavits as well as numerous
receipts and other evidence. Moreover, it was only
after the case was set for arraignment when petitioners
raised the issue of delay. Their silence may, therefore,
be interpreted as a waiver of the right.

TAU MU
TAU MU

DELA PENA vs. SANDIGANBAYAN


360 SCRA 470 (2001)

The Fraternal
Ateneo de Davao

TAU MU
TAU MU

Ruling: No. The right to speedy disposition of cases is


deemed violated only when the proceeding is attended
by vexatious, capricious and oppressive delays. It was
not so in this case. In 1988, the Zaldivar vs. Gonzales
case was decided where it was held that the incumbent
Tanodbayan lost his right to conduct preliminary
investigation so that resolutions remained pending
awaiting the appointment of an Ombudsman. When
one was appointed, it took some time before his Office
could be fully operational. Finally, this Court has taken
notice that the Sandiganbayan has a heavy case load.
The structural reorganization of prosecutorial agencies,
procedural changes brought about by frequent
amendments, as well as the Sandiganbayans case
load are valid reasons for the delay in disposition of
petitioners case.

KITY

TAU MU TAU MU TAU MU


TAU MU TAU MU TAU MU

CONSTITUTIONAL LAW II
Order of Saint Thomas More
Atty. Philip John Pojas/Atty. Rovyne G. Jumao-as, RN
University College of Law

TAU MU
TAU MU

Facts: On Feb. 19, 1993, the police caught a mini-truck


owned by petitioner, then Mayor of Salcedo, Eastern
Samar, loaded with illegally cut logs. A complaint for
violation of the Anti-Graft and Corrupt Practices Act
was files on Sept. 1, 1993 before the Ombudsman, but
an information was filed with the Sandiganbayan only
on Feb. 17, 1995. She moved for a reinvestigation on
July 4, 1995, but the court denied it on March 5, 1999.
Meanwhile the complain for violation of PD No. 705 was
file with the Ombudsman on May 11, 1993; but the
corresponding information was filed with the
Sandiganbayan only on April 28, 1997. She filed a
motion for reinvestigation which was resolved only on
ACADCOM 2010; Contributors: Gene Geocaniga, Jarissa
TAU MU Page 170 of 179

January 4, 1999. Was the right of accused to speedy


disposition of cases violated?
Ruling: No. The right to a speedy disposition of cases
is deemed violated only when the proceedings is
attended by vexations, capricious and oppressive
delays. A mere mathematical reckoning of the time
involved would not be sufficient. In this case, reliance
by petitioner in Tatad is misplaced.
There is no
showing that the charge against her is politically
motivated. In addition, unlike in Tatad, the established
procedure for preliminary investigation was observed
in this case. Finally, petitioner contributed to the delay
by filing a motion for reinvestigation.
The bare
allegation that it took the Ombudsman more than 3
years to terminated the preliminary investigation is not
enough. A mere mathematical reckoning of the time
involved is insufficient.
Section 17. No person shall be compelled to be a
witness against himself.
PRIVILEGE
OF
RIGHT
AGAINST
SELFINCRIMINATION
The right against self-incrimination has its roots in
the common law and is based on humanitarian and
practical considerations. Humanitarian because it is
intended to prevent the State, with all its coercive
powers, from extracting from the suspect testimony
that may convict him. Practical because a person
subjected to such compulsion is likely to perjure
himself for his own protection.
This right is available not only criminal
prosecutions but also in all other government
proceedings, including civil actions and administrative
or legislative investigations. It may be claimed not only
by the person accused of an ofense but by any witness
to whom an incriminating question is addressed.
SCOPE OF RIGHT
1. Testimonial self-incrimination when one is
required to answer a question which will incriminate
him. This involves the use of intelligence, or acts which
are not merely mechanical.
2.
Exempts the accused from producing
documents or articles demanded of him which
are incriminating.
Exceptions:
a. When the state has the right to inspect the
same under police power;
b. Corporate documents, even if the same
would incriminate the corporate officers;
c. Public documents which are matters of
public concern (CF: right of people to information)
3. It does not extend to non-testimonial acts,
physical acts like blood test to determine if the
accused was infected with a sexual disease.
EXCEPTION:
a. If the test will subject the accused to
unnecessary and existent humiliation
b. If the accused is required to submit a
specimen of his handwriting to be used as
evidence against him to a charge of forgery
4. It also exempts the accused to do a
reenactment of the crime he is being charged of.

TAU MU

NON-TESTIMONIAL PHYSICAL ACTS ARE ALLOWED


AS EVIDENCE SO LONG AS:
1. They are purely mechanical and does not involve
the use of intelligence or imagination
Guiani, Darlene Magabilen

At the heart of Section 17 is the right to remain

TAU MU
TAU MU
TAU MU
TAU MU

NOTE:
a. Beltran vs. Samson The accused was asked to give
a sample of his handwriting. This is not only a
mechanical act because it still needs the use of
intelligence;
b. When the accused is asked to take the lie detector
test even if the questions asked are unrelated to the
crime not merely a mechanical act;
c. When the accused is asked to re-enact the crime, the
right attaches as this is not a mechanical act;
d. Cases where the right was not violated:

TAU MU

WHAT ARE NOT COVERED BY THIS RIGHT:


Non-testimonial acts such as blood tests to determine
if the accused has STD. In other words, acts which do
not involve the use of intelligence or mechanical acts;
unless the act is communicative in nature, or if the test
will subject the accused to unnecessary and existent
humiliation.

TAU MU

DISTINCTION BETWEEN AN ACCUSED AND AN


ORDINARY WITNESS
1. An accused can refuse to take the witness stand by
invoking the right against self-incrimination.
2. An ordinary witness cannot refuse to take the
stand. He can only refuse to answer specific questions
which would incriminate him in the commission of an
ofense.

TAU MU

WHEN IS A QUESTION INCRIMINATING:


A question tends to incriminate when the answer of the
accused or the witness would establish a fact which
would be a necessary link in a chain of evidence to
prove the commission of a crime by the accused or the
witness.

TAU MU

silent --- so that accused cannot be forced to testify --in accord with the rules of fair play.
There is no right against incrimination per se, only
self-incrimination.
This right guarantees that you cannot be forced to
present evidence against yourself.
It is important for a person to know when a question
is incriminating and when it is not.
An incriminating question is a question that leads to
an admission to the commission of a crime.

TAU MU
TAU MU

BASES OF THIS RIGHT:


1. Principle of humanity, to prevent the extortion of
confessions under duress or compulsion;
2. Public policy, to prevent the witnesses or the
accused from committing perjury.

The Fraternal
Ateneo de Davao

TAU MU
TAU MU

What is PROHIBITED is the use of physical or moral


compulsion to extort communication from the witness
or to otherwise elicit evidence which would not exist
were it not for the actions compelled from the witness.
The right does NOT PROHIBIT the examination of
the body of the accused or the use of findings with
respect to his body as physical evidence. Hence, the
fingerprinting of an accused would not violate the right
against self-incrimination.
However, obtaining a
sample of the handwriting of the accused would violate
this right if he is charged for falsification.
The accused cannot be compelled to produce a
private document in his possession which might tend
to incriminate him. However, a third person in custody
of the document may be compelled to produce it.

KITY

TAU MU TAU MU TAU MU


TAU MU TAU MU TAU MU

CONSTITUTIONAL LAW II
Order of Saint Thomas More
Atty. Philip John Pojas/Atty. Rovyne G. Jumao-as, RN
University College of Law
2. And if it will not subject the accused to unnecessary
and causes humiliation

1. US vs. Tan: where the accused was examined for


gonorrhea which might have been transmitted
to the victim.
2. When the accused was asked to strip so he can
be examined for scratch marks.
3. When accused was asked to go through a
pregnancy test.
4. When one is dusted with florescent powder
5. When placed in a police line-up
WHEN THE RIGHT CAN BE IINVOKED:
1. In criminal cases
2. In administrative proceedings if the accused is
liable to a penalty (Ex. Forfeiture of property)
Note: This right can be waived, however, the SC has
held that there is no implied waiver of this right.
WHO CAN INVOKE THE RIGHT:
Only natural persons. Judicial persons are subject to
the visitorial powers of the state in order to determine
compliance with the conditions of the charter granted
to them.
SCOPE
OF
THE
RIGHT
AGAINST
SELFINCRIMINATION IN VARIOUS PROCEEDINGS:
1. In CRIMINAL CASES, this is a prohibition against
inquiry. It means that the accused cannot be compelled
to take the witness stand. But if you are a witness and
not the accused, the right extends only to refusal to
answer incriminating questions. The witness has no
right to refuse to take the witness stand.
o Editors Note: Under the rules on
evidence, there can be waiver if the accused takes
the witness stand. But he can refuse to answer
questions which are not related to his direct
testimony.
MEANING:

SCOPE: absolute/total prohibition of inquiry


(Chavez Case).

The accused cannot be forced to take the


witness stand and testify.

Accused can refuse to answer questions at the


outset.

Witnesses in a criminal case can invoke this


right optionally only --- right is not absolute for them --otherwise there would be a violation of accuseds right
to compulsory process.

If the accused chooses to testify, he is deemed


to have waived the protection of this right.

Waiver is only to the extent of the matter he


testified to --- problematic --- waiver as to matter x:
accused should answer questions only as to that
matter, if he is asked about other matters, he can
invoke right.
2. In CIVIL CASES, the right is just an option to refuse
since there is no accused in civil cases. Anybody can
be called to the witness stand, the only thing they have
is the option to refuse questions which would tend to
incriminate them.
MEANING:

The right is only an option to refuse to answer


an incriminating question at the moment it is asked.

Defendant must take the witness stand and


wait for the incriminating question to be asked before
he can refuse to answer it otherwise he can be jailed
for contempt --- thus the right is not absolute in civil
cases.

TAU MU

3. In ADMINISTRATIVE PROCEEDINGS, same as rule


in civil cases. In short, as a general rule, proceedings
other than criminal, the right against self incrimination

ACADCOM 2010; Contributors: Gene Geocaniga, Jarissa Guiani, Darlene Magabilen


TAU MU Page 171 of 179

MEANING:

The same principle for civil cases applies to


administrative proceedings.

Right can be invoked only in cases when the


proceeding is penal in nature.

KITY

TAU MU
TAU MU

Q:
WHEN
IS
A
QUESTION
CONSIDERED
INCRIMINATING?
A: When the question tends to expose the person to
the accusation of a crime or tend to establish guilt
against him.

TAU MU
TAU MU

TESTIMONIAL/NON-TESTIMONIAL
I. ACCUSED IS EXEMPT FROM TESTIFYING

This right covers only testimonial rights


except:
1.
acts which require the use of
intelligence;
example: writing your signature, copying
certain words
2.
acts that communicate in nature.
II. THE ACCUSED IS EXEMPT FROM PRODUCING
DOCUMENTS

EXCEPTIONS:
1. Corporation or their officers (PASECO
Case);
2. Public
officials
(ALMONTE
vs.
VASQUEZ).
Corporations are only creature of the state,
they cannot resist the mandate of the state if they
should be asked to produce certain documents.
Corporation officers cannot invoke this right
when asked to produce documents of the corporation
which may incriminate them as well because they
are only acting in behalf of the corporation --- it is not
self-incrimination because it is the corporation that is
incriminated.
Public officials cannot invoke section 17
because the public has the right to information.

TAU MU

Q: WHAT IF A LIE DETECTOR IS USED?


A: No jurisprudence yet on this.
Atty. DELA BANDA: it is safe to assume that it
would violate section 17 because answering the
questions would require the use of intelligence

TAU MU

Q: WHAT ABOUT DRUG-TESTING?


A: It is non-testimonial because the
technologist will only get your urine sample.

TAU MU

TAU MU
TAU MU
TAU MU
TAU MU

DOJ:
a. employees cannot be compelled to take it --because it would be a compulsion on their part;
b. people applying for a job in the government
can be required to be tested --- if they refuse, the
choice is still theirs, there is no compulsion

TAU MU

4. In LEGISLATIVE PROCEEDINGS one can refuse to


answer incriminating questions.

The principles for civil and administrative


proceedings apply.

medical

The Fraternal
Ateneo de Davao

TAU MU TAU MU TAU MU


TAU MU TAU MU TAU MU

CONSTITUTIONAL LAW II
Order of Saint Thomas More
Atty. Philip John Pojas/Atty. Rovyne G. Jumao-as, RN
University College of Law
is merely an option to refuse to answer incriminating
questions.
Exceptions in administrative proceedings:
a. PASCUAL vs. BOARD An administrative
proceeding against a doctor for malpractice.
The SC ruled that he cannot be forced to take
the witness stand without his consent since the
penalty is so severe;
b. CABAL vs. CAPUNAN when the penalty
involves forfeiture of ones property.

A: Atty. DELA BANDA: it might fall in the exception


where section 17 applies because it requires the use of
intelligence --- imagination!
GALMAN CASE
IMMUNITY IS THE ONLY WAY TO DEFEAT
THE RIGHT GIVEN BY SECTION 17.
1. USE AND FRUIT IMMUNITY Prohibits the use of
the testimony and its fruits in any manner in
connection with the criminal prosecution of the
witness. It merely grants the witness immunity fro use
of any statement given before the court, but not
immunity from prosecution by reason of or on the basis
thereof.
Ex. A, B, C, D are the accused. D is the state
witness. Now D's testimony cannot be used in
any manner in connection with the criminal
prosecution in that particular case. His
testimony cannot be used to convict him in
that particular case. But this does not mean
that he cannot be convicted, his testimony
cannot be used to as evidence against him in
that
particular
case.
2. TRANSACTIONAL IMMUNITY makes the witness
immune from criminal prosecution for the ofense for
which his compelled testimony relates. So this is
immunity from prosecution.
CASES: RIGHT AGAINST SELF-INCRIMINATION
PEOPLE vs. GALLARDE
325 SCRA 835 (2001)
Facts: Accused was a suspect in a rape with homicide
case. After his arrest, he was asked by a Kagawad to
pull down his shorts at police headquarters and he
complied. Photographs of him were also taken after
the incident and presented during trial.
Are the
pictures admissible in evidence?
Ruling: Yes. The constitutional right of an accused
against self-incrimination proscribes the use of physical
or moral compulsion to extort communications and not
the inclusion of his body in evidence when it may be
material. Purely mechanical acts are not included in
the prohibition as the accused does not thereby speak
his guilt, hence the assistance and guiding hand of
counsel is not required. The essence of the right
against self-incrimination is testimonial compulsion,
that is the giving of evidence against himself through a
testimonial act. Hence, the taking of pictures of an
accused even without the assistance of counsel, being
a purely mechanical act, is not a violation of his
constitutional right against self-incrimination.
PEOPLE vs. RONDERO
320 SCRA 383 (1999)
Facts: Accused was charged with rape with homicide.
When the corpse of the 9-year old victim was found,
tightly gripped in her right hand were hair strands. To
enable the National Bureau of Investigation to conduct
an examination on the hair strands, it sent a fax
message to the police that hair strands be pulled, no
cut, from the suspect, from the four regions of his head
for comparison with the specimen.
By then the
accused was detained and he now claims that his hair
strands were taken by the police without his consent.
Was his right against self-incrimination violated?

TAU MU

Ruling: No. What is proscribed by the right against


self-incrimination is the use of physical or moral
compulsion to extort communication from the accused,
ACADCOM 2010; Contributors: Gene Geocaniga, Jarissa Guiani, Darlene Magabilen
TAU MU Page 172 of 179
Q: WHAT ABOUT SPERM TESTING?

PEOPLE vs. YATAR


428 SCRA 504 (2004)

TAU MU
TAU MU
TAU MU
TAU MU

Facts: Senator Miriam Defensor Santiago introduced


Senate Resolution 455, "directing an inquiry in aid of
legislation on the anomalous losses incurred by the
Philippines Overseas Telecommunications Corporation
(POTC),
Philippine
Communications
Satellite
Corporation (PHILCOMSAT), and PHILCOMSAT Holdings
Corporation (PHC) due to the alleged improprieties in
their operations by their respective Board of Directors.
Chief of Staf Inocencio, under the authority of Senator
Gordon, wrote Chairman Sabio of the PCGG inviting
him to be one of the resource persons in the public
meeting jointly conducted by the Committee on
Government Corporations and Public Enterprises and
Committee on Public Services. The purpose of the
public meeting was to deliberate on Senate Res. No.
455.
Chairman Sabio declined the invitation because
of prior commitment and at the same time, he invoked
Section 4(b) of E.O. No. 1 which provides that No
member or staf of the Commission shall be required to
testify or produce evidence in any judicial, legislative
or administrative proceeding concerning matters within
its official cognizance. Several notices by virtue of a
subpoena ad testificandum was issued to the
petitioner-directors but still, they did not comply with
the notice.

TAU MU

IN RE: SABIO
504 SCRA 704

TAU MU
TAU MU

Ruling: The contention is untenable. The kernel of the


right is not against all compulsion but against
testimonial compulsion. The right against selfincrimination is simply against the legal process of
extracting from the lips of the accused an admission of
guilt. It does not apply where the evidence sought to
be excluded is not an incrimination but as part of
object evidence. A person may be compelled to submit
to fingerprinting, photographing, paraffin, blood and
DNA, as there is no testimonial compulsion is involved.
It must also be noted that appellant in this case
submitted himself for blood sampling which was
conducted in open court on March 30, 2000, in the
presence of counsel.

The Fraternal
Ateneo de Davao

TAU MU
TAU MU

Facts: Yatar was sentenced to death for special


complex crime of rape with homicide. Subsequent
testing showed that the DNA of the sperm specimen
from the vagina of the victim was identical to the
semen of the appellants gene type.
Issue: Whether the blood sample taken from appellant
as well as the DNA tests were conducted in violation of
his right against self-incrimination.

KITY

TAU MU TAU MU TAU MU


TAU MU TAU MU TAU MU

CONSTITUTIONAL LAW II
Order of Saint Thomas More
Atty. Philip John Pojas/Atty. Rovyne G. Jumao-as, RN
University College of Law
and not the inclusion of his body in evidence when it
may be material. For instance, substance emitted from
the body of the accused may be received as evidence
in prosecution for rape, and morphine forced out of his
mouth may also be used as evidence against him.
Consequently, although accused insists that hair
samples were forcibly taken from him and submitted to
the NBI for forensic examination, the hair samples may
be admitted in evidence against him for what is
proscribed is the use of testimonial compulsion or any
evidence communicative in nature acquired from the
accused under duress.

officers of Philcomsat Holdings Corporation only when


the incriminating question is being asked, since they
have no way of knowing in advance the nature or
efect of the questions to be asked of them. That this
right may possibly be violated or abused is no ground
for denying respondent Senate Committees their power
of inquiry. The consolation is that when this power is
abused, such issue may be presented before the
courts. At this juncture, what is important is that
respondent Senate Committees have sufficient Rules to
guide them when the right against self-incrimination is
invoked. Sec. 19 reads: Privilege Against SelfIncrimination - A witness can invoke his right against
self-incrimination only when a question tends to elicit
an answer that will incriminate him is propounded to
him. However, he may ofer to answer any question in
an executive session. No person can refuse to testify or
be placed under oath or affirmation or answer
questions before an incriminatory question is asked.
His invocation of such right does not by itself excuse
him from his duty to give testimony. In such a case, the
Committee, by a majority vote of the members present
there being a quorum, shall determine whether the
right has been properly invoked. If the Committee
decides otherwise, it shall resume its investigation and
the question or questions previously refused to be
answered shall be repeated to the witness. If the latter
continues to refuse to answer the question, the
Committee may punish him for contempt for
contumacious conduct.
The same directors and officers contend that
the Senate is barred from inquiring into the same
issues being litigated before the CA and the
Sandiganbayan. Suffice it to state that the Senate
Rules of Procedure Governing Inquiries in Aid of
Legislation provide that the filing or pendency of any
prosecution of criminal or administrative action should
not stop or abate any inquiry to carry out a legislative
purpose. So long as the constitutional rights of
witnesses, like Chairman Sabio and his Commissioners,
will be respected by respondent Senate Committees, it
is their duty to cooperate with them in their eforts to
obtain the facts needed for intelligent legislative
action. The unremitting obligation of every citizen is to
respond to subpoenae, to respect the dignity of the
Congress and its Committees, and to testify fully with
respect to matters within the realm of proper
investigation.
LASERNA vs. DANGEROUS DRUGS BOARD (G.R.
No. 158633)
SJS vs. DANGEROUS DRUGS BOARD (G.R. No.
157870)
NOVEMBER 3, 2008

TAU MU

Facts: Section 36 of RA 9165 (Comprehensive


Dangerous Drugs Act of 2002) provides that Authorized
drug testing shall be done by any government forensic
laboratories or by any of the drug testing laboratories
accredited and monitored by the DOH to safeguard the
quality of the test results. The drug testing shall
employ, among others, two (2) testing methods, the
screening test which will determine the positive result
as well as the type of drug used and the confirmatory
test which will confirm a positive screening test. The
following shall be subjected to undergo drug testing:
(c) Students of secondary and tertiary schools; (d)
Officers and employees of public and private office; (f)
All persons charged before the prosecutor's office with
Issue: WON the subpoenae violated petitionera criminal ofense having an imposable penalty of
directors right againt self-incrimination.
imprisonment of not less than six years and one day
shall undergo a mandatory drug test; and (g) All
Ruling: The subpoenae did not violate their right
candidates for public office whether appointed or
against self-incrimination. It must be emphasized that
elected both in the national or local government shall
this right maybe invoked by the said directors and
undergo a mandatory drug test.
ACADCOM 2010; Contributors: Gene Geocaniga, Jarissa Guiani, Darlene Magabilen
TAU MU Page 173 of 179

TAU MU

TAU MU

TAU MU

TAU MU
TAU MU
TAU MU
TAU MU
TAU MU
TAU MU
TAU MU
TAU MU
TAU MU

Facts: Paynor was charged with the crime of murder.


On the strength of a positive identification of a witness,
appellant was detained at the police station.

TAU MU
TAU MU

Ruling: Unlike the situation covered by Sec. 36(c) and


(d) of RA 9165, the Court finds no valid justification for
mandatory drug testing for persons accused of crimes.
In the case of students, the constitutional viability of
the mandatory, random, and suspicionless drug testing
for students emanates primarily from the waiver by the
students of their right to privacy when they seek entry
to the school, and from their voluntarily submitting
their persons to the parental authority of school
authorities. In the case of private and public
employees, the constitutional soundness of the
mandatory, random, and suspicionless drug testing
proceeds from the reasonableness of the drug test
policy and requirement. However, the SC find the
situation entirely diferent in the case of persons
charged before the public prosecutor's office with
criminal ofenses punishable with six (6) years and one
(1) day imprisonment. The operative concepts in the
mandatory drug testing are "randomness" and
"suspicionless." In the case of persons charged with a
crime before the prosecutor's office, a mandatory drug
testing can never be random or suspicionless. The
ideas of randomness and being suspicionless are
antithetical to their being made defendants in a
criminal complaint. They are not randomly picked;
neither are they beyond suspicion. When persons
suspected of committing a crime are charged, they are
singled out and are impleaded against their will. The
persons thus charged, by the bare fact of being haled
before the prosecutor's office and peaceably
submitting themselves to drug testing, if that be the
case, do not necessarily consent to the procedure, let
alone waive their right to privacy. To impose mandatory
drug testing on the accused is a blatant attempt to
harness a medical test as a tool for criminal
prosecution, contrary to the stated objectives of RA
9165. Drug testing in this case would violate a persons'
right to privacy. Worse still, the accused persons are
veritably forced to incriminate themselves. Thus, in this
case, Section 36 (f) of RA 9165 was declared
unconstitutional and Section and (d) were declared
constitutional..
PEOPLE vs. PAYNOR
261 SCRA 615 (1996)

The Fraternal
Ateneo de Davao

TAU MU
TAU MU

Issue: WON the aforesaid provision violated the


constitutional right against self-incrimination.

KITY

TAU MU TAU MU TAU MU


TAU MU TAU MU TAU MU

CONSTITUTIONAL LAW II
Order of Saint Thomas More
Atty. Philip John Pojas/Atty. Rovyne G. Jumao-as, RN
University College of Law
Petitioner Laserna, Jr., as citizen and taxpayer,
seeks in his Petition for Certiorari and Prohibition that
the aforementioned provision of RA9165 be struck
down as unconstitutional for infringing on the
constitutional right to privacy, the right against
unreasonable search and seizure, and the right against
self - incrimination, and for being contrary to the due
process and equal protection guarantees.
On the other hand, Social Justice Society, a
registered political party, seeks to prohibit the
Dangerous Drugs Board and the Philippine Drug
Enforcement Agency from enforcing paragraphs (c),
(d), (f), and (g) of Sec. 36 of RA 9165 on the ground
that they are constitutionally infirm. For one, the
provisions constitute undue delegation of legislative
power when they give unbridled discretion to schools
and employers to determine the manner of drug
testing. For another, the provisions trench in the equal
protection clause inasmuch as they can be used to
harass a student or an employee deemed undesirable.
And for a third, a person's constitutional right against
unreasonable searches is also breached by said
provisions.

Issue: Whether there was a violation of his


constitutional right when the police stripped him of his
clothing and personal items and the same were later
introduced as evidence during trial.
Ruling: The protection of the accused under custodial
investigation, which is invoked by the appellant, refers
to testimonial compulsion. Section 12, Article 3 of the
Constitution provides that such accused shall have the
right to be informed of his right to remain silent, the
right to counsel and the right to waive the right to
counsel in the presence of the counsel, and that any
confession or admission obtained in violation of his
rights shall be inadmissible in evidence against him. As
held in People vs. Gamboa, this constitutional right
applies only against testimonial compulsion and not
when the body of the accused is proposed to be
examined. In fact, an accused may validly be
compelled to be photographed or measured, or his
garments or shoes removed or replaced, or to move his
body to enable the foregoing things to be done,
without running afoul of the proscription against
testimonial compulsion.
CHAVEZ vs. CA
Facts: The accused was charged with theft of a motor
vehicle. During the trial the prosecutor called on the
accused as a witness. The lawyer objected but was
overruled. According to the judge, you only object
when the statement is under scrutiny for trial. The
accused was convicted and as stated in the decision,
the accused was described as the star witness against
himself.
Ruling: The court held that in criminal cases the
accused may refuse altogether to take the witness
stand and answer all questions. The accused can
invoke Sec. 17. Neither a witness can be compelled to
take the witness stand. In contrary to the theory of the
judge, accused need wait until the incriminating
question is asked. He can refuse altogether to take the
witness stand.
This is called a PROHIBITION OF INQUIRY wherein a
witness can refuse altogether to take the witness stand
or to answer all questions whether incriminating or not.
The court also stated that it should be only if the
accused is allowed to take the stand since the purpose
of the fiscal is mainly to incriminate him. This would be
contrary to the rights of the accused.
NOTE: The right can also be waived if the accused
willingly takes the stand.
VILLAFLOR vs. SUMMERS
Facts: The accused was charged for adultery and upon
motion of the prosecutor, the judge ordered her
examined to find out if she is pregnant. She refused
and was detained for contempt. She went to the SC.
Ruling: The court ruled that the right extends to
testimonial self incrimination only. Order of the Court
was only for an ocular inspection of the body of the
accused. So as not to embarrass the witness, the court
ordered the physicians not to do any other means more
than necessary. (NOTE: If this case happened now, the
woman can also invoke the right to privacy and it will
most probably be ordered in favor of the woman)
BELTRAN vs. SAMSON
Facts: Accused was charged with falsification and the
fiscal ordered him to take a specimen of his writing and
afterwards will compare his handwriting with the

ACADCOM 2010; Contributors: Gene Geocaniga, Jarissa Guiani, Darlene Magabilen


TAU MU Page 174 of 179

Ruling: The court held this as a violation of the right to


self-incrimination. Asking the accused to write is a
positive estimated act. Writing is more than moving the
hand, the body or the fingers. It is more than just a
mechanical act because it requires the use of
intelligence and attention.
DISTINCTION BETWEEN
VILLAFLOR CASES:

THE

BELTRAN

AND

TAU MU
TAU MU
TAU MU

SECTION 18 CANNOT BE INVOKED IN THE


FOLLOWING:
1. In times of war, citizens can be forced to render
military service or civil service as required by the
Constitution [Art. II Sec. 4];
2. In return-to-work orders;

If an employee participates in a strike, the


employer can force him to resume work,
otherwise he is deemed resigned.
This NOT compulsion because if this is not allowed
the employer would be prejudiced because employees
refuse to work for flimsy reasons.
3. If you are appointed as counsel de officio of the
accused;

TAU MU

INVOLUNTARY SERVITUDE
It is every condition of enforced or compulsory service
of one to another no matter under what form such
servitude may be disguised.

TAU MU

NOTE: There is no law authorizing as a penalty for a


crime to be sufered to work. If in any case, the judge
has to embody in the decision the order to work.

TAU MU

SECOND PARAGRAPH
Freedom from Involuntary Servitude
It is the liberty not to be compelled
against your will to work whether paid or not.

TAU MU

Political Prisoner a person detained solely for what he


believes in (caveat: not a definite meaning).
Under Amnesty International - the definition is broader
such that they are those represented for acts
done pursuant to his political beliefs.

TAU MU

FIRST PARAGRAPH
The first Part speaks of the freedom of conscience or
the freedom of thoughts. What is protected by this
section are the political beliefs (the principles are
similar to freedom of religion).

TAU MU

Section 18. (1) No person shall be detained


solely by reason of his political beliefs and
aspirations.
(2) No involuntary servitude in any form shall
exist except as a punishment for a crime
whereof the party shall have been duly
convicted.

TAU MU
TAU MU

Evidence
Villaflor was only asked to explain something already in
existence, whereas in Beltran, accused was asked to
produce evidence not yet in existence.

The Fraternal
Ateneo de Davao

TAU MU
TAU MU

Classify the act


In the Villaflor case, the accused was not asked to
perform a positive act. She was only asked to perform
a negative act, or to present an examination of herself,
whereas, in the Beltran case, the accused was asked to
perform a positive act.

KITY

TAU MU TAU MU TAU MU


TAU MU TAU MU TAU MU

CONSTITUTIONAL LAW II
Order of Saint Thomas More
Atty. Philip John Pojas/Atty. Rovyne G. Jumao-as, RN
University College of Law
writing of the falsified documents. The accused refused
invoking the right against self-incrimination.

4. Punishment for a crime for which the party has


been duly convicted
5. When he is enlisted in the services of a merchant
vessel; (ROBERTSON vs. BALDWIN)
6. Posse Comitatus for the apprehension of
criminals. The state may require some groups of
people to assist in the pursuit of some criminals under
the Police power
7. "Patria Potestas" unemancipated minors under
parental power. (They must be minors under parental
authority)
Q: Can a person who ate in a restaurant and did
not pay, be compelled to render service for the
establishment?
A: No. He can still invoke section 18 which prohibits
involuntary servitude.
CASES: INVOLUNTARY SERVITUDE
ACLARACION vs. GATMAITAN
Facts: Petitioner was a stenographer with the CFI.
After working one year with the court, he transferred to
another government entity, but he left some
untranscribed notes of some cases on appeal. The
court ordered him to finish the transcription, but he
refused. He was reprimanded fro his refusal. (This case
became moot and academic because there was a
settlement subject to the condition that he will
transcribe the notes)
Ruling: The court held that the RTC stenographer can
still be compelled to transcribe notes on cases on
appeal without violating the right against involuntary
servitude. This is part of the inherent power of the
court necessary for the ordinance and efficient
administration of justice.
SARMIENTO vs. TUICO
162 SCRA 676 (1988)
Facts: Petition for certiorari to review the order of the
NLRC. Asian Transmission Corporation terminated the
services of the petitioner for allegedly carrying deadly
weapon in the premises. The striking workers were
ordered to return to work immediately.
Issue: Whether the return to work order results to
involuntary servitude.
Ruling: while one purpose of the return-to-work order
is to protect the workers who might otherwise be
locked out by the employer for threatening or waging
the strike, the more important reason is to prevent
impairment of the national interest in case the
operations of the company are disrupted by a refusal of
the strikers to return to work as directed. The return to
work order imposes a duty. It is not considered violative
of the right against involuntary servitude.
Section 19. (1) Excessive fines shall not be
imposed, nor cruel, degrading or inhuman
punishment inflicted. Neither shall death penalty
be imposed, unless, for compelling reasons
involving heinous crimes, the Congress hereafter
provides for it. Any death penalty already
imposed shall be reduced to reclusion perpetua.
(2) The employment of physical, psychological,
or degrading punishment against any prisoner or
detainee or the use of substandard or
inadequate penal facilities under subhuman
conditions shall be dealt with by law.
Section 19 is not self-executing.

ACADCOM 2010; Contributors: Gene Geocaniga, Jarissa Guiani, Darlene Magabilen


TAU MU Page 175 of 179

TAU MU
TAU MU
TAU MU
TAU MU

INSTANCES WHEN PUNISHMENT MAY ALSO BE


DEEMED CRUEL AND UNUSUAL:
1. If it inherently involves so much pain and
sufering that civilized people cannot tolerate it.
2. If in the sense of being previously unknown it
is imposed for a given ofense

TAU MU

To be cruel and unusual or excessive punishment


w/i the meaning of the constitution, the penalty must
be flagrantly disproportionate to the ofense no matter
under what circumstances the ofense may be
committed.
If the punishment involves torture or lingering
death.
Unconstitutional punishment implies something
inhuman and barbarous, something more than the
mere extinguishment of life.

TAU MU

CRUEL, DEGRADING and INHUMAN PUNISHMENT

TAU MU

2
CONCEPTS
OF
CRUEL
AND
UNUSUAL
PUNISHMENT:
1. Those which public sentiment would regard as
cruel and obsolete to law, refers to form, not on the
severity or seriousness of the punishment. An example
crucifixion
2. Those which are disproportionate to the
ofense as to shock the moral senses.

TAU MU

II. Cruel, degrading or inhuman punishment. So


when is punishment considered cruel, inhuman or
degrading? If it is flagrantly and plainly oppressive,
wholly disproportionate to the ofense as to shock the
moral sense of the community. When it involves torture
or lingering death.

TAU MU

So this only applies to criminal cases because fines are


imposed only in criminal cases. And it only applies to
the fine imposed. If the court goes beyond the range
provided for by law, then that is excessive fines. So
what is the yardstick to measure that the fine imposed
is not excessive. (what must the judge consider when
imposing a fine)
1. The financial condition of the convict;
2. The amount fixed should be within the limit
established by law;
3. The mitigating and aggravating circumstances
attending the crime

TAU MU
TAU MU

I. Excessive fines shall not be imposed. What is a


fine? Fine is a penalty. So for example A was convicted
of Arson with homicide. And the court awarded moral
damages of 500 million, can A invoke his rights against
excessive fines? No. Fines are provided by RPC,
damages are not fines. Damages are civil liability.

The Fraternal
Ateneo de Davao

TAU MU
TAU MU

3. Death penalty unless for compelling reasons


involving heinous crimes;
4. Right to be protected against physical, psychological
or degrading punishment;
5. That any prisoner or detainee shall be entitled to
adequated, standard penal facilities.

KITY

TAU MU TAU MU TAU MU


TAU MU TAU MU TAU MU

CONSTITUTIONAL LAW II
Order of Saint Thomas More
Atty. Philip John Pojas/Atty. Rovyne G. Jumao-as, RN
University College of Law
5 RIGHTS GUARANTEED BY SECTION 19:
1. Right to be protected against cruel and inhuman
punishments;
2. Excessive fines shall not be imposed;
Yardstick to measure that the fine imposed is
not excessive. (What the judge must consider when
imposing a fine)
a. The financial condition of the
convict;
b. The amount fixed should be within
the limit established by law;
c. The mitigating and aggravating
circumstances attending the crime

3. If it is excessive and serves a legislative


purpose.
4. If popular sentiments abhors it even if not
excessive or with a legislative purpose.
DEATH PENALTY
Congress can re-impose death penalty (conditions):

a.
b.
c.
SC: If
enough to

by enacting a law;
for heinous crimes;
there must be compelling reason.
a crime is heinous, it is compelling reason
impose the death penalty

EXCESSIVE FINE
A fine is excessive, when under any circumstance, it is
disproportionate to the ofense.
Note: Fr. Bernas says that the accused cannot be
convicted of the crime to which the punishment is
attached if the court finds that the punishment is cruel,
degrading or inhuman.
Reason: Without a valid penalty, the law is not a penal
law.
STANDARDS USED
1. The punishment must not be so severe as to be
degrading to the dignity of human beings.
2. It must not be applied arbitrarily.
3. It must not be unacceptable to contemporary
society
4. It must not be excessive, i.e. it must serve a
penal purpose more efectively than a less severe
punishment would.
Penalty is cruel, degrading and inhuman if it is
flagrantly
and
plainly
oppressive,
wholly
disproportionate to the ofense as to shock the moral
sense of the community. When it involves torture or
lingering death.
Q: What is the yardstick to measure if the
penalty is justified?
A: The penalty imposed should be proportional to the
ofense committed. Take note that public officers may
be meted out graver penalties because of public office
is a public trust.
LIM vs. CA: Estafa under PD 818 is punished as
severely as reclusion perpetua.
Q: Is the penalty disproportionate to the offense
committed?
A: No, because this kind of penalty is actually
prescribed by law. It is actually provided for in PD 818
that for this kind of estafa, this is the penalty.
Punishment authorized by a statute is not cruel,
degrading or disproportionate to the nature of the
ofense unless it is flagrantly and plainly oppressive.
ZALDIVAR
vs.
SANDIGANBAYAN:
indefinite
suspension is not cruel, degrading or inhuman.
III. Death Penalty. Under the Constitution, there is no
death penalty. When the Constitution was ratified in
1987, it had no death penalty. But it had a provision
that said that: Neither shall the death penalty be
imposed, so that is the general rule, unless for
compelling reasons involving heinous crimes, the
Congress hereafter provides for it. So the Congress
enacts a law providing for the death penalty. (RA 7659)
The first person sentenced to die under RA 7659 was
Echegaray. His lawyers questioned the death penalty
claiming that this is cruel and inhuman punishment.
The SC said that all punishments are cruel. What the
Constitution means is something inhuman, and
barbarous. Something more than just extinguishment

ACADCOM 2010; Contributors: Gene Geocaniga, Jarissa Guiani, Darlene Magabilen


TAU MU Page 176 of 179

TAU MU
TAU MU
TAU MU
TAU MU
TAU MU
TAU MU
TAU MU
TAU MU

Ruling: The evil of a crime may take various forms.


There are crimes, like murder, rape, kidnapping, which
are, by their nature despicable, either because life was
callously taken or the victim is utterly dehumanized.
There are crimes, like bribery, destructive arson, drug
ofenses involving government officials, in which the
abdomination lies in the significance and implications
of the subject criminal acts in the scheme of the larger
socio-political and economic context. There is no need
for Congress to state the compelling reason for each
and every heinous crime and statistical proof that such
compelling reason actually exists. The elements of
heinousness and compulsion are inseparable and are in
fact interspersed with each other. The Court has no
doubt that insofar as heinousness is concerned
Congress has correctly identified the crimes warranting
the death penalty.
The claim of accused that the death penalty for
the crime of rape is cruel and inhuman is based on a
decision of the United States Supreme Court. In turn,
said Court made a finding that (1) the public has
manifested its rejection of the death penalty as a
proper punishment for the crime of rape by the willful
omission by state legislatures to include rape in their
death penalty statutes, and (2) while the crime
involves a dastardly contemptuous violation of a
womans spiritual integrity and psychological balance,
it does not involve the taking of life. The first ground,
however has no bearing on Philippine experience and
the context of our culture. As to the second, we
disagree with the notion that whether or not the crime
warrants the death penalty is the attendance of the
circumstance of the death of the victim because it goes
back to the idea of retributive justice, of an eye for an
eye. The death penalty is imposed because the
perpetrators have committed unforgivable acts causing
irreparable and substantial injury to the victim and
society so that they must be permanently prevented
form repetition of their acts.

TAU MU

Facts: Accused was convicted of incestuous rape and


sentenced to death under R.A. No. 7659 which
reimposed the death penalty. He now argues that the
death penalty law is unconstitutional for having been
imposed in the absence of compelling reasons
therefore and that the death penalty for rape is a cruel,
excessive and inhuman punishment.

TAU MU
TAU MU

PEOPLE vs. ECHEGARAY


267 SCRA 682 (1997)

The Fraternal
Ateneo de Davao

TAU MU
TAU MU

CASES: PROHIBITED PENALTIES

KITY

TAU MU TAU MU TAU MU


TAU MU TAU MU TAU MU

CONSTITUTIONAL LAW II
Order of Saint Thomas More
Atty. Philip John Pojas/Atty. Rovyne G. Jumao-as, RN
University College of Law
of life. The death penalty is not cruel and inhuman
because cruelty should be inherent in the method and
not the punishment itself. The death penalty requires
two reasons: so there must be a compelling reason and
it only involves heinous crimes. So when is a crime
considered heinous? They are crimes that by their
nature are despicable. There are other crimes like
bribery, destructive arson and drug ofenses, the
abomination lies in the significance and the implication
of the subject criminal act in the scheme of the larger
socio-political and economic context. (What is the
implication or the efect on society). What is this
compelling reason? That the crime is heinous is reason
compelling enough to bring back the death penalty.
Heinous crime=compelling reason = death penalty
justified. Kasi diba sabi ng SC it is not the penalty, it is
the method. So balik na naman ang lawyers ni
Echegaray saying that lethal injection is cruel and
inhuman daw. The SC said that any infliction of pain in
the lethal injection is incidental to the execution of the
death penalty.

ECHEGARAY vs. SECRETARY


297 SCRA 754 (2002)
Facts: In 1997, Congress passed R.A. No. 8177,
designating death by lethal injection as the method of
carrying out the death penalty. Pursuant to the law,
the Secretary of Justice promulgated the implementing
rules and directed the Director of the Bureau of
Corrections to prepare the Lethal Injection Manual.
Echegaray, a death convict, argues that death by the
lethal injection is unconstitutional for being a cruel,
degrading and inhuman punishment.
Ruling: All punishments are cruel but of course the
Constitution does not mean that crime, for this reason,
is to go unpunished. It implies something inhuman and
barbarous,
something
more
that
the
mere
extinguishments of life. The cruelty against which the
Constitution protects a convicted man is cruelty
inherent in the method of punishment, not the
necessary sufering involved in any method employed
to extinguish life humanely. Any infliction of pain in
lethal injection is merely incidental in carrying out the
execution of the death penalty.
Courts in other
countries have found that the few minutes of pain does
not rise to a level of constitutional violation. Likewise,
what is cruel and unusual must draw its meaning from
the evolving standards of decency, and the widespread
of lethal injection indicates that it comports with
contemporary norms.
LIM vs. PEOPLE
390 SCRA 194 (2002)
Facts: The constitutionality of PD 818, a decree which
amended Article 315 of the Revised Penal Code by
increasing the penalties for estafa committed by
means of bouncing checks, is being challenged in this
petition for certiorari, for being violative of the due
process clause, the right to bail and the provision
against cruel, degrading or inhuman punishment
enshrined under the Constitution. Petitioner spouses
issued to private respondent two postdated checks.
One check was dishonored upon presentment for
having been drawn against insufficient funds. When
petitioners reneged on their promise to cover the
amount, PR filed a complaint for estafa against them.
The trial court ruled against petitioner spouses.
Issue: WON PD 818 was enacted in contravention of
Section 19 of Article III of the Constitution
Ruling: A punishment authorized by statute is not
cruel, degrading or disproportionate to the nature of
the ofense unless it is flagrantly and plainly oppressive
and wholly disproportionate to the nature of the
ofense as to shock the moral sense of the community.
It takes more than merely being harsh, excessive, out
of proportion or severe for a penalty to be obnoxious to
the Constitution. Based on this principle, the Court has
consistently overruled contentions of the defense that
the penalty of fine or imprisonment authorized by the
statute involved is cruel and degrading. In one case,
the SC ruled that prohibition against cruel and unusual
punishment is generally aimed at the form or character
of the punishment rather than its severity in respect of
its duration or amount, and applies to punishments
which never existed in America or which public
sentiment regards as cruel or obsolete. This refers, for
instance, to those inflicted at the whipping post or in
the pillory, to burning at the stake, breaking on the
wheel, disemboweling and the like. The fact that the
penalty is severe provides insufficient basis to declare
a law unconstitutional and does not, by that
circumstance alone, make it cruel and inhuman.

ACADCOM 2010; Contributors: Gene Geocaniga, Jarissa Guiani, Darlene Magabilen


TAU MU Page 177 of 179

TAU MU
TAU MU

2) Thus, if an accused fails to pay the fine imposed


upon him, this may result in his subsidiary
imprisonment because his liability is ex delicto and not
ex contractu.

TAU MU

DEFINITION OF DEBT UNDER SECTION 20:


1) Debt refers to a CONTRACTUAL obligation, whether
express or implied, resulting in any liability to pay
money. Thus, all other types of obligations are not
within the scope of this prohibition.

TAU MU

In non-payment of a debt, the debt assured in this


section refers only to CONTRACTUAL DEBT (that which
arises from contracts).
Reason: this kind of contract is civil in nature.

TAU MU

NOTE: However, poll tax, as a condition precedent for


the right to vote might be contrary to the
Constitutional prohibition on literacy, property, or other
substantive requirement as precedent to the right to
vote.

TAU MU

According to Fr. Bernas, poll tax refers to residence


certificate or cedula and the tax paid is a precedent to
the exercise of the right of sufrage.

TAU MU

POLL TAX
Tax paid as a condition precedent for the exercise of
sufrage.

TAU MU

SEC. 20: No person shall be imprisoned for


nonpayment of debt or poll tax (cedula)

TAU MU

Ruling: The court held that malversation is a serious


ofense as it involves betrayal of public trust so the
penalty cannot be considered unduly severe. At any
rate, the remedy is to ask Congress to amend the law.

TAU MU
TAU MU

Facts: Accused who was a municipal treasurer was


accused of misappropriating the amount of P21,940.70.
He was sentenced to 11 to 16 years of imprisonment.
On appeal, he argues that the Revised Penal Code was
enacted in 1952 and since then the peso has
devaluated so that the penalty imposed on him has
become obsolete, excessive and oppressive inflation.
He pointed out that the penalty was severe as to shock
the moral senses.

The Fraternal
Ateneo de Davao

TAU MU
TAU MU

AGBANLOG vs. PEOPLE

KITY

TAU MU TAU MU TAU MU


TAU MU TAU MU TAU MU

CONSTITUTIONAL LAW II
Order of Saint Thomas More
Atty. Philip John Pojas/Atty. Rovyne G. Jumao-as, RN
University College of Law
In this case, the increase in the penalty, far
from being cruel and degrading, was motivated by a
laudable purpose, namely, to efectuate the repression
of an evil that undermines the countrys commercial
and economic growth, and to serve as a necessary
precaution to deter people from issuing bouncing
checks. The fact that PD 818 did not increase the
amounts corresponding to the new penalties only
proves
that
the
amount is
immaterial
and
inconsequential. What the law sought to avert was the
proliferation of estafa cases committed by means of
bouncing checks. Taking into account the salutary
purpose for which said law was decreed, we conclude
that PD 818 does not violate Section 19 of Article III of
the Constitution. Moreover, when a law is questioned
before the Court, the presumption is in favor of its
constitutionality. To justify its nullification, there must
be a clear and unmistakable breach of the Constitution,
not a doubtful and argumentative one. T he burden of
proving the invalidity of a law rests on those who
challenge it. In this case, petitioners failed to present
clear and convincing proof to defeat the presumption
of constitutionality of PD 818.

3) A FRAUDULENT debt may result in the


imprisonment of the debtor if:
A. The fraudulent debt constitutes a crime
such as estafa and
B. The accused has been duly convicted.
NON-PAYMENT OF POLL TAX
poll tax can be imposed but you cannot be
imprisoned for not paying it
imposition of tax on right to vote --- BERNAS: you
still cannot go to jail for not paying it because its
imposition is prohibited by the Constitution
no jurisprudence yet
NON-IMPRISONMENT FOR NON-PAYMENT OF
DEBT
contractual debt only
you cannot be imprisoned for non-payment of
contractual debt but you can be proceeded against for
estafa
LABOR CODE: minimum wage --- if you pay your
employee the amount less than that of the minimum
wage, you cannot be imprisoned for it
CASES: NON-PAYMENT OF DEBT
VERGARA vs. GEDORIO
402 SCRA 520(2003)
Facts: Petitioners are tenants of property and were
arrested for indirect contempt for not complying with
the probate courts orders directing them to pay
monthly rentals. Some of the petitioners, together with
the other tenants, informed the probate court that they
are freezing their monthly rentals as they are in a
quandary as to whom to pay the rentals. The probate
court found them guilty and ordered them to pay a fine
and undergo imprisonment until they comply with the
order to pay rentals.
Issue: Whether the order to undergo imprisonment
violates the right to non-imprisonment of dent.
Ruling: The Constitution provides that no person shall
be imprisoned for debt. Debt refers to a civil debt or
one not arising from a criminal ofense. It means that
any liability to pay arising out of a contract, express or
implied. The payment of rentals is covered by the
Constitutional guarantee against imprisonment.
PEOPLE vs. DELA CRUZ
Facts: The accused sold meat for P0.30 and at that
time, there was a price ceiling of P0.20 established by
law. He was charged and convicted for overpricing. The
sentence was 5 years imprisonment plus P5,000.00
fine. Also he cannot exchange in retail trade for 5 years
(It was 195). The conviction was challenged as violative
of his right against cruel and unusual punishment.
Ruling: The court held the punishment as not shocking
nor unusual. When seen with the policy of the
government in protecting the public welfare, it is not
disproportionate. Although the accused only earned
P0.10, it is possible that he will earn a lot more if he
will not be prosecuted. The damage to the government
cannot be measured in terms of what he earned. This
does not fall under the meaning of cruel and unusual
act according to the 2 concepts.
PEOPLE vs. DACUYCUY
Facts: Violation of the Magna Carta for public school
teachers provided for penalty of fine (P500-P1000) and

ACADCOM 2010; Contributors: Gene Geocaniga, Jarissa Guiani, Darlene Magabilen


TAU MU Page 178 of 179

Ruling: the court ruled giving the 2 concepts for cruel


and unusual. It is cruel an unusual when it is
barbarous, obsolete or unknown to law, and when it is
disproportionate as to shock the moral conscience. The
law does not violate the right against cruel and unusual
punishment based on the 2 concepts. If one of the 2 is
violated, the law is unconstitutional. However, it is
unconstitutional for being an undue delegation of
legislative power.
LOZANO vs. MARTINEZ

Trust Receipt Law


Issue: Whether the trust receipt law violates the right
on non-imprisonment for non-payment of debt.

TAU MU
TAU MU

Ruling: The court held that the accused is wrong.


Indemnity arises ex-delicto and not ex-contractu. The
right only applies to debts arising from contracts. Also
the judge was wrong because subsidiary imprisonment
applies only to FINES and not to indemnity.

TAU MU

Facts: Accused was sentenced to imprisonment and


ordered to pay an indemnity of P200.00 to the event
that he is insolvent, he will serve subsidiary
imprisonment. Accused challenged this for being
violative of the right against imprisonment for nonpayment of debt.

TAU MU

AJENO vs. INSERTO

TAU MU

Ruling: The court rules No. The law does not seek to
enforce payment of loan under the pain of
imprisonment. The law punishes dishonesty and abuse
of confidence in handling money or goods to the
prejudice of others. What is protected is not the loan,
which is a separate contract, but the security or
property given.

TAU MU

PEOPLE vs. NITAFAN

TAU MU
TAU MU

Ruling: The court held BP 22 does not permit the


failure of the maker to pay his debt but only the
making and issuance of a worthless check. The
purpose of the law is not to force people to pay their
debts under the pain of imprisonment. The purpose is
to prohibit the issuance of worthless checks. This was
likened to introducing garbage to the bloodstream of
the economy.

The Fraternal
Ateneo de Davao

TAU MU
TAU MU

Bouncing Checks Law


Issue: Whether or not BP 22 violates the right against
non-imprisonment for non-payment of debt.

KITY

TAU MU TAU MU TAU MU


TAU MU TAU MU TAU MU

CONSTITUTIONAL LAW II
Order of Saint Thomas More
Atty. Philip John Pojas/Atty. Rovyne G. Jumao-as, RN
University College of Law
imprisonment at the discretion of the Court. The
accused challenged this as cruel and unusual.

TAU MU
TAU MU
TAU MU

ACADCOM 2010; Contributors: Gene Geocaniga, Jarissa Guiani, Darlene Magabilen


TAU MU Page 179 of 179

You might also like